Está en la página 1de 245

EX-2022-988209-UNC-ME#FP

ANEXO
calificación será individual; en él se interroga sobre el programa trabajado durante el año.
Alumno libre: trabajo práctico y examen final escrito y oral (ver 6.3 Régimen de
cursado).Debera aprobar el TP antes para pasar a la instancia escrita y luego oral

Evaluaciones de recuperación
Se podrá recuperar, en caso de ausencia o reprobación, uno solo de los parciales
establecidos por condición y un trabajo práctico evaluativo.
Serán escritos a desarrollar.

Criterios de evaluación

Se tendrán en cuenta los siguientes criterios:


● Posicionamiento crítico y análisis reflexivo.
● Establecimiento de relaciones entre las nociones fundamentales del programa con
las situaciones prácticas trabajadas en clase, las entrevistas realizadas, el
proyecto y el diseño curricular.
● Elaboración de conceptualizaciones, argumentaciones consistentes y
posicionamientos teórico metodológicos, en forma oral y escrita y participación
grupal.
● Análisis crítico del material sobre el cual se trabaja
● Pertinencia de la respuesta en relación a lo que se pregunta.
● Claridad conceptual.
● En el caso de las evaluaciones escritas, se agrega:
● La coherencia y la legibilidad del texto.
● Capacidad de síntesis

BIBLIOGRAFÍA (Obligatoria y de consulta)

1-El Oficio de enseñar en contexto: Dimensiones política, histórica, social, cultural,


institucional, subjetiva. La tarea de enseñar a través del tiempo y los desafíos que
imponen las condiciones de época. La escuela secundaria y sus transformaciones.
La relevancia de las condiciones de escolarización en las trayectorias escolares.

● Alterman Nora y Coria Adela Coord (2014) “Cuando de ensenar se trata” Cap. 1 Ed
Brujas. Córdoba
● Bauman Z. “Modernidad líquida” Conferencia (1999) disponible en
http://www.oei.org.ar/edumedia/pdfs/T14_Docu1_La modernidad
líquida_Bauman.pdf
● Dubet, F. (2006). El declive de la institución. Profesiones, sujetos e individuos en la
modernidad. Barcelona. Gedisa. Introducción y Cap. 1
● Feldman, D. (2010) Enseñanza y escuela. Buenos Aires. Paidós. Cap.1
● Maldonado, H compilador (2013) Aportes para mejorar los aprendizajes en la
Universidad. UNC.

IF-2023-00101291-UNC-SHCD#FP 9

página 9 de 13
EX-2022-988209-UNC-ME#FP
ANEXO
● Merieu P. “El significado de educar en un mundo sin referencias” en Ministerio de
Educación, Ciencia y Tecnología de la Nación Dirección Nacional de Gestión
Curricular y Gestión Docente Área de Desarrollo Profesional Docente junio de 2006
Ministerio de Educación, Ciencia y Tecnología
● Romero Cecilia,(2013) Construyendo justicia educativa en “Hacia la innovación en
la escuela secundaria” Ed Comunicarte. Córdoba
● Ministerio de Educación Ciencia y Tecnología de la Nación Rev. Explora “Docentes
en la tarea de cruzar fronteras y tender puentes” Revista del Ministerio de
Educación de la Nación “El monitor” Nº 25 Junio 2010. Dossier “Ser Docente hoy”.
● Kaplan, C. V. . (2021) La implicación afectiva en tiempos de pandemia y en la
postpandemia. Educar para una sociedad de reciprocidades Anales De La
Educación Común, 2(1-2), 104-113. Recuperado a partir de
https://cendie.abc.gob.ar/revistas/index.php/revistaanales/article/view/490.
● Ministerio de Educación de la Nación Argentina Pensar los vínculos en tiempos de
pandemia : la escuela como un lugar de cuidado. - 1a ed - Ciudad Autónoma de
Buenos Aires : Ministerio de Educación de la Nación, 2022. (hasta pág. 20)

Ampliatoria
● Alliaud, A. (2004) La escuela y los docentes: ¿Eterno retorno o permanencia
constante? Apuntes para abordar una particular relación desde una perspectiva
biográfica”. Cuaderno de Pedagogía Rosario. Año 7 Nro. 12. Rosario. El Zorzal
● Alliaud, A. y Antelo, E. (2009). Los gajes del oficio. Enseñanza, pedagogía y
formación. Buenos Aires. Aique.
● Dussel, I. Brito, A. Núñez, P. (2007) Más allá de la crisis. Visión de alumnos y
profesores de la escuela secundaria argentina. Buenos Aires. Fundación
Santillana.
● Introducción.
● Southwell, M. (Comp.) (2012) La educación secundaria frente a la obligatoriedad:
una ecuación compleja. Entre generaciones. Exploraciones sobre educación,
cultura e instituciones. FLACSO Argentina. Buenos Aires.Homo Sapiens Editorial
● Frigerio, G. y Diker, G. (2005). Educar, ese acto político. Buenos Aires. Del estante.

2- Sujeto que enseña y sujeto que aprende: relaciones posibles. Deseo de


saber, deseo de enseñar, procesos transferenciales. Confianza en el maestro
confianza .en el discípulo. Enseñar, cuidar, acompañar.
● Cornú, Laurence (2004) Transmisión e institución del sujeto. Transmisión simbólica,
sucesión, finitud. En Frigerio G y Diker G. (Comps) La transmisión en las
sociedades, las instituciones y los sujetos. Un concepto de la acción educativa” Bs
As
● Cornú Laurence,( 2004) “La ética de la oportunidad” en Frigerio, Graciela y Diker,
Gabriela, coordinadores. Una ética en el trabajo con niños y adolescentes: la
habilitación de la oportunidad Novedades Educativas. Bs As
● Freud, S. (1914) Sobre la psicología del colegial, en Obras Completas - Editorial
Biblioteca Nueva, Madrid.
● Lajounquiere, Leandro (2011). La educación para la realidad del deseo. Pag 53.

IF-2023-00101291-UNC-SHCD#FP10

página 10 de 13
EX-2022-988209-UNC-ME#FP
ANEXO
Ed. Nueva Visión.
● La palabra educativa entre el laissez faire y la frustración. Pag 65
● Maldonado Horacio (2001) “Escritos sobre Psicología y Educación.” Espartaco.
Córdoba. Cap.5 y 7.
● Skliar C. (2003) ¿ Y si el otro no estuviera allí? Notas para una pedagogía
(improbable) de la diferencia. Miño y Dávila. Bs. As. Cap. ¿Por qué nos
transformamos tanto?

Ampliatoria
● Cordie Anny (1998) Malestar en el docente. Nueva Visión. Bs as.
● Greco, M.B. (2012). Emancipación, educación y autoridad. Prácticas de formación
y transmisión democrática. Buenos Aires. Noveduc.

3- El currículum: enfoques y tradiciones. Claves de lectura y análisis crítico


de los contenidos curriculares en nivel secundario y de la formación docente.
Criterios de selección, organización y secuenciación de los contenidos. El
curriculum oculto.

● Bernstein, B. (1998). Pedagogía, Control simbólico e identidad. Madrid. Morata.


Cap.1 y 2
● Chevallard, Y. (1991) La transposición didáctica. Buenos Aires, Aique. Cap. 1
● Dussel I. y Southwell M. (2009) El curriculum. Explora. Programa de capacitación
multimedial. Ministerio de Educación.
● Feldman Daniel, (2009) “La innovación escolar en el currículum de la escuela
media” En Romero C, Claves para mejorar la escuela secundaria. Noveduc. Bs As.
● Gvirtz S y Palamidesi M.() “El ABC de la tarea docente” AIQUE. Bs As
● Terigi F (2009) : “Los cambios en el formato de la escuela secundaria argentina por
qué son necesarios, por qué son tan difíciles”. Revista Propuesta Educativa N 19
● Terigi (1999) Curriculum. Itinerarios para aprehender un territorio. Buenos Aires;
Santillana. Capítulo 2 y 3

Ampliatoria
● Alterman, N. (2012) Desarrollo Curricular centrado en la escuela y en el aula.
Aportes para reflexionar sobre nuestras prácticas. Unión de Educadores de la Prov.
de Córdoba. Ministerio de Gobierno, Educación y Cultura. Disponible en:
http://www.uepc.org.ar/conectate/wp-content/uploads/2012/05/Desarrollo-curricular-
centrado-en-la-escuela-y-en-el-aula.pdf
● Frigerio, G y Diker, G (comps.). (2010). Educar: saberes alterados. Buenos Aires.
Del estante.• Gimeno Sacristán, J., Feito Alonso, R., Perrenoud, P., y Clemente
Linuesa, M. (2011). Diseño, desarrollo e innovación del currículum. Madrid. Morata.
● Connell, R. 1999. Escuelas y Justicia Social. Justicia curricular cap. IV. Madrid.
Morata

4- La construcción de propuestas de enseñanza: La planificación como

IF-2023-00101291-UNC-SHCD#FP11

página 11 de 13
W
ha
ts
ap Ale
p: jan
35 dr
17 ía

-5-
61 Im
17 pre
27 si
- 3 one
51 s
76
11
63
2
W
ha
ts
ap Ale
p: jan
35 dr
17 ía

-9-
61 Im
17 pre
27 si
- 3 one
51 s
76
11
63
2
W
ha
ts
ap Ale
p: jan
35 dr
17 ía

- 10 -
61 Im
17 pre
27 si
- 3 one
51 s
76
11
63
2
W
ha
ts
ap Ale
p: jan
35 dr
17 ía

- 11 -
61 Im
17 pre
27 si
- 3 one
51 s
76
11
63
2
W
ha
ts
ap Ale
p: jan
35 dr
17 ía

- 12 -
61 Im
17 pre
27 si
- 3 one
51 s
76
11
63
2
W
ha
ts
ap Ale
p: jan
35 dr
17 ía

- 13 -
61 Im
17 pre
27 si
- 3 one
51 s
76
11
63
2
W
ha
ts
ap Ale
p: jan
35 dr
17 ía

- 14 -
61 Im
17 pre
27 si
- 3 one
51 s
76
11
63
2
W
ha
ts
ap Ale
p: jan
35 dr
17 ía

- 15 -
61 Im
17 pre
27 si
- 3 one
51 s
76
11
63
2
W
ha
ts
ap Ale
p: jan
35 dr
17 ía

- 16 -
61 Im
17 pre
27 si
- 3 one
51 s
76
11
63
2
W
ha
ts
ap Ale
p: jan
35 dr
17 ía

- 17 -
61 Im
17 pre
27 si
- 3 one
51 s
76
11
63
2
W
ha
ts
ap Ale
p: jan
35 dr
17 ía

- 18 -
61 Im
17 pre
27 si
- 3 one
51 s
76
11
63
2
W
ha
ts
ap Ale
p: jan
35 dr
17 ía

- 19 -
61 Im
17 pre
27 si
- 3 one
51 s
76
11
63
2
W
ha
ts
ap Ale
p: jan
35 dr
17 ía

- 20 -
61 Im
17 pre
27 si
- 3 one
51 s
76
11
63
2
W
ha
ts
ap Ale
p: jan
35 dr
17 ía

- 21 -
61 Im
17 pre
27 si
- 3 one
51 s
76
11
63
2
W
ha
ts
ap Ale
p: jan
35 dr
17 ía

- 22 -
61 Im
17 pre
27 si
- 3 one
51 s
76
11
63
2
W
ha
ts
ap Ale
p: jan
35 dr
17 ía

- 23 -
61 Im
17 pre
27 si
- 3 one
51 s
76
11
63
2
Traducción de ZYGMUNT BAUMAN
MIRTA ROSENBERG
en cola~oración con
JAIME ARR.iTh1BIDE SQUIRRU

MODERNIDAD
LÍQUIDA

FONDO DE CULTURA ECONÓMICA

MÉXICO - ARGENTINA - BRASIL - COLOMBIA - CHILE - ESPAÑA


ESTADOS UNIDOS DE AMÉRICA - GUATEMALA - PERÚ - VENEZUELA
Primera edición en inglés, 2000
Primera edición en español (FCE, México), 2003
Primera reimpresión (FCE, Argentina), 2003
Segunda reimpresión (FCE, Argentina), 2003
Tercera reimpresión (FCE, Argentina), 2004
Prólogo
Acerca de lo leve y lo líquido>

La interrupción, la incoherencia, la sorpresa son las


condiciones habituales de nuestra vida. Se han conver-
tido incluso en necesidades reales para muchas perso-
nas, cuyas mentes sólo se alimentan [... ] de cambios
súbitos y de estímulos permanentemente renovados
[oo.] Ya no toleramos nada que dure. Ya no sabemos có-
mo hacer para lograr que el aburrimiento dé fruto.
Entonces, todo el tema se reduce a esta pregunta:
¿la mente humana puede dominar lo que la mente hu-
mana ha creado?
PAUL VALÉRY

La "fluidez" es la cualidad de los líquidos y los gases. Según nos infor-


Título original: Liquid Modernity ma la autoridad de la Encyclopcedia Britannica, lo que los distingue de
los sólidos es que "en descanso, no pueden sostener una fuerza tangen-
© 2000. Zygmunt Bauman
cial o cortante" y, por lo tanto, "sufren un continuo cambio de forma
© 2000. Polity Press y Blackwell Publishers Ltd.
ISBN de la edición original: 0-7456-2409-X cuando se los somete a esa tensión".
D.R. © 2002, FONDO DE CULTURA ECONÓMICA DE ARGENTINA. S. A.
El Salvador 5665; 1414 Buenos Aires, Argentina Este continuo e irrecuperable cambio de posición de una parte del
www.fce.com.ar/fondo@fce.com.ar material con respecto a otra parte cuando es sometida a una ten-
Av. Picacho Ajusco 227: 14200 México D. F. sión cortante constituye un flujo, una propiedad característica de
los fluidos. Opuestamente, las fuerzas cortantes ejercidas sobre
ISBN: 950-557-513-0 un sólido para doblarlo o flexionarlo se sostienen, y el sólido no
fluye y puede volver a suforma original.
Fotocopiar libros está penado por la ley.
Los líquidos, una variedad de fluidos, poseen estas notables cualidades,
Prohibida su reproducción total o parcial por cualquier hasta e! punto de que "sus moléculas son preservadas en una disposición
medio de impresión o digital, en forma idéntica, extractada 'ordenada solamente en unos pocos diámetros moleculares"; en tanto, "la
o modificada. en castellano o en cualquier otro idioma, amplia variedad de conductas manifestadas por los sólidos es resultado
sin autorización expresa de la editorial. . directo de! tipo de enlace que reúne los átomos de los sólidos y de la dis-
posición de los átomos". "Enlace", a su vez, es e! término que expresa
IMPRESO EN LA ARGENTINA - PRINTED IN ARGENTINA la estabilidad de los sólidos -la resistencia que ofrecen "a la separación
Hecho el depósito que previene la ley 11.723 de los átornos"-.

7
8 MODERNIDAD LíQUIDA PRÓLOGO 9

Hasta aquí lo que dice la Encyclopiedia Britannica, en una entrada logro? En otras palabras, ¿acaso la modernidad no ha sido "fluida" desde
que apuesta a explicar la "fluidez" como una metáfora regente de la eta- e! principio? .
pa actual de la era moderna. Éstas y otras objeciones son justificadas, y parecerán más justificadas
En lenguaje simple, todas estas características de los fluidos implican aun cuando recordemos que la famosa expresión "derretir los sólidos",
que los líquidos, a diferencia de los sólidos, no conservan fácilmente su acuñada hace un siglo y medio por los autores de! Manifiesto comunista,
forma. Los flúidos, por así decirlo, no se fijan al espacio ni se atan al se refería al tratamiento con que e! confiado y exuberante espíritu mo-
tiempo. En tanto los sólidos tienen una clara dimensión espacial pero derno aludía a una sociedad que encontraba demasiado estancada para
neutralizan e! impacto -y disminuyen la significación- de! tiempo (resisten su gusto y demasiado resistente a los cambios ambicionados, ya que to-
efectivamente su flujo o lo vuelven irrelevante), los fluidos no conservan das sus pautas estaban congeladas. Si e! "espíritu" era "moderno", lo era
una forma durante mucho tiempo y están constantemente dispuestos (y en tanto estaba decidido a que la realidad se emancipara de la "mano
proclives) a cambiarla; por consiguiente, para ellos lo que cuenta es e! muerta" de su propia historia ... y eso sólo podía lograrse derritiendo los
flujo de! tiempo más que e! espacio que puedan ocupar: ese espacio que, sólidos (es decir, según la definición, disolviendo todo aquello que per-
después detodo, sólo llenan "por unmomento". En cierto sentido los só- siste en el tiempo y que es indiferente a su paso e inmune a su fluir). Esa
lidos cancelan el tiempo; para losIíquidos, pare! contrario, lo que im- intención requería, a su vez, la "profanación de lo sagrado": la desauto-
porta es el tiempo. En la descripción de los sólidos, es posible ignorar rización y la negación del pasado, y primordialmente de la "tradi-
completamente e! tiempo; en la descripción de los fluidos, se cometería' ción" -es decir, elsedimento y el residuo de! pasado en el presente-o Por
un error grave si el tiempo se dejara de lado. Las descripciones de un lo tanto, requería asimismo la destrucción de la armadura protectora
fluido son como instantáneas, que necesitan ser fechadas al dorso. forjada por las convicciones y lealtades que permitía a los sólidos resis-
Los fluidos se desplazan con facilidad. "Fluyen';, "se derraman", "se tirse a la "licuefacción".
desbordan", "salpican", "se vierten", "se filtran", "gotean", "inundan", . Recordemos, sin embargo, que todo esto no debía llevarse a cabo para
"rocían", "chorrean", "manan", "exudan"; a diferencia de los sólidos acabar con los sólidos definitivamente ni para liberar al nuevo mundo de
no es p~sible detenerlos fácilmente -sortean algunos obstáculos, disuelve~ ellos para siempre, sino para hacer espacio a nuevos y mejores sólidos;
otros o se filtran a través de ellos, ernpapándolos-, Emergen incólumes para reemplazar el conjunto heredado de sólidos defectuosos y deficientes
de sus encuentros con los sólidos, en tanto que estos últimos -si es que si- por otro, mejor o incluso perfecto, y por eso mismo inalterable. Al leer el
guen siendo sólidos tras el encuentro- sufren un cambio: se humedecen o Ancien Régime [El Antiguo Régimen y la Revolución] de De Tocqueville,
empapan. La extraordinaria movilidad de los fluidos es lo que los asocia podríamos preguntarnos además hasta qué punto esos "sólidos" no es-
con la idea de "levedad", Hay líquidos que en pulgadas cúbicas son más taban de antemano resentidos, condenados y destinados a la licuefac-
pesados que muchos sólidos, pero de' todos modos tendemos a visualizar- ción, ya que se habían oxidado y enmohecido, tornándose frágiles y poco
los como más livianos, menos "pesados" que cualquier sólido. Asociamos confiables. Los tiempos modernos encontraron a los sólidos premodernos
."levedad" o "liviandad" con movilidad e inconstancia: la práctica nos de- en-un estado bastante avanzado de desintegración; y uno de los motivos
muestra que cuanto menos cargados nos desplacemos, tanto más rápido más poderosos que estimulaba su disolución era el deseo de descubrir o
será nuestro avance. inventar sólidos cuya solidez fuera -por una vez- duradera, una solidez en
Estas razones justifican que consideremos que la "fluidez" o la "liqui- la que se pudiera confiar y de la que se pudiera depender, volviendo al
dez" son metáforas adecuadas para aprehender la naturaleza de la fase mundo predecible y controlable.
actual -en muchos sentidos nueva- de la historia de la modernidad. Los primeros sólidos que debían disolverse y las primeras pautas sagra-
. Acepto que esta proposición pueda hacer vacilar a cualquiera que esté das que debían profanarse eran las lealtades tradicionales, los derechos y
familiarizado con e! "discurso de la modernidad" y con e! vocabulario obligaciones acostumbrados que ataban de piesy manos, obstaculizaban
empleado habitualmente para narrar la historia moderna. ¿Acaso la mo- los movimientos y constreñían la iniciativa. Para encarar seriamente la
dernidad no fue desde e! principio un "proceso de licuefacción"? ¿Acaso tarea de construir un nuevo orden (¡verdaderamente sólido!), era nece-
"derretir los sólidos" no fue siempre su principal pasatiempo y su mayor sario deshacerse del lastre que e! viejo orden imponía a los constructores.
10 MODERNIDAD LíQUIDA
PRÓLOGO 11

"Derretir los sólidos" significaba, primordialmente, desprenderse d; las


obligaciones "irrelevantes" que se interponían en el camino de ~n calcu- admite opciones; ni siquiera está claro cuáles podrían ser esas opciones,
lo racional de los efectos; tal como lo expresara Max Weber, liberar la y aun menos claro cómo podría hacerse real alguna opción viable, en el
iniciativa comercial de los grilletes de las obligaciones domésticas y de la improbable caso de que la vida social fuera capaz de concebirla y ges-
densa trama de los deberes éticos; o, según Thomas Carlyle, de todos los tarla. Entre el orden dominante y cada una de las agencias, vehículos y
· estratagemas de cualquier acción efectiva se abre una brecha -un abismo.
vínculos que condicionan la reciprocidad humana y la mutua responsa-
cada vez más infranqueable, y sin ningún puente a la vista-o
bilidad conservar tan sólo el "nexo del dinero". A la vez, esa clase de
A diferencia de la mayoría de los casos distópicos, este efecto no ha
"disolución de los sólidos" destrababa toda la compleja trama de las re-
sido consecuencia de un gobierno dictatorial, de la subordinación, l¡l
laciones sociales, dejándola desnuda, desprotegida, desarmada.y expues-
opresión o la esclavitud; tampoco ha sido consecuencia de la "coloniza-
ta, incapaz de resistirse a las reglas del juego y .a los cnrenos de
ción" de la esfera privada por parte del "sistema". Más bien todo lo con-
racionalidad inspirados y moldeados por el comercio, y menos capaz
trario: la situación actual emergió de la disolución radical de aquellas
aun de competir 'con ellos de manera efectiva. . .
amarras acusadas -justa o injustamente- de limitar la libertad individual
Esa fatal desaparición dejó el campo libre a la invasión y al dominio
de elegir y de actuar. La rigidez del orden es el artefacto y el sedimento
de (como dijo Weber) la racionalidad instrumental, o (como l~ artic~ló
de la libertad de los agentes humanos. Esa rigidez es el producto general de
. Marx) del rol determinante de la economía; las "bases" de la vida social
"perder los frenos"; de la desregulación, la liberalización, la "flexibiliza-
infundieron a todos los otros ámbitos de la vida el status de "superes-
cióri", la creciente fluidez, la liberación de los mercados financiero, labo-
tructura" --es decir, un artefacto de esas "bases" cuya única función era
ral e inmobiliario, la disminución de las cargas impositivas, etc. (como
contribuir a su funcionamiento aceitado y constante-o La disolución de
señalara Offe en "Binding, shackles, brakes", publicado por primera vez
los sólidos condujo a una progresiva emancipación de la economía de sus
en 1987); o (citando a Richard Sennett en Flesh and Stone [Carne y pie-
tradicionales ataduras políticas, éticas y culturales. Sedimentó un nuevo
draJ), de las técnicas de "velocidad, huida, pasividad" --en otras pala-
orden, definido primariamente en términos económicos. Ese nuevo or~en
bras, técnicas que permiten que el sistema y los agentes libres no se
debía ser más "sólido" que los órdenes que reemplazaba, porque -a dife-
comprometan entre sí, que se eludan en vez de reunirse-o Si ha pasado
rencia de ellos- era inmune a los embates de cualquier acción que no fue-
la época de las revoluciones sistémicas, es porque no existen edificios pa-
ra económica. Casi todos los poderes políticos o morales capaces de
ra alojar las oficinas del sistema, que podrían ser invadidas y capturadas
trastocar o reformar ese nuevo orden habían sido destruidos o incapacita-
por los revolucionarios; y también porque resulta extraordinariamente
dos, por debilidad, para esa tarea. Y no porque el orden económico, una
difícil, e incluso imposible, imaginar qué podrían hacer los vencedores,
vez establecido, hubiera colonizado, reeducado y convertido a su gusto el
una vez dentro de esos edificios (si es que primero los hubieran encon-
resto de la vida social, sino porque ese orden llegó a do,minar la totalidad
trado), para revertir la situación y poner fin al malesta.r que los im~ulsó
de la vida humana, volviendo irrelevante e inefectivo todo aspecto de la
a rebelarse. Resulta evidente la escasez de esos potenciales revoluciona-
vida que no contribuyera a su incesante y continua reproducción.
rios, de gente capaz de articular el deseo de cambiar su situ~ción indivi-
Esa etapa de la carrera de la modernidad ha sido bien descripta por
dual como parte del proyecto de cambiar el orden- de la SOCiedad.
Claus Offe (en "The utopia of the zero option", publicado por primera
La rarea de construir un nuevo orden mejor para reemplazar al viejo
vez en 1987 en Praxis International): las sociedades complejas "se han
y defectuoso no forma parte de ning.u~a agend.~ actua,l .-al men~,s ~o de
vuelto tan rígidas que el mero intento de renovar o pensar normativa-
la agenda donde supuestamente se sitúa la aceren pol~tlca-. La dl~~lu-
mente su 'orden' --es decir la naturaleza de la coordinación de los pro-
·ción de los sólidos", el rasgo permanente de la modernidad, ha adquirido
cesos que se producen en 'ellas- está virtualmente obturado en funció?
por lo tanto un nuevo significado, y sobre todo ha sido redirigida hacia
de su futilidad práctica y, por lo tanto, de su inutilidad esencial". Por li- un nuevo blanco; uno de los efectos más importantes de ese cambio de
bres y volátiles que sean, individual o grupalmente, los "subsistemas" de
dirección ha sido la disolución de las fuerzas que podrían mantener el
ese orden se encuentran interrelacionados de manera "rígida, fatal y sin ·tema del orden y del sistema dentro de la agenda polític~.. Los sólidos
ninguna posibilidad de libre elección". El orden general de las cosas no que han sido sometidos a la disolución, y que se están derritiendo en este
12 MODERNIDAD líqUIDA PRÓLOGO 13

momento, el momento de la modernidad fluida, son los vínculos entre fielmente las reglas y modalidades de conducta correctas y adecuadas a
las elecciones individuales y los proyectos y las acciones colectivos -las esa ubicación.
estructuras de comunicación y coordinación entre las políticas de vida Sin embargo, esos códigos y conductas que uno podía elegir como pun-
individuales y las acciones políticas colectivas-o tos de orientación estables, y por los cuales era posible guiarse, escasean
En una entrevista concedida a Jonathan Rutherford el 3 de febrero de .cada vez más en la actualidad. Eso no implica que nuestros contemporá-
1999, Ulrich Beck (quien hace pocos años acuñó el término "segunda mo- neos sólo estén guiados por su propia imaginación, ni que puedan decidir
dernidad" para connotar la fase en que la modernidad "volvió sobre sí a voluntad cómo construir un modelo de vida; ni que ya no dependan de
misma", la época de la soi-disant "modernización de la modernidad") ha- la sociedad para conseguir los materiales de construcción o planos autori-
bla de "categorías zombis" y de "instituciones zornbis", que están "muer- zados. Pero sí implica que, en este momento, salimos de la época de los
tas y todavía vivas". Nombra la familia, la clase y el vecindario como "grupos de referencia" preasignados para desplazarnos hacia una era de
ejemplos ilustrativos de este nuevo fenómeno. La familia, por ejemplo: "comparación universal" enla que el destino de la labor de construcción
individual está endémica e irremediablemente indefinido, no dado de an-
¿Quées una familia en la actualidad? ¿Qué significa? Por supuesto, temano, y tiende a pasar por numerosos y profundos cambios antes de al-
hay niños, mis niños, nuestros niños. Pero hasta la progenitura, el canzar su único final verdadero: el final de la vida del individuo.
núcleo de la vida familiar, ha empezado a desintegrarse con el di- En la actualidad, las pautas y configuraciones ya no están "determi-
vorcio r...] Abuelas y abuelos son incluidos y c!kcluidos sin recur- nadas", y no resultan "autoevidentes" de ningún modo; hay demasia-
sos para participar en las decisiones de sus hijos e hijas. Desde el das, chocan entre sí y sus mandatos se contradicen, de manera que cada
punto de vista de los nietos, el significado de los abuelos debe de-
una de esas pautas y configuraciones ha sido despojada de su poder
terminarse por medio de decisiones y elecciones individuales,
coercitivo o estimulante. Y, además, su naturaleza ha cambiado, parlo
Lo que se está produciendo hoyes, por así decirlo, una redistribución y cual han sido reclasificadas en consecuencia: como ítem del inventario
una reasignación de los "poderes de disolución" de la modernidad. Al de tareas individuales. En vez de preceder a la política de vida y de en-
principio, esos poderes afectaban las instituciones existentes, los marcos cuadrar su curso futuro, deben seguirla (derivar de eIJa), y reformarse y
que circunscribían los campos de acciones y elecciones posibles, como remoldearse según los cambios y giros que esa política de vida experi- .
los patrimonios heredados, con su asignación obligatoria, no por gusto. mente. El poder de licuefacción se ha desplazado del "sistema" a la "so-
Las configuraciones, las constelaciones, las estructuras de dependencia e ciedad", de la "política" a las "políticas de vida" .,. o ha descendido del
interacción fueron arrojadas en el interior del crisol, para ser fundidas y "macronivel" al "rnicronivel" de la cohabitación social.
después remodeladas: ésa fue la fase de "romper el molde" en la histo- Como resultado, la nuestra es una versión privatizada de la moder-
ria de la transgresora, ilimitada, erosiva modernidad. No obstante, los nidad, en la que el peso de la -construcción de pautas y la responsabili-
individuos podían ser excusados por no haberlo advertido: tuvieron que dad del fracaso caen primordialmente sobre los hombros del individuo.
enfrentarse a pautas y configuraciones que, aunque "nuevas y mejores", La licuefacción debe aplicarse ahora a las pautas de dependencia e inte-
seguían siendo tan rígidas e inflexibles como antes. racción, porque les ha tocado el turno. Esas pautas son maleables hasta
Por cierto, todos los moldes que se rompieron fueron reemplazados un punto jamás experimentado ni imaginado por las generaciones ante-
por otros; la gente fue liberada de sus viejas celdas sólo para ser censu- riores, ya que, como todos los fluidos, no conservan mucho tiempo su
rada y reprendida si no lograba situarse -por medio de un esfuerzo de- forma. Darles forma es más fácil que mantenerlas en forma. Los sólidos
dicado, continuo y de por vida- en los nichos confeccionados por el nuevo . son moldeados una sola vez. Mantener la forma de los fluidos requiere
orden: en las clases, los marcos quettan inflexiblemente como los ya di- muchísima atención, vigilancia constante y un esfuerzo perpetuo ... e in-
sueltos estamentos) encuadraban la totalidad de las condiciones y perspec- cluso en ese caso el éxito no es, ni mucho menos, previsible.
tivas vitales, y condicionaban el alcance de los proyectos y estrategias de Sería imprudente negar o menospreciar el profundo cambio que el
vida. Los individuos debían dedicarse a la tarea de usar su nueva liber- advenimiento de la "modernidad fluida" ha impuesto a la condición hu-
tad para encontrar el nicho apropiado y establecerse en él, siguiendo mana. El hecho de que la estructura sistémica se haya vuelto remota e
14 MODERNIDAD LíQUIDA

PRÓLOGO 15
inalcanzable, combinado con el estado fluido y desestructurado del en-
cuadre de la política de vida, ha cambiado la condición humana de rno-
d? radical y exi?e repensar los viejos conceptos que solían enmarcar su los instrumentos naturales de movilidad -Ios miembros inferiores hu-
discurso narrativo. manos o equinos- solían reducir los movimientos de los cuerpos prerno-
· . Como zombis, esos conceptos están hoy vivos y dernos. Cuando la distancia recorrida en una unidad de tiempo pasó a
muertos a l rrusmo tiempo. La pregunta es si su resurrección -aun en una
nueva forma o encarnación- es factible; o, si no lo es, cómo disponer para depender de la tecnología, de los medios de transporte artificiales exis-
ellos un funeral y una sepultura decentes. . tentes, los límites heredados de la velocidad de movimiento pudieron
Este libro está dedicado a esa pregunta. Hemos elegido examinar cinco transgredirse. Sólo el cielo (o, como se reveló más tarde, la velocidad de
conceptos básicos en torno de los cuales ha girado la narrativa ortodoxa la luz) empezó a ser e! límite, y la modernidad fue un esfuerzo constante,
de la .condición humana: emancipación, individualidad, tiempo/espacio, imparable y acelerado por alcanzarlo.
trabaJO. y com~ni?ad. Se ha~ explorado (aunque de manera muy frag- Gracias a sus recientemente adquiridas flexibilidad y capacidad de ex-
menta:la.y preliminar) sucesrvos avatares de sus significados y aplicacio- pansión, e! tiempo moderno se ha convertido, primordialmente, en e! arma
nes practicas, con la esperanza de salvar a los niños del diluvio de aguas para la conquista de! espacio. En la lucha moderna entre espacio y tiempo,
. contaminadas. e! espacio era e! aspecto sólido y estólido, pesado e inerte, capaz de enta-
blar solamente una guerra defensiva, de trincheras ... y ser un obstáculo
La modernidad significa muchas cosas, y su advenimiento y su avance para las flexibles embestidas del tiempo. El tiempo era e! bando activo y
pueden eval~arse empleando diferentes parámetros. Sin embargo, un dinámico del combate, el bando siempre a la ofensiva: la fuerza invaso-
rasgo de la Vida moderna y de sus puestas en escena sobresale particu- ra, conquistadora y colonizadora. Durante la modernidad, la velocidad
larmente, como "diferencia que hace toda la diferencia" como atributo de movimiento y el acceso a medios de movilidad más rápidos ascendie-
crucial del que derivan todas las demás características. E~e atributo es el ron hasta llegar a ser e! principal instrumento de poder y dominación.
cambio en la relación entre espacio y tiempo. Miche! Foucault usó e! diseño de! panóptico de Jeremy Bentham co-
mo archimetáfora del poder moderno. En e! panóptico, los internos es-
, L~ mo~ernidad empieza cuando el espacio y el tiempo se separan de la
taban inmovilizados e impedidos de cualquier movimiento, confinados
practica Vital y entre sí, y pueden ser teorizados como categorías de es-
dentro de gruesos muros y murallas custodiados, y atados a sus camas,
trategia y acción mutuamente independientes, cuando dejan de ser -corno
celdas o bancos de trabajo. No podían moverse porque estaban vigila-
s~lían ~erIo en los siglos premodernos- aspectos entrelazados y apenas
dos; debían permanecer en todo momento en sus sitios asignados por-
discernibles de la experiencia viva, unidos por una relación de corres-
que no sabían, ni tenían manera de saber, dónde se encontraban sus
pondencia estable y aparentemente invulnerable. En la modernidad, el
vigilantes, que tenían libertad de movimiento. La facilidad y la disponi-
ti~mpo tiene historia, gracias a su "capacidad de contención" que se arn-
bilidad de movimiento de los guardias eran garantía de dominación; la
ph~ permane~temente: la prolongación de los tramos de espacio que las
"inmovilidad" de los internos era muy segura, la más difícil de romper
Unidades de tiempo permiten "pasar", "cruzar", "cubrir" ... o conquis-
entre todas las ataduras que condicionaban su subordinación. El domi-
tar. El tiempo adquiere historia cuando la velocidad de movimiento a
nio de! tiempo era e! secreto del poder de los jefes... y tanto la inmoviliza-
través del espacio (a diferencia del espacio eminentemente inflexible, que
ción de sus subordinados en el espacio mediante la negación de! derecho
no puede ser ampliado ni reducido) se convierte en una cuestión de in- a moverse como la rutinización del ritmo temporal impuesto eran las
genio, imaginación y recursos humanos. principales estrategias del ejercicio del poder. La pirámide de poder es-
La idea misma de velocidad (y aun más conspicuarnenre, de acelera- taba construida sobre la base de la velocidad, el acceso a los medios de
ción), referida a la relación entre tiempo y espacio, supone su variabili- transporte y la subsiguiente libertad de movimientos.
dad, y sería difícil que tuviera algún sentido si esa relación no fuera El panóptico era un modelo de confrontación entre los dos lados de la
cambiante, si fuera un atributo de la realidad inhumana y prehumana en relación de poder-,Las estrategias de los jefes -salvaguardar la propia vo-
vez de estar condicionada a la inventiva y la determinación humanas, y latilidad y rutinizar e! flujo de tiempo de sus subordinados- se fusionaron.
si no hubiera trascendido e! estrecho espectro de variaciones a las que Pero existía cierta tensión entre ambas tareas. La segunda tarea ponía
límites a la primera: ataba a los "rutinizadores" al lugar en el.cual habían
PROLOGO 17
16 MODERNIDAD LíqUIDA
citos en guerra. La principal técnica de poder es ahora la huida, el escurri-
sido confinados los objetos de esa rutinización temporal. Los "ruriniza- miento, la elisión, la capacidad de evitar, el rechazo concreto de cualquier
dores" no tenían una verdadera y plena libertad de movimientos: era im- confinamiento territorial y de sus engorrosos corolarios de construcción y
posible considerar la opción de que pudiera haber "amos ausentes". mantenimiento de un orden, de la responsabilidad por sus consecuencias
El panóptico tiene además otras desventajas. Es una estrategia costosa: y de la necesidad de afrontar sus costos.
conquistar el espacio y dominarlo, así como mantener a los residentes en Esta nueva técnica de poder ha sido ilustrada vívidamente por las es-
el lugar vigilado, implica una gran variedad de tareas administrativas en- trategias empleadas durante la Guerra del Golfo y la de Yugoslavia. En
gorrosas y caras. Hay que construir y mantener edificios, contratar y pa- la conducción de la guerra, la reticencia a desplegar fuerzas terrestres fue
gar a vigilantes profesionales, atender y abastecer ·la supervivencia y la notable; a pesar de lo que dijeran las explicaciones oficiales, esa reticen-
capacidad laboral de los internos. Finalmente, administrar significa, de cia no era producto solamente de! publicitado síndrome de "protección
una u otra manera, responsabilizarse del bienestar general del lugar, de los cuerpos". El combate directo en e! campo de batalla no fue evita-
aunque sólo sea en nombre del propio interés... y la responsabilidad sig- do meramente por su posible efecto adverso sobre la política doméstica,
nifica estar atado al lugar. Requiere presencia y confrontación, al menos sino también (y tal vez principalmente) porque era inútil por completo e
bajo la forma de presiones y roces constantes. incluso contraproducente para los propósitos de la guerra. Después de to-
Lo que induce a tantos teóricos a hablar del "fin de la historia", de do, la conquista del territorio, con todas sus consecuencias administra-
posmodernidad, de "segunda modernidad" y "sobrernodernidad", o ar- tivas y gerenciales, no sólo estaba ausente de la lista de objetivos bélicos,
ticular la intuición de un cambio radical en la cohabitación humana y en sino que era algo que debía evitarse por todos los medios y que era con-
las condiciones sociales que restringen actualmente a las políticas de vi- siderado con repugnancia como otra clase de "daño colateral" que, en
da, es el hecho de que el largo esfuerzo por acelerar la velocidad del mo- esta oportunidad, agredía a la fuerza de ataque.
vimiento ha llegado ya a su "límite natural". El poder puede moverse con Los bombardeos realizados por medio de casi invisibles aviones de
la velocidad de la señal electrónica; así, el tiempo requerido para el mo- combate y misiles "inteligentes" -lanzados por sorpresa, salidos de la
vimiento de sus ingredientes esenciales se ha reducido a la instantaneidad. nada y capaces de desaparecer inmediatamente- reemplazaron las inva-
En la práctica, el poder se ha vuelto verdaderamente extraterritorial, y ya siones territoriales de las tropas de infantería y el esfuerzo por despojar
no está atado, ni siquiera detenido, por la resistencia del espacio (el ad- al enemigo de su territorio, apoderándose de la tierra controlada y admi-
venimiento de los teléfonos celulares puede funcionar como el definitivo
nistrada por el adversario. Los atacantes ya no deseaban para nada ser
"golpe fatal" a la dependencia del espacio: ni siquiera es necesario acce-
"los últimos en el campo de batalla" después de que e! enemigo huyera o
der a una boca telefónica para poder dar una orden y controlar sus efec-
fuera exterminado. La fuerza militar y su estrategia bélica de "golpear y
tos. Ya no importa dónde pueda estar el que emite la orden -la distinción
huir" prefiguraron, anunciaron y encarnaron aquello que realmente es-
entre "cerca" y "lejos", o entre lo civilizado y lo salvaje, ha sido prácti-
camente caricelada-). Este hecho confiere a los poseedores de poder una taba en juego en el nuevo tipo de guerra de la época de la modernidad
oportunidad sin precedentes: la de prescindir de los aspectos más irritan- líquida: ya no la conquista de un nuevo territorio, sino la demolición de
tes de la técnica panóptica del poder. La etapa actual de la historia de la los muros que impedían e! flujo de los nuevos poderes globales fluidos;
modernidad -sea lo que fuere por añadidura- es, sobre todo, pospanop- sacarle de la cabeza al enemigo todo deseo de establecer sus propias re-
tica. En el panóptico lo que importaba era que supuestamente las perso- gIas para abrir de ese modo un espacio -hasta entonces amurallado e
nas a cargo estaban siempre "allí", cerca, en la torre de control. En las inaccesible- para la operación de otras armas (no militares) del poder. Se
relaciones de poder pospanópticas, lo que importa es que la gente que .podría decir (parafraseando la fórmula clásica de Clausewitz) que la
maneja el poder del que depende el destino de los socios menos volátiles guerra de hoy se parece cada vez más a "la promoción de! libre comercio
de la relación puede ponerse en cualquier momento fuera de alcance... y mundial por otros medios".
volverse absolutamente inaccesible. . Recientemente, Jim MacLaughlin nos ha recordado (en Sociology, 1/99)
El fin del panóptico augura el fin de la era del compromiso mutuo: en- que e! advenimiento de la era moderna significó, entre otras cosas, e! ataque
tre supervisores y supervisados, trabajo y capital, líderes y seguidores, ejér- consistente y sistemático de los "establecidos", convertidos al modo de
18 MODERNIDAD LíQUIDA
PRÓLOGO 19

vida sedentario, contra los pueblos y los estilos de vida nómades,.comple-


tarnenre adversos a las preocupaciones territoriales y fronterizas del emer- sión de "esclarecer", "reformar las costumbres", "levantar la moral",
gente Estado moderno. En el siglo XN, Ibn Khaldoun podía cantar sus "civilizar" y cualquier cruzada cultural. El compromiso activo con la vi-
alabanzas del nomadismo, que hace que los pueblos "se acerquen más a da de las poblaciones subordinadas ha dejado de ser necesario (por el
la bondad que los sedentarios porque (...] están más alejados de los ma- contrario, se lo evita por ser costoso sin razón alguna y poco efectivo),
los hábitos que han infectado los corazones sedentarios", pero la febril y por lo tanto lo "grande" no sólo ha dejado de ser "mejor", sino que
construcción de naciones y estados-nación que se desencadenó poco tiem- ha perdido cualquier sentido racional. Lo pequeño, lo liviano, lo más
po después en toda EUropa puso el "suelo" muy por encima de la "san- portable significa ahora mejora y "progreso". Viajar liviano, en vez de
gre" al sentar las bases del nuevo orden legislado, que codificaba los aferrarse a cosas consideradas confiables y sólidas -por su gran peso, so-
derechos y deberes de los ciudadanos. Los nómades, que menospreciaban lidez e inflexible capacidad de resistencia-, es ahora el mayor bien y sím-
las preocupaciones territoriales de los legisladores y que ignoraban abso- bolo de poder. .
lutamente sus fanáticos esfuerzos por establecer fronteras, fueron presen- Aferrarse al suelo no es tan importante si ese suelo puede ser alcan-
tados como los peores villanos de la guerra santa entablada en nombre del zado y abandonado a voluntad, en poco o en casi ningún tiempo. Por otro
progreso y de la civilización. Los modernos "cronopolíticos" no sólo los lado, aferrarse demasiado, cargándose de compromisos mutuamente in-
consideraron seres inferiores y primitivos, "subdesarrollados" que necesi- quebrantables, puede resultar positivamente perjudicial, mientras las
taban ser reformados e ilustrados, sino también retrógrados que sufrían nuevas oportunidades aparecen en cualquier otra parte. Es comprensible
"retraso cultural", que se encontraban en los peldaños más bajos de la es- que Rockefeller haya querido que sus fábricas, ferrocarriles y pozos pe-
cala evolutiva y que eran, por añadidura, imperdonablemente necios por troleros fueran grandes y robustos, para poseerlos durante mucho, mucho
su reticencia a seguir "el esquema universal de desarrollo". tiempo (para toda la eternidad, si medimos el tiempo según la duración
Durante toda la etapa sólida de la era moderna, los hábitos nómades de la vida humana o de la familia). Sin embargo, Bill Gates se separa sin
fueron malconsiderados. La ciudadanía iba de la mano con el sedenta- pena de posesiones que ayer lo enorgullecían: hoy, lo que da ganancias
rismo, y la falta de un "domicilio fijo" o la no pertenencia a un "Estado" es la desenfrenada velocidad de circulación, reciclado, envejecimiento,
implicaba la exclusión de la comunidad respetuosa de la ley y protegida descarte y reemplazo -rio la durabilidad ni la duradera confiabilidad del
por ella, y con frecuencia condenaba a los infractores a la discriminación producto-o En una notable inversión de la tradición de más de un mile-
legal, cuando no al enjuiciamiento. Aunque ese trato todavía se aplica a nio, los encumbrados y poderosos de hoy son quienes rechazan y evitan
la "subclase" de los sin techo, que son sometidos a las viejas técnicas de lo durable y celebran lo efímero, mientras los que ocupan el lugar más
control panóptico (técnicas que ya no se emplean para integrar y discipli- bajo -contra todo lo esperable- luchan desesperadamente para lograr
nar a la mayoría de la población), la,época de la superioridad incondicio- que sus frágiles, vulnerables y efímeras posesiones duren más y les rin-
nal del sedentarismo sobre el nomadismo y del dominio de lo sedentario dan servicios duraderos. Los encumbrados y los menos favorecidos se
sobre lo nómade tiende a finalizar. Estamos asistiendo a la venganza del encuentran hoy en lados opuestos de las grandes liquidaciones y en las
nomadismo contra el principio de la territorialidad y el sedentarismo. En ventas de autos usados.
la etapa fluida de la modernidad, la mayoría sedentaria es gobernada por
una elite nómade y extraterritorial. Mantener los caminos libres para el La desintegración de la trama social y el desmoronamiento de las agen-
tráfico nómade y eliminar los pocos puntos de control fronterizo que cias de acción colectiva suelen señalarse con gran ansiedad y justificarse
quedan se ha convertido en el rnetaobjetivo de la política, y también de como "efecto colateral" anticipado de la nueva levedad y fluidez de un
las guerras que, tal como lo expresara Clausewitz, son solamente "la ex- poder cada vez más móvil, escurridizo, cambiante, evasivo y fugitivo.
pansión de la política por otros medios". Pero la desintegración social es tanto una afección como un resultado
La elite global contemporánea sigue el esquema de los antiguos de la nueva técnica del poder, que emplea como principales instrumentos
"amos ausentes". Puede gobernar sin cargarsecon las tareas administra- el descompromiso y el arte de la huida. Para que el poder fluya, el mundo
tivas, gerenciales o bélicas y, por·añadidura, también puede evitar la mi- debe estar libre de trabas, barreras, fronteras fortificadas y controles.
Cualquier trama densa de nexos sociales, y particularmente una red es-
20 MODERNIDAD LÍQUIDA

trecha con base territorial, implica un obstáculo que debe ser eliminado.
Los poderes globales están abocados al desmantelamiento de esas redes,
en nombre de una mayor y constante fluidez, que es la fuente principal
de su fuerza y la garantía d~ su invencibilidad. Y el derrumbe, la fragili-
dad, la vulnerabilidad, la transitoriedad y la precariedad de los vínculos 1. Emancipación
y redes humanos permiten que esos poderes puedan actuar.
Si estas tendencias mezcladas se desarrollaran sin obstáculos, hom-
bres y mujeres serían remodelados siguiendo la estructura del mol elec- Hacia fines de las "tres gloriosas décadas" que siguieron al final de la
trónico, esa orgullosa invención de los primeros años de la cibernética Segunda Guerra Mundial v-tres décadas de crecimiento sin precedentes y
que fue aclamada como un presagio de los años futuros: un enchufe por- de afianzamiento de la riqueza y de la seguridad económica del próspe-
tátil, moviéndose por todas partes, buscando desesperadamente tomaco- ro Occidente-, Herbert Marcuse protestaba:
rrientes donde conectarse. Pero en la época que auguran los teléfonos
celulares, es probable que los enchufes sean declarados obsoletos y de En cuanto al presente y a nuestra propia situación, creo que nos
mal gusto, y que tengan cada' vez menos calidad y poca oferta. Ya aho- enfrentamos a un nuevo momento de la historia, porque hoy de-
ra, muchos abastecedores de energía eléctrica enumeran las ventajas de bemos liberarnos de una sociedad relativamente funcional, rica y
poderosa [... ] El problema al que nos enfrentamos esla necesidad
conectarse a sus redes y rivalizan por el favor de los buscadores de en-
de liberarnos de una sociedad que atieríde en gran medida a las
chufes. Pero a largo plazo (sea cual fuere el significado que "á largo pla- demandas materiales e incluso culturales del hombre -una socie-
zo"pueda tener en la era de la instantaneidad) lo más probable es que dad que, por usar un eslogan, reparte sus mercancías a un amplio
los enchufes desaparezcan y sean reemplazados por baterías descarta bies sector de la población-o Y esto implica que nos enfrentamos a la
que venderán los kioscos de todos los aeropuertos y todas las estaciones liberación de una sociedad en donde la .liberación no tiene el apa-
de servicio de autopistas y caminos rurales. ' rente sustento de las masas.r
Parece una diotopía hecha a la medida de la modernidad líquida ...
adecuada para reemplazar los temores consignados en las pesadillas al Para Marcuse, que debamos y tengamos que emanciparnos, "liberarnos
estilo Orwell y Huxley. de la sociedad", no representaba un problema. Lo que sí era un proble-
ma -el problema específico de una sociedad que "reparte sus mercan-
Junio de 1999. cías"- es que esa liberación carecía de un "sustento de las masas". Para
decirlo más sencillamente: pocos individuos deseaban liberarse, todavía
menos estaban 'dispuestos a actuar para lograrlo, y prácticamente nadie
sabía con certeza en qué medida esa "liberación de lasociedad" sería di-
ferente de la situación en la que ya se hallaban todos ellos.
"Liberarse" significa literalmente deshacerse de las ataduras que im-
piden o constriñen el movimiento, comenzar a sentirse libre de actuar y
moverse. "Sentirse libre" implica no encontrar estorbos, obstáculos, re-
sistencias de ningún tipo que impidan los movimientos deseados o que
. puedan llegar a desearse. Tal como lo observara Arthur Schopenhauer,
la "realidad" es creada por el acto del deseo; la empecinada indiferencia

1 Herbert Marcuse, "Liberarion from the affluent society", en: Stephen Eric Bronner
y Douglas MacKay Kellner (cornps.), Critical Theory and 'Society: a Reader, Londres,
Routledge, 1989, p. 227.

21
El declive y las mutaciones de la institución

The Institution’s Decline and Mutations

Le déclin et les mutations de l’institution

François DUBET
Cadis. Université de Bordeaux 2. EHESS
francois.dubet@sociologie.u-bordeaux2.fr

Recibido: 6 de junio de 2006


Aceptado: 4 de noviembre de 2006

Resumen
Este artículo trata sobre la forma institucional de la educación escolar. Durante mucho tiempo se
construyó esta forma como un programa estructurador de una economía simbólica caracterizada
por cuatro principios: valores “sagrados”, vocación de los maestros, santuario escolar, liberación
a través de la disciplina. Las transformaciones de la escuela en las sociedades modernas conlle-
van el declive de este programa institucional y engendra un profundo sentimiento de crisis en los
actores. Más allá de esta crisis es importante definir lo que está en juego en esta transformación
de la institución y la obligación que tiene de volverse más democrática y más política.

Palabras clave: educación, institución, socialización, vocación.

Abstract
This paper deals with the institutional form of school education. For a long time, this form was built
as a program structuring a symbolic economy, itself characterized by four principles: “sacred”
values, vocation of the masters, school sanctuary, liberation through discipline. The transformations
of school in modern societies induce the decline of this institutional program and generate a deep
feeling of crisis for the actors. Beyond this crisis, it is important to define the issues linked to this
transformation and the obligation that the institution has to become more democratic and more poli-
tical.

Key words: education, institution, socialization, vocation.

Résumé
Cet article porte sur la forme institutionnelle de l’éducation scolaire. Longtemps, cette forme a
été construite comme un programme structurant une économie symbolique caractérisée par qua-
tre principes: valeurs «sacrées», vocation des maîtres, sanctuaire scolaire, libération par la disci-
pline. Les transformations de l’école dans les sociétés modernes entraînent le déclin de ce pro-

Revista de Antropología Social 39 ISSN: 1131-558X


2007, 16 39-66
François Dubet El declive y las mutaciones de la institución

gramme institutionnel et engendrent un profond sentiment de crise chez les acteurs. Au-delà de
cette crise, il importe de définir les enjeux liés à cette transformation de l’institution et l’obliga-
tion qui lui est faite de devenir plus démocratique et plus politique.

Mots clé: éducation, institution, socialisation, vocation.

SUMARIO: 1. Introducción. 2. Socialización y subjetivación. 3. El programa institucional. 4. La


fuerza de un programa. 5. El declive del programa institucional. 6. Crisis y decepción. 7. Del pro-
grama institucional a la experiencia social. 8. Conclusión. 9. Referencias bibliográficas.

1. Introducción
“Institución” forma parte de esas palabras de las que uno no puede desha-
cerse a pesar de o a causa de su polisemia y de sus ambigüedades. Se pueden
distinguir tres grandes significaciones. La primera, heredada de la tradición
antropológica, especialmente de Mauss, define como instituciones todas las
prácticas sociales más o menos rutinarias y ritualizadas, todos los sistemas
simbólicos, como las religiones y las lenguas, que se imponen a los actores
como “hechos” que preceden a su acción. En este caso la lista de institucio-
nes es casi infinita: las maneras, las costumbres, los hábitos alimenticios, los
códigos de cortesía... son instituciones. El segundo significado proviene de la
sociología política. Las instituciones son un conjunto de marcos y procedi-
mientos constituyentes de la soberanía, que permiten que se regulen los con-
flictos y que se tomen las decisiones legítimas: las constituciones políticas,
los parlamentos, los procedimientos de arbitraje, los sistemas políticos... El
tercer significado de esta noción identifica las instituciones con organizacio-
nes y empresas beneficiarias de una antigüedad legítima: las empresas y las
administraciones identificadas con el interés general y con la nación accede-
rían al estatus de institución a causa de su fuerte legitimidad –FIAT sería una
institución turinense, y la SNCF* una institución francesa...–.
En este texto utilizaremos la noción de institución en un sentido particu-
lar: el que tiene la función de instituir y de socializar. La institución es defi-
nida entonces por su capacidad de hacer advenir un orden simbólico y de for-
mar un tipo de sujeto ligado a este orden, de instituirlo. En este sentido, la
Iglesia, la Escuela, la Familia o la Justicia son instituciones porque inscriben
un orden simbólico y una cultura en la subjetividad de los individuos, porque

* (N. del T.) El equivalente en español sería la RENFE, es decir, la institución pública encargada del
transporte ferroviario.

40 Revista de Antropología Social


2007, 16 39-66
François Dubet El declive y las mutaciones de la institución

“institucionalizan” valores, símbolos, porque “instituyen” una naturaleza


social en la naturaleza “natural” de los individuos. Según esta acepción, la
noción de institución no designa solamente un tipo de aparato o de organiza-
ción, sino que también caracteriza un tipo específico de socialización y de
trabajo sobre el otro.
Si nos interesamos por estas instituciones, y más particularmente por la
escuela, es porque a día de hoy muchas de ellas parecen estar en crisis. Nues-
tra hipótesis central es que esta crisis no es sólo una dificultad de adaptación
a un entorno en movimiento, sino que es una crisis del propio proceso de
socialización, una crisis inscrita en una mutación profunda del trabajo sobre
el otro. Esta mutación está ligada a las transformaciones de la modernidad
que trastocan el ordenamiento simbólico de la socialización, de la formación
de los individuos, y, por consiguiente, de la manera de instituir a los actores
sociales y a los sujetos.

2. Socialización y subjetivación
La sociología clásica otorga un lugar central a la socialización en la expli-
cación de la estabilidad de la vida social. Cuando, para los sociólogos, el
orden social no está fundado en el designio de los dioses y cuando no es
reductible al orden natural engendrado por el mercado, se explica por el
hecho de que los individuos interiorizan unas normas y unos valores que les
llevan a reproducir el orden social en el curso mismo de su acción. La socia-
lización “formatea” a los individuos de tal manera que se conducen, sin que
a menudo ellos mismos lo sepan, conforme a las necesidades del sistema. De
acuerdo con esto, la integración social y la integración sistémica vienen jun-
tas (Lockwood, 1964).
En cualquier caso, esta concepción de la socialización descansa sobre una
paradoja. Por un lado, afirma que siendo todo social, los actores interiorizan
modelos, valores, sentimientos, marcos cognitivos colectivos, habitus que
terminan por “clonar” a los individuos. En definitiva, sólo le quedaría a uno
un estilo personal debido a los vaivenes de su historia singular y, aún más, si
le prestamos más atención, este estilo probablemente no sería más que una
mera ilusión. Pero, por otro lado, las sociedades modernas también son socie-
dades individualistas en las que cada uno se percibe como un sujeto libre y
autónomo, como un individuo capaz de ser el autor de sus propios juicios y
de sus propias acciones. Cuando, según la fórmula de Durkheim, el individuo
se convierte en el “dios” de la sociedad moderna, debe ser definido por su

Revista de Antropología Social 41


2007, 16 39-66
François Dubet El declive y las mutaciones de la institución

autonomía y por su libertad. Así, la socialización de los individuos está domi-


nada por una ambivalencia fundamental puesto que supone, a la vez, que los
actores estén perfectamente socializados y que, al mismo tiempo, cada uno
de ellos disponga de un sentimiento de libertad fundamental, aquel que lo
constituye como sujeto singular y no simplemente como un caso particular de
un modelo social general. En gran medida la sociología clásica se ha esforza-
do por responder a la paradoja según la cual la socialización es también una
subjetivación.
La primera serie de respuestas es la de Durkheim y Parsons y, en cierta
medida, la de Freud (Durkheim, 1924; Parsons, 1964; Freud, 1992). Ésta afir-
ma que a la modernidad subyacen valores universales y generales, a los cua-
les se adhieren los individuos sin que, sin embargo, estos valores constituyan
códigos rígidos que determinen las respuestas de los individuos a todas las
situaciones. Aunque existe un gran número de conductas rutinarias, los indi-
viduos también son capaces de producir acciones singulares que ponen a prue-
ba su autonomía y su libertad en el seno de una división de trabajo compleja.
Puesto que los valores son universales y “racionales”, ellos engendran una
“intro-determinación”, un fuero interno gracias al cual el personaje social no
se corresponde totalmente con la subjetividad de cada uno (Riesman, 1964).
Del mismo modo, para Freud existe una distancia entre el Yo y el Ideal del Yo,
distancia que se funda en la autonomía y la capacidad crítica de los actores:
para ser un sujeto, hace falta dejar de identificarse con su padre a fin de iden-
tificarse con los marcos simbólicos con los cuales es, a su vez, identificado su
propio padre, es el pasaje de la identificación particular a la ley simbólica. Esta
concepción de la socialización lleva a afirmar que cuanto más se socializa el
individuo en valores y principios universales, más distancia puede poner con
el mundo y más libre puede ser. Después de todo, es la misma socialización la
que engendra la subjetivación y, según Durkheim, dos grandes desgracias
amenazan al individuo: la anomia cuando la socialización es demasiado débil;
el “fatalismo” cuando es demasiado fuerte. En pocas palabras, el individuo
está amenazado por un déficit de socialización que lo desgarra de la sociedad,
y por un exceso de socialización que lo sofoca y lo conduce a la “neurosis”.
Otra tradición sociológica, más sensible a los efectos de la Reforma y de
la Ilustración insiste en el hecho de que la socialización genera subjetivación
cuando el control social cambia de naturaleza. La obra de Elias (1991) puede
ser la más claramente identificada con esta perspectiva. Con la modernidad y
particularmente con la modernidad política instaurada por las costumbres de

42 Revista de Antropología Social


2007, 16 39-66
François Dubet El declive y las mutaciones de la institución

la corte, el control social ya no descansa sobre el juicio de los otros, sobre la


vergüenza y el castigo, sino sobre la interiorización profunda de las normas,
sobre la culpabilidad y sobre el dominio de sí como virtud esencial. El indi-
viduo tiene menos la sensación de obedecer a los demás que la de obedecer-
se a sí mismo, de ser a la vez libre y su propio censor. En la sociedad de los
individuos, cada uno se constituye como el autor de su propia moral y de sus
propias conductas, incluso cuando las normas que lo guían son las de la
sociedad en su conjunto. También en este caso, puesto que la socialización es
eficaz y conseguida, puede llevar a que el individuo se perciba como un suje-
to capaz de autocontrolarse y de asumir su propia libertad. El mundo social
se presenta entonces como un paisaje de fondo mientras que el individuo se
refiere a su propia moral y a su propia subjetividad. Así, la modernidad, la
“civilización” según Elias, produce a la vez una socialización y una subjeti-
vación; el individuo es enteramente social y, sin embargo, se puede percibir
como un individuo relativamente “libre” en sus elecciones y en sus gustos1.
En todo los casos la socialización moderna desarrolla un “fuero interno”, una
distancia entre el personaje social identificado con sus roles sociales, y una
subjetividad propia porque el individuo está determinado desde el interior y
actúa en nombre de principios generales que le permiten simultáneamente
adaptarse al mundo y criticarlo, estar totalmente dentro del mundo y percibir-
lo como distante y no reducible a las expectativas sociales. De esta manera,
la tradición sociológica consigue conciliar lo irreconciliable, afirmar que la
subjetivación resulta de la socialización y que esta no puede ser una clona-
ción pese a ser totalmente social.

3. El programa institucional
La realización de esta socialización paradójica y, hay que decirlo, algo
milagrosa, exige un dispositivo, una “técnica” de socialización cuyos oríge-
nes se encuentran en la Iglesia y más concretamente en la forma escolar tal
como la ha inventado la Iglesia. Como subrayan Durkheim (1990) y Dumont
(1983), si la Iglesia ha “inventado” la escuela, si ella ha racionalizado la
pedagogía sacando a los alumnos de las familias, creando clases, niveles, dis-
ciplinas, profesionales de la educación, reuniendo y racionalizando en un
solo lugar lo que los preceptores llevaban a cabo individualmente en las fami-

1 Esto va en la misma línea que las críticas que se desarrollan, particularmente las de Foucault, quien
identifica subjetivación y sujeción convirtiendo así al sujeto en un ardid de la sujeción.

Revista de Antropología Social 43


2007, 16 39-66
François Dubet El declive y las mutaciones de la institución

lias aristócratas, fue para crear un tipo particular de sujeto: un creyente, un


hombre capaz de actuar estando guiado por la fe. Desde el punto de vista de
la Iglesia, la socialización de los cristianos no se podía reducir a su adhesión
a las costumbres y a los hábitos de las familias, hacía falta sacar a los niños
de la “pequeña sociedad” como decía Durkheim, y elevarlos hasta la gran
sociedad, aquella cuyos valores universales dan forman a los creyentes autén-
ticos capaces de aplicar los principios del dogma transformando los dogmas
generales en fe personal. Para alcanzar este objetivo, la Iglesia ha inventado
una forma, un dispositivo simbólico, un tipo de trabajo educativo que yo
denomino “un programa institucional”. Digamos de entrada que si fue la
Iglesia la que construyó este programa a partir del siglo IX, este programa se
ha ido distanciando progresivamente de su marco inicial hasta el punto de ser
retomado por aquellos que han querido combatir a la Iglesia, como los repu-
blicanos franceses o los revolucionarios rusos... Más allá de los valores y de
los principios que promueva, más allá de los dioses o las razones a las que se
asocie, el programa institucional es una composición simbólica y práctica al
que se pliegan aquellos que están encargados de llevarlo a cabo. ¿Cómo defi-
nir el tipo ideal de este programa? (Dubet, 2002).

Un conjunto de valores y de principios sagrados


El primer elemento del programa institucional radica en su identificación
con un conjunto de principios y de valores tenidos por sagrados. Sagrado sig-
nifica aquí que esos valores son considerados como estando fuera del mundo,
situados por encima de la sociedad y que parecen indiscutibles dentro del
marco de la institución. Se comprende fácilmente esta afirmación cuando se
trata de la Iglesia y de principios religiosos considerados trascendentes y
dados a los hombres en la forma de una obligación ética y de un conjunto de
dogmas universales y objetivos. Pero la fuerza del programa institucional
desborda la influencia de lo que es sagrado en sentido religioso, como indica
de manera luminosa el caso de la escuela republicana francesa encargada, sin
embargo, de combatir la influencia de la Iglesia. Al igual que la Iglesia, esta
escuela afirma el carácter “sagrado”, no negociable, de un cierto número de
valores planteados como a priori, los de la Razón, los de la nación, los del
progreso y la ciencia. En este sentido, el programa institucional se desarrolla
en el seno de un “templo” inscrito en la arquitectura y la iconografía de las
escuelas, los hospitales, los cuarteles y los tribunales cuya estética se ha
tomado prestada durante mucho tiempo de la de los monasterios. Sin duda la

44 Revista de Antropología Social


2007, 16 39-66
François Dubet El declive y las mutaciones de la institución

discusión de los principios es posible, pero esta crítica es siempre interna al


orden institucional y adquiere una forma teológica más que pragmática, se
discuten creencias en nombre de estas creencias, principios en nombre de los
principios. El carácter universal y sagrado de los principios que dirigen la ins-
titución no puede ser impugnado sin crear un escándalo y sin desbordarse
hacia la violencia y la represión.
Concebida como universal, la cultura escolar se sitúa por encima de la
sociedad y, durante mucho tiempo, ha valorado la abstracción y “la inutili-
dad” social de los aprendizajes distinguidos de la formación profesional,
hecha ésta en la sociedad. Durante mucho tiempo las lenguas muertas, las
matemáticas y la filosofía han sido consideradas como las disciplinas escola-
res más dignas y más selectivas, mientras que las disciplinas más enraizadas
en la vida social, como la tecnología y las lenguas vivas, tenían una dignidad
inferior. Notemos que este tipo de jerarquía que va de la abstracción gratuita
a la práctica “útil” ha dominado mucho tiempo la escuela republicana y laica
francesa; de la misma manera que las escuelas religiosas están encargadas de
elevar a los alumnos hacia Dios, ella ha querido elevar a los alumnos hacia la
Razón y la cultura, relegando a las formaciones más instrumentales a aque-
llos que son menos dignos de los valores más universales de la institución,
valores que se consideran capaces de fomentar el espíritu crítico y la libertad.

El trabajo sore el otro como vocación


En el programa institucional, la principal virtud de los maestros es su
vocación, el hecho de que crean en los principios de la institución y que los
encarnen. Esto es evidente para el sacerdote, pero también vale para el maes-
tro de escuela que encarna los valores de la escuela desde el momento en que
enseña. Durkheim podía así afirmar que el maestro de escuela es tan sagrado
como el sacerdote porque, como él, transmite unos principios superiores. Se
espera del maestro y del profesor que representen los valores de la institución
sacrificándose a ellos, adoptando una vida virtuosa y ejemplar.
En el programa institucional, la autoridad del maestro –sacerdote, maestro
de escuela, médico, juez...– es muy carismática en la medida en la que la obe-
diencia que se le debe trasciende a su propia persona. Al obedecer al maes-
tro, el alumno obedece antes que nada a aquello que el maestro representa y
encarna. Si duda su carisma está “rutinizado”, banalizado, pero eso no quita
que el maestro se apoye sobre los principios que le sobrepasan. Y, porque
encarna una figura de lo sacro, al obedecerle el alumno obedece a esos mis-

Revista de Antropología Social 45


2007, 16 39-66
François Dubet El declive y las mutaciones de la institución

mos valores. No obedezco al maestro o al médico porque tenga una persona-


lidad simpática o autoritaria, sino porque actúa en nombre de la Razón, de la
Nación, de la Cultura, de la Ciencia... Así, al someterse a una autoridad sin-
gular, el alumno se pliega a la autoridad de los valores de la institución. De
hecho, tanto en la escuela como en la Iglesia, se espera del maestro que sea
modesto y virtuoso, y en los dos casos, el alumno y el creyente no deben
“amar” demasiado al sacerdote y al maestro para que, mediante esta distan-
cia y esta frustración, el alumno y el creyente se eleven hacia los principios
trascendentales de la institución.
En el programa institucional, la principal virtud del actor encargado de
socializar al otro no es su competencia técnica y profesional, sino el hecho de
que “crea”, que tenga una vocación, que encarne los principios y se sacrifi-
que por ellos. Así, el individuo socializado no está preso en una simple rela-
ción particular como la que propone la familia, por ejemplo, sino en una rela-
ción que lo eleva hacia los valores que el maestro representa con una cierta
majestad. Esto no impide recurrir ni a la simpatía, ni a veces a la violencia,
pero por la mediación de la vocación, la educación no es ni una pura seducción
ni un simple adiestramiento.
Durante un largo periodo, la escuela republicana francesa ha formado a sus
maestros como la Iglesia formaba a sus sacerdotes. Se aseguraba más de sus
convicciones y de su moralidad personal que de sus competencias pedagógi-
cas propiamente dichas. Los profesores de primaria franceses estaban forma-
dos como una especie de sacerdotes civiles capaces de sacrificarse totalmente
por su vocación y la República, siguiendo de esta manera las recomendacio-
nes de Condorcet, la república se ha opuesto desde hace mucho tiempo al
derecho de los profesores a sindicarse, a hacer valer sus intereses particulares
en detrimetno de su vocación.

El santuario
En la medida en la que los valores de la institución están fuera del mundo,
hace falta que la institución misma esté fuera del mundo, que ella sea un san-
tuario protegido de los desórdenes, los intereses y las pasiones de la sociedad.
La escuela es entonces concebida como una Orden Regular, como un conven-
to separado de la sociedad. De hecho la disciplina escolar se inspira en la
Regla de los conventos, en su regularidad, en su dominio del tiempo y de los
ejercicios escolares, por su repetición y precisión, están acompasados como
los ritos religiosos (Vincent, 1980).

46 Revista de Antropología Social


2007, 16 39-66
François Dubet El declive y las mutaciones de la institución

La conversión en santuario de la escuela exige que la naturaleza social


del alumno sea ella misma “santuarizada”. En la iglesia, durante la misa, los
creyentes se despojan de sus adscripciones sociales, de sus pasiones y de
sus intereses; el rito se dirige a unas almas separadas de la sociedad y de los
cuerpos, y son almas iguales entre sí las que se dirigen a Dios. Se encuen-
tra la misma separación en la escuela republicana francesa que distinguía
radicalmente a los niños y a los adolescentes, atrapados por su psicología y
la sociedad, de los alumnos, considerados sujetos participantes de la razón
e iguales entre sí en la medida en que “la razón es lo más compartido” del
mundo como escribía Descartes. Entonces, el maestro encarnando la Razón
se dirige a la razón de los alumnos por encima de sus singularidades psico-
lógicas y sociales y cada actor escolar es, dentro de una cierta medida, san-
tuarizado porque está dividido entre una parte sagrada, el alma o la razón, y
una parte profana, las adscripciones y los intereses sociales. La santuariza-
ción exige una distinción latente entre el sujeto y el individuo psicológico y
social.
La escuela republicana francesa ha impulsado muy lejos su santuariaza-
ción al negarse a que los padres participen en ella, al negarse a que los empre-
sarios y los sindicatos de asalariados y de patrones intervengan en los asun-
tos escolares. De la misma manera, esta escuela, a pesar de estar fundada en
la igualdad, separó mucho tiempo a los alumnos en función de su sexo con el
fin de que los deseos y las “pasiones” no pervirtiesen el orden de un santua-
rio construido al margen de la vida social “normal”.

La socialización escolar es una subjetivación


A primera vista, el programa institucional se presenta como una empresa
de enderezamiento religioso o laico. Es hacia esta interpretación a la que nos
invitaría una lectura foucaultiana del programa institucional. Sin embargo, se
puede rechazar esta lectura en la medida en la que esta obediencia y este
adiestramiento tienen por objetivo construir la libertad y la autonomía del
sujeto. El alumno no se libera por, sino en tanto obedece, las reglas que le
permiten acceder a los universales de la fe y la Razón. Cuando aprende estos
principios universales, cuando los hace suyos, el alumno puede alejarse de
los ritos, de las disciplinas y de la obediencia ciega con el fin de ser el maes-
tro de su propia acción. Y se espera del buen alumno que termine por alejar-
se de los ejercicios escolares porque estos ejercicios le han dado armas a su
libertad. De la misma manera, en el espacio religioso se considera con fre-

Revista de Antropología Social 47


2007, 16 39-66
François Dubet El declive y las mutaciones de la institución

cuencia que la fe auténtica es vivida como una creencia libre en la que obe-
dezco más a mi propia regla que a la de los ritos y las rutinas religiosas.
El programa institucional reposa sobre una concepción fuerte de la sociali-
zación, la que Pascal ya había descrito al escribir “Rezad y atontaos, la razón
vendrá por añadidura”. A través del entrenamiento de los ritos y las discipli-
nas se forja una libertad subjetiva, la de la fe que es un sentimiento personal.
Los profesores defienden el mismo razonamiento cuando explican a los alum-
nos que su libertad, su inteligencia y su espíritu crítico surgirán de su larga
sumisión a las disciplinas escolares. Es en este sentido que la escuela, como
la Iglesia, es una institución, porque instituye un sujeto arrancado de su pro-
pia naturaleza y de su propia naturaleza social. Se podría mostrar fácilmente
como Platón y Freud, por ejemplo, uno anterior a la Iglesia y el otro desde
fuera, desarrollaron concepciones cercanas de la educación en las cuales la
religión no desempeñaba ningún papel, aunque suponen también que lo
“sacro”, la cultura y lo universal se articulan en disciplinas que instituyen un
sujeto capaz de decir “yo” y de conducir por sí mismo su vida dentro de la
sociedad gracias a “la escuela liberadora” como dicen los maestros de escue-
la franceses. Por supuesto, el programa institucional está adaptado necesaria-
mente a la sociedad y dirigido a producir actores conformes a sus demandas.
Pero también está construido en tensión con esta sociedad y esta tensión es la
que separa la socialización y la subjetivación; generalmente el ejercicio tiene
éxito cuando el alumno es capaz de criticar la escuela y su formación y, sobre
todo, cuando lo hace en nombre de los principios de la escuela porque ésta no
será jamás totalmente digna de los principios que promueve.

4. La fuerza de un programa
El hecho de que el programa institucional haya desbordado ampliamente
el mero marco de la Iglesia, que se haya ampliado a la escuela republicana y
a otras instituciones como el hospital o la justicia, indica que posee una fuer-
za y una plasticidad particulares. Tres grandes características pueden explicar
esta persistencia y esta extensión.

La legitimidad del trabajo sobre el otro


La relación pedagógica, pero también la relación de cuidado o la relación
de ayuda, implica, a la vez, una autoridad y una voluntad de liberar al otro y
de convertirle en igual. Esta paradoja está superada por el programa institu-
cional que confiere una fuerte legitimidad a los profesionales de la educación

48 Revista de Antropología Social


2007, 16 39-66
François Dubet El declive y las mutaciones de la institución

y promueve el asentimiento de los alumnos. Los maestros no tienen que


exponer su personalidad y sus sentimientos porque están preservados por la
institución y por su rol incuestionable. Además, están protegidos por el san-
tuario escolar que los dispensa de rendir cuentas a los usuarios, ya que ellos
sólo deben explicaciones a sus superiores e, incluso, a su sola conciencia.
En estas circunstancias, se comprende por qué los profesores, pero también
los médicos y los jueces, se pueden adherir a un programa que les otorga un
prestigio y una legitimidad que exceden a menudo sus ingresos y sus con-
diciones de vida. En gran medida, el programa institucional participa de un
encanto carismático que silencia la violencia real de las instituciones por-
que esta violencia es, en cierta medida, legítima. Ahora bien, hay que recor-
dar que siempre hay una cara sombría de las instituciones, la de los golpes,
las humillaciones, las exclusiones que durante mucho tiempo han parecido
aceptables porque estaban encubiertas por los valores de la institución.

La capacidad crítica
La fuerza del programa institucional procede también de su plasticidad
puesto que no cambia fundamentalmente de forma incluso cuando cambian
los valores que lo dirigen. Uno de los ejemplos más claros de esta resistencia
podría ser tomado prestado de la concepción de la escuela de Bourdieu y
Passeron (1970). A priori esta teoría es un esfuerzo de crítica y un desencan-
tamiento radical de la escuela, definida como un aparato de reproducción de
las desigualdades sociales, de la cultura dominante mediante una especie de
adiestramiento de los individuos. Pero Bourdieu y Passeron observan preci-
samente que la escuela distribuye “bienes de salvación” y que tiene la capa-
cidad de enmascarar su propio trabajo en nombre de los principios que pos-
tula, de la “gratuidad” de los aprendizajes e incluso, del espíritu crítico ritual
que intenta promover.
Considerada más detalladamente la descripción de la institución escolar
propuesta por estos dos sociólogos se amolda al modelo de programa institu-
cional, visión crítica e “invertida” sin duda, pero visión de la escuela como
institución. Allí donde los creyentes sitúan los valores y los principios sagra-
dos, sitúan “la arbitrariedad cultural” enmascarando la cultura dominante
detrás de la imagen de la cultura universal. Ahí donde está la vocación se da
“la ilusión pedagógica” de la autonomía de los profesores y de su crítica a los
valores de la sociedad y del mundo. El santuario se mantiene porque, para
funcionar eficazmente, la escuela debe afirmar su distancia respecto a los

Revista de Antropología Social 49


2007, 16 39-66
François Dubet El declive y las mutaciones de la institución

intereses sociales. Al final, la teoría de la socialización de habitus es una


manera sociológica de describir la ilusión de un aprendizaje de la libertad
detrás de la fuerza de la socialización.
Si el programa institucional se invierte desde el punto de vista normativo,
sin embargo se mantiene desde el punto de vista de su forma. Es lo que mues-
tra la recepción social de este análisis. En un principio considerado por los
profesores franceses como una crítica intolerable a la escuela porque destro-
zaba las ilusiones, acababa siendo defendida por los profesores y los sociólo-
gos más cercanos a la tradición republicana en nombre de la preservación de
un modelo institucional hoy en día amenazado por las mutaciones liberales
de la sociedad. Como lo que importa es la estructura del programa institucio-
nal más que el contenido, el alcance crítico del análisis ha acabado por des-
dibujarse tras la defensa de una forma institucional, ya que cada uno sabe que
lo esencial está ahí: poco importa que se enseñe Racine o Beckett, Kant o
Nietzsche, lo esencial es la forma de transmisión.

La relación regulada
De manera general, el programa institucional es una economía simbólica
y no sólo un marco organizado. Esta economía estructura profundamente la
subjetividad de los individuos. Las relaciones entre los roles sociales y la per-
sonalidad están establecidos fuertemente con una predominancia de los roles
que se imponen a las personalidades, pero forjan las subjetividades singula-
res en una tensión entre la adhesión y la resistencia a los roles sociales. El
maestro y el alumno desaparecen detrás de sus roles y se construyen al abri-
go de éstos y bajo su protección.
El sistema de motivaciones que compromete a los individuos también está
fuertemente establecido por el sistema institucional, que ya ofrece motivos y sig-
nificados y trasciende las razones particulares para actuar. La legitimidad de la
cultura escolar se impone pues, como la autoridad del maestro. Claro está, todas
las instituciones incurren en desviaciones y deben dejarles un espacio. Pero en
el programa institucional, se podría decir que la mayor parte de estas desviacio-
nes son parte del programa. Por ejemplo, el que unos alumnos armen jaleo es
ritual y a menudo se tolera, funcionando como una válvula de escape, sin poner
verdaderamente a la institución en peligro (Testanière, 1967). Se puede incluso
formar una contra-institución subterránea que revierta los valores del programa
institucional, a la vez que proviene del propio mecanismo y que permite a los
actores una autonomía crítica “prevista” por el programa. De la misma manera

50 Revista de Antropología Social


2007, 16 39-66
François Dubet El declive y las mutaciones de la institución

en que a la Iglesia no le gustan mucho los creyentes “demasiado” santurrones,


al hospital no le gustan los enfermos “demasiado” pasivos y a la escuela los
alumnos “demasiado” buenos.
Desde el punto de vista de los actores, el programa institucional es “eco-
nómico” en la medida en la que las conductas de los distintos actores son pre-
visibles. Como diría Bernstein (1975), la escuela es una organización “serial”
en la que los actores comparten los mismos códigos, los mismos valores y las
mismas connivencias, de tal manera que la organización misma es “liviana”
porque los actores se adaptan espontáneamente y porque durante mucho
tiempo la división del trabajo en ella ha sido relativamente relajada. También,
cuando decae el programa institucional, se sobrecarga el peso de la organiza-
ción, del control y de la burocracia.

5. El declive del programa institucional


Si en la versión republicana y laica el programa institucional participó
del ingreso en una modernidad que se podría caracterizar como clásica, a
día de hoy este programa parece prisionero en un proceso de declive que
afecta a la naturaleza misma del trabajo sobre el otro tal y como se practi-
ca en la escuela, pero también en otras instituciones, como el hospital.
Desde hace una treintena de años, parece evidente que el modelo bosqueja-
do más arriba se resquebraja y se derrumba. El análisis más compartido res-
pecto a esta cuestión, atribuye esencialmente este declive a factores exte-
riores a la escuela. El relato canónico repetido con todos los tonos, tanto
desde la derecha como desde la izquierda, desde la cima a la base de la ins-
titución, remite a la caída de un templo de la cultura, de la igualdad y de la
virtud republicana, amenazado y luego invadido por la barbarie del mundo.
El capitalismo y su crisis, los medios de comunicación, la pobreza y el
paro, y la crisis de la familia, han acabado por romper la alianza entre la
escuela y la sociedad. Como en todo relato institucional, la institución es
pura y el mal viene de fuera, del “mundo”. Esta representación no es total-
mente falsa pues el mundo ha cambiado y no necesariamente en el mejor
sentido: por ejemplo, es cierto que la convergencia de una masificación
escolar rápida y de una crisis social ha confrontado a la escuela a proble-
mas que hasta ahora había logrado mantener a distancia. Pero, en lo esen-
cial, el declive del programa institucional es un proceso endógeno introdu-
cido por los “virus” de la modernidad “nueva”, “tardía”, “post”, poco
importa aquí como se la llame.

Revista de Antropología Social 51


2007, 16 39-66
François Dubet El declive y las mutaciones de la institución

La poliarquía de los valores


Sea cual sea la manera en la que se conciba, “la modernidad tardía” ha
destruido la modernidad clásica. Como somos “cada vez más modernos” nos
separamos de la imagen según la cual subyace a nuestra acción un bloque de
valores sagrados y homogéneos. El “desencantamiento” con el mundo no
proviene del declive de los valores, sino del declive de su identificación con
un conjunto de principios sagrados, homogéneos y trascendentes que se
imponen a todos. Progresivamente el espíritu crítico que anima a la ciencia y
a la misma Razón se ha vuelto en contra de la confianza en la ciencia y la
razón misma y todos admiten que la autenticidad de las creencias y de las
convicciones tiene tanto valor como aquella autoridad. Bajo el asedio de la
crítica sociológica, la gran cultura que se ha impuesto a la escuela se percibe
progresivamente como un conjunto de elecciones de curricula más o menos
contingentes. Del mismo modo que el espíritu de la Ilustración ha terminado
por debilitar la autoridad de los principios religiosos, la modernidad crítica
erosiona hoy los principios del progreso y de la Razón.
La escuela debe instituir la nación pero, al menos en Francia, la misma
nación no sigue percibiéndose como la expresión más conseguida de lo uni-
versal; ya no resulta creíble explicar a todos los alumnos de Francia que sus
antecesores eran Galos. La escuela ya no es el templo encargado de construir
una nación homogénea cuando las distintas minorías exigen que sus singula-
ridades sean reconocidas en el espacio escolar, como muestra el episodio del
pañuelo islámico. También desde este punto de vista, se encomienda a la
escuela ser más democrática y menos republicana.
Junto a la masificación escolar, las finalidades de la institución se han
diversificado. La presión por la igualdad se ha incrementado con el modelo
de la igualdad de oportunidades y el aumento de la lógica competitiva de la
escuela (Dubet, 2004). La necesidad de adaptar la escuela a las demandas del
mercado de trabajo ha sido a su vez intensificada, y la institución fundamen-
ta menos su legitimidad sobre sus valores que sobre su utilidad. En otro plano
completamente distinto, los valores sagrados de la escuela han perdido algo
de su buena imagen frente a la influencia de los medios de comunicación que
ofrecen imágenes del mundo alternativas, desordenadas sin duda, pero extre-
madamente poderosas. Siempre se puede pensar que estos medios de comu-
nicación son vulgares, mediocres y anárquicos, están aquí por mucho tiempo
y proponen a los alumnos otras maneras de comprender el mundo distintas
de las de la escuela. Los medios de comunicación ofrecen hoy otras imáge-

52 Revista de Antropología Social


2007, 16 39-66
François Dubet El declive y las mutaciones de la institución

nes del mundo, otros modelos culturales y se sabe que los adultos y los alum-
nos dedican más tiempo a la televisión que a los estudios. A la larga, el rela-
to weberiano de la modernidad se ha cumplido a través del desencantamien-
to con el mundo, la instauración de la “guerra de los dioses”: los principios
del programa institucional no encarnan ya la unidad de la sociedad, no son
más que un conjunto de principios entre otros y, sobre todo, ya no se asien-
tan sobre una visión homogénea, vertical y trascendente de los valores.

De la vocación al dominio del oficio


Cuando decae lo sagrado institucional, la vocación se transforma. Ya no es
la vocación “católica” del sacrificio de sí mismo mediante el que el individuo
participa de lo sagrado que encarna; ya no es la vocación del sacerdote o del
monje que se olvida de si mismo por una causa superior y se separa por tanto
de la sociedad. Hoy la vocación de los profesionales del trabajo sobre el otro se
parece cada vez más al beruf de la ética protestante: es a través del mismo tra-
bajo, más que en una relación directa con los ritos religiosos como el individuo
cumple su vocación. Entonces la vocación se convierte en profesión y la profe-
sión es percibida como un modo de realización personal. El individuo quiere
estar cualificado y ser competente y, en un nivel más subjetivo, quiere realizar-
se con su profesión. Ya no se pregunta a los profesores si “creen” en Dios o en
la Razón, sino si se sienten hechos para este oficio, si les gusta enseñar, si
aguantan a los niños, si piensan que pueden realizarse con este oficio. Ya no se
pide a los enfermeros que aguanten la muerte y el sufrimiento de los enfermos
en nombre de la compasión cristiana, sino en nombre de un ethos estrictamen-
te profesional. Este desplazamiento de la vocación a la profesión y esta laiciza-
ción de la profesión están asociadas a una transferencia de legitimidad. La auto-
ridad del profesor no proviene ya de su vocación “sagrada” y del carisma que
la acompaña, sino de sus competencias, de sus cualificaciones, de su eficacia.
Está formado para ejercer eficazmente su oficio, para que los alumnos salgan
adelante y debe rendir cuentas con respecto a ello ante los alumnos, ante los
padres y ante la administración.
A medida que la escuela se aleja del programa institucional, el mundo
simple y homogéneo, porque todos comparten a priori los mismos valores, es
reemplazado por una organización cada vez más compleja. En todas partes, la
división del trabajo escolar se ha visto considerablemente intensificada. Se han
formado decenas de itinerarios y de especialidades nuevas, se han separado
progresivamente el trabajo educativo y el trabajo de transmisión, el control

Revista de Antropología Social 53


2007, 16 39-66
François Dubet El declive y las mutaciones de la institución

social y la escucha singular de personas, y la gestión se ha vuelto mucho más


fina y mucho más engorrosa que hace treinta años. “La Orden Regular” católi-
ca o republicana se ha vuelto una gigantesca burocracia profesional en la que
trabajan administrativos, especialistas de la pedagogía, psicólogos, orientado-
res, expertos como los sociólogos, políticos... A medida que la escuela deja de
ser una institución, se va convirtiendo en una organización “como las demás”.
Las mismas observaciones valdrían para el hospital y la magistratura.

El fin del santuario


El cambio más espectacular inducido por la propia escuela es el derrumba-
miento del santuario delante de los batallones de nuevos alumnos que provie-
nen de la masificación. Puesto que la escuela creaba movilidad social y jus-
ticia en una sociedad injusta, hacía falta ampliar a todos la oferta escolar
antes reservada sólo a los Herederos y a los Becarios. También se pensaba
que el aumento del nivel de escolarización desarrollaría el capital humano
convertido en la riqueza principal de las sociedades modernas. Esta política
proviene del propio mundo escolar y no parece agotarse. La masificación
escolar ha hecho explotar literalmente el santuario escolar. Los problemas
sociales no se han podido mantener fuera de la escuela y la han invadido. Las
culturas juveniles también han penetrado el templo acarreando con ellas un
desorden desconocido hasta entonces. La generalización de lo mixto ha con-
vertido la escuela en un espacio juvenil situado con frecuencia en el margen
del aprendizaje escolar. De manera general, cada vez más alumnos pasan
cada vez más tiempo en la escuela, los muros que protegían la institución se
han derrumbado.
Sobre todo, al abandonar el maltusianismo escolar, la escuela se ha vuel-
to una gran productora de títulos, hasta el punto de que la ausencia de título
constituye una verdadera desventaja social. Desde entonces la competitividad
escolar ha aumentado por el simple hecho de la multiplicación del número de
competidores. De esto se derivan dos consecuencias principales que los pro-
fesores condenan pero que la masificación que defienden ha producido: la
formación de un mercado escolar en el seno mismo de la escuela pública, y
el desarrollo del utilitarismo de las familias y de los alumnos, que todos con-
denan pero en el que todos participan, sin que los profesores se queden atrás
comportándose como consumidores ilustrados. Todos los ciudadanos que
pueden hacerlo eligen el establecimiento, los itinerarios y las clases particu-
lares de sus hijos.

54 Revista de Antropología Social


2007, 16 39-66
François Dubet El declive y las mutaciones de la institución

Mientras que el programa institucional conseguía protegerse de la deman-


da social e imponer sus reglas a los usuarios, el proceso se invierte y es la
escuela la que debe responder a múltiples demandas: las de la economía, las
de las familias que persiguen la mayor rentabilidad escolar, las de las diver-
sas comunidades culturales que ya no quieren diluirse en el molde institucio-
nal. Probablemente es esta disgregación del santuario lo que peor viven los
profesores porque les expone a la sociedad, porque les banaliza, incluso si al
mismo tiempo a menudo son los propios profesores los que han incluido
entre sus deseos la masificación escolar en nombre de las propias virtudes del
programa institucional.

La afirmación del sujeto


Como hemos defendido la tesis según la cual la Iglesia ha sido la matriz
general del programa institucional, evoquemos por un momento las transfor-
maciones de la relación para con lo religioso antes de hablar de la escuela.
Podemos considerar que el rito católico tradicional era una forma particular-
mente nítida de poner en relación la identificación de la obediencia con la sub-
jetivación (Hervieu Léger, 2003). El sacerdote, un mediador entre Dios y el fiel,
da la espalda a los creyentes para decir misa en un lenguaje desconocido para
la mayoría de ellos, el latín; para dirigirse a los creyentes, los domina hablan-
do desde el púlpito y los creyentes son definidos como “fieles”, cuyo primer
deber es el de sumisión y el de obediencia. Al arrodillarse delante del sacerdo-
te, los fieles se arrodillan ante Dios. El nuevo rito instaurado por el concilio
Vaticano segundo cambia la posición y la definición del fiel: el fiel se convier-
te en un laico y el sacerdote se pone frente a él para dar la misa en la lengua de
los asistentes; el sacerdote no se sube ya al púlpito y comparte la prédica con
algunos laicos. Este cambio de forma también es un cambio de fondo en el cual
el laico se considera un sujeto autónomo que tiene una demanda de creencia y
de fe, a la cual es importante atender y a cuyas preguntas hay que responder. El
ideal de obediencia del fiel queda sustituido por un modelo de diálogo, desde
luego con jerarquías, pero un diálogo a pesar de todo. Evidentemente este rito,
mucho menos mágico y encantado que el antiguo, indica claramente que nos
adentramos en una sociedad donde el sujeto ya no es considerado como el pro-
ducto de la obediencia y de la socialización, sino como un individuo que ya
existe, que tiene una personalidad, que tiene necesidades y que debe adherirse
libremente a la creencia. Su autonomía es el comienzo del proceso de sociali-
zación, ya no es simplemente su fin.

Revista de Antropología Social 55


2007, 16 39-66
François Dubet El declive y las mutaciones de la institución

Podemos evocar también el caso de la relación médica donde la norma del


abandono y de la confianza del enfermo analizado por Parsons (1955) va sien-
do progresivamente reemplazada por la de su adhesión razonada, de su infor-
mación, de su responsabilidad y de su compromiso con los cuidados. El enfer-
mo no se concibe simplemente como un objeto pasivo de cuidados y técnicas,
sino que es invitado a comportarse como actor de su tratamiento; su nivel de
información aumenta, sus quejas también, incluso ante la justicia.
La escuela ha sufrido una evolución similar pero más radical todavía con la
transformación del estatus del niño y del adolescente que ya no se distinguen
del de alumnos. Éste pasa a ser considerado teniendo una personalidad, un jui-
cio, una singularidad que la escuela debe reconocer y a partir de ahí asegurar
su desarrollo. En general, no esperamos de la escuela que discipline a los
niños para hacerles crecer, sino que les ayude a convertirse en lo que ya son.
Igualmente, la relación del maestro con el alumno ha cambiado inadvertida-
mente de naturaleza. Se insiste sobre la actividad de los alumnos, sobre su
compromiso con el aprendizaje, esperamos que se expresen y que los conoci-
mientos tengan sentido para ellos. Todas las nuevas pedagogías combaten los
aprendizajes automáticos, la simple memorización y el simple conformismo.
Muy a menudo este desplazamiento hacia una nueva pedagogía no es perci-
bido por los profesores como un proyecto verdaderamente nuevo, sino como
una adaptación a las demandas de los alumnos que ya no aceptan desempe-
ñar el papel tradicional de alumno “pasivo” y “fiel” porque su educación
familiar y las representaciones ideales del niño y del adolescente creativos e
iguales ya no les atraen (Singly, 1993). Así es como todo un estilo disciplina-
rio tradicional y toda una distancia pedagógica se replantean cuando se con-
sidera de antemano al alumno como un sujeto, antes incluso de entrar en la
escuela.

6. Crisis y decepción
El oficio es difícil
Esta larga mutación ha incrementado apreciablemente las dificultades del
oficio de profesor. En un contexto post-institucional, el profesional del trabajo
sobre el otro ya no es un “monje” o un “sacerdote” encargado de tomar el rele-
vo de la economía simbólica de la institución; se convierte en un trabajador
encargado de construir él mismo el marco simbólico de su actividad. El viejo
acuerdo pedagógico organizado por el programa institucional se ha roto y la
carga de trabajo se ha visto desplazada hacia los individuos. Parece poco dis-

56 Revista de Antropología Social


2007, 16 39-66
François Dubet El declive y las mutaciones de la institución

cutible que los oficios de sacerdote, de médico, de trabajador social y de pro-


fesor se han vuelto más exigentes y más pesados cuando ya no descansan sobre
un sistema de creencias implícitas e iguales para todos, cuando las demandas
de la sociedad se exacerban, cuando el usuario puede oponerse legítimamente,
cuando hace falta convencer a los individuos de la validez de su acción. El régi-
men de justificación pasa entonces de la institución al individuo.
En Francia, muchos profesores tienen la impresión de que la escuela ya no
está hecha para los alumnos de hoy y aún más a menudo que estos alumnos
no están hechos para la escuela. No sólo hace falta adaptarse a los alumnos
percibidos como cada vez más dispares, sino que este trabajo ya no está sos-
tenido por el dispositivo simbólico de la institución. Por ejemplo, el declive
del programa institucional hace que la autoridad del maestro ya no esté apo-
yada sobre un principio superior, sobre el hecho de que el maestro represen-
te una cosa más o menos sagrada a la que se le debe obediencia. Le corres-
ponde al profesor construir él mismo su autoridad movilizando aquello que,
a falta de otro concepto llamamos su personalidad. Ahí donde la subjetividad
del maestro desaparecía tras su rol, se antepone a éste, provocando un replan-
teamiento profundo y un agotamiento puesto que la clase ya no es una ruti-
na. Se puede hacer el mismo razonamiento con respecto a la motivación de
los alumnos ya que ésta ya no es adquirida en una escuela de masas que impi-
de excluir precozmente a los alumnos.
Ahora bien, la motivación en el trabajo escolar, ésta que algunos llaman
su sentido, se vuelve incierta cuando la cultura escolar entra en competencia
con otras culturas más seductoras, cuando la utilidad de los estudios ya no
está garantizada o está muy pospuesta, cuando las distintas culturas juveniles
y sociales invaden la escuela. Entonces es cuando el maestro tiene que moti-
var a los alumnos y también en este caso poner en juego su personalidad.
Finalmente, la apertura del santuario escolar ha desarrollado una demanda de
accountability, los profesores deben rendir cuentas ante otros distintos de
ellos mismos. Evidentemente esto se vive como un descenso de estatus, y con
toda la razón, pues al haber perdido el estatus su sacralidad, parece decaer
incluso cuando no se degrade desde el punto de vista material. No sólo el ofi-
cio es más difícil, sino que ha perdido su aura.

Decepciones
A estas dificultades profesionales se suma una profunda decepción ya que
la escuela democrática de masas está lejos de haber cumplido todas sus pro-

Revista de Antropología Social 57


2007, 16 39-66
François Dubet El declive y las mutaciones de la institución

mesas al tiempo que ha contribuido a destruir el programa institucional. La


esperanza de formar un mundo de cultura y de justicia parece haber sido trai-
cionada en numerosos países a pesar de que la nación ha consagrado muchos
medios a la educación. Las desigualdades sociales siguen pesando mucho en
las trayectorias escolares e incluso desde hace algunos años se han acentuan-
do. Y en este asunto, la escuela no puede considerarse perfectamente inocen-
te, como totalmente víctima de las desigualdades sociales, puesto que las
estrategias de los establecimientos, los itinerarios, las composiciones de cla-
ses parecen reforzar las desigualdades sociales (Duru-Bellat, 2004). Muchos
alumnos ya no creen en ella y, sobre todo muchos de los propios profesores
han perdido la fe “naíf” en la justicia de la escuela. Tanto en la escuela públi-
ca como en la privada, los padres se comportan como usuarios racionales en
un mercado escolar y buscan los itinerarios y los establecimientos más pres-
tigiosos, más eficaces y más rentables. En gran medida también la escuela ha
desarrollado un espíritu crítico que se vuelve contra ella y muchos profeso-
res se sienten profundamente afectados, incluso insultados por las críticas
que la sociedad dirige a la escuela. También es verdad que las comparaciones
internacionales que empiezan a difundirse desmitifican muy profundamente
las imágenes que a los sistemas escolares les gusta dar de sí mismos2. Criticar
el sistema, es cuestionar al personal educativo y aquí también, el sentimien-
to del declive del estatus es total.
Todo esto engendra un sentimiento de crisis endémica, una impresión de
caída continua que arrastra consigo todas las dimensiones del estatus de los
profesores. Cambiar los títulos, los servicios, los programas, las obligaciones
diversas, es alterar una dignidad que no es reductible al “honor” de los distin-
tos “Cuerpos” que componen la educación nacional, es afectar la dimensión
simbólica de este estatus. Más allá, es poner a la nación, la cultura y la Razón
en peligro. Quizás se podrá considerar estos propósitos excesivos. Sin embar-
go, los debates y las discusiones que han ido surgiendo a lo largo de los últi-
mos veinte años están dominados por esta tendencia a la generalización casi
sistemática según la cual cualquier reforma pretendería “rebajar” y “reducir” los
cuerpos profesionales y las disciplinas. En la medida en la que los profesores
son ampliamente identificados con las disciplinas que enseñan, todo cambio
pesa como una amenaza más o menos enmascarada contra el orden y la jerar-

2 Para restaurar la dignidad del sistema escolar francés ¡algunos intelectuales proponen suprimir la esta-
dística escolar!

58 Revista de Antropología Social


2007, 16 39-66
François Dubet El declive y las mutaciones de la institución

quía que fundamentan la situación de las disciplinas en la teología imaginaria


de los saberes. Al final, la Razón y la Cultura se verían amenazadas.
En Francia, donde el programa institucional republicano ha sido afirmado
con mucha fuerza, su declive provoca desde hace algunos años reacciones muy
conservadoras que podrían ser comparadas con ciertas formas de integrismo
católico que defiende los ritos tradicionales y la fidelidad. Estos movimientos de
profesores se califican a sí mismos de “republicanos” y se oponen a los “peda-
gogos” que serían, a sus ojos, los agentes de la destrucción de la escuela. Ellos
reclaman la vuelta a la escuela de antaño, la de la obediencia y el santuario, con-
tra los nuevos métodos y la presencia creciente de las demandas de la sociedad.
Pero mientras que generalmente los movimientos conservadores se sitúan casi
siempre a la derecha en el abanico político, aquellos se sitúan a la izquierda por
la defensa del ideal de una institución que fue identificada con una República
progresista. Pero la limitación de estos movimientos radica en que acentúan el
cierre de la escuela sobre sí misma y que implican que serán rápidamente exclui-
dos todos los alumnos que no estén ya dispuestos a seguir el juego de la institu-
ción y a creer en sus valores. Sin embargo, sigue siendo cierto, aunque se con-
sidere el declive del programa institucional algo irreversible o que es una “pro-
videncia”, como habría dicho Tocqueville, que esta duradera tendencia provoca
movimientos tanto en la escuela como en la religión, donde se multiplican las
sectas y los movimientos integristas. A veces estas contra-tendencias participan
ellas mismas de la modernidad que critican, porque apelan a una emoción y a
una fe más auténticas como lo indican ciertos fundamentalismos que no son sólo
anti-modernos, puesto que apelan a la libertad de ser “reaccionarios”.

¿“Providencia” o liberalismo?
Muchos actores y sociólogos que analizan las transformaciones de las ins-
tituciones observan tendencias próximas a las que acabamos de describir.
Pero la mayor parte de ellos, especialmente en Francia, las explican en fun-
ción de la expansión del liberalismo, del mercado y de la mundialización. De
este modo, es el cambio del entorno el primero en “atacar” al programa ins-
titucional. Este análisis es válido en la medida en la que el declive del san-
tuario enfrenta directa y brutalmente las instituciones con el mercado y la
sociedad. Sin embargo, este análisis confunde los efectos con las causas, por-
que, en lo esencial, el cambio institucional proviene de la misma modernidad
y de la amplia mutación social y cultural que implica.
Las instituciones escolar y hospitalaria se identifican desde hace mucho con

Revista de Antropología Social 59


2007, 16 39-66
François Dubet El declive y las mutaciones de la institución

la modernidad, con la ciencia y con la Razón, al mismo tiempo que somete el


programa institucional al servicio de estos valores. Ahora bien, estos principios
han erosionado ellos mismos el programa institucional al desarrollar el espíritu
crítico y la autonomía. Todo sucede como si el programa institucional no hubie-
se resistido a los valores de la modernidad. La mayor parte de las transforma-
ciones de la escuela, por ejemplo, provienen de la misma escuela. La demanda
de profesionalización proviene de los profesores y de sus sindicatos más que de
la sociedad. La nueva concepción de los niños y de los adolescentes está enrai-
zada en las familias de las clases medias escolarizadas donde los profesores tie-
nen una posición esencial. La crítica de los valores es el resultado de un mundo
intelectual producido por la escuela y que se vuelve contra ella. En el viejo len-
guaje de la dialéctica, se podría decir que el programa institucional ha entrado
en contradicción con la modernidad a la que inicialmente servía, de la misma
manera en la que el marco nacional ha entrado en contradicción con el mercado
capitalista que inicialmente había tenido por objetivo constituir.
El carácter “providencial” del declive del programa institucional puede ser
ejemplificado por las transformaciones de la familia “burguesa”. Afirmando
la igualdad y la libertad de los individuos, sin embargo ha seguido siendo una
institución estable en tanto que las mujeres y los niños se mantuvieron en
posición de dominados. Cuando las mujeres accedieron al mercado de traba-
jo y a la contracepción, cuando la juventud se prolongó con los estudios, la
institución “burguesa” quedó desequilibrada por la realización de los princi-
pios que perseguía, desarrollando así el divorcio y las recomposiciones fami-
liares, instaurando la “crisis” de la familia.
En consecuencia se puede lamentar el declive del programa institucional a la
vez que se observa que tiene algo de “fatal”. Al mismo tiempo, el retorno al pro-
grama institucional exigiría sacrificar libertades y autonomía que pocos de noso-
tros aceptaríamos de buen grado. Al fin y al cabo, esta es la tragedia de la con-
dición moderna dominada por el cambio continuo y por la nostalgia que todos
los “padres fundadores” de la sociología habían anunciado y descrito al hablar
de “tragedia de la cultura”, de anomia y de desencantamiento del mundo... El
sentimiento de crisis se impone entonces como la manera más evidente de apre-
hender la vida social ya que todo en ella parece inestable y efímero.

7. Del programa institucional a la experiencia social


La sociología debe resistir al tema de la crisis y no percibir ella también la
vida social como una larga descomposición. Pues al fin y al cabo, la sociedad

60 Revista de Antropología Social


2007, 16 39-66
François Dubet El declive y las mutaciones de la institución

siempre existe y continúa “funcionando”, los actores actúan y se adaptan: más


allá de los distintos malestares, se constituye una forma de vida social que sabe-
mos bien que mañana parecerá una versión integrada y “normal” de la vida
social desde una perspectiva nostálgica. Incluso si es difícil proyectar una ima-
gen estabilizada de lo que sustituye al programa institucional, hace falta asumir
el riesgo de intentar esbozar algunos trazos.

Del rol a la experiencia social


El trabajo de socialización se ha desplazado hacia los actores, en la medi-
da en que deben, en cierto modo, hacer lo que la institución hacía por ellos.
De este modo, la socialización institucional ya no depende fundamentalmen-
te del aprendizaje de los roles sociales, de los alumnos, de los maestros, de
los enfermos, de los médicos: porque estos roles no cristalizan la totalidad de
la institución, o, por decirlo de otra manera, porque estos roles se fraccionan
y tienen que ser habitados por la persona mientras que se esperaría más bien
que el rol constituyese a esta persona. Se puede ejemplificar este cambio
retornando al problema de las motivaciones.
No sólo con la afirmación del sujeto “a la baja” y el desencantamiento de
los valores “al alza”, las motivaciones subjetivas y el sentido de la acción ya
no son evidentes sino que los registros de éste último se multiplican y se
escinden. De ahí que el trabajo en las instituciones –el de los profesores y los
alumnos, el de los médicos y los enfermos, el de los sacerdotes y los creyen-
tes...– se desplaza desde las instituciones hacia los individuos que están
menos sostenidos por el sistema simbólico de un programa. Esto no implica
el declive de los roles sociales. No porque haya más roles, sino porque no basta
con desempeñar su papel para que el trabajo de socialización se cumpla. Los
individuos deben comprometerse subjetivamente con su trabajo, deben moti-
varse y motivar a los otros cuando el sistema de motivaciones no es evidente
y unánimemente compartido. Es más, con el declive del programa institucio-
nal, a los individuos se les priva de consuelos que podrían aportar, sea las
creencias de la institución, sea incluso su mismo cierre. Ni las enfermedades
en el hospital, ni los “dones” de los alumnos para los estudios son concebi-
dos como fatalidades deseadas por los dioses o por la naturaleza, y ni los
usuarios, ni los profesionales, los aceptan ya sin indignación o sin quejas. Las
instituciones tenían la capacidad simbólica de ser percibidas como espacios
protegidos de las impurezas del mundo y atribuían sus fracasos y debilidades
a la sociedad. Por ejemplo, el mundo del profesorado aceptó fácilmente las

Revista de Antropología Social 61


2007, 16 39-66
François Dubet El declive y las mutaciones de la institución

teorías que atribuyen los fracasos escolares a las simples desigualdades socia-
les porque la escuela seguía siendo “inocente”. Pero cuando el santuario se
erosiona, la culpabilidad se cuela en el trabajo; los usuarios protestan legíti-
mamente y los profesionales de las instituciones se ven privados de todas las
explicaciones que les liberaban de sus responsabilidades: ¿no hay fallos
médicos y fallos pedagógicos detrás de los fracasos?
Este trabajo es más difícil también porque el retroceso del programa institu-
cional deja expuestos a los individuos a una obligación de justificación y de
acountability. En el programa institucional, los profesionales del trabajo sobre
los otros rinden cuentas a unos superiores jerárquicos que comparten con ellos
los valores de la institución y todo se juega “por dentro” en la Iglesia, en la
Escuela, en el hospital y en el ejército... A día de hoy, cuando los muros del san-
tuario se desmoronan, los actores tienen que rendir cuentas ante los usuarios,
ante los clientes, ante los que financian que pueden en cualquier momento
abandonar el mercado de la educación, de la salud y de la religión... La opinión
espera transparencia, donde antes se conformaba con el secreto.
A fin de cuentas el trabajo de socialización ya no puede ser concebido
como consecución de roles sociales. Exige un compromiso de los actores que
tienen que socializarse a través de la construcción de sus propias experien-
cias. La teoría de la socialización más adecuada a este modo de socialización
es, probablemente, la de Mead (1963) porque parte del postulado según el
cual los individuos actúan en un mundo heterogéneo en términos de normas,
de valores y de roles sociales. Por ello, el problema de los actores es el de
construir la unidad de su experiencia y, a partir de ahí, de construirse a sí mis-
mos. Los actores se constituyen a través de “pruebas” que deben a la vez
construir y superar. ¿Cómo movilizarse a sí mismo para lograr hacer su tra-
bajo más allá de la multiplicidad de roles, de expectativas y de alternativas a
las cuales cada uno se ve confrontado? El problema es idéntico en el caso de
los usuarios de las instituciones: ¿cómo movilizarse y construirse mediante
las pruebas escolares, cómo construirse a sí mismo estando enfermo?

¿Qué instituciones?
El declive del programa institucional no significa que salgamos de las ins-
tituciones, que éstas se hayan vuelto inútiles o que sólo puedan ser concebi-
das salvo como sistemas de regulación de la acción tal como se habla hoy en
día de las instituciones económicas que regulan el mercado. Es difícil imagi-
nar que las organizaciones y los profesionales que intervienen sobre el otro,

62 Revista de Antropología Social


2007, 16 39-66
François Dubet El declive y las mutaciones de la institución

que forman la subjetividad de los individuos, no puedan mantener algo de las


instituciones y en particular la adhesión a principios fundamentales percibi-
dos como “superiores” a los individuos y capaces de dar sentido a una acción.
Si se admite que estamos irremediablemente comprometidos con un pro-
ceso de desencantamiento del mundo y del declive de lo sagrado, se pensará
que las instituciones están hoy confrontadas al problema que se ha presenta-
do a los regímenes democráticos después de la caída de los regímenes monár-
quicos, cuya legitimidad procedía de principios sagrados, en los que el rey
recibía su autoridad de dios y el maestro de escuela, la suya, del rey... Note-
mos que las figuras republicanas que han sucedido a estos regímenes han pro-
cedido a una especie de traslación de lo sagrado a la “virtud”, a la “Razón” y
a la nación al hacer de la soberanía popular la encarnación de esta unidad de
valores que reflejan la unidad de un pueblo (Renaut, 2004; Rosanvallon,
2001). Pero hoy, este desencanto ha dado un paso más, ya que la trascenden-
cia de los principios republicanos –en Francia y en los Estados Unidos los
republicanos son los más fieles defensores del programa institucional– se
enfrenta a la heterogeneidad de las demandas democráticas y al derecho sobe-
rano de los individuos. A pesar de ello, no estamos condenados a renunciar a
todo acuerdo sobre los valores y la definición del bien soberano siempre que se
piense que puede haber acuerdos sobre unos principios de justicia que preser-
van unos intereses particulares y unas identidades múltiples. En este sentido,
las teorías de la justicia nos pueden ayudar a definir los marcos de una acti-
vidad institucional en la medida en la que admiten de manera general que los
principios de justicia son heterogéneos y que las elecciones democráticas
pueden permitir jerarquizarlos de manera legítima. Evidentemente, desde el
punto de vista del programa institucional, esta solución da vértigo del mismo
modo que la democracia dio vértigo a los que vieron deshacerse el vínculo
entre lo soberano y lo sagrado. Se concluye de este razonamiento que las ins-
tituciones no tienen más remedio que ser más democráticas y más políticas si
uno no quiere dejarse encerrar en la nostalgia de una época dorada. La virtud
cardinal de las instituciones no es la de ser sagradas, sino la de ser justas.
Las instituciones son necesarias porque protegen a aquellos que trabajan
en ellas de una inestabilidad y de un compromiso subjetivo excesivo. En este
ámbito, se da un desplazamiento desde la legitimidad del “carisma” y de la
tradición hacia la eficiencia y la eficacia. El problema proviene de que, si
bien la eficacia es colectiva, el trabajo de las instituciones es de una natura-
leza mucho más relacional que técnica. A este respecto, parece que las insti-

Revista de Antropología Social 63


2007, 16 39-66
François Dubet El declive y las mutaciones de la institución

tuciones deberían formar a los trabajadores en términos profesionales, en tér-


minos de oficio y de competencias propias, más que diluir este trabajo en
unos procedimientos excesivamente formales y burocráticos. Cuando ya no
puede apoyarse en las vocaciones, la institución debe apoyar el oficio y a los
colectivos de trabajo. De hecho, sabemos que el equipo, el servicio hospita-
lario o el establecimiento escolar, son los verdaderos marcos de trabajo de los
profesionales.
Las instituciones ya no pueden protegerse de las demandas sociales, pero
por otro lado, no pueden ser simples organizaciones de servicios encargadas
de satisfacer estas demandas, aunque sólo fuera porque hay una tensión entre
sus principios, su profesionalidad y las demandas de los colectivos y de los
individuos. Por lo tanto es importante redefinir las fronteras del santuario ins-
titucional mediante mecanismos de filtro que, ellos también, deben ser con-
cebidos como procesos políticos.
En definitiva, las instituciones se ven confrontadas a unos sujetos que no
pueden ignorar y sobre todo, a individuos que deben transformar en sujetos.
Esto no supone abolir ni las reglas, ni las disciplinas, pero implica que la ins-
titución permita a los individuos elaborar experiencias singulares. Aunque la
institución construya pruebas, debe permitir a los individuos superarlas, no
necesariamente triunfar en ellas, pero por lo menos no ser destruidos. Hoy en
día, todos saben que la buena escuela no es sólo la escuela más eficaz sino
que es también la más justa y la que permite a los alumnos construirse de
manera singular. Todos saben también que el buen hospital no es sólo el que
cura mejor a los enfermos, sino el que los trata también como personas y no
sólo como cuerpos enfermos.

8. Conclusión
Mientras la regulación general de la vida social parezca globalizarse por la
extensión de mercados y de medios de comunicación, el riesgo de desaparición
de las instituciones es real y con él viene también el riesgo de que una serie de
bienes como la educación, la salud y la justicia ya no escapen de las lógicas de
mercado que erosionan la economía simbólica de la formación de los sujetos.
Esta evolución provoca entonces sentimientos de miedo y de defensa de una
tradición institucional que a veces no es más que nostalgia. Ahora bien, el decli-
ve del programa institucional no significa la muerte de las instituciones, sino la
transformación de las instituciones enfrentadas a un mundo más moderno, más
desencantado, más democrático y más individualista hace falta ver una muta-

64 Revista de Antropología Social


2007, 16 39-66
François Dubet El declive y las mutaciones de la institución

ción a la medida de la acentuación del proceso de modernización. En esta trans-


formación, es evidente que las instituciones –la escuela, el hospital, la justicia,
el ejército...– perderían su grandiosidad y sus protecciones. Se volverían a la
vez más “prácticas” y más democráticas. Está claro también que algunas de
ellas conocerían un cambio de escala y se dividiría en unidades más pequeñas,
más especializadas, más ligadas a sus comunidades de referencia y, de esta
manera, más fuertes.
Traducción: Laura Martínez Alamillo

9. Referencias bibliográficas
BERNSTEIN, B.
1975 Classes et pédagogies: visibles et invisibles. Paris: OCDE.

BOURDIEU, P., PASSERON, J.-C


1970 La reproduction. Paris: Ed. de Minuit.

DUBET, F.
2004 L’école des chances. Qu’est-ce qu’une école juste? Paris: Seuil. (La escue-
la de las oportunidades. Barcelona, 2006)
2002 Le déclin de l’institution. Paris: Seuil. (El Déclive de la institucion. Barce-
lona: Gedisa, 2006)

DUMONT, L.
1983 Essais sur l’individualisme. Paris: Seuil.

DURKHEIM, E.
1974 L’Education morale. Paris: PUF.
1990 L’évolution pédagogique en France. Paris: PUF.

DURU-BELLAT, M.
2004 Les inégalités sociales à l’école. Paris: PUF.

FREUD, S.
1992 [1929] Malaise dans la civilisation. Paris: PUF.

ELIAS, N.
1991 La société des individus. Paris: Fayard.

HERVIEU, D.
2003 Catholicisme, la fin d’un monde. Paris: Bayard.

Revista de Antropología Social 65


2007, 16 39-66
W
ha
ts
ap Ale
p: jan
35 dr
17 ía

- 69 -
61 Im
17 pre
27 si
- 3 one
51 s
76
11
63
2
W
ha
ts
ap Ale
p: jan
35 dr
17 ía

- 73 -
61 Im
17 pre
27 si
- 3 one
51 s
76
11
63
2
W
ha
ts
ap Ale
p: jan
35 dr
17 ía

- 74 -
61 Im
17 pre
27 si
- 3 one
51 s
76
11
63
2
W
ha
ts
ap Ale
p: jan
35 dr
17 ía

- 75 -
61 Im
17 pre
27 si
- 3 one
51 s
76
11
63
2
W
ha
ts
ap Ale
p: jan
35 dr
17 ía

- 76 -
61 Im
17 pre
27 si
- 3 one
51 s
76
11
63
2
W
ha
ts
ap Ale
p: jan
35 dr
17 ía

- 77 -
61 Im
17 pre
27 si
- 3 one
51 s
76
11
63
2
W
ha
ts
ap Ale
p: jan
35 dr
17 ía

- 79 -
61 Im
17 pre
27 si
- 3 one
51 s
76
11
63
2
W
ha
ts
ap Ale
p: jan
35 dr
17 ía

- 81 -
61 Im
17 pre
27 si
- 3 one
51 s
76
11
63
2
W
ha
ts
ap Ale
p: jan
35 dr
17 ía

- 82 -
61 Im
17 pre
27 si
- 3 one
51 s
76
11
63
2
W
ha
ts
ap Ale
p: jan
35 dr
17 ía

- 83 -
61 Im
17 pre
27 si
- 3 one
51 s
76
11
63
2
W
ha
ts
ap Ale
p: jan
35 dr
17 ía

- 84 -
61 Im
17 pre
27 si
- 3 one
51 s
76
11
63
2
W
ha
ts
ap Ale
p: jan
35 dr
17 ía

- 85 -
61 Im
17 pre
27 si
- 3 one
51 s
76
11
63
2
W
ha
ts
ap Ale
p: jan
35 dr
17 ía

- 86 -
61 Im
17 pre
27 si
- 3 one
51 s
76
11
63
2
W
ha
ts
ap Ale
p: jan
35 dr
17 ía

- 87 -
61 Im
17 pre
27 si
- 3 one
51 s
76
11
63
2
W
ha
ts
ap Ale
p: jan
35 dr
17 ía

- 88 -
61 Im
17 pre
27 si
- 3 one
51 s
76
11
63
2
W
ha
ts
ap Ale
p: jan
35 dr
17 ía

- 89 -
61 Im
17 pre
27 si
- 3 one
51 s
76
11
63
2
W
ha
ts
ap Ale
p: jan
35 dr
17 ía

- 90 -
61 Im
17 pre
27 si
- 3 one
51 s
76
11
63
2
W
ha
ts
ap Ale
p: jan
35 dr
17 ía

- 91 -
61 Im
17 pre
27 si
- 3 one
51 s
76
11
63
2
W
ha
ts
ap Ale
p: jan
35 dr
17 ía

- 92 -
61 Im
17 pre
27 si
- 3 one
51 s
76
11
63
2
W
ha
ts
ap Ale
p: jan
35 dr
17 ía

- 93 -
61 Im
17 pre
27 si
- 3 one
51 s
76
11
63
2
W
ha
ts
ap Ale
p: jan
35 dr
17 ía

- 94 -
61 Im
17 pre
27 si
- 3 one
51 s
76
11
63
2
W
ha
ts
ap Ale
p: jan
35 dr
17 ía

- 95 -
61 Im
17 pre
27 si
- 3 one
51 s
76
11
63
2
W
ha
ts
ap Ale
p: jan
35 dr
17 ía

- 96 -
61 Im
17 pre
27 si
- 3 one
51 s
76
11
63
2
W
ha
ts
ap Ale
p: jan
35 dr
17 ía

- 97 -
61 Im
17 pre
27 si
- 3 one
51 s
76
11
63
2
W
ha
ts
ap Ale
p: jan
35 dr
17 ía

- 98 -
61 Im
17 pre
27 si
- 3 one
51 s
76
11
63
2
W
ha
ts
ap Ale
p: jan
35 dr
17 ía

- 99 -
61 Im
17 pre
27 si
- 3 one
51 s
76
11
63
2
Ministerio de Educación, Ciencia y Tecnología de la Nación
Dirección Nacional de Gestión Curricular y Gestión Docente
Área de Desarrollo Profesional Docente

Martes 27 de junio de 2006 Ministerio de Educación, Ciencia y Tecnología, Pizzurno 935.

Institutos de Formación Docente

El significado de educar en un mundo sin referenciasi


Por Philippe Meirieu

La reflexión que deseo presentarles es Renunciar al porvenir es, sin embargo, la


tanto la del investigador en el campo de las tentación de la modernidad, la de instalarse
ciencias de la educación, como la del en el presente, en la inmediatez, en el de
profesor y la del ciudadano y la del padre de todo y rápido, en el “nuestros deseos son
familia también. Es entonces una reflexión órdenes”, en la organización del mundo en el
general sobre la educación, sobre la corto plazo, sin un relato del día siguiente, sin
situación educativa de la modernidad; es una un futuro posible y es esta reinvención del
reflexión que pretende encarar con fuerza los futuro, esa capacidad para recrear nuestro
problemas con los que nos encontramos futuro, esa capacidad para recrear juntos un
cotidianamente, que no desprecia ninguna de futuro, para darle a los chicos de hoy la
las dificultades con las que se topa cada posibilidad de pensar un futuro y de pensar
educador en su día a día y que sin embargo un futuro distinto al de hoy, un futuro de no
no hace ningún impasse con la reflexión esté determinado de antemano, un futuro que
filosófica. no sea un destino. Esa es la ambición de
todo educador. No podemos renunciar al
Vivimos, aunque sea una banalidad porvenir, no tenemos derecho a hacerlo y no
decirlo hoy, en un periodo de crisis en es esa una de las menores paradojas de la
materia educativa. En una crisis que se educación, el hecho de que la presencia de
desarrolla ya desde hace unos cuantos años, los chicos nos obligue a levantar cabeza. La
ya que desde 1991, en una hermosa novela existencia misma de chicos educados nos
de Rosenbaum, un famoso novelista impone abandonar esa única obsesión por el
norteamericano contemporáneo, escribe: presente e imaginar un mundo para ellos,
imaginar que ellos podrán imaginar el mundo.
“Todos hemos perdido a nuestros hijos, a Los chicos en ese aspecto nos brindan más
nuestros chicos, para nosotros es como si servicios que aquellos que nosotros les
todos los chicos de América hubieran muerto. dispensamos, porque ellos nos imponen el
Véanlos violentos en las calles, comatosos mundo y al respecto el educador es el que
en los centros comerciales, hipnotizados porque mira al niño, mira hacia el futuro. Es
delante de la TV. En el correr de mi lo que el filósofo Jonas llama “el principio de
existencia si algo ha pasado de terrible es responsabilidad”: somos responsables del
que nos han secuestrado a nuestros hijos. Yo futuro primero y solo el hombre puede ser
ignoro cuáles son las causas y cuáles son los responsable de aquello que no existe y
efectos, pero los chicos han desaparecido, puede hacer existir aquello que todavía no
eso yo lo sé, y lo mejor que podamos hacer existe. Maria Montessori, la gran pedagoga,
por ellos y por nosotros es morirnos de rabia decía “el hombre degeneraría sin este niño
con aquellos que los han tomado, aun que lo ayuda a elevarse, el hombre ayuda al
cuando no sepamos a que se parecerá todo niño a elevarse porque el hombre educa y
esto cuando el humo vaya disipándose.” eleva al niño”.

Sabemos que esa rabia para lo mejor y Les voy a presentar entonces, tres series
para lo peor engendra un porvenir y el autor de reflexiones en torno a la temática de
termina diciendo “las víctimas son aquellos educar en tiempos de crisis. Voy a recordar
que han renunciado al porvenir”. algunos de los elementos que constituyen la
crisis de la educación, luego evocaré la
necesidad de volver a los fundamentales
educativos antes de situar algunas Esta crisis de la educación se ve
referencias y algunas perspectivas. reforzada por algunos fenómenos
sociológicos, en particular, la desligazón
La crisis de la educación es una realidad entre generaciones. Podemos decir que
que está ligada, en cierto sentido, al vivimos una formidable aceleración de la
surgimiento de la democracia. Nunca hay historia que hace que la transmisión que
crisis de la educación en sociedades tradicionalmente se efectuaba por una
totalitarias, no hay crisis de educación en las superposición de generaciones ya no pueda
sociedades teocráticas ni en la sociedad efectuarse por medio de esta superposición.
dictatorial; la crisis es el reverso del vacío Con lo cual las generaciones se separan
que instalamos en el corazón mismo de la cada vez más una de otras y lo que separa
sociedad. La democracia afirma que el lugar hoy en día, en occidente, a los padres de 40
del poder esta intrínsecamente vacío, nadie años con respecto a un chico de 15, una
en sí está habilitado a ocupar ese lugar de generación, es eso que separaba a esa
poder, ni el intelectual, ni el hombre de dios, generación de 7 generaciones, hace un siglo.
ni el hombre providencial: el lugar del poder Pasan más cosas en 25 años que las que
está vacío y debe seguir quedando vacío y pasaron en 7 generaciones a lo largo de un
solo puede ser ocupado provisoriamente por siglo. Esto trae aparejada la aparición de
hombres que acepten ser los mandantes de problemas totalmente novedosos, para los
aquellos que les confían provisoriamente el cuales en nuestra propia historia no tenemos
poder. Entonces tenemos que alegrarnos de solución. Los padres están totalmente
la crisis de la educación, en la Unión carenciados porque no pueden usar con sus
Soviética de los años 50 no había ninguna hijos los métodos que sus propios padres
crisis de la educación, en el Irán del ayatolá utilizaron con ellos. Ningún padre puede hoy
Jomeini, en todos los países que están en en día buscar en sus recuerdos para
manos de una dictadura no hay reflexión preguntarse a qué edad hay que comprarle a
educativa. La crisis de la educación es el un chico un celular, evidentemente en
precio que pagan las democracias por la aquella época la cuestión esta no se
incertidumbre que asumen, en términos de planteaba. Esta aceleración de la historia, de
poder político, moral y social. Cuando una la aparición de nuevas tecnologías, nos
democracia afirma que no hay poderes en sí pone ante problemas inéditos para los cuales
y que son los hombres quienes asumen el no hay ningún catecismo escrito y tenemos
poder, no puede entonces imponer a la que intentar soluciones para esos problemas
educación una dirección única, una inéditos. Es por eso que la propia
trayectoria que sea la misma para todos. En parentalidad plantea problemas en la medida
la dictadura los padres que no educan a sus en que precisamente los padres de hoy no
chicos correctamente son considerados tiene escrito su oficio en ninguna parte y
disidentes y, en las sociedades totalitarias, tampoco tienen soluciones por ningún lado
incluso se les retiran sus hijos, ya que la para los problemas que les plantean sus
educación está considerada como un objetivo propios hijos. Y a esto debe agregarse
común, cosificado, indiscutible e indiscutido. además un medio ambiente mediático y
Entonces, en cierta forma, no solamente hay comercial que exaspera el infantilismo en la
que aceptar sino también reivindicar que hay propia sociedad. La publicidad, el conjunto de
y que haya crisis de la educación. Eso quiere los medios de comunicación, reducen al
decir que nadie detenta la verdad educativa, individuo a la condición de consumidor y el
que nadie sabe ni puede imponer en lugar consumidor es aquel que está en estado de
nuestro aquello en que debemos educar a regresión infantil. El motor de la economía es
nuestros hijos y lo que tenemos que hacer de el capricho, es la pulsión de compra, como
y con nuestros hijos. La cuestión de la crisis dicen los psicoanalistas. El motor de nuestras
de la educación está fundamentalmente sociedades es justamente eso, de las cuales
ligada a nuestra visión democrática. En el educador debe liberar al chico. Vivimos en
algunos momentos, incluso preferimos volver un mundo que constantemente dice a todos
a las certezas del pasado, porque tenemos “tus deseos son órdenes”. Mientras que
que asumir, como lo dice Milan Kundera, “la nosotros tenemos que enseñarle al chico que
insoportable levedad del ser”. sus deseos no son órdenes, los adultos
somos constantemente solicitados/ terrible y sabe que si distribuye su sonrisa y
requeridos para regresar a nuestra propia los besos que nos da y hasta nos da las
infantilización para comprar por ejemplo gracias, sabiendo que es él quien tiene todo
montones de cosas que no necesitamos para el poder sobre nosotros (a pesar de que
nada, pero que simplemente son el objeto de nosotros pensemos que tenemos todo el
nuestros caprichos, para tomar decisiones poder sobre él), él podrá hacer lo que quiera
rápidas, muy rápidas, mientras que con nosotros, porque basta con hacernos
tendríamos necesidad de tiempo y de pensar que él no nos quiere para que
serenidad para reflexionar. caigamos deshechos. Ese niño rey, que por
definición es un tirano, vive la totalidad del
Lo que hoy hace difícil la educación, es mundo de acuerdo a su propia subjetividad,
que está a contracorriente de lo que es el es un brujo, es un mago, no es por
carburante económico de la sociedad, del casualidad que en los cuentos para los
consumo individual, la pulsión inmediata y la chicos el brujo y el mago tengan un lugar tan
satisfacción de todos nuestros deseos. especial, hasta en Harry Potter.
Respecto a ello, me parece importante volver
a eso que yo llamo los fundamentos El niño siempre es un brujo que piensa
educativos. Entre esos fundamentos voy a que le basta con imaginar muy fuerte unos
citar brevemente algunos: el nacimiento, por bombones delante de él para que los
supuesto, “el hombre –dice Hannah Arendt– bombones aparezcan, de a poco va a tener
es un ser para el nacimiento, “el nacimiento que ir comprendiendo que su deseo no hace
es la continuidad del mundo”; el nacimiento la ley, que su deseo choca con la existencia
es también para cada uno de nosotros un de los demás y va a tener que aceptar salir
arranque permanente y continuo de la de su omnipotencia. Es difícil y doloroso salir
nostalgia de una felicidad solitaria y prenatal. de la omnipotencia, sobre todo cuando uno
Tenemos que hacer nuestro duelo, siempre, vive en un mondo que además nos invita a la
constantemente, de la satisfacción de todos omnipotencia todo el tiempo, y nos distribuye
nuestros deseos y todas nuestras pulsiones y objetos que son objetos de omnipotencia
este duelo no termina nunca y en este punto como el control remoto, por ejemplo, que es
nacemos y renacemos a cada momento por excelencia el objeto de la omnipotencia
hasta el momento final, hasta el momento de ya que en décimas de segundo uno puede
nuestra muerte. El nacimiento en realidad es optar por el mundo que quiere ver. Para
el surgimiento del sujeto, de un sujeto capaz emplear una metáfora de la teoría
de dotarse de proyectos y por tanto de psicoanalítica, podríamos decir que el control
proyectarse en el porvenir, de hacer remoto es un falo de alta tecnología, en todo
elecciones, de tomar decisiones, de eliminar, caso es un instrumento de alta tecnología. Y
de descartar, de dejar de lado y de darse vemos las consecuencias pedagógicas en las
prioridades, y la prioridad, por supuesto, es clases mismas: en los países desarrollados
salir de lo que los psicólogos llaman el los chicos llegan a clase con un control
egocentrismo inicial, el egocentrismo del niño remoto insertado en la cabeza y lamentan
rey. Todo niño que llega al mundo y que ha profundamente no poder hacer zapping en
sido deseado es un niño rey. Tiene a los clase. Ahí están obligados a quedarse en el
adultos a criterio suyo, porque los chicos mismo canal, y como el docente no puede
saben que lo primero que quieren los adultos rivalizar con la televisión, viven esa situación
es ser amados y amarnos, que haríamos como una profunda frustración. Ahora bien, el
todo por tener el amor de nuestros hijos y crecer es aceptar que el mundo existe por
que eso que se denomina el círculo de fuera de nosotros, que no somos
familia es siempre un círculo amenazado omnipotentes, que el mundo nos ofrece
porque cada uno, en el seno de una familia resistencia y que no depende de nuestra
así sea la más unida, quiere de todas propia voluntad, y que debemos renunciar a
maneras ser querido por el chico más de lo interpretarlo todo.
que el chico quiere a otro, aunque la familia
sea la más solidaria. Siempre estamos ahí En una clase, el chico generalmente
tratando de tener la atención del chico, de piensa que si la maestra lo mira es porque lo
que nos sonría, de reivindicarnos con su está espiando y si no lo mira es porque no lo
amor y el chico sabe que tiene un poder quiere, él piensa eso porque no puede
imaginar que si la maestra no lo mira es una tarea gigantesca que es asignada a cada
simplemente porque está haciendo otra cosa, hombre en todo momento: se trata de tener
está convencido de que todo lo que pasa bajo control esas prevenciones llenas de
está destinado a él y poco a poco va a tener esperanza, de interés, de manera tal que el
que ir aprendiendo que todo no puede ser otro no se vuelva invisible, que no
interpretado, que las cosas son y están fuera permanezca invisible. Él dice además algo
de él y que no siempre están en contra de él, que a mí me parece importantísimo para
que el niño no siempre está en el centro del nosotros los educadores y es que le pueda
mundo. dar razón al otro, que uno puede estar
equivocado respecto de sí mismo y
En una hermosa novela de otro autor equivocado respecto de sus propios
americano, hay un hermoso diálogo entre intereses y es una cosa que no es fácil de
una madre y su hijo. Ellos están en la orilla entender pero que es absolutamente
del río y ven pescaditos y el chico está un necesaria. Y esto me lleva a una idea: que la
poco nervioso y le pregunta a la madre y le educación es aprendizaje para la renuncia a
pregunta: “¿los pescados nos comerían si la omnipotencia. El niño cree que su deseo
estuviéramos muertos?” Y la mamá es ley, no soporta que su deseo pueda verse
responde: “me imagino, bajo el agua los trabado, siempre está a punto de su pasaje al
muertos se ponen blanditos y no sirven para acto; este niño, que yo llamo el “niño bólido”
nada, pero recordá una cosa, ellos no lo en mi trabajo, es el chico que no se queda
hacen por maldad, son así, nada más.” Es nunca en el mismo lugar; “es como un
muy importante –agrega la madre- “conocer resorte continuo” nos dicen los maestros, no
las cosas como son, no como quiero que le interesa nada, se levanta continuamente y
sean ni como querría que fueran, ni una cosa si tiene ganas de tomar agua va y toma agua,
ni la otra, como son. Tenés que descubrir si le molesta otro chico va y lo toma de los
que el mundo no está pensando en vos, que pelos, siempre está en el pasaje al acto,
no hace nada esperando hacerte mal, aun siempre en la inmediatez. No ha construido
cuando haya gente que lo piense, como pasa este espacio interno entre el pasaje y el acto.
con los chicos. El mundo tampoco intenta Ningún deseo está prohibido, aún el deseo
hacerte el gusto, el mundo está ahí y vos de matar; sabemos desde Freud que aquel
tenés que arreglarte con eso.” que no desea matar a alguien es porque
tiene un electroencefalograma chato, que lo
Este es un aprendizaje muy difícil para que está prohibido no es desear matar a
los chicos: el aprendizaje de la alteridad. El alguien sino hacerlo, porque entre el deseo y
aprendizaje del rostro del otro, como dice el acto hay una caja negra que unos la
Manuel Levinas, aparece progresivamente llaman conciencia, otros le dicen alma, otros
como una interpelación a la vez imperativa y razón. No importa como la llamen, para el
misteriosa porque jamás sé quien es y la educador es simplemente el aplazamiento
conciencia del otro me escapa radicalmente. del acto. Aplazar el pasaje al acto, aceptarlo
Y el chico tiene que aprender para tomarse el tiempo de analizar, de
progresivamente a entrar en relación con el encarar las consecuencias de los actos. Un
otro, a colaborar, a reconocerlo como su pedagogo que me gusta mucho que se llama
semejante pero también como un ser distinto. Janusz Korczac, un pedagogo polaco que
Allí hay algo muy complicado para los chicos, murió en Treblinka en el año 1942, había
el otro le da miedo, lo pone nervioso, lo creado en Varsovia orfelinatos para chicos de
inquieta y Levinas dice que en la presencia padres deportados, y en esas instituciones
del otro hay como un llamado a la identidad, donde había mucha violencia y los chicos se
porque su existencia misma me obliga a salir peleaban todo el tiempo, él intentó una
de mi propia identidad, a escuchar otra cosa. cantidad de métodos para que dejaran de
Y ahí hay todo un trabajo permanente de pelearse: dijo que los iba a castigar, que los
aceptación de la alteridad que es iba a dejar sin comer, que los iba a golpear.
consustancial a la educación. Encontramos Nada de eso funcionó, la violencia era más
en un pensador alemán, Georg Gadamer, fuerte. Un día se le ocurrió algo
esta fórmula que me parece muy justa y muy extraordinario, dijo: “a partir de hoy
bien escrita en cuanto al imperativo cualquiera puede agarrar a golpes a
educativo. Él dice que verdaderamente es cualquiera con la condición de que lo
prevenga por escrito 24 horas antes”, e podría ser ese lugar, podría ser ese lugar en
instaló la caja de peleas que era como un donde todos dejan la espada en la puerta, su
buzón donde los chicos escribían “quiero espada física pero también su espada
agarrarte a golpes mañana”, ese buzón se mental, la escuela podría ser ese lugar donde
vaciaba y se volvía a llenar y los chicos la búsqueda de la verdad, de la precisión, del
contestaban “¿por qué me querés pegar? Y rigor, vaya delante de la ley del más fuerte.
eso Korczac se lo impuso a chicos más Marcel Mauss dice que para empezar fue
chiquitos que no sabían leer ni escribir y que preciso primero dejar las lanzas; así ocurrió
tenían que encontrar a alguien que les con el clan, las tribus, los pueblos y así será
escribiera esa carta o descifrara lo que otros mañana en nuestro mundo considerado
habían intentado escribir. Cuando el civilizado: las naciones y los individuos deben
pedagogo inventa esta caja de peleas confrontarse sin sacrificarse unos a otros, en
inventa, a la vez, la educación y la un registro que no es el de la violencia. Las
democracia y sobre todo muestra que el crónicas de Arturo cuentan de qué manera el
desarrollo psicológico y ciudadano es el rey Arturo, con la ayuda de un carpintero,
mismo. Hay una perfecta simetría entre inventó esa maravilla del patio milagroso en
acceder al estado adulto y acceder al estado torno del cual los caballeros ya no se
ciudadano. La modernidad descubre esto: batieron más.
descubre que el ciudadano es precisamente
aquel que renuncia a lo infantil, es aquel que Para nosotros, educadores, nuestra
sabe tomarse el tiempo de examinar las misión hoy es refabricar la mesa redonda, es
consecuencias de sus actos, que no está en crear espacios donde los seres puedan
la inmediatez, que está en el tiempo de la comunicarse sin pelear y en eso hay algo
reflexión y esto es lo que me hace decir que fundamental que me parece ser el gran
toda educación es una educación para el desafío de la modernidad. En las sociedades
aplazamiento, no es una educación para la tradicionales podía esperarse que la gente
frustración. Como decía Freud, yo no creo dejara de pelearse o bien por el miedo al
que la cuestión pase por decirle al chico que castigo, a la sanción o porque estaban bajo
trate de renunciar a sus deseos, sino que hay influencia de una ideología única. En una
que examinar sus deseos, pasarlos por un democracia que acepta la diversidad, que
tamiz, por el tamiz de su conciencia, anticipar acepta la pluralidad para que la gente no se
las consecuencias de sus actos y examinar, enfrente, va a tener que aprender primero a
mas allá de su interés individual, el interés encontrarse, y no como en las sociedades
colectivo. Por eso es que la educación y la tradicionales que funcionaban de manera
democracia se inscriben en el mismo fusional, como una identificación clánica.
movimiento: es la renuncia al narcisismo. Cuando aceptamos la diversidad, el
Educar a un chico es ayudarlo a renunciar a encuentro y la creación de la posibilidad del
su narcisismo y educarnos como pueblo encuentro hacen al fundamento mismo de la
democrático es educarnos para renunciar sociedad, solo hay sociedad en torno de la
cada uno de nosotros a nuestros intereses mesa redonda o bajo la influencia de un jefe,
individuales, para reflexionar acerca de lo pero como queremos democracia tenemos
que podría ser el bien común y el interés que construir esas mesas redondas. Hemos
colectivo. En una democracia la escuela no construido eso que se llaman las grandes
es otra cosa que el lugar de proyección instituciones del Estado, el parlamento, etc.,
posible del aprendizaje de la democracia, etc., que funcionan más o menos bien pero
justamente. Es ese lugar en el cual se rehace que funcionarán mejor si justamente
la experiencia fundadora que describe Marcel construimos a todos los niveles y desde la
Mauss en la conclusión de su ensayo sobre infancia mesas redondas en donde los seres
el don, cuando el autor evoca a los puedan encontrarse.
caballeros de la Mesa Redonda y evoca ese
acto fundante que consiste en colocar la Como tercera y última serie a partir de
lanza a la entrada de la sala diciendo “oh, tú este momento, ¿qué referencias podemos
que entras aquí, renuncias a tu violencia”: darnos, mas concretamente para la
entras en un espacio en donde está la educación?. Cinco referencias rápidas:
meditación, la reflexión, y en donde todo esto
lo hace la ley y no tu espada. La escuela
Nacer al mundo, nacer a la ley, nacer a lo que hay una diferencia fundamental entre la
posible, nacer a la voluntad, nacer a lo muñeca y la hermanita. El problema hoy es
político. que para muchos chicos el mundo entero se
Nacer al mundo es difícil porque para el transforma en un juguete y vemos a chicos
niño el mundo no existe: el mundo es la TV. jugar con el cuerpo de los demás sin
La ficción y la realidad se mezclan imaginar que ese cuerpo está habitado por
constantemente: ¿dónde está la realidad?, alguien, así como los generales lanzan
¿dónde está la ficción? En cierta forma hoy bombas a los países sin imaginar que esos
existir es salir en TV, el que no aparece en países están habitados por gente. La
TV no existe y el chico sabe de esa idea. El capacidad de comprender que la acción
mundo ya no es mundo, es lo que la TV humana no es un juego porque está inscripta
muestra del mundo y lo que la TV no puede en una historia irreversible, porque en la
mostrar, no existe. Lo que no es televisado ni realidad las heridas aun cuando se cierren no
televisable es negado en su existencia. El se olvidan nunca y no se le puede hacer mal
ejercicio de la razón no es demasiado a nadie impunemente, implica un aprendizaje
televisable, es mucho menos televisable, es muy complejo para el chico. Para conocer el
mucho menos espectacular que muchos mundo hace falta salir de la relación dual de
otros acontecimientos. Y llegamos a la idea conflictos y deseos, hay que introducir eso
que lo que no es demasiado espectacular no que denominamos el tercero mediador, hay
tiene derecho a existir. La meditación no es que hacer juntos. Creo que lo esencial de
espectacular, sin embargo ¿puede tener una educación democrática consiste en
derecho a la existencia? Tenemos que aprender a hacer en conjunto. Hacer junto
acompañar al chico en la idea de que con los otros para descubrir que nuestro
Superman no va a llegar a ultimo momento deseo no es omnipotente, que la autoridad
para salvar al mundo, que eso es ficción. verdadera no es la competencia. Creo que la
Que en la realidad Bruce Willis no existe, es democracia debe insuflar el deseo de hacer
una creación, es ficción. Tenemos que salir cosas en conjunto, tiene que ayudar a todas
de lo imaginario para no confundir al mundo las iniciativas donde haya individuos que se
con una película o un juguete. Uno de mis pongan juntos en un proyecto y aprendan a
libros lo titulé “El mundo no es un juguete” y trabajar y a realizar cosas juntos, porque
lo llame así porque salió algunos meses actuar y hacer en conjunto permite ponerse a
después de iniciada la guerra de Irak y la escucha, construir una autoridad que
después de la caída de los norteamericanos encarna al mundo y que ya no encarna en
en Afganistán. Me había impactado ver a los capricho de uno: es en ese hacer juntos que
generales del Pentágono comandar aviones superamos el capricho. Por supuesto, hay
sin pilotos y lanzar bombas a distancia sobre que acompañar a los chicos en ese hacer
blancos virtuales, como si no hubiera ninguna juntos porque es muy difícil, impone
población ahí y como si se tratara renuncia, disciplina y una gestión de la
simplemente de un juego electrónico. Para temporalidad, pero todo esto es lo que
los políticos el mundo se había convertido en permite salirse de la omnipotencia y llegar al
una gigantesca Playstation manejada desde mundo: esto es lo que abre las puertas al
el Pentágono, y esa transformación del mundo.
mundo en juguete, a través de los adultos Un pedagogo que me gusta mucho, les
mismos, por ellos mismos, es evidentemente suele decirle a los educadores “tenés que
el anverso de lo que le tenemos que enseñar saber lo que querés, si lo que querés es
a los chicos. Les tenemos que enseñar que hacerte amar llevale bombones. Pero el día
tiene derecho a romper ese castillo que ha que llegues con las manos vacías te van a
hecho con sus cubos porque la característica tratar de lo peor. Si verdaderamente querés
del juego es justamente la reversibilidad: uno hacer tu trabajo de educador, llevale una
puede romper su castillo porque puede volver cuerda para que tiren de ella”. Estamos
a reconstruirlo y no hay daño ahí. Pero en la hablando de 1945, hoy podríamos decirle
realidad, cuando uno destruye el bien común llevales cosas para hacer, proponeles que
está dañando la posición colectiva y se le hagan cosas en vez de intentar satisfacer
crea una herida a la colectividad. El chico constantemente el deseo de ellos, después
sabe bastante rápido que le puede sacar los vendrá el amor y ahí no está tu recompensa.
ojos a la muñeca pero no a su hermanita, Yo soy muy sensible a esta idea, el chico
necesita ser útil como el adulto. El mismo manera de vestir, de hablar, de comportarse;
pedagogo cuenta una historia de un chico es en el seno de los grupos que la influencia
con grandes dificultades, que estaba al borde que se ejerce hace que esté prohibido amar
del suicidio, que viene a verlo y le dice “hacé algo más que se halle por fuera del grupo,
algo por mí, te pido que hagas algo por mí” y que esté prohibido salir del grupo, que esté
él le responde “no puedo hacer nada por vos, prohibido distinguirse del grupo. Y hoy
nada. Pero vos si podés hacer algo por mí, asistimos a un ascenso en las últimas
podés ayudarme” y lo empleó al chico en su generaciones del fenómeno tribal. El centro
asociación y se convirtió en el brazo derecho de esta tribu, la autoridad que se ejerce
de este hombre. Él se negó a hacer algo por sobre cada uno de los miembros, es de
el chico, por eso yo le pedió que hiciera algo enorme fuerza y nuestra responsabilidad de
por él, le dio un lugar. En una patria, en una adulto es ayudar al chico a liberarse de esa
clase, en una sociedad, cuando alguien presión de la norma que se ejerce incluso a
quiere ocupar todo el lugar es porque no través de las marcas de comercio, de la ropa
tiene lugar, eso vale para el hermano, para la que hace que haya que ser igual a los
hermana, para los alumnos, eso es verdad demás, que haya que ser conforme al
para aquel que ocupa todo el espacio: prototipo. Eso es algo viscoso para los
cuando alguien quiere ocupar todo el espacio chicos, eso les impide acceder a nuevas
es porque no tiene lugar. Darle un lugar en posibilidades, eso los hace sentirse seguro
una obra colectiva es permitirle ya no tener de sí mismo en los proyectos. El chico
más la voluntad de ocupar todo el lugar, necesita que el adulto haga una alianza con
también es así como el chico aprende a él para escapar a todas las formas de
entrar en la ley, así es como aprende que las influencia y de fatalidad. Puedo ayudarte a
reglas no caen del cielo sino que son que seas distinto de lo que te imponen ser y
requeridas por el proyecto que perseguimos mi rol es abrirte posibles, abrirte posibilidades
en común. El adulto no es la regla, él es insospechadas, proponerte objetos culturales
quien lleva al chico hacia la regla. Si el adulto que ignorás, proponerte posibilidades
es la regla, cuando el adulto se va ya no hay profesionales que ni siquiera sospechás,
más regla, la regla no puede ser impuesta, ya proponerte trayectorias de vida que te
está construida, y está construida desde el permitan dejar de lado todas las fatalidad
momento en que se descubre que la sociales y económicas y resistirte a todos
prohibición autoriza, desde el momento en estos tipos de imperio sobre vos. Esto
que se descubre que todas nuestras impone también ayudar al chico a forjarse la
prohibiciones están hechas para autorizar. La voluntad, la libertad solo puede llegar si el
prohibición del incesto autoriza la existencia chico se pone en juego él mismo, si pone en
de la sociedad, la prohibición de hablar todos juego su propia voluntad. Esto supone una
al mismo tiempo permite expresarse, la verdadera formación de la voluntad, una
prohibición de pasar la luz roja: la prohibición voluntad que no niega el deseo sino que lo
autoriza. La verdadera prohibición es aquella pone a prueba de la ley. Para trabajar la
que le permite ver al chico todo eso que la voluntad hay que trabajar siempre la relación
prohibición va a autorizar, y cuando el chico entre deseo y ley y el deseo y la ley no
obedece tiene que saber que no es por pueden articularse más si el niño simboliza
sumisión sino porque es una promesa de ese deseo, si lo representa. Todos tenemos,
libertad, nacer a la ley es también nacer a lo el niño también, todos tenemos pulsiones
posible, porque lo que impide la libertad es el arcaicas, todos tenemos corazones pero
encierro y ese encierro es tanto más también tenemos el deseo de apropiarnos del
poderoso cuando es manejado por grupos otro, amamos a los demás, y al mismo
que ejercen influencia sobre las personas. tiempo amándolos puede que no soportemos
Uno de los problemas más importantes para quererlos libres, por eso es tan importante el
las nuevas generaciones no es la ogro en la mitología. Porque el ogro es aquel
desaparición de la autoridad, sino que el que tanto nos quiere que nos come, y
ejercicio de la autoridad sobre los jóvenes es nosotros los humanos nunca vamos a
hoy mucho más potente, poderoso y tiránico resolver la cuestión de saber cómo amar
que en el caso de las autoridades tanto a alguien sin comérnoslo, o cómo que
tradicionales. Es el ejercicio de la autoridad alguien nos ame tanto pero sin que nos
por estrellas de los medios que imponen su coma, cómo amar a alguien sin privarlo de su
libertad. Todas estas cuestiones no las ha objetos culturales que encarnen esas
resulto nadie y lo mejor que hay que pensar contradicciones y que le ofrezcan la
es que es mejor que nadie las resuelva, mediación necesaria para que él pueda, en
porque no habría más literatura, no habría cierto punto, domar al mundo.
mas cine, no habría nada más.
Nosotros somos portadores de cosas
La única cuestión que nos ocupa es espantosas y el chico también es portador de
cómo ser amado por alguien sin correr el ellas, no podemos hacer como que lo
riego de que nos coma. El chico tiene ese ignoramos. El chico también es un ser
problema y también tiene miedo de no estar violento, pero el objetivo no es prohibirle la
a la altura del deseo de sus padres, tiene violencia sino metabolizar la violencia. La
miedo a ser abandonado, y lo que tengo que finalidad de la cultura es metabolizar la
ofrecerle son objetos culturales donde esos violencia y a partir de ahí podemos llegar a
miedos, esos deseos, encuentren formas puntualizar la actividad del niño para
psicológicamente manipulables. Eso es la progresivamente nacer a lo político.
simbolización, es la capacidad de un objeto
cultural de nombrar las propias fuerzas Lo político es hacer nacer la sociedad,
internas, esas contradicciones, esas una sociedad que no es una comunidad. En
tensiones; es nombrar todo eso para poder una comunidad vivimos juntos porque
manipularlo: o manipulamos nuestras compartimos el mismo pasado, los mismos
pulsiones o son ellas las que nos manipulan. gustos, las mismas elecciones; puede haber
La capacidad de manipular nuestras una comunidad de pescadores, una
pulsiones pasa por representaciones comunidad de gente que le gusta leer los
simbólicas, sobre todo a través de los primeros escritos de Trotzky, o una
cuentos, los mitos. En nuestras sociedades comunidad de gente que le gusta el rap. Una
desarrolladas hay un déficit de lo simbólico sociedad es un conjunto de comunidades
muy fuerte, este déficit deja la puerta abierta que acepta que existen leyes que
al uso mercantil del símbolo. Cuando no hay trascienden su pertenencia comunitaria. A tal
más cuentos está Walt Disney, cuando ya no título la escuela no es una comunidad, es
hay mitología esta Hollywood. No todo es una sociedad que enseña que más allá de
malo, cuando hace algo como la Guerra de las comunidades existen reglas societales
las Galaxias es porque encuentra los que les permiten coexistir a las comunidades,
fundamentos de la mitología, pero esto tiene que les permite a cada uno hacer sus
que alertarnos sobre la necesidad, en una elecciones, tener sus gustos, tener sus
sociedad laica, de no abandonar la forma de propios deseos pero que también permite
expresión cultural que permite a los chicos vivir juntos y darse un marco común. La
pensar la propia historia. El hombre lobo, sociedad impone superar o ir más allá de las
esta persona que de noche se transforma en individualidades comunes educamos para el
lobo, el chico necesita hablar de este lobo bien común, para la polis griega y ahí la
porque este chico tiene que saber que sus educación es educación a lo político. Cuando
padres pueden amarlo, pero que ese padre y yo le pido al chico que renuncie a ser el
esa madre pueden también ser muy centro del mundo le pido como ciudadano
violentos, que pueden transformarse en ese que se inscriba en un colectivo que renuncia
lobo, que todos podemos transformarnos en a que su comunidad le imponga su ley a lo
ese lobo, que todos podemos tener colectivo. Renunciar a ser el centro del
momentos de cólera en los cuales una mundo es a la vez la condición para
cantidad de cosas se nos escapa, la violencia aprender, aprender una lengua extranjera,
se nos escapa. Si el chico no entendió que aprender historia, geografía, matemática,
ese lobo es el hombre que puede pero también es la condición para vivir en la
transformarse en lobo y luego volver a ser sociedad democrática. Por eso el aprendizaje
hombre, no va a poder entender por qué su de saberes es condición para la ciudadanía,
padre que tanto lo quiere un día se va a no son dos cosas diferentes, es lo mismo. El
poner tremendo y va a decir cosas muy feas, aprendizaje de la alteridad es la renuncia a
cómo un chico puede pensar las estar en el centro, es el hecho de hacer
contradicciones de comportamiento de su existir la democracia reconociendo siempre el
propia familia. Solo puede pensarlas si hay espacio vacío del centro. Es un esfuerzo
permanente de los hombres mantener ese emancipación, una pedagogía apoyada en un
espacio vacío en el seno de la familia, de la método experimental donde el chico haga
clase, del barrio, de la ciudad, del país, del hipótesis y las verifique, una pedagogía de la
mundo. Mantener el espacio vacío, diciendo investigación documentaria donde el chico no
que nadie tiene derecho a instalarse en el crea ciegamente en la palabra de nadie y que
centro del mundo: ni el chico en el centro de sí vaya a verificar, vaya a buscar fuentes
la familia ni el tirano en el centro de la ciudad. para seguir buscando la verdad y buscar la
La democracia es eso. verdad por sí mismo
Para decir las cosas rápidas, y ya voy a ir
terminando, la educación en lo político Tenemos la posibilidad de educar a
proviene en primer lugar del reconocimiento nuestros chicos para que sean buscadores
de los demás; pasa simplemente en las de verdades y no para que queden atónitos
formas ritualizadas del saludo: el saludo ante cualquier ídolo que haya. Pienso que la
implica decir te reconozco. Esto también desaparición de lo que llamamos referencias
inicia el aprendizaje del respeto por el otro, puede ser una magnífica oportunidad para
no vivirlo al otro como un peligro sino como construir nuevas referencias, nuevas
un ser a la vez semejante y distinto. Aquí referencias en torno a nuevos valores que
tocamos realmente el corazón de una quedan por inventar: esa es la razón por la
educación para el futuro, de una educación cual en este misterio no tenemos que
que permita a cada cual existir mas allá de volvernos siempre hacia el pasado.
una relación de poder. En cierta manera lo
que hay de formidable hoy es que vivimos la Desconfío de aquellos que pretenden
muerte de los dioses. Vivimos la muerte de que estamos en decadencia, prefiero
los ídolos y estamos en los inicios de la aquellos y aquellas que nos dicen que
invención de algo que es la posibilidad de un podemos construir un porvenir juntos y que
mundo fundado en la cooperación, en la nada está jugado definitivamente y que ese
solidaridad, en la confrontación y no en la porvenir puede ser mejor que el presente y el
adoración de ídolos. Por eso es que no soy pasado.
nostálgico del pasado. Pienso que hay muy
grandes razones para inquietarse por el En occidente hoy está muy bien visto
porvenir, pero también creo que hay muy estar desesperado, hay como una estética de
buenas razones para esperar. El hecho que la desesperación, de la falta de esperaza, y
el cielo esté vacío quizás quiera decir que ha los que no están desesperados pasan por
llegado el tiempo de los hombres, el tiempo idiotas, pero yo prefiero pasar por idiota
de que los hombres hagan su ley, que la porque pienso que en la desesperanza no
hagan ellos mismos y les enseñen a sus hijos hay porvenir.
que son los hombres los que hacen la ley y
que la hacen juntos y no por separado. Muchas gracias por haber venido.

Dije al principio que había una crisis de


autoridad, una crisis de la educación, pero i
Desgrabación de la traducción simultánea
creo que esa crisis es una posibilidad, una
posibilidad de fundar una sociedad a la altura
del hombre y una posibilidad de crear una
educación de individuos que sean
ciudadanos libres. Esto es muy difícil, lo es
porque siempre hay mucha gente que quiere
ocupar el centro y siempre tenemos que
hacer extraordinarios esfuerzos de negarnos
a confiarle nuestro destino a alguien que se
reivindique en la omnipotencia y eso también
se lo tenemos que enseñar a nuestros
chicos. Podemos hacerlo, podemos
enseñárselo en la medida en que pongamos
en ejercicio una pedagogía que haga una
alianza entre la transmisión y la
W
ha
ts
ap Ale
p: jan
35 dr
17 ía
61 Im

- 115 -
17 pre
27 si
- 3 one
51 s
76
11
63
2
W
ha
ts
ap Ale
p: jan
35 dr
17 ía
61 Im

- 116 -
17 pre
27 si
- 3 one
51 s
76
11
63
2
W
ha
ts
ap Ale
p: jan
35 dr
17 ía
61 Im

- 117 -
17 pre
27 si
- 3 one
51 s
76
11
63
2
W
ha
ts
ap Ale
p: jan
35 dr
17 ía
61 Im

- 118 -
17 pre
27 si
- 3 one
51 s
76
11
63
2
W
ha
ts
ap Ale
p: jan
35 dr
17 ía
61 Im

- 119 -
17 pre
27 si
- 3 one
51 s
76
11
63
2
W
ha
ts
ap Ale
p: jan
35 dr
17 ía
61 Im

- 120 -
17 pre
27 si
- 3 one
51 s
76
11
63
2
PROGRAMA
EXPLORA
LAS CIENCIAS EN EL MUNDO CONTEMPORÁNEO
DE CAPACITACIÓN
MULTIMEDIAL

PEDAGOGÍA

DOCENTES: LA TAREA DE CRUZAR


FRONTERAS Y TENDER PUENTES

Introducción | Una relación con la cultura, una autoridad cultural | Método y vocación. Pensar pedagogías
para este tiempo | La política y la sociedad constituyendo la tarea | El empleo docente | Una relación con el mun-
do del trabajo | Conclusiones: los educadores frente a los desafíos del mundo contemporáneo

Autores: Dra. Myriam Southwell (UNLP / CONICET / FLACSO), con la colaboración de la Lic. Silvia Storino | Coordinación Autoral:
Dra. Myriam Southwell (UNLP / CONICET / FLACSO)
2 EXPLORA PEDAGOGÍA

INTRODUCCIÓN

El monitor de la educación
La profesión docente es el resultado de la construcción de una serie de rasgos, funciones y representaciones a lo largo de la historia.

L a docencia es hoy un trabajo en el que


se centran grandes expectativas y que,
a la vez, es frecuentemente puesto en
nes pesan sobre ella? ¿Qué aspectos de la
sociedad y la cultura contemporáneas
revisan y renuevan los rasgos históricos en
saberes y las formas de su enseñanza; una
relación con los otros y lo que ellos gene-
ran en uno, con la política y la sociedad,
cuestión. En ocasiones, parece ser una los que se asentó el enseñar? ¿Qué cons- con el mundo del trabajo y las múltiples
profesión en permanente crecimiento; en trucción identitaria se desarrolló y cuáles estrategias que desarrollamos para ubi-
otras, muchos perciben que su trabajo son las nuevas identidades que se están carnos en él.
adquiere características muy distintas de gestando? ¿Cómo desarrollar una pers- Pensar un trabajo docente enriquecido
las conocidas. Nos gustaría reflexionar pectiva renovada para la transmisión para este tiempo, probablemente implique
sobre cómo está cambiando el oficio intergeneracional que asegure el pasaje dejar atrás algunos modelos que ataron la
docente, tomando como punto de partida de la cultura de adultos a jóvenes? En el certeza de lo instrumental a una relación
tanto las transformaciones en la estructu- texto que sigue nos proponemos abrir autoritaria y empobrecida con el conoci-
ra del sistema educativo y las nuevas estos interrogantes a partir de la síntesis miento. El papel del profesor no puede ser
pedagogías como los cambios sociales, entre algunos rasgos construidos a lo lar- pensado hoy como un rol escrito de ante-
culturales y políticos que envuelven a las go de la historia y los dilemas de hoy. mano. En un sentido similar, será producti-
escuelas. Se nos abre, así, la necesidad de Enseñar es −a riesgo de ser un poco vo revisar cuánto hemos ganado en las
pensar el presente y plantear nuevas esquemáticos− establecer una relación, es transformaciones que el oficio ha ido des-
pedagogías en diálogo con la sociedad en decir, construir una posición que no está plegando, analizar como productivas inclu-
la que vivimos. situada en coordenadas predefinidas, fijas so algunas incertezas que ponen a prueba
¿Qué sentido tiene hoy enseñar? ¿Qué y definitivas sino que sufre alteraciones y nuestro juicio profesional, dotándolo así de
dilemas presenta la época a la fisonomía busca e inventa respuestas. Esa relación se mayor autonomía en su vinculación con el
más conocida del enseñar? ¿Qué tradicio- establece con la cultura, el poder, los conocimiento y con el mundo.
DOCENTES: LA TAREA DE CRUZAR FRONTERAS Y TENDER PUENTES 3

UNA RELACIÓN CON LA CULTURA, UNA AUTORIDAD CULTURAL

S in lugar a dudas, lo que funda el sentido


del trabajo de enseñar es la relación con
la cultura, esto es, la relación propia y la que
ción del Estado en una amplia gama de fun-
ciones, en competencia con otros actores
sociales como las instituciones religiosas o las
pia, previa y diferente del mundo escolar.
Un ejemplo puede ayudarnos a ilustrar esta
idea. La pedagoga mexicana Elsie Rockwell
propiciamos para los otros. Cuando habla- organizaciones familiares. Se trataba de crear estudió la construcción de cerramientos,
mos de relación propia, pensamos que antes una nueva red institucional local −ya no inter- verjas y llaves en las escuelas rurales de su
que docentes somos ciudadanos que nos nacional como la Iglesia−, que ordenara y país. En esa historia, ella encuentra muchos
vinculamos a una sociedad y nos insertamos regulara los intercambios entre las personas conflictos entre las comunidades (sobre
en ella poniéndonos en diálogo con sus ten- en una forma nueva, con nuevos "apósto- todo las comunidades indígenas) y el
dencias, sus problemas, sus urgencias, sus les". Se institucionalizó, así, una de las piezas Estado nacional mexicano, por definir de
dilemas. (Claro está, cuando decimos diálogo clave de la "maquinaria escolar", la forma- quién era la escuela, quién podía usar ese
no nos referimos a obedecer un mandato ción de docentes, bajo el imperio del control territorio comunal, y cómo debían incluirse
inapelable, sino a una práctica que involucra político del Estado y el control científico de la las familias y las culturas locales. A veces,
la crítica, el aporte propio, el compromiso, las pedagogía (Diker y Terigi, 1997). esa pelea llegaba a los tribunales, donde se
múltiples perspectivas, la ética, etc.) Pero ¿Es este el único modo de vinculación que discutía quién tenía derecho a poseer la lla-
también es necesario subrayar que a partir de podemos pensar con la cultura y con un ve de la escuela y quién y cuándo podía dis-
la propia relación habilitamos, facilitamos, orden social y cultural más amplio? ¿Cómo poner del uso del espacio común. En este
abrimos, acompañamos una relación de los nos posicionamos los adultos, los educado- caso, la escuela era "del Estado", enten-
otros −fundamentalmente nuestros alumnos res, facilitadores de esa interrelación con la diendo a este como un aparato exterior,
y alumnas− con una cultura y una sociedad cultura y con el mundo? ¿Qué claves para ajeno, superior, que se declaraba propieta-
en las que viven y que les pertenecen. Sobre entender el mundo estamos enseñando? rio exclusivo de la razón, la fuerza y los inte-
esas bases, asentamos nuestro trabajo a par- ¿Qué saberes sobre el presente y perspectivas reses de la comunidad, y cuyo máximo
tir de un sentido que se nutre permanente- de futuro les estamos abriendo a nuestros representante era la escuela. Esa forma de
mente, que genera crecimiento para nos- alumnos? Los educadores de las escuelas escuela era entendida como una especie de
otros como docentes y también a nuestro normales, a fines del siglo XIX y comienzos injerto que venía a "civilizar" esas poblacio-
alrededor. del XX, pensaron que la escuela debía civilizar nes, a incorporarlas a la sociedad nacional
Ahora bien, esta vinculación y sus efectos, el mundo, formar sujetos nuevos −"ciudada- con la promesa del bienestar, a costa de
¿son siempre tan cristalinos? Podemos decir nos letrados"− desconociendo aquello que expropiar sus territorios, sus saberes y su
que la relación pedagógica es una relación los individuos traían como experiencia pro- participación activa.
asimétrica −y es necesario que así sea−, por-

Archivo General de la Nación


que ambos miembros de la díada no están en
igual relación con el saber, las normas, las res-
ponsabilidades, los frutos del trabajo, etc.
Además, el trabajo de la enseñanza supone
una construcción de formas de autoridad: el
currículum constituye una autoridad cultural;
el Estado y las instituciones donde desarrolla-
mos nuestro trabajo establecen formas de
autoridad; el conocimiento científico se cons-
tituye en una autoridad; un docente esfor-
zándose por desarrollar puentes que no sólo
son con su saber específico sino también con
la sociedad en la que vivimos y en la que que-
remos vivir, construye una autoridad.
¿Qué nos provee la historia sobre este pro-
blema? La expansión de la escolarización de
masas requirió la formación de un cuerpo
profesional que fuera difusor de los nuevos
A fines del siglo XIX y principios del siglo XX, los docentes formaban parte de una escuela
valores del Estado-nación. Esa figura fue concebida como un espacio separado del mundo exterior, y caracterizado por el dominio
absorbiendo también formas de representa- del conocimiento y la racionalidad.
4 EXPLORA PEDAGOGÍA

Colección Museo de la Ciudad

como las disciplinas escolares. Así lo expre-


saba: "La cultura es un resultado natural
del ejercicio de la curiosidad y del interés,
siendo realmente milagroso el que no haya
sucumbido a consecuencia de los refina-
mientos de una mal llamada educación,
que parece empeñada en hacer odioso
todo aprendizaje" (Nelson, 1919: 518).
Hoy, con el declive de las instituciones
fuertes se hace imposible demarcar el aden-
tro y el afuera de forma taxativa. No se trata
de incorporar todo lo que sucede, o renun-
ciar a que la escuela sea un lugar de media-
ción y de transformación de los saberes en
función de lógicas de más largo plazo que
no se agotan ni se definen sólo desde lo
inmediato. Pensar el presente no significa ni
más ni menos que eso: pensar, reflexionar,
decidir cómo los docentes nos ubicamos
frente a él y qué selección hacemos de ese
presente para transmitir a nuestros alum-
nos. Educamos para que ellos aprendan a
vivir en el mundo, para que tengan más
Durante mucho tiempo, el docente fue visto como una figura de autoridad a la que distin- herramientas que los ayuden a ser felices, y
guía un manejo privilegiado del saber letrado. para que puedan hacer algo mejor con lo
que reciben de nosotros. Ahí es donde
Paradójicamente, la escuela concebida de diarios, la democracia−, y sobre los saberes cobran sentido las preguntas sobre qué
esa forma también llevó adelante una uto- y sujetos que lo anunciaban, fue un ele- saberes acerca del mundo estamos transmi-
pía transformadora que nos legó muchas mento que perduró mucho más largamente tiendo, con qué actitud nos posicionamos
cosas, algunas muy democráticas y otras de lo que persistieron los ecos de Sarmiento ante ese mundo, y cuáles de sus rasgos
con consecuencias menos alentadoras. y sus discípulos (Dussel y Southwell, 2006). ofrecen potencialidades nuevas.
Dado que se pensaba a la escuela como Esto, por supuesto, generó una forma de ¿Qué posibilidades tiene hoy la escuela de
una institución renovadora y transformado- autoridad construida en torno a una perso- dar claves para interpretar este presente y
ra de la sociedad, no debe sorprendernos na que encarnaba una figura fuerte, de de no transmitir imágenes de desencanto?
que los límites entre el afuera y el adentro mando. La docencia se pensó como un tra- ¿Qué opciones habilita, por la vía del acceso
estuvieran allí rígidamente marcados, y que bajo individual, personal, en cuya definición a la cultura, para construir la imagen de un
el adentro se percibiera como superior al los elementos del carácter y la personalidad mundo distinto a como ha sido en el pasa-
afuera. Por esta razón, surge la idea de un eran muy influyentes. El docente se definía do, y al que vale la pena explorar, con sus
espacio que era propiedad de determinada por un dominio del saber letrado, lo que le misterios, sus deudas y sus rasgos promiso-
manifestación cultural −la cultura letrada otorgaba una autoridad legítima e inapela- rios? ¿Cómo los adultos que trabajamos en
de cuño europeo− y el injerto en una comu- ble para ponerse frente al aula y ser digno la escuela nos podemos constituir en "cru-
nidad que debía "abandonar su naturale- de imitar. Además, esta autoridad gozaba zadores de fronteras" entre épocas, tradu-
za" para educarse. Trasponer la puerta de de gran prestigio en la sociedad, y sobre esa ciendo, articulando y ayudando a construir
la escuela era entrar en "otro mundo", un base se consolidaba una sólida alianza con referencias nuevas? Si retomamos las cues-
mundo donde el conocimiento y la raciona- las familias en pos de la educación de las tiones de la asimetría y la autoridad, proba-
lidad eran la moneda corriente. El afuera nuevas generaciones. También hubo otros blemente uno de los rasgos más producti-
sobre el que se recortó la escuela argentina posicionamientos en la historia de nuestras vos para refundar esas ideas sea un diálogo
fue planteado como una fuente de conta- instituciones. Ernesto Nelson −inspector de −con más curiosidad que sospecha− con la
minación, una amenaza o un problema. Es enseñanza media en las primeras dos déca- cultura contemporánea. Podemos autorizar-
cierto que había un mundo mejor al que se das del siglo XX− fue un impulsor de las nos como intérpretes, como puentes que
miraba, y era el de una Europa idealizada e ideas de John Dewey, a las que vinculó al dibujen otros cruces entre las generaciones;
inexistente, el de las letras, y a veces el de currículum de la escuela media: introdujo el la escuela sigue siendo, en efecto, un puen-
las ciencias. La sospecha sobre lo contem- fútbol, el cine, la prensa y los viajes de estu- te valiosísimo para configurar este y otros
poráneo −el tango, el fútbol, el cine, los dio como formas educativas tan valiosas mundos posibles.
DOCENTES: LA TAREA DE CRUZAR FRONTERAS Y TENDER PUENTES 5

EL ESCENARIO EDUCATIVO PARA LAS NUEVAS GENERACIONES

¿Qué transmitirles a las nuevas gene- guen. ¿De qué modo? El laicismo que permitan construir la humanidad
raciones? ¿Cómo pensar una transmi- francés entendía que enseñarles a en el hombre, que inscriban a los
sión emancipadora y no reproducto- pensar por sí mismos era una de niños en un proceso de universaliza-
ra? En las sociedades teocráticas, la esas formas. Históricamente, se bus- ción, que los religuen. Esto implica
transmisión siempre significaba repro- caba escapar de las influencias del trasmitirles los sentidos de los saberes,
ducción; mientras que en las socieda- pensamiento religioso; hoy, pode- darles los medios para perforar el mis-
des democráticas, la transmisión mos pensar en otros modos de suje- terio, medios que les permitan encon-
implica subversión, creación de un ción que están en expansión y de los trar respuestas parciales a cuestiones
presente y futuro diferentes. En la que es necesario enseñarles a los antropológicas. ¿Cómo? Hablándole
medida en que pensemos la educa- niños a desligarse: las influencias de al alumno de sus preguntas antropo-
ción como el surgimiento de una los medios de comunicación, los lógicas: su lugar en el mundo, su rela-
libertad y no como inculcación, educa- comerciales, las marcas. La educa- ción con los demás, consigo mismo,
ción y democracia estarán vinculadas. ción tiene el rol de promover la sus pulsiones. La clave −y la dificultad−
La escuela debe transmitir a los liberación de las distintas formas de reside en transformar saberes fosiliza-
niños, entonces, saberes que los sujeción del sujeto, de permitirle dos en saberes vivientes, para que el
liguen, los desliguen y los religuen. En interrogarse y emanciparse. La alumno se los apropie. Esto permite
el primer sentido, es necesario inte- emancipación pone en discusión el religar a todos los alumnos, en la reu-
grar a los niños a nuestro mundo en estatuto del discurso. En este senti- nión de lo íntimo y lo universal, a tra-
común, transmitirles saberes que les do, la transmisión escolar puede vés de la cultura. Ella es una forma de
permitan integrarse a la sociedad permitir al alumno implicarse −arti- simbolizar lo interior, donde se conju-
(como la lengua, pautas de comporta- cular conocimientos nuevos y ya gan el amor y la libertad.
miento, formación para los oficios), estabilizados, ser activos− y, al mis-
saberes que, en definitiva, los liguen. mo tiempo, desprenderse, separar-
En segundo lugar, tenemos que se, poder tener una mirada crítica. Philippe Meirieu:
transmitirles saberes que les habili- En un tercer momento, inescindible conferencia en "La Mutua",
ten modos de escapar a toda forma de los otros dos, los docentes deben Ciudad de Buenos Aires,
de influencia, saberes que los desli- transmitirles a los alumnos saberes 26 de junio de 2006.

MÉTODO Y VOCACIÓN. PENSAR PEDAGOGÍAS PARA ESTE TIEMPO

E n los orígenes de la organización de una


carrera profesional para el magisterio,
se buscó no dejar librada su práctica a la
117). Es así como el Estado dio un lugar cen-
tral a una serie de políticas para sostener la
expansión educacional: fundación de escue-
todavía precaria acción estatal en ese territo-
rio de la enseñanza, alcanzó entonces su
cauce privilegiado: la prescripción metodoló-
buena voluntad de los individuos que se las, prescripción de los métodos y enseñan- gica. Si se lograba establecer y difundir con
desempeñaran en ese papel, y se apuntó a zas considerados válidos, políticas de lectura claridad y precisión cuáles eran las metodo-
un riguroso proceso de formación para cir- y del texto escolar, entrega de títulos y certi- logías adecuadas, el progreso escolar parecía
cunscribir su tarea a condiciones precisas. En ficaciones, control de los modos correctos estar garantizado. Esto, además, se unía a la
ese marco, los futuros profesores aprende- de ser alumno y de ser docente, entre otras. preocupación ligada a que ningún aspecto
rían a ocupar "el lugar del que sabe, del que En ese despliegue de un gobierno escolar, del ejercicio docente quedara fuera de con-
vigila, del que es capaz de contribuir a la docentes, directivos e inspectores fueron trol, para alcanzar una homogeneización
producción de saberes en la institución esco- componentes clave para la mediación entre cultural y moral puesta en manos de esos
lar de una manera correcta". Allí es donde la micropolítica escolar y la política más funcionarios civilizadores llamados "maes-
surge la profesionalización de la docencia: la amplia, en tanto ciudadanos de un momen- tros". Decía Leopoldo Lugones, escritor
escuela deja de ser un asunto estrictamente to preciso. argentino que además fue inspector de
eclesiástico para participar de manera direc- El problema del grado de preparación de Enseñanza Secundaria y Normal, en 1910:
ta del orden público (Narodowski, 1994: los docentes, de su situación laboral y de la "[la docencia] no es una profesión liberal [...]
6 EXPLORA PEDAGOGÍA

[sino] una carrera esencialmente burocrática dos, cuaderno guía para la maestra novel que, con relación entre decisiones metodológicas y
cuya demanda depende de las necesidades sólo consultarlo, tenía la clave para su trabajo dia- decisiones políticas e ideológicas? Aquel
del Estado". rio [...]. Tenemos uno a la vista del año 1894 que modo prescriptivo de operar sobre la reali-
Esta forma de construir una relación con la contiene desde los consejos de preparación del dad educativa sintetiza el pensamiento polí-
prescripción precisa del método a seguir maestro para su trabajo, determinación precisa de tico y social sobre el cual se basó la escuela
también establece una relación con la autori- la idea o fin de la lección, resumen de un bosquejo, moderna. Incluía una mirada particular de
dad, con la autonomía profesional ante la principios, preparación de lecciones, clase de pre- las disciplinas escolares que mostraba, entre
serie de aspectos que deben ser considera- guntas, actitud, lenguaje, énfasis, inflexión de la otros aspectos, un fuerte relieve moral para
dos al buscar las mejores condiciones para voz y expresión de la cara de la maestra, etcétera la enseñanza, una concepción del conoci-
enseñar, para incluir a los otros en las mane- (Centro Mary O. Graham, 1938: 26). miento que lo entendía de modo acabado y
ras más convenientes de participar del cono- permanente en el tiempo, y una pretensión
cimiento. Sin embargo, no hay nada de Probablemente, este nivel de prescripción de atrapar y "formatear" las diversas com-
natural, esencial o "escrito en piedra" acerca nos resultará desmesurado, pero nos pone plejidades de las realidades educativas.
de los mejores modos de enseñar. El método frente a la ocasión de pensar qué cons- La producción pedagógica, psicológica y
no es sino un artificio que establece una trucción de sujeto con conocimientos, habi- sociológica del siglo XX contribuyó a desarro-
determinada relación del individuo −tanto el lidades y toma de decisiones involucra un llar nuevos modelos e intervenciones sobre
alumno como el docente− con el conoci- planteo así. ¿Qué se añora cuando se tiene esos modelos. De este modo, la formación
miento que aprende o que prepara para nostalgia de un tiempo donde la docencia necesaria para el docente se concentró en las
acercar a otros. contaba con marcos de referencia más cla- particularidades metodológicas de su rol y la
Pensemos en el siguiente ejemplo: ros? ¿Qué la caracterizaba además de la comunicación con los alumnos, pero ya no
Después de esa práctica y a medida que se llega- claridad? ¿Es ese el tipo de formación profe- eran igualmente necesarios los conocimientos
ba a cada conclusión se trasladaba al papel, en el sional que puede ubicarnos en las mejores generales que orientaban las prácticas peda-
cuaderno de métodos, las síntesis del procedi- condiciones para nuestro ejercicio crítico gógicas. Guías e instrucciones para los
miento empleado con los sucesivos pasos a seguir. frente a la serie de aspectos que deben ser docentes y minuciosísimas planificaciones
En forma tan segura, al final de la carrera, cada contemplados para diseñar las mejores expe- empezaron a conformar una "industria" de
alumna-maestra poseía su Cuaderno de Métodos riencias de aprendizaje posibles para nues- la educación que ha sido perdurable. Más
que registraba los empleados para la enseñanza de tros alumnos? ¿No se trata la docencia de un cerca de nuestros días, procesos autoritarios
cada materia, lacónica pero claramente consigna- trabajo irreductible a técnicas? ¿Cuál es la como el de la última dictadura militar se asen-
taron sobre aquellos rasgos preexistentes, a los
Museo de las escuelas/UNLu

que les sumaron el componente del Estado


constructor de terror. El control ideológico y la
utilización de nuevas tecnologías como garan-
tía de modernización se articularon con una
noción de "profesionalidad" entendida como
eficiencia y vocación, con creencias ético-reli-
giosas y con la adhesión al régimen.
Cabe recordar que la escisión entre pro-
ducción de conocimientos y docencia ha
sido uno de los más tenaces esfuerzos de las
didácticas positivistas, que reapareció en las
concepciones psicológicas restringidas que
estaban en la base del desarrollismo tecno-
crático y autoritario (Southwell, 2003). Esto
representó una continuidad con la concep-
ción restringida de la ciencia social propia de
los regímenes autoritarios. En suma, la tarea
docente era pensada en un espacio de
determinación que reunía las ideas de efi-
ciencia y orden, a través de una propuesta
tecnocrático-moralizadora, sobre la base de
algunos valores que se pretendían "salvado-
res" (Kaufmann y Doval, 1999).
Hacia mediados del siglo XX, se impuso en la escuela una visión técnico-pedagógica que,
frente a las perspectivas humanistas y espiritualistas, vinculó la docencia a la planificación y En la actualidad, la cuestión de la forma-
a un conjunto de aprendizajes instrumentales y técnicas. ción técnica e instrumental ha tomado otros
DOCENTES: LA TAREA DE CRUZAR FRONTERAS Y TENDER PUENTES 7

Dirección General de Cultura y Educación. Gobierno de la Provincia de Buenos Aires


La idea de vocación representa, antes que un atributo individual, los efectos de la dinámica social y cultural de un momento histórico
determinado sobre la tarea docente.

caminos. Un aspecto que suele discutirse querida, cultivada y muy extendida en el culares, entiende el trabajo de socialización
con frecuencia en cuanto a la formación de campo de la educación. Nos interesa poner- como una vocación en tanto se encuentra
los docentes es la relación con la práctica. la aquí en un marco más amplio que deje de fundado en valores, y cree que ese trabajo
Hay una creencia instalada en el sentido verla como un atributo individual −frecuen- inculca normas que configuran al individuo
común, según la cual la práctica −en sus temente como algo con lo que se nace o no− y simultáneamente lo vuelven autónomo y
diferentes denominaciones− es el momento para mirarla en términos más colectivos, libre (Dubet, 2006: 22).
en que el nuevo docente entra en contacto como efecto de una dinámica social. La perdurable noción de vocación, puede
con "la realidad" y esta es la que −en forma En su libro El declive de la institución, el ser puesta en diálogo, a su vez, con nuevos
de choque− establece las adecuaciones que sociólogo François Dubet enmarca el trabajo sentidos, identidades y problemas. Mirar la
esa persona deberá operar sobre su forma- docente −entre otros trabajos que caracteri- tarea de enseñar nutrida de responsabilida-
ción. Esta creencia supone, por un lado, una za como "trabajo sobre los otros"− en lo des, saberes diversos y sensibilidad por el
mirada que refuerza la dicotomía entre la que denomina un "programa institucional", mundo y por los otros, nos pone algunos
aproximación conceptual y la vinculación expresión que alude al proyecto de la pasos más adelante de una idea de vocación
más empírica. Pero además, lo que allí surge modernidad: entendida como don personal que remite a
es la idea de que la práctica dictamina, y que El programa institucional se funda sobre valores, restaurar algunos de los modelos más cono-
por lo tanto esa es la experiencia ante la cual principios, dogmas, mitos, creencias laicas o reli- cidos. Los docentes, "en tanto herederos de
deben subordinarse los demás saberes. Esta giosas pero siempre sagradas, siempre situadas una historia, no son custodios de un templo
manera de entender la interrelación entre más allá de la evidencia de la tradición o de un institucional" (Dubet, 2006: 139). Nuestros
diferentes aprendizajes puede contribuir a mero principio de utilidad social [...] invoca princi- modos de operar con la realidad son el
prácticas conservadoras al entender que sólo pios o valores que no se presentan como simples resultado de un proceso de producción cul-
debe operarse con "la realidad", entendida reflejos de la comunidad y sus costumbres, se tural y social que se tramita entre las urgen-
esta como "lo que hay", desestimulando así construye sobre un principio universal y más o cias, prioridades, dilemas y certidumbres de
perspectivas más transformadoras. menos "fuera del mundo" (Dubet, 2006: 35). cada momento histórico. Ese diálogo sensi-
El modo de posicionar al docente frente a ble y autónomo será un terreno productivo
la tarea que venimos describiendo fue carac- Este programa institucional considera el para profesoras y profesores nutridos de
terizada −y continúa siendo caracterizada− trabajo sobre el otro como una mediación saberes sólidos y −seguramente− heterogé-
por la idea de vocación. Esta es una idea entre valores universales e individuos parti- neos. El entramado institucional que pro-
8 EXPLORA PEDAGOGÍA

porcionaba un suelo o punto de partida ve de las instituciones modernas, no es posi- con mayor incertidumbre pero que pueden
común se debilita, pero no implica que por ble renunciar a la tarea de socialización, una ser, al mismo tiempo, menos opresivos y
ello se produzca un "vacío". Aun en el decli- socialización en suelos menos conocidos y más distributivos.

LA POLÍTICA Y LA SOCIEDAD CONSTITUYENDO LA TAREA

E n el mundo y el país en los que vivimos,


donde los saberes circulan y la comuni-
cación crea nuevos vínculos, nuestras vidas
tían en robustecer técnica y metodológica-
mente su trabajo, en procura de alcanzar
de modo eficiente la finalidad establecida
Sabemos que nuestras instituciones, las
familias, nuestros alumnos y las comunida-
des se encuentran en procesos de fragilidad
están atravesadas por la desigualdad y la desde la prescripción estatal. social. Hoy, las experiencias de ser padre o
injusticia; muchas deudas de larga data Cabe preguntarnos aquí cómo tiene que madre, docente, joven o niña están atrave-
siguen presentes y otras nuevas muestran el ser, cómo tiene que lucir un individuo para sadas por transformaciones muy cruciales,
límite de algunos sueños pasados. En ese que la escuela lo adopte como sujeto digno de quiebre y fragmentación. Pero también
territorio, volvemos a preguntarnos por el rol del derecho a la educación. ¿Puede cons- están bajo condiciones que no son definiti-
político −transformador− de las instituciones truirse una relación de enseñanza entre vas ni inexorables, que tienen aspectos ambi-
que tienen como objeto lo común, lo colecti- docente y alumno que presente una asime- guos y abiertos, continuamente desafiados.
vo, y entre las cuales ubicamos a la escuela. tría respecto de los saberes y responsabili- Todos los días, muchas iniciativas y acciones
Para nosotros, resulta hoy obvio que la edu- dades pero que sea de semejanza en los entre familias e instituciones, grupos comuni-
cación y la enseñanza tienen una dimensión aspectos humanos y ciudadanos? Creemos tarios y docentes, distintas instituciones en
política que aloja derechos, que produce que sí, y mucho de esto forma parte de los asociación, familias y organizaciones sociales,
sujetos, que favorece u obstaculiza la partici- desvelos de quienes hacen cotidianamente muestran diversas posibilidades de interven-
pación, la democratización y la transforma- nuestras escuelas. Eso implica sostener una ción y cambio, sin rendirse frente a una pers-
ción de las instituciones y la sociedad. Pero mirada cautelosa para que la asimetría de la pectiva de crisis como pérdida total o devas-
¿cómo se ha llevado la escuela con esta posición de quien orienta con respecto a tación que nos dejaría sin posibilidad de
idea?; ¿cuánto de político reconoce en sus quien es orientado no se deslice hacia una
actos?; ¿qué ha incluido la formación política desigualdad irremediable que elimine toda

Biblioteca Nacional de Maestros


de la escuela?; ¿qué cambia al reintroducir el dignidad. ¿Puede haber cuidado sin digni-
carácter político de la cotidianidad escolar? dad del que cuidamos? Marcar este terreno
Nos interesa aquí explorar qué vinculación se implica dar lugar a la pregunta sobre el víncu-
establece entre política y trabajo docente, lo entre escuela, ética y justicia, recordando
cómo eso incluye perspectivas acerca de que nadie −ni jóvenes ni adultos− puede
quiénes, cómo y para quiénes enseñar en un dentro de la escuela poseer menos dere-
escenario social y público más amplio. chos que los que tiene fuera de ella
Pensarse a sí misma como ámbito político (Southwell, 2004). Este y otros problemas
no ha sido fácil para la escuela; sólo a partir éticos y políticos se ponen en juego cuando
de permanentes análisis y de la confluencia buscamos ayudar a los estudiantes a pensar
de perspectivas se ha podido poner de su futuro de manera más esperanzada e inte-
manifiesto esa cualidad. Por el contrario, el grada a la sociedad. ¿Qué pasa si considera-
sistema educacional consolidado desde mos, como lo hace la política democrática,
fines del siglo XIX había incluido una des- que en las sociedades actuales hay pluralidad
cripción sobre la escuela y el trabajo docen- de actores, cuyos intereses no siempre coinci-
te que los ligaba a la neutralidad y la objeti- den? ¿Cuáles pueden ser las implicancias de
vidad como dos de sus pilares centrales un diálogo pleno entre la comunidad y la ins-
(Dussel, 1997: 12). La docencia se había titución, un diálogo de mutuo reconocimien-
consagrado como una actividad intelectual to y participación ampliada? ¿Qué pasa, ade-
a la vez que subordinada: las finalidades más, cuando las sociedades son desiguales y
políticas de su trabajo se resolvían fuera de la capacidad de hacerse escuchar y de impo- La maestra normal, de Manuel Gálvez, repre-
senta una época en la que los aspirantes al
su alcance; su fortaleza, y la autonomía ner posiciones no es equiparable para ricos y magisterio estaban motivados por la perspecti-
profesional que le estaba destinada, consis- pobres? ¿Cómo se construye allí lo común? va de una movilidad social ascendente.
DOCENTES: LA TAREA DE CRUZAR FRONTERAS Y TENDER PUENTES 9

Ministerio de Educación de la Ciudad de Buenos Aires


modificar nada. Contrariamente, esas ini-
ciativas y situaciones concretas muestran
las posibilidades que brinda el hecho de
buscar alianzas para organizar a madres y
padres, pensar en organizaciones estatales
y comunitarias que puedan sumarse a
mejorar el bienestar de la población y crear,
en la práctica, otros lazos compartidos, de
sostén y dependencia mutua. Quizás en esa
cadena de dependencias mutuas pueda
articularse una relación más igualitaria,
donde los unos necesiten de los otros, de
manera que en esa mutua protección pue-
da funcionar una verdadera sociedad
humana.
Dialogar acerca del lugar de la justicia y
su vínculo con la transmisión de la cultura
supone también estar incidiendo en la
posición que el docente asume, dándole
Las iniciativas y prácticas que se generan entre padres, instituciones, grupos comunitarios y
concreción cotidiana a ese terreno dilemá- docentes, dan cuenta de las posibilidades de crear lazos sociales transformadores, aun en un
tico al que solemos aludir como "lo políti- contexto de crisis.
co". La escuela y el trabajo docente han
estado y están siempre tensionados, preo- de familias que lograban el acceso al nivel soportes materiales, restricción de cargos,
cupados por la producción y la reproduc- medio de enseñanza. Una de las motivacio- deterioro salarial); por el otro, crecientemente
ción de desigualdad e injusticia. La no nes fue la perspectiva de una movilidad buscado ante la vulnerabilidad producida por
naturalización de la injusticia es algo que social ascendente, que impregnaba el ima- el desmoronamiento de la condición salarial
les toca a las instituciones políticas, entre ginario de progreso que se sostuvo durante y, fundamentalmente, de la protección social
las que se incluye la escuela. Esto implica buena parte del siglo XX. Los docentes fue- que ella incluía. Se configura allí una resignifi-
no perder de vista que la escuela es un ron el ejemplo del discurso que transmitían: cación de las motivaciones "vocacionales"
ámbito específico y particular donde se en ellos se encarnaban la posibilidad del para la tarea docente (apostolado, voluntaris-
produce la socialización de las nuevas ascenso social y la posesión de un capital mo, optimismo pedagógico) y su adecuación
generaciones en la cultura letrada y donde cultural por medio de la educación. Con el a los parámetros más generales que caracte-
se construye una relación con la cultura y la tiempo, otro de los rasgos que distinguieron rizan la opción por cualquier empleo. Por
política no solamente a través de los espa- al trabajo docente fue la estabilidad en el ello, para las perspectivas de los aspirantes a
cios curriculares destinados a ello, sino cargo, en el contexto de un país con pleno la docencia, el trabajo ocupa un nuevo lugar
además mediante el modo en que la justi- empleo. Pero ni el pleno empleo ni la expan- donde se reacomodan lo vocacional, lo
cia y la ética circulan por los pasillos y por sión de los cargos docentes continúan redentor, lo laboral, la enseñanza (Birgin,
los patios, sin perder de vista las palabras vigentes. Alejandra Birgin (2000) relata 1999). La preocupación marcada por la bús-
que allí se ponen en juego. Como sabe- cómo al finalizar el siglo XX el panorama queda de un empleo genera conflictos y
mos, más allá de las prescripciones existen- mostraba significativas diferencias: se redujo rechazos en el interior de la institución forma-
tes, la manera en que las formas de la justi- la heterogeneidad de la población que acce- dora, que muchas veces sigue a la espera de
cia y la protección de niños y adultos día a las carreras docentes, con un peso cre- un perfil de estudiante para la docencia que
entran en juego en la vida escolar encierra ciente de los sectores más empobrecidos. ya no existe. Frecuentemente, la valoración
una serie de cuestiones que no tienen res- Confluyendo con la característica anterior, de lo que sucede en la escuela y del rol de
puesta prefijada, sino que cobran sentido las dinámicas generales del mercado de tra- quienes trabajan en ella se hace desde un
en el devenir de la práctica en un terreno bajo regulan los movimientos en el interior peculiar encapsulamiento, y pese a que se
de decisión que no está previamente carto- del mercado de trabajo docente; el incremen- reconoce que fuera de las escuelas las diná-
grafiado. to de la precariedad del empleo ha generado micas social, familiar, de género y laboral se
motivaciones en relación con el ingreso en la han transformado notoriamente, se sigue
EL EMPLEO DOCENTE carrera docente, así como el retorno de quie- proponiendo un "deber ser" de la escuela y
nes no ejercían (Birgin, 1999). De este modo, de los docentes, con la idea de reponer un
Históricamente, los aspirantes al magisterio el empleo docente se instaló en una parado- modelo que se estableció y fue eficaz tiempo
provenían de sectores sociales heterogéneos, ja: por un lado, amenazado por las medidas atrás. Allí, más que un diálogo entre épocas y
en general de las primeras generaciones de ajuste estructural (degradación de los prácticas, a la descripción de transformacio-
10 EXPLORA PEDAGOGÍA

Biblioteca Nacional de Maestros/Programa Historia de la Educación Argentina


nes de época parece sobreimprimirse un
imperativo que actúa a contrapelo dentro de
las instituciones educativas.
En esta situación, las instituciones forma-
doras se lamentan porque los estudiantes no
poseen una "vocación" (a la que consideran
indispensable para el ejercicio del rol docen-
te), o porque no cuentan con las disposicio-
nes intelectuales y físicas que se consideran
inherentes a la función. Lo que se busca
identificar es si el estudiante presenta esas
disposiciones o no, pero no si la institución
las forma (Birgin, 2000). En muchos casos,
en estas instituciones formadoras se constru-
ye un discurso alrededor del supuesto déficit
que traen los estudiantes, tanto por sus ren-
dimientos académicos como porque se ale-
jan del patrón cultural y social que se consi-
dera valioso y pertinente para la docencia.
Esto puede devenir discriminación social por
lo menos en dos sentidos: por los procesos
de estigmatización que se generan y porque
se van construyendo nuevos circuitos en fun-
En nuestro país, la docencia fue representada desde su origen por imágenes ligadas a la
ción del perfil de docente que se visualiza en abnegación y el sacrificio, como parte de una concepción que veía al maestro como un servi-
los alumnos. A su vez, esa perspectiva se dor público.
absolutiza en una epistemología academicis-
ta que sólo reconoce como conocimientos a 2002). Se refuerzan, así, la reproducción y tituciones en las que nos formamos y trabaja-
aquellos legitimados en el medio escolar y la fragmentación sociales, y cae uno de los mos. ¿En qué medida estas tradiciones son
descarta otras formas de conocer, otras baluartes de la escuela pública, que era el buenos puntos de partida y en qué medida
oportunidades, otros saberes mediante los docente homogéneo e "intercambiable"; requieren ser transformadas para desarrollar
cuales los sujetos actúan en la sociedad. Es se pierde a su vez la dimensión de una pedagogías actuales, que nos permitan reto-
decir, a partir de rígidas clasificaciones que escuela que apueste a la construcción de mar viejas deudas con la igualdad, con la
solamente dan lugar al conocimiento que una esfera pública incluyente y plural democracia, con una construcción profesio-
proviene de la cultura letrada legitimada en (Tiramonti, 1997). Ahora bien, esto no sólo nal menos subalterna, con una posición polí-
el ámbito escolar, los jóvenes son tipificados sucede a partir de la inequidad distributiva tica enriquecida? Por el carácter paradojal,
(Birgin, 2000). sino también como consecuencia de patro- problemático y sin equilibrio inmanente de
Esta situación acarrea otro problema, que nes institucionalizados de interpretación y esta situación, se trata de plantear procesos
consiste en que estos jóvenes se forman evaluación que constituyen a alguien como de interrogación política más que de proble-
visualizando sólo determinados circuitos del no merecedor de estima, respeto o legitimi- mas lógicamente resolubles o caminos técni-
sistema educativo como los ámbitos de tra- dad para ocupar determinada posición camente prefigurados.
bajo deseables y posibles para ellos. En (Nancy Fraser, citada por Birgin, 2002).
algunos casos, esto es vivenciado como un Probablemente sea una buena ocasión para
modo de "devolver" a la propia localidad autorizar la palabra docente, la palabra de UNA RELACIÓN CON EL MUNDO
las oportunidades recibidas, pugnando por mujeres y hombres que busquen vincularse DEL TRABAJO
que las fronteras sociales no se configuren de modo nuevo, adulto y profundo a los
en fronteras educativas (Redondo y Thisted, saberes, a la cultura que se despliega fuera de El trabajo docente se constituyó a partir de
1999). En otros casos, es el único ámbito la escuela, a la sociedad de la que son parte, la conformación de una fuerza de trabajo
donde los estudiantes sienten que la distan- mas allá de la sospecha, de rígidos preceptos económica y disciplinada sobre la base de la
cia cultural que está en la base del vínculo morales y de relaciones autoritarias con el subordinación de género en la sociedad del
pedagógico tradicional se mantiene y les conocimiento. ¿Cuánto nos configuran las siglo XIX y comienzos del XX. Feminización
hace posible alcanzar una posición de ense- tradiciones? ¿Cuántos rasgos invisibles hay de la enseñanza, abnegación, sacrificio,
ñantes. Muchos de ellos no se consideran de ellas en nuestras perspectivas y decisiones entrega, neutralidad, trabajadores/as bara-
capacitados para enfrentar otros entornos cotidianas? Tal vez sea una suerte de "incons- tos/as, fueron rasgos propios del origen de la
culturales que anticipan hostiles (Birgin, ciente colectivo" que se encuentra en las ins- docencia. Cien años después, las característi-
DOCENTES: LA TAREA DE CRUZAR FRONTERAS Y TENDER PUENTES 11

cas del modelo docente que fundó el sistema conservado una significativa presencia en en función de incrementar su competencia
educativo argentino presentaban importan- amplios sectores. intelectual y social y, sobre todo, su respon-
tes cambios. La pretensión de neutralidad Las referencias al profesional crítico, al pro- sabilidad por la obtención de resultados
resultó quebrada. Los docentes −hasta ese fesional eficiente, al profesional autónomo, favorables (Popkewitz y Pereyra, 1994). El
entonces considerados misioneros ascéticos al profesional democrático, establecen énfa- uso del concepto de accountability, de difícil
del saber− realizaron durante la segunda sis particulares en los múltiples significados. traducción al castellano, da cuenta de este
mitad del siglo XX un proceso de asunción Si bien la profesionalización de la tarea cambio que, por una parte, define la necesi-
de su identidad como trabajadores de la edu- docente fue uno de los principios más dad de evaluar la tarea de los profesores en
cación. salientes de la política para el sector en el términos de "rendir cuentas" de su trabajo
En este contexto, el concepto de profesio- marco de las reformas educativas de la déca- (Nóvoa, 2002) y, por otra parte, admite la
nalización docente ha sido objeto de innu- da de 1990, las discusiones acerca del carác- necesidad de regular los procesos de ense-
merables debates teóricos y disputas prácti- ter profesional del trabajo de los docentes ya ñanza y las identidades de los docentes a fin
cas, particularmente en las últimas décadas. estaban presentes en el contexto de la crea- de conseguir una estandarización profesio-
Las reformas educativas latinoamericanas se ción de los sistemas educativos nacionales, nal y una creciente responsabilización indivi-
han incluido en una transformación social y como una de las piezas clave que hicieron dual (Fenwick, 2003; Burchell, 1996).
cultural más amplia. En la mayoría de los posible la educación de masas. La identidad Aunque no en forma precisa ni homogé-
casos nacionales, la temática del desarrollo profesional de los profesores se definió bajo nea, las recomendaciones para el sector pro-
había instalado en las décadas del 50 y el 60 la tutela del Estado nacional, en un proceso venientes de diferentes organismos interna-
una preocupación por la expansión del sis- que supuso la consecuente independencia cionales dan cuenta de estas nuevas formas
tema educativo. Más tarde, la problemática de los intereses propios de las comunidades de entender el trabajo y la identidad de los
de la debilidad de los regímenes políticos locales y la definición de cualidades, virtudes docentes. En algunos casos, relacionan estas
agregó al tema de la expansión el de la for- y capacidades que debían ajustarse al pro- formas con una mejora en la formación ini-
mación y consolidación del sujeto político yecto nacional (Nóvoa, 2002; Lawn, 2001). cial y la capacitación en servicios, con el fin
democrático. En años recientes, las trans- En la década de 1990, la retórica de la pro- de garantizar una educación de calidad que
formaciones culturales caracterizadas por el fesionalización también supuso una serie de responda a las nuevas demandas del actual
advenimiento de la llamada "sociedad del estrategias para racionalizar y normalizar la contexto global. En otros casos, las relacio-
conocimiento" y sus derivaciones en socie- formación y el trabajo de los docentes, aun- nan con la proclama por la dignificación de
dades profundamente desiguales confor- que, a diferencia del anterior escenario, las los docentes ligada a la mejora en las condi-
maron una agenda educativa compleja, con políticas educativas impulsadas no supusie- ciones laborales y a un aumento en sus
yuxtaposición de problemas y temáticas. ron ya un lugar central para los Estados en ingresos. A estas afirmaciones se suman
En Argentina, desde la década de 1950, esta materia. Por el contrario, la tendencia otras que admiten la necesidad de imple-
coincidiendo con los cambios en el modelo fue definir nuevas exigencias para su tarea, mentar mecanismos de evaluación de la
de desarrollo, la consolidación de la socie-

María Gabriela López Trück


dad salarial y la difusión del pensamiento
económico de la CEPAL −que promulgaba
una estrecha relación entre desarrollo edu-
cativo y desarrollo económico−, el Estado
diseñó nuevas políticas tendientes a regular
la tarea de enseñar. En 1958 fue sanciona-
do el Estatuto del Personal Docente
Nacional, a partir del cual se reguló el ingre-
so, ascenso y estabilidad de la carrera
docente, se fijó la necesidad de contar con
cursos de perfeccionamiento y se definieron
criterios para la remuneración salarial. Los
dirigentes sindicales entendieron la sanción
del Estatuto como un avance en materia de
organización gremial por la defensa de los
derechos de los docentes. La competencia
técnica que les era conferida a los profeso-
res implicaba un desempeño laboral preciso
que se oponía a la idea de un apostolado
guiado por la vocación. Sin embargo, esta Desde los inicios mismos del Estado nacional, hubo una clara tendencia a la feminización del
mirada apostólica sobre el propio trabajo ha trabajo docente.
12 EXPLORA PEDAGOGÍA

Dirección General de Cultura y Educación. Gobierno de la Provincia de Buenos Aires


tarea, como una forma de asignación de
incentivos monetarios y de ligar los ingresos
a una no muy especificada productividad.
Frecuentemente, la profesionalidad está
asociada a la defensa de la autonomía, la
responsabilidad por el trabajo y el compro-
miso social con una mejora en la calidad de
la enseñanza. Así, el discurso profesionali-
zante ha buscado prescribir lo que significa
ser un "buen docente" como un "construc-
tor de su capacidad" y poseedor de destre-
zas "no rígidas", o ligadas a la adaptación
flexible, como las de ser administrador y ges-
tor organizacional. Por ello, las políticas
docentes impulsadas desde ese consenso
han encerrado una tendencia para regulari-
zar los procesos de enseñanza y las identida-
des dentro de la preocupación general por
una estandarización profesional y por una
instancia de responsabilización.
Las diferentes interpretaciones dadas al
término "profesionalización" involucran
un proceso de normalización que, a su
vez, liga una variedad de presupuestos En los últimos años, los discursos sobre la profesionalización docente incluyeron la necesidad
acerca de disposiciones que deberían for- de la actualización constante.
mar las competencias de los docentes en
tanto profesionales (Popkewitz, 1995). XX se manifestaron en enunciados destina- Como sabemos, la reformas llevadas a
Bajo una pretendida autonomía de los dos al ejercicio individual, a la actualización cabo durante los años noventa tuvieron
profesores y el aliento a su desarrollo pro- de cada uno de los docentes, al fortaleci- también importantes resistencias. En líneas
fesional, estas nuevas formas de regula- miento de su autonomía y a la gestión del generales puede decirse que se pusieron en
ción ligan la libertad otorgada al colectivo propio riesgo. Algunos investigadores afir- discusión los proyectos de profesionaliza-
docente con su responsabilidad individual man que el individuo está sujeto al riesgo y, ción docente, que suponían una mejora
por la obtención de resultados. Como sos- a la vez, es sujeto de riesgo, con lo cual se salarial condicionada por la reforma del
tienen Popkewitz y Pereyra (1994), estos redefinen subjetividades, se redistribuyen Estatuto y las condiciones de trabajo. Los
usos remiten a discusiones presentes en el costos y responsabilidades, y la persona dirigentes sindicales se opusieron a este
momento de la creación de los sistemas queda "libre" de administrar su riesgo proyecto por entenderlo como un progra-
educativos nacionales, es decir, al carácter (Palamidessi, 2003). ma de ajuste y flexibilización laboral para el
vocacional o apostólico de la tarea de ¿Cómo contrasta esta retórica con el coti- sector. Así, intervinieron en el debate sobre
enseñar y, consecuentemente, a la respon- diano trabajo de enseñar? Frente a las reco- la profesionalización poniendo las condicio-
sabilidad casi mesiánica del docente con la mendaciones y prescripciones a favor de nes laborales en el centro de la discusión.
comunidad. Se conjuga así la ya clásica este tipo de profesionalismo, los docentes En este sentido, muchas expresiones sindi-
interpelación por la vocación con la de la debieron llevar a cabo su tarea en contextos cales no toman distancia de aspectos rele-
profesionalización. cada vez más desfavorables, producto de vantes como la actualización y la capacita-
A partir de la consolidación de sociedades las mismas políticas económicas que postu- ción, pero plantean para éstas un marco de
de mercado, la responsabilidad por la efica- laban su responsabilidad aludiendo a su protección y mejoramiento de condiciones
cia y la eficiencia de la tarea docente −de compromiso en la consecución de una edu- para que el docente pueda participar de
difícil caracterización y cuantificación− ha cación de calidad. La evaluación de los esas instancias en una mejor situación en lo
comenzado a constituir nuevas identidades docentes, los incentivos ligados a los resul- referente a disponibilidad horaria, estado
a las que se les exige capacidad para el tra- tados obtenidos y la diferencia salarial de salud, etcétera.
bajo colectivo pero también compromiso según el desempeño son respuestas indivi- El siglo XXI ha traído otros enfoques sobre
individual por los resultados y la calidad de duales frente a una situación que implica estos problemas. En los últimos años se fue
la educación. En un sentido similar, las polí- aspectos colectivos y condiciones socioeco- construyendo otra perspectiva que busca
ticas más frecuentes destinadas a la profe- nómicas más allá del desempeño de un pro- imbricar la formación con la carrera, esto
sionalización en los últimos años del siglo fesor aislado. es, construir una perspectiva de desarrollo
DOCENTES: LA TAREA DE CRUZAR FRONTERAS Y TENDER PUENTES 13

profesional que comience con la formación "profesional" ello involucra conjuntos parti- profesional es producto de una mejora de la
inicial y continúe con el enriquecimiento de culares de disposiciones sobre cómo se debe formación; el docente profesional no sólo
saberes de la experiencia, buscando situar hablar, pensar, actuar y "ver". De este remite a un conjunto de saberes sino a con-
al docente como voz autorizada a través de modo, la profesionalización constituye una diciones personales. Se trata de un campo
la recreación de la herencia cultural y del interpelación que construye al docente pro- de significados que a la vez funciona como
fortalecimiento de sus vínculos con la políti- fesional no sólo como modelo, sino como mecanismo de regulación. En la medida en
ca y con el mundo contemporáneo. reivindicación de una tradición siempre exis- que la perspectiva de la profesionalización
Las palabras actúan para ligar una varie- tente. Sobre este campo se teje, entonces, se va asociando de manera predominante a
dad de presupuestos sobre lo que es social- una variedad de posiciones que son objeto alguno o algunos de los significados posi-
mente apropiado y sobre cómo un individuo de disputa: el docente profesional es res- bles, las políticas adoptadas en consecuen-
debe comprender su propia competencia ponsable primario de la calidad educativa; el cia establecen una norma, una trayectoria
en acciones futuras. Aunque nunca es un docente profesional requiere de condiciones deseable, una serie de condiciones ideales
asunto monolítico, si uno es considerado de trabajo acordes a su estatus; el docente que caracterizan al docente profesional.

CONCLUSIONES:
LOS EDUCADORES FRENTE A LOS DESAFÍOS DEL MUNDO CONTEMPORÁNEO

¿ Cuáles serán las principales característi-


cas de un docente que actúe como
culturales y políticos. Para esto, la posición
que se construyó para el docente, mano efi-
por eso convalidar la desigualdad y la injus-
ticia. Observar lo que aprenden los chicos y
puente −cruzando fronteras entre genera- caz de esa imposición, tuvo mucho que ver construir pronósticos desde una posición
ciones y entre épocas y saberes−, como con el desarrollo efectivo de esa escuela preocupada por generar igualdad para que
intérprete y traductor entre vivencias y tiem- que se llamaba "común y pública", y que accedan al patrimonio cultural que les per-
pos diferentes? Hemos hecho referencia a entendía lo común y lo público de maneras tenece por derecho, o −a diferencia de
que el trabajo docente, como práctica espe- que hoy se hace necesario problematizar y esto− pensar que hay condiciones de parti-
cializada y referente de transmisión cultural, ampliar. Como hemos afirmado en otro tra- da que los inhabilitan, construye claras
ha sido, y es, objeto de interpelación por su bajo, la igualdad debería empezar a pensar- posiciones determinantes acerca del éxito o
inscripción en escenarios atravesados por se como una igualdad compleja, como una fracaso y, por lo tanto, acerca del futuro
profundas e inéditas transformaciones políti- igualdad que habilita y valora las diferencias que se les augura. Este dilema se hace más
cas, económicas, sociales y culturales. que cada uno porta como ser humano, sin agudo en la medida en que se endurecen
Quisiéramos −para cerrar− puntualizar bre-

Puentes Escolares/Ministerio de Educación de la Ciudad de Buenos Aires


vemente dos aspectos: los dilemas que plan-
tean la fragmentación y la igualación social
al trabajo docente, y la potencialidad de la
tarea de transmisión de los/as profesores/as y
maestros/as en este nuevo tiempo, con las
nuevas generaciones.
La situación caracterizada por acelerados
cambios, precarización y débiles apoyos,
junto a la alta fragmentación y exclusión
social y la configuración de nuevas subjeti-
vidades, pone frecuentemente en cuestión
la potencialidad de la escuela −y del trabajo
docente− como espacio privilegiado de
traspaso del orden cultural. El sistema edu-
cativo argentino abrazó una idea de igual-
dad que generó un gran crecimiento y
movilidad social para amplios sectores; a la
vez, como efecto del mismo proceso, cons-
truyó una concepción de igualdad homo-
geneizante y autoritaria que dejó fuera de
consideración muchos aspectos humanos, Los docentes llevan a cabo su tarea en contextos muchas veces desfavorables.
14 EXPLORA PEDAGOGÍA

las condiciones económicas y sociales de ridad sustentada en la tradición se ha esfu- de cualquiera de esas posiciones, que se la
niños y jóvenes, lo que hace que se consolide mado en el mundo moderno y, por lo tanto, vuelve propia. Esto implica que la tradición
la presunción sobre la imposibilidad de una la tradición no es el fundamento del poder es centralmente una transmisión de con-
buena experiencia educativa. Que haya suje- político, entendido como el juego entre cepciones, de una mirada del mundo que
tos que puedan educarse depende de lo que hombres libres e iguales. En términos de encierra valores y les da sentido a las institu-
hagamos con ellos en la escuela, no sólo de una autoridad pedagógica, estos planteos ciones. Dentro de ellas, modela también
lo que hagan la familia o la sociedad: depen- brindan la posibilidad de preguntarse por la posiciones para los individuos; allí, enton-
de de cómo los recibamos y los alojemos en fuente de su potencia y su significado. ces, la tradición es transmisión para que
una institución que los considere iguales, ¿Sobre qué se sustenta la autoridad de los otros la hagan suya al renovarla.
con iguales derechos a ser educados y a maestros y los profesores cuando la tradi- El mundo en el que vivimos posee difíciles
aprender (Dussel y Southwell, 2004). ción ya no parece ser la fuente de la autori- condiciones de vida pero también promiso-
Además, esta dimensión nos pone frente dad política? ¿De qué forma se puede rios horizontes nuevos, potencialidades y
a la pregunta acerca de los alcances y los hacer conocer el mundo y transmitir la tra- posibilidades que abren otras ventanas
límites del trabajo docente y de la acción dición a los "recién llegados"? ¿Cómo se hacia nuestro presente y nuestro futuro.
escolar. Retomemos algunos conceptos que genera la autoridad del maestro? Qué transmitir, por qué y cómo, son pre-
hemos usado: autoridad, tradición, manda- En este sentido, conviene hacer una dis- guntas fundamentales del quehacer de los
to, renovación, autorización. ¿Sobre qué tinción entre tradición y mandato. Este últi- enseñantes, preguntas cuyas respuestas
bases pensar la autoridad y la potencialidad mo se relaciona con la aceptación y el cum- han ido variando a lo largo de la historia de
de nuestro trabajo como docentes? plimiento. En la tradición, en cambio, hay acuerdo a condiciones políticas, sociales,
Hannah Arendt recuerda que la palabra rasgos en los cuales reconocerse, y es así económicas y culturales, que a su vez segui-
"autoridad" proviene de auctoritas, que como se renueva un vínculo de autoridad rán variando con la historia humana. La
remite a augere (aumentar) y a auctor respecto del presente y del futuro. La expe- docencia es un trabajo social que se consti-
(autor, fundador). En el contexto del riencia que se transmite y se deja a otros es tuye en el entramado de diversas experien-
Imperio Romano, los ancianos, el Senado o también una suma de pasado y autoridad; cias −escolares y extraescolares−, y ese diá-
los patres eran acreedores de una autoridad es sabiduría recogida de la vida que se logo con la cultura de su tiempo es central
sustentada en la tradición de sus anteceso- transfiere a los nuevos para que la hagan para los puentes que los docentes habilitan
res. Así, la autoridad de los vivos no estaba suya. Esa autoridad de la tradición puede para sus alumnos. Este es el nuevo suelo en
entre los vivos, sino entre los ancestros del construir posiciones conservadoras, renova- el que puede nutrirse nuestro trabajo −ade-
pasado. De acuerdo con Arendt, esta auto- doras o críticas, y es a través de ese gesto, más de hacerlo en el diálogo con las nuevas
generaciones−, con problemas clásicos, con
Ministerio de Educación, Ciencia y Tecnología

otros nuevos, y con herramientas que tie-


nen experiencia, que han sido revisadas
pero que a la vez tienen una potencialidad
renovada. Se hará necesario recurrir a nue-
vas preguntas, a hacer visible lo oculto y
permitirse entrar en contradicción con el
origen de las prácticas docentes para incluir
nuevas miradas que contemplen la nove-
dad de situaciones, la pluralidad de infan-
cias, adolescencias y juventudes que carac-
terizan nuestro tiempo, y estar preparados
además para acompañar circunstancias has-
ta ahora inéditas. Esto, sin lugar a dudas, es
una tarea compleja que requiere formación
y reflexión acerca de la experiencia. Para ello
es necesario reconocer la centralidad de
algunas cuestiones en la tarea docente, y
redefinir nuestro trabajo y nuestro papel por
medio de la recuperación de la responsabili-
dad y la importancia del oficio de enseñar.
El escritor Eduardo Galeano provee una
imagen que siempre me ha resultado muy
El trabajo docente se construye en un entramado de experiencias que establecen diálogos iluminadora para pensar la posición que
con el universo de los alumnos. construyen los docentes:
DOCENTES: LA TAREA DE CRUZAR FRONTERAS Y TENDER PUENTES 15

El monitor de la educación
La docencia implica siempre una instancia que se constituye en relación con otros y con el propio presente.

A orillas de otro mar, otro alfarero se retira en fuera distinto. En la educación, esta responsabili- la voluntad oficial. Las instituciones, a tra-
sus años tardíos. Se le nublan los ojos, las manos dad con respecto al mundo adopta la forma de vés de sus muchos actores (maestros,
le tiemblan, ha llegado la hora del adiós. Entonces autoridad. [...] La calificación del profesor consis- alumnos, equipos técnicos, padres, gre-
ocurre la ceremonia de iniciación: el alfarero viejo te en conocer el mundo y ser capaz de darlo a mios, etc.) dejan de ser meras reproducto-
ofrece al alfarero joven su pieza mejor. Así manda conocer a los demás, pero su autoridad descansa ras para transformarse en productoras,
la tradición, entre los indios del noroeste de en el hecho de que asume la responsabilidad con creadoras y recreadoras de aquellos proce-
América: el artista que se va entrega su obra respecto a ese mundo. Ante el niño, el maestro es sos. De esta manera, aquello que en el
maestra al artista que se inicia. Y el alfarero joven una especie de representante de todos los adul- marco institucional entró en crisis, que
no guarda esa vasija perfecta para contemplarla y tos [...] (Arendt, 1996: 201). resultó insuficiente o improductivo, puede
admirarla, sino que la estrella contra el suelo, la ser renovado.
rompe en mil pedacitos, recoge los pedacitos y los Recuperar el protagonismo de quienes ¿Qué es, entonces, enseñar? Podemos
incorpora a su arcilla (Galeano, 1993). componemos las instituciones no significa concluir que es un trabajo profesional que
dejar de reconocer el papel privilegiado de posee una tradición en la cual se respalda
La mejor obra se entrega, y en ese traspa- quienes dirigen, administran y gobiernan pero a la que puede revisar porque entien-
so hay enseñanza y responsabilidad. Pero en la organización de la producción y de que no se trata de un mandato inapela-
también ese gesto inaugura un camino reproducción cultural de la sociedad. Aun ble. Por el contrario, la docencia toma esa
nuevo que será distinto del anterior, y al que sin renunciar al lugar de peso que le cabe tradición para renovarla, y en esa renova-
dará forma quien lo protagonice recorrién- al Estado en esto, la perspectiva que busca- ción cultural se autoriza. La potencialidad
dolo con luz nueva. Sobre esta manera de mos abrir tiene presente que la acción edu- que adquiere su voz autorizada puede resi-
concebir el trabajo docente como un modo cativa es propia de espacios que tienen dir en la necesidad de investigar el mundo,
de legar a los otros la mejor obra, nos dice particularidades específicas. Involucra la de interpretarlo y darlo a conocer. Se trata,
mucho la siguiente afirmación de la filósofa relación entre los intereses de los distintos en definitiva, de brindar ese conocimiento
Hannah Arendt: actores grupales e individuales que inter- como un puente, ayudando a cruzar fronte-
Como el niño no está familiarizado aún con el vienen en cada uno de los procesos. Desde ras en una relación mediada que liga y tam-
mundo, hay que introducirlo gradualmente en él; esta concepción, la ampliación de las opor- bién habilita para la renovación, que genera
[...] los educadores representan para el joven un tunidades educativas para todos los chicos responsabilidades −micro y macro, indivi-
mundo cuya responsabilidad asumen, aunque y chicas de las escuelas y la propia selec- duales, colectivas e institucionales− en la
ellos no son los que lo hicieron y aunque, abierta ción de valores, conocimientos y habilida- construcción de una posición con otros y
o encubiertamente, preferirían que ese mundo des que se enseñan no dependen sólo de con nuestro tiempo.
ANALES DE LA EDUCACIÓN COMÚN VOL 2 · N° 1-2

Carina V. Kaplan
Universidad Nacional de Buenos Aires - Universidad Nacional de La Plata -
CONICET

La implicación afectiva en
tiempos de pandemia y
postpandemia. Educar para
una sociedad de reciprocidades

RESUMEN

La experiencia escolar inédita que se origina a partir de la pandemia por el


virus SARS-CoV-2 nos interpela a una pedagogía del trauma que posibilite la
tramitación y reparación del sufrimiento social. En este ensayo esbozo una serie
de reflexiones sobre la necesidad de que la escuela se constituya en soporte
emocional y funcione como refugio para las infancias y juventudes, habilitando
la posibilidad de constitución del lazo social. Los vínculos emocionales en las
sociedades modernas refieren a las formas de compromiso que establecen los
sujetos entre sí y con su realidad. Muestran el lado más humano de la existencia.
Tal como nos recuerda Agnes Heller, sentir significa estar implicado en algo o
con alguien. Si aspiramos a construir una sociedad de reciprocidades, ella se
crea mediante la educación para el reconocimiento y el respeto mutuos que
consiste en: cuidar de sí, hacer algo por sí mismo y ayudar a los demás.

Palabras clave: Afectividad, Sistema educativo, Lazo social, Subjetividad,


Cuidado.

Anales de la Educación Común, 2021, Vol. 2, N° 1-2.


Carina V. Kaplan - La implicación afectiva en tiempos de pandemia y postpandemia. Educar
para una sociedad de reciprocidades. 104
pp. 104--113
· 2021

En este ensayo esbozo una serie de reflexiones sobre la necesidad de que


la escuela se constituya en soporte emocional y funcione como refugio
para las infancias y juventudes, habilitando la posibilidad de constitución
del lazo social en tiempos atravesados por el sufrimiento social. La noción
de soporte se define a partir de conjuntos heterogéneos de elementos,
reales o imaginarios, que se despliegan a través de un entramado de vín-
culos en virtud de los cuales los individuos nos amarramos (Martuccelli,
2007). Esta trama contribuye a contrarrestar los sentimientos de soledad y
crea un tejido existencial que da sentido a nuestro mundo. Las relaciones
afectivas en las sociedades modernas refieren a las formas de compromi-
so que establecen los sujetos entre sí y con su realidad. Muestran el lado
más humano de la existencia. Tal como nos recuerda Agnes Heller (1999),
sentir significa estar implicado en algo o con alguien.
La experiencia escolar inédita que se origina a partir de la pandemia por
el virus SARS-CoV-2 nos interpela a una pedagogía del trauma que posibili-
te la tramitación de narrativas del dolor social. El dolor es una experiencia
humana altamente simbólica. El sentir del dolor, es decir el sufrimiento, es
consecuencia de una relación afectiva y significante con una situación. La
relación con el dolor es siempre una cuestión de significación y de valor.
Se entrama en la afectividad, que da la medida de su intensidad y su tona-
lidad. Se trata de un hecho situacional, aislable en un sujeto que lo padece,
pero modelizado por la trama social, cultural, relacional que impregna ese
sufrimiento (Le Breton, 1999).
El sufrimiento puede ser interpretado, entonces, como una experiencia
comunitaria y que puede ser colectivizado a partir de los relatos biográfi-
cos personales. Estar disponible para la escucha y comprender la perspec-
tiva de los demás es una condición de posibilidad para la socialización de
los sentires hacia la construcción de soportes socio-psíquicos. El lenguaje
y la narración adquieren una importancia fundamental en el proceso de
elaboración del padecimiento de los actores de la comunidad educativa.
Educar para la sensibilidad hacia los demás es un imperativo ético peda-
gógico en sociedades desiguales atravesadas por el sufrimiento individual
y colectivo que la pandemia agudizó. Si aspiramos a construir una socie-
dad de reciprocidades, ella se crea mediante la educación para el respeto
mutuo que consiste en: cuidar de sí, hacer algo por sí mismo y ayudar a
los demás (Sennet, 2003). La trama vincular estructura subjetividades en
el marco de una pedagogía del cuidado.

105
ANALES DE LA EDUCACIÓN COMÚN VOL 2 · N° 1-2

La mirada del cuidado es el signo más amoroso de la relación pedagógica


y funciona como de analizador de la desigual distribución de bienes materia-
les y simbólicos en cada sociedad y tiempo particular, y de la necesidad de
reparación. La sensibilidad hacia sí mismo y hacia los demás ayuda a percibir
el padecimiento e involucrarse en procesos de elaboración del dolor social.
Uno de los interrogantes que surge es acerca de cómo educar para tratar
a los demás con respeto, y posibilitar que se fabrique el sentimiento de
auto respeto cuando el contacto se produce en circunstancias tan des-
iguales. Es decir, el desafío que interpela a la institución escolar es acerca
de cómo contrarrestar el sentimiento de inferioridad que incide en la cons-
trucción de lo que Goffman (2009) ha caracterizado como un proceso de
fabricación de identidades deterioradas. La construcción de autoestima
es un eje vertebrador de la experiencia escolar.
La escuela constituye un territorio simbólico de esperanza en la medida
en que allí o desde allí pueden tejerse lazos de solidaridad y reconocimiento
mutuo. La institución educativa participa contrarrestando los muros emo-
tivos que nos dividen, nos separan, nos segregan y nos excluyen. Permite
atenuar las marcas de la desigualdad durante el proceso de escolarización.
La reciprocidad en el cuidado del otro constituye un elemento central de
los procesos educativos que se despliegan en una situación de emergencia
sanitaria. La pedagogía del cuidado supone la estructuración emocional
de vínculos de confianza y compromiso mutuos. Elias (2000) afirma que la
imagen del “nosotros” forma parte de la imagen del “yo” que se pretende
construir. La escuela es un escenario privilegiado para la construcción de
lazos de pertenencia toda vez que se erige como lugar habitable, sin dis-
tinción. Es el espacio simbólico donde prima la promesa de un “nosotros”
que rompe con la instantaneidad de la individualización (Kaplan, 2017).

L A E M O C I Ó N C O M O C AT E G O R Í A P E D A G Ó G I C A

El giro afectivo, relativamente reciente y que adquiere fuerza en el cam-


po de la educación, consiste en la confirmación de la inscripción de las
emociones como categoría interpretativa para poder acceder al corazón
de las prácticas pedagógicas. El lenguaje de las emociones es una gra-
mática fértil para comprender la construcción de vínculos y las vivencias
subjetivas de los actores de la comunidad durante la experiencia inédita de

106
· 2021

escolarización en pandemia, donde se han transformado las ritualidades y


las formas de interacción. En la excepcionalidad de la experiencia escolar
que nos toca vivir, las coordenadas de espacio y tiempo se resquebraron.
Las transformaciones de la vida educativa poseen efectos en la organiza-
ción socio afectiva que suponen los oficios de docente y estudiante, en lo
concerniente a las formas conocidas del cotidiano escolar.
Asumimos que los sentimientos funcionan como organizadores del lazo
social. No se trata de sentimientos sueltos ni apriorísticos, sino que es
dable pensar que lo que se conforma es una red o estructura emotiva que
posibilita producir trama escolar; aún en una escuela sin paredes, aún en
procesos de virtualización forzosa de la enseñanza y el aprendizaje, aún
en una presencialidad distinta a la habitual. Una cultura afectiva forma un
tejido apretado donde cada sentimiento está situado en perspectiva den-
tro de un conjunto.
No se pueden comprender las experiencias emocionales sin ponerlas en
contacto directo con una situación específica y con un entramado, con la
forma con la cual una cultura afectiva escolar se mezcla directamente con
el tejido social. La experiencia emocional de cada quien de nosotros como
seres concretos está imbricada con la realidad emocional de las socieda-
des en que nacemos y en las que nos toca habitar. Las transformaciones
en los modos de vivir modelan nuestra estructura emotiva.
Bajo las consideraciones precedentes, resulta importante puntualizar
qué se entiende por las emociones en la vida escolar desde un enfoque
sociocultural e histórico. Una perspectiva de análisis sobre la construcción
social del orden afectivo escolar debe partir de la premisa de que ninguna
de las formas de comportamiento ni las disposiciones para sentir (habitus
emotivo) pueden catalogarse como naturales, aunque así se perciban y se
vivencien (característico de la dinámica de la naturalización). Las dispo-
siciones para sentir son producto del aprendizaje y se interiorizan bajo la
forma del inconsciente social que lleva impreso las marcas de la memoria
biográfica y los signos de época.
La emotividad requiere ser interpretada a partir de la interrelación entre
lo biológico y lo social en el marco de ciertos patrones culturales e histó-
ricos. Los comportamientos sociales y las experiencias emocionales son
producto de continuos movimientos inconstantes. De ello se desprende
que la estructura emotiva es cambiante y necesita ser anclada en proce-
sos sociohistóricos y culturales más amplios. Las experiencias emociona-

107
ANALES DE LA EDUCACIÓN COMÚN VOL 2 · N° 1-2

les y los acontecimientos sociohistóricos se imbrican mutuamente. Para


pensar la vida afectiva, es preciso considerar que los procesos de cons-
trucción y transformación psicológica y social sólo pueden ser entendidos
en conexión; ligando las transformaciones de largo alcance de la estructu-
ra social y la estructura emotiva.
A los fines de alcanzar una comprensión profunda de las experiencias
escolares nos posicionamos en un horizonte epistemológico bajo el su-
puesto de que ni las emociones pueden ser abordadas sin tener en cuenta
la dimensión estructural material de lo social, ni de que esta última puede
ser interpretada si no se pone en juego la producción de la subjetividad.
Las emociones son construcciones culturales situadas que tienen una his-
toria. En la medida en que están condicionadas por los contextos sociales,
no es posible comprenderlas si no atendemos la perspectiva relacional de
los sujetos. Así, el yo siento se complementa con el nosotros y nosotras
sentimos, en configuraciones singulares y en los vínculos con los otros. La
emoción no es una naturaleza descriptible sin contexto, ni independiente
de las interacciones y significados atribuidos por los actores escolares.
Por otro lado, se parte del supuesto de que los individuos en sociedad
construimos un conjunto de soportes afectivos, materiales y simbólicos,
mediante una red vincular en el contexto social e institucional donde se
despliega nuestra experiencia biográfica. Pueden presentarse movilizados
conscientemente por el sujeto, o bien de un modo inconsciente como un
efecto indirecto. Los soportes simbólicos operan como una protección
donde la existencia individual y colectiva se superponen.
Es indudable el carácter constitutivo de los otros (sean personas, sím-
bolos o instituciones) en la producción de toda emotividad. Para autoafir-
marse el sujeto precisa de la mirada del otro, pues éste se organiza sub-
jetivamente en la relación interpersonal con otros sujetos, en una relación
intercultural y social (Wieviorka, 2009). De allí la importancia de ayudar
desde la escuela a tramitar los sentimientos que producen las prácticas de
humillación y exclusión para fortalecer la valía social.
En términos de Bericat Alastuey, “existe un vínculo necesario entre subje-
tividad afectiva y situación social objetiva” (2000:152). Es preciso entonces
pensar de un modo integral los límites objetivos que marcan de entrada a
las y los estudiantes en su experiencia y trayectoria. Partir de sus constric-
ciones o determinaciones estructurales que configuran un sentido de los
límites subjetivos, esto es, una suerte de cálculo simbólico anticipado de

108
· 2021

lo que los actores pueden o no pueden proyectar para la propia trayectoria


social y educativa. En virtud de sus condiciones de clase, género, etnia y
discapacidad, se limitan y autoexcluyen de aquello que ya están excluidos,
o bien se proyectan hacia el éxito que creen les espera. En este proceso de
construcción de autoestima o de descrédito, la trama afectiva escolar de
aceptación o rechazo juega un papel estructurante.
Por medio de los juicios, las clasificaciones y los veredictos que la ins-
titución educativa realiza, cada estudiante va incorporando de un modo
inconsciente sus límites y sus posibilidades simbólicas, estableciéndose
como efecto simbólico una conciencia de los límites o efecto de destino
(Kaplan, 2017). No obstante, instituciones y docentes tienen márgenes de
autonomía y creatividad para inclinar el péndulo a favor de la ampliación
de las posibilidades simbólicas con miras a subvertir el orden injusto.
Se trata de romper ciertas creencias sociales muy naturalizadas sobre la
inevitabilidad de ciertos destinos. Aún bajo la evidencia de la existencia de
una multiplicidad de determinantes externos, actúa allí donde los límites
objetivos parecen sentenciar al estudiantado y sus familias; contribuyendo
en ocasiones a tensionar el sentido de los límites subjetivos, es decir, ayu-
dándolos a no ajustar mecánicamente sus deseos y horizontes futuros a
los límites que les son dados y esperables por sus constricciones sociales
de origen o de otro tipo.
El orden escolar colabora en la producción de ciertas tramas y prácticas
de afectividad (Kaplan, 2018). En un escenario de virtualización forzada y
una redefinición de la relación yo-otros en las diversas formas que asume
la presencialidad en las instituciones en el proceso de escolarización en
pandemia, la escuela puede funcionar como un lugar de encuentro. Ello
es así debido a que la escuela opera como una presencia material, pero
también, como una presencia simbólica en la constitución del lazo social.
La institución escolar es un lugar, físico y simbólico, donde desde la in-
fancia, y a través de la interacción con compañeras y compañeros y con
la autoridad, cada quien va formándose ideas, imágenes y sentimientos
acerca de sí mismo y de los demás. Cabe afirmar que “la fabricación cul-
tural de emociones y sentimientos ligados a la valía social nos constituye
en nuestro proceso de subjetivación” (Kaplan, 2013: 47). La mirada escolar
posee una innegable fuerza simbólica en la constitución de la autoestima
ya que las y los estudiantes se miran en ese espejo.
Es preciso posicionarse bajo el supuesto de que los seres humanos esta-

109
ANALES DE LA EDUCACIÓN COMÚN VOL 2 · N° 1-2

mos constituidos para la vida en compañía de otros (Elias, 1998). Somos se-
res interdependientes precisamente porque tenemos esa necesidad de entrar
en contacto, de comunicarnos, de convivir, de simbolizar y de aprender. Por
tanto, resulta relevante generar las condiciones para que la comunidad educa-
tiva pueda objetivar sentimientos de duelo, de ruptura de vínculos, de pérdida
de soportes afectivos, que marcan los sentidos del proceso de escolarización
en el transcurrir de una experiencia traumática. Se trata de transformar los
miedos y los sentimientos de pérdida en esperanza, dado que la escuela re-
presenta algo del orden de lo vital y de proyección del porvenir.
Los sentimientos de pérdida, miedo y soledad se agudizaron en el con-
texto de pandemia. Se observa que en esta experiencia de trauma social se
expresa una red emotiva que transita desde el miedo frente a la posibilidad
de perder las oportunidades académicas que prometían las expectativas
subjetivas respecto del ciclo escolar, hasta los miedos frente a la muerte
y la enfermedad. La posibilidad de la pérdida de una persona que ocupa
un lugar significativo en el entramado de relaciones socioafectivas se con-
vierte en un acontecimiento que faculta la pregunta por la propia vida.
Tengamos en cuenta que el miedo constituye una emoción vertebradora
de la condición humana. Aun cuando la posibilidad de sentir miedo sea
un rasgo invariable de la naturaleza humana, “la intensidad, el tipo y la es-
tructura de los miedos que laten o arden en el individuo [...] aparecen de-
terminados siempre por la historia y la estructura real de sus relaciones
con otros humanos, por la estructura de su sociedad y se transforman con
ésta” (Elias, 1987: 528).
Al mismo tiempo, la soledad es un sentimiento vinculado a la falta de
contacto. Si las profundas relaciones de dependencia emocional que es-
tablecemos en la vida se convierten en una necesidad vital, el sentimiento
de soledad que se experimenta en el mundo social suele estar en la base
del miedo a no existir para los demás. Al mismo tiempo, la soledad es un
sentimiento vinculado a la falta de estima y reconocimiento.
Si las profundas relaciones de dependencia emocional que establece-
mos en la vida se convierten en una necesidad vital, el sentimiento de so-
ledad suele estar en la base del miedo a no existir en los ojos de los otros.
En La soledad de los moribundos, Elias sostiene que “es bastante fútil el
intento de descubrir en la vida de una persona un sentido que sea indepen-
diente de lo que esa vida significa para otros” (Elias 2009:70). El aislamiento
emocional entre los individuos estructura la sensación de que la existencia

110
· 2021

carece de significación ante la pérdida de vinculaciones afectivas que fun-


dan el apego mutuo en el tejido humano. De allí que experiencia de escola-
rización en pandemia y en las condiciones de la postpandemia se necesita
crear y recrear puentes afectivos que operen como anclajes intersubjetivos.

U N A R E F L E X I Ó N A B I E R TA

Si bien es cierto que nada reemplaza a los vínculos que se generan en


las modalidades de la presencialidad, la mirada protectora de la escuela,
incluso en la continuidad pedagógica no presencial, simboliza un sostén
emocional, un lugar simbólico donde amarrarse subjetivamente. En el par-
ticular cruce entre la presencialidad y la no presencialidad, así como de
otras múltiples formas de la experiencia escolar que iremos construyendo
en lo sucesivo, necesitamos abrazar la idea de reparación simbólica.
Uno de los mayores desafíos con miras hacia la postpandemia es la de
avanzar en la producción de condiciones educativas, curriculares y so-
cio-afectivas, que contribuyan a revertir las profecías de fracaso que se
ciernen, en especial, sobre niñas, niños y jóvenes de sectores sociales des-
favorecidos. Para ello, ampliando recursos simbólicos que permitan armar
nuevas escenas frente a una realidad compleja de tramitar en soledad, y
también, abandonando por fin viejas antinomias entre el contenido acadé-
mico y el contenido socializador del proceso de escolarización. Si hay una
lección que nos deja esta experiencia inédita es que no se puede escindir
lo académico de lo vincular.
Sin afectividad, sin afectación subjetiva, sin movilización emocional, no
hay posibilidad de estructurar una trama que promueva los procesos co-
laborativos y fraternales de enseñanza y aprendizaje. Para ello es preciso
reponer una mirada del estudiante que le otorgue valía social. La mirada
escolar posee una innegable fuerza simbólica en la constitución de la au-
toestima ya que las y los estudiantes se miran en esa imagen, y tienden a
reconocerse o a negarse. La experiencia intersubjetiva de reconocimiento
significa la reivindicación de la identidad, mediante la mirada del otro que
produce autoestima.
La mirada social tiene la capacidad de formular juicios de valor (otorga
y quita valor) porque se dirige a las raíces inconscientes, en un sentido so-
ciológico, de un sentimiento de identidad que depende de la aprobación de

111
ANALES DE LA EDUCACIÓN COMÚN VOL 2 · N° 1-2

los otros. El reconocimiento, del orden de lo simbólico, implica así la con-


firmación de la aceptación y el cuidado del otro, tanto en el plano cognitivo
académico, como en el socioafectivo.
Esta pandemia nos deja como lección aprendida la necesidad de traba-
jar colectivamente sobre la afectación subjetiva de los procesos de escola-
rización. Podemos afirmar que las interacciones escolares se estructuran
a través de circuitos afectivos que trazan horizontes de posibilidad. Si as-
piramos a construir una sociedad de reciprocidades, ella se crea y recrea
mediante la educación para el cuidado socioafectivo en la trama escolar.
Se trata de educar para que el sufrimiento del otro nos conmueva y que el
otro se movilice por nuestro dolor.

REFERENCIAS BIBLIO GRÁFICAS

Bericat Alastuey, E. (2000). La sociología de la emoción y la emoción


en la sociología. Papers, 62. https://idus.us.es/handle/11441/48381

Elias, N. (1987). El proceso de la civilización. Investigaciones sociogenéticas


y psicogenéticas. Fondo de Cultura Económica.

Elias, N. (1998). Los seres humanos y sus emociones: un ensayo sociológico


procesual. En La civilización de los padres y otros ensayos (pp. 290-329).
Editorial Norma.

Elias, N. (2000). La sociedad de los individuos. Península.

Elias, N. (2008). Sociología Fundamental. Gedisa.

Elias, N. (2009). La Soledad de los Moribundos. Fondo de Cultura Económica.

Goffman, E. (2009). Estigma. La identidad deteriorada. Amorrortu.

Heller, A. (1999). Teoría de los sentimientos. Coyoacán.

Kaplan, C. V. (2013). El miedo a morir joven. Meditaciones de los estudiantes


sobre la condición humana. En C.V. Kaplan (dir.), Culturas estudiantiles.

112
Pensar
los vínculos
en tiempos
de pandemia:
La escuela
como un
lugar de
cuidado

la educación Ministerio de Educación


nuestra bandera
Introducción
Desde la conformación de las instituciones educativas a nuestros días, la escuela ha lo-
grado sobreponerse a importantes crisis y transformaciones sociales, constituyendo un
lugar de resguardo, cuidado, encuentro, enseñanza y aprendizaje. Reconocer su fortaleza
nos presenta el desafío de trabajar mancomunadamente en la búsqueda de consensos
que permitan valorar su rol en el marco de los nuevos escenarios que presenta el mundo
contemporáneo.

La irrupción de la pandemia ha generado tal conmoción que puso en el centro la pregun-


ta por la vida en común y la dimensión colectiva del cuidado. El contexto de emergencia
sanitaria del COVID-19 y las políticas de cuidado que se han implementado -y continúan
implementándose-, modificaron las formas de abordaje e intervención de las/os docentes,
presentándonos nuevos desafíos. De este modo, la pandemia y su atravesamiento nos
condujo a pensar de otra manera las condiciones de enseñanza y aprendizaje, el entrama-
do institucional y las formas de acompañamiento a niñas, niños, adolescentes, jóvenes y
familias instaurando -de algún modo- una reformulación del formato escolar.

En este marco, la pregunta por la función social de la escuela se renueva debido a lo diná-
mico y cambiante de la coyuntura actual, invitándonos a pensar: ¿Cuáles son los efectos
que la pandemia ha generado en las subjetividades y en los vínculos en niñas, niños, ado-
lescentes y jóvenes? ¿Cuáles son los modos de alojarlos y acompañarlos desde las escue-
las? ¿Cómo enseñar cuidando y cómo cuidar enseñando? ¿Qué recursos puede ofrecer la
escuela para poner palabras al sufrimiento, al dolor y la pérdida que emergieron ante lo
ocurrido? ¿Resulta necesaria la búsqueda de una perspectiva común para pensar la expe-
riencia? ¿Cómo contribuir a que las políticas públicas puedan promover un pensamiento
o conceptualización de la pandemia que pueda introducir diversas miradas? ¿Debemos
abordar la pandemia en la escuela? ¿Puede ser entendida como un tema de enseñanza?

“Pensar los vínculos en tiempos de pandemia: la escuela como un lugar de cuidado”, es un


material elaborado por el Ministerio de Educación, a través de la Dirección de Educación para
los Derechos Humanos, Género y ESI, dependiente de la Subsecretaría de Educación Social
y Cultural, Secretaría de Educación. En particular, desde el Área de Convivencia Escolar y el
Programa de Prevención y Cuidados en el Ámbito Educativo. Se trata de una propuesta de
acompañamiento a la reflexión y el trabajo en las aulas para docentes, equipos directivos,
equipos de orientación y otras personas que asumen roles pedagógicos en las escuelas.

Este material es una invitación a la reflexión compartida, a la búsqueda de palabras para


nombrar la experiencia. No pretende plantear una interpretación unívoca de lo acontecido,

6.
tampoco realizar generalizaciones, ni mucho menos recetas para superar lo ocurrido, sino
más bien abrir diálogos y espacios de escucha. Para elaborarlo, los equipos de trabajo
mantuvieron reuniones e intercambios con investigadores, investigadoras y docentes. A la
vez, se buscaron testimonios y expresiones de niñas, niños, adolescentes y jóvenes quie-
nes comparten también en estas páginas situaciones vivenciadas en estos tiempos.

La pregunta por el rol de la escuela frente a la pandemia no está saldada y es resignificada


de forma constante. Sin embargo esperamos, a través de este material, brindar algunas
claves que aporten a:

contribuir en la construcción de una pedagogía del cuidado en las escuelas, fortale-


ciendo las propuestas de enseñanza para brindar a las nuevas generaciones saberes
necesarios con los que habitar el mundo;
propiciar espacios para compartir experiencias;
elaborar narrativas singulares y comunes, ofreciendo relatos y sentidos a partir de los
cuales significar la pandemia;
promover el ejercicio de una autoridad pedagógica que facilite la construcción de un
porvenir frente a contextos de incertidumbre;
favorecer espacios que den lugar a la tramitación de las pérdidas individuales y co-
lectivas.

En función de estos propósitos, el material se encuentra organizado en cuatro apartados.


En primera instancia invitamos a una reflexión en torno a los efectos en los modos de
ser y de actuar que las condiciones de época ligadas a la pandemia están produciendo,
para luego adentrarnos sobre el rol de la escuela a la hora de repensar las condiciones
para enseñar y aprender, desde una pedagogía del cuidado. A continuación, encontrarán
algunas propuestas para trabajar en el aula de acuerdo a los distintos niveles educativos
y, por último, incluimos algunas experiencias que intentan dar respuesta a problemáticas
complejas suscitadas en tiempos de pandemia.

Esperamos que se encuentren en la lectura de estas páginas con preguntas comunes y con
nuevos interrogantes para seguir haciendo escuela hoy.

.7
Apartado I: La escuela en
tiempos de pandemia
1. La escuela en tiempos de pandemia

Cuando comenzó el aislamiento en marzo del 2020, ante la urgencia de encontrar los modos
de hacer escuela en esa coyuntura, las tareas y el envío de actividades quedaron en el
centro de la escena procurando encontrar la forma de sostener la continuidad pedagógica
y el vínculo con las/os estudiantes. Frente a la extensión del confinamiento, rápidamente,
advertimos que muchas de nuestras prácticas docentes no podían trasladarse de
manera directa a una versión mediatizada por la virtualidad. Tampoco la proliferación de
plataformas -que habilitaban encuentros sincrónicos- eran en sí mismas una respuesta
para el ejercicio de nuestra tarea pedagógica. Si a alguien le quedaban dudas, la pandemia
vino a corroborar que la función de la escuela trasciende la acreditación de saberes,
por lo que no se trataría solo de la transmisión de contenidos sino de la oportunidad de
construirlos entre docentes y estudiantes, armando experiencias educativas que resulten
significativas.

Hoy, ya desde otro posicionamiento y a partir de la experiencia que podemos relevar en


este tiempo, nos volvemos a preguntar, ¿qué nos dejó la pandemia como aprendizajes para
ensayar otras formas de hacer escuela?, ¿cuáles son las experiencias que podemos recu-
perar?, ¿qué oportunidades nos brindó para repensarnos?

La pandemia nos vino a mostrar, una vez más, que la escuela co-construye objetos de co-
nocimiento y deviene, asimismo, en un espacio con tiempos propios para el despliegue de
formas singulares en las que cada estudiante y cada grupo se expresa, aprende y se vin-
cula con otras y otros. En ese sentido, nuestra responsabilidad como docentes supone el
ejercicio de una autoridad pedagógica que promueva aprendizajes y, a la vez, habilite
procesos de subjetivación a partir del ofrecimiento de un espacio material y simbólico,
con los instrumentos necesarios para ocuparlo. Para incluirse en el mundo las/os niñas,
niños, adolescentes y jóvenes precisan de personas adultas que confíen en ellas y ellos,
que las y los desafíen, y que establezcan límites humanizantes. De este modo, nuestra
posición implicaría la responsabilidad de hacerse cargo de otras/os, en el marco de una
relación de confianza que es instituyente.

La no concurrencia a los edificios escolares resultó un desafío a la hora de desplegar


nuevas configuraciones para el ejercicio de una autoridad habilitante y para la cons-
trucción de otras formas de acompañar, sostener y enseñar. Lo que en otro momento
implicaba la mirada en el aula, se convirtió en un seguimiento a través de un mail,
una llamada telefónica o un mensaje. La propuesta didáctica hizo de puente para sa-

8.
ber quién acompañaba a cada estudiante del otro lado de la pantalla, cómo esta-
ban transitando el confinamiento y qué extrañaban de la cotidianeidad escolar previa.

La pandemia lo que hizo fue hacer visible la centralidad del vínculo pedagógico, en
el cual reside -por supuesto- la autoridad pedagógica, pero la autoridad entendida
no en el sentido de una imposición, o de una cuestión que baja desde arriba, sino la
autoridad pedagógica entendida como habilitación, como la posibilidad de abrirle al
estudiante y a la estudiante nuevas posibilidades y ofrecerle las herramientas para
conocer el mundo. A partir de la pandemia muchos y muchas docentes advirtieron
que ese vínculo es central en la transmisión del conocimiento, o de los saberes1.

María Beatriz Greco2

En este sentido, podríamos decir que la escuela demanda el armado de escenarios de


ternura (Ulloa, 2005) sobre la base de una pedagogía del cuidado, que permitan el des-
pliegue de experiencias subjetivantes3 posibilitando dar sentidos a lo que nos pasó. Desde
este enfoque, cuidar no se limita a resolver situaciones “emergentes” o enseñar conteni-
dos relativos al reconocimiento del propio cuerpo y de las/os otras/os, sino que se vincula
también con garantizar a las nuevas generaciones la distribución y el acceso a saberes
necesarios para conocer y participar del mundo que las/os rodea.

1- Para ampliar,
Posicionarse desde una perspectiva de cuidados implica, entonces, un movimiento a partir invitamos a la lectura del
material Autoridades que
del cual la/el otra/o no es un sujeto al que prevenir o del cual prevenirse, sino un sujeto habilitan a través del cual
se propone repensar la
de derecho que habita la escuela mientras construye su propia identidad. Ello requiere autoridad pedagógica a
partir de nuevos desafíos
necesariamente de una propuesta educativa integral que comprenda que la formación, en las formas de
abordarla y construirla,
además de avanzar en el dominio de la lectura, la escritura y el cálculo, también promueve en una perspectiva
de derechos que nos
el desarrollo de la conciencia de sí misma o sí mismo, del propio cuerpo, del cuerpo de las permita fortalecer
prácticas democráticas
otras personas, de las emociones, de los distintos niveles de autonomía, del propio lugar contemporáneas en
torno a la enseñanza y la
en la sociedad. En ese sentido, resulta clave ofrecer experiencias en las que las/os es- convivencia.

tudiantes puedan decidir, organizarse, implicarse y participar activamente poniendo a 2- Doctora en Filosofía
y Ciencias Sociales.
disposición horizontes culturales diversos y plurales. Profesora de la UBA.
Directora de Proyectos
de Investigación
(UBACyT)
De este modo, el acompañamiento no se configura desde el ejercicio de un lugar que coloca
3- Entendemos
a las/os estudiantes en una posición pasiva, dejándolas/os en el rol de “ser acompañadas/ por experiencias
subjetivantes aquellas
os”, sino que -por el contrario- intenta brindar herramientas, cada vez más complejas, para situaciones en las que
se generan condiciones
que ellas/os mismas/os puedan pensar, reflexionar, cuestionar, imaginar, e interpretar sus para una transformación
de la subjetividad.
experiencias. Tienen que ver con
una ampliación de
las maneras de mirar
y de posicionarse
que posibilitan el
enriquecimiento, la
interpelación y la
transformación en la
medida que se mira de
otra forma, se producen
reposicionamientos y/o
se habilita la producción
de nuevos sentidos.

.9
Se trata de restituir a un sujeto activo. Es decir, no solo acompañar, porque
acompañar tiene a veces ese desliz de que una/o es la/el que acompaña y otra/o es
la/el acompañada/o y eso deja al sujeto en un lugar pasivizado, victimizado. Resulta
mucho más interesante poder buscar recursos que restituyan a un sujeto activo.
Es decir, que pueda ser activo también en el lazo con otros y otras, en armar una
red solidaria, tener una función más activa en lo que es ese acto solidario con la/el
otra/o que le devuelva también algo de la vitalidad que se ha visto afectada.

Paula Husni4

En este punto cobra relevancia el trabajo sobre la afectividad en las escuelas pensando
en el afecto entramado con el acto pedagógico. Generar espacios de circulación de la pala-
bra y de participación democrática en los que niñas, niños, adolescentes y jóvenes puedan
ser escuchadas/os. Espacios que rompan con una posición y forma de explicación adulto-
céntrica (que muchas veces presupone lo que las/os estudiantes pueden estar pensando
o sintiendo y genera conjeturas sobre ello) y que promuevan el protagonismo de las/os
estudiantes para la narración de sus propias vivencias y experiencias.

Se trata de advertir, en palabras de Sara Ahmed (2016), la relevancia política de los afectos,
distanciándonos de prácticas que pretenden regular, educar e individualizar las emocio-
nes, reconociendo su complejidad y posibilitando su despliegue. Las emociones no son
“buenas” ni “malas”, ni tenemos que acompañar a las/os estudiantes a que expresen o al-
cancen determinada emoción, ni a que eliminen alguna en particular; sino que ese desplie-
gue de la afectividad nos habla de encuentros y/o desencuentros con otras/os en formas
bien diversas de atravesar situaciones conflictivas, con ellas/os mismas/os y con el mundo
circundante. Esto impactaría en las formas de convivencia en las escuelas y en el desafío
que supone un aprendizaje que es indisociable de los deseos, temores y malestares que se
suscitan en momentos de conmoción como los vivenciados durante estos tiempos.

Entonces una de nuestras funciones sería la de intermediar, acompañando a que niñas, ni-
ños, adolescentes y jóvenes construyan interpretaciones y posicionamientos propios, con la
prudencia de no apresurarnos frente a la preocupación misma que nos genera, ni intervenir
con preguntas directas sobre lo que sienten y piensan. Es posible, así, validar y reconocer el
tiempo subjetivo en el cual cada persona puede significar lo que sucedió a través del ofreci-
4- Licenciada en
Psicología. Analista miento de representaciones diversas (como la escritura, el arte, el cine, la literatura, el juego,
Miembro de la Escuela de
la Orientación Lacaniana proyectos científicos), en soportes y dispositivos diferentes en pos de promover la tramita-
(EOL). Coordinadora del
equipo de orientación ción y simbolización de lo ocurrido, de forma individual y también colectiva.
psicológica del
bachillerato trans “Mocha
Celis”.

5- Doctor en Psicología,
Profesor Titular de Tenemos que generar propuestas de participación activa de los y las estudiantes para
Psicología Social en las
Facultades de Psicología que transmitan lo que ha sido la experiencia de la pandemia a través de producciones
y Ciencias Sociales artísticas, culturales. Espacios que posibiliten la representación de las niñeces y las
de la Universidad
Nacional de Córdoba adolescencias, que nos permitan corrernos de un lugar adultocéntrico a través del
(UNC) y Coordinador del cual queremos hasta incluso coordinar cómo deberían expresar sus sentimientos, sus
Programa de Promoción
de Convivencia Escolar y emociones y vivencias.
Ciudadanía (PROCONVI),
Facultad de Psicología,
UNC. Horacio Paulin5

10.
2. Subjetividad y escuela, ¿cómo alojar los efectos de la pandemia?

Consideramos a la subjetividad como un proceso de formación de un modo de ser, pensar


y hacer, en vínculo con otras/os dentro de un orden social y discursivo determinado. Por
tanto la subjetividad, no es un estado a alcanzar, definitivo o cerrado. Tampoco se trata de
algo meramente individual, sino de una construcción que se da en la relación con otras/
os, en tiempos y lugares, en donde circulan saberes, objetos de conocimiento, pensamien-
tos, sentires. Así, la subjetividad no es sinónimo de interioridad -algo que le compete sola-
mente a un sujeto- sino que es la resultante de una construcción socio-histórica que habla
de nosotras/os, de nuestras/os estudiantes, pero también de las formas de vincularnos y
las marcas propias de cada época. Dentro de la escuela las formas de subjetivación refie-
ren a un “entre”, que se construye en la trama vincular: docentes-docentes, docentes-es-
tudiantes, estudiantes-estudiantes, docentes-familias, familias-estudiantes. Las infancias
y adolescencias resultan etapas particularmente importantes en la constitución subjetiva
e implican profundos trabajos psíquicos y simbólicos que no podrían desplegarse sin el
acompañamiento de personas adultas, de otras/os pares y de espacios institucionales
como formas de sostén.

Toda época presenta características que dejan marcas y moldean los modos de ser,
vincularse, actuar, sentir, a los que podríamos llamar “subjetividades de época”. Este con-
cepto tiene la potencialidad de referirse tanto a los aspectos singulares de cada persona,
como a los construidos colectivamente. Somos únicas/os, irrepetibles y diversas/os, y a la
vez, portamos marcas de la época en la que vivimos. Sin duda, una de las marcas de época
más preponderante de los últimos tiempos es la situación de pandemia.

La situación epidemiológica de la pandemia ha reeditado sentimientos y sensaciones aso-


ciadas al desamparo estructural de la condición humana. Pérdida o miedo a la pérdida,
incertidumbre, ansiedad, temor, frustración, sensación de ausencia de un futuro posible
son, entre otros, padecimientos de esta experiencia. Niñas, niños, adolescentes y jóvenes
vivieron la interrupción de los vínculos físicos con seres queridos y, al mismo tiempo, la
interrupción de espacios de socialización en la escuela y fuera de ella, entre otras situacio-
nes. Esas experiencias produjeron sufrimientos particulares por las característica del ciclo
vital que se encuentran atravesando.

En algunas zonas de nuestro país, las condiciones de conectividad eran muy


complejas. Las/os docentes para poder conectarse se encontraban con las/os
chicas/os, por ejemplo, en algunos lugares como la plaza del pueblo. Se tuvieron 6- Licenciada y
Profesora en Ciencias de
que generar infinidad de estrategias para llegar a estudiantes de las diferentes la Educación. Docente
zonas en cada provincia. e investigadora en la
Universidad Nacional
de Jujuy, Facultad de
Beatriz Norma Lambrisca6 Humanidades y Ciencias
Sociales.

.11
Durante el año 2020, UNICEF junto a otras organizaciones llevó a cabo un estudio sobre los
efectos de la pandemia en niños, niñas y adolescentes, con el fin de conocer las principales
necesidades emocionales y los cambios en los comportamientos durante ese primer año
de pandemia. A través de la investigación se pudo conocer que:

Algunas/os niñas, niños y adolescentes atravesaron emociones de soledad, tristeza,


ansiedad, miedo.
Quienes estuvieron cercanos a la finalización del ciclo secundario, manifestaron vi-
vencias ligadas a la incertidumbre en relación a proyectos futuros.
En adolescentes de sectores vulnerables la angustia se profundizó debido a las priva-
ciones materiales.
La posibilidad de soñar con un futuro posible se recupera como una cuestión funda-
mental para la construcción identitaria.

El trabajo de indagación permitió reconocer también algunos factores asociados a es-


tos padecimientos como pueden ser deficiencias o insuficiencia del acompañamiento de
personas adultas significativas, déficit habitacional, situaciones de violencia, precaria si-
tuación económica, violencias institucionales, ausencia de espacios donde sentirse escu-
chadas/os y/o comprendidas/os, dificultades o insuficiencia de contacto con la escuela,
distanciamiento en el vínculo con las y los docentes, sentimientos de discriminación y
estigmatización, entre otros.

Pablo di Napoli7, Adriana Goglino8 e Iñaki Bardin9 indagaron sobre los sentires
de jóvenes estudiantes durante la pandemia, y advirtieron que la virtualización
forzada de la escolarización, y las consecuencias que trajo aparejadas, fue una
7- Doctor en Ciencias de las cuestiones a las que más les costó adaptarse. El estudio señala que una
Sociales. Investigador del gran proporción de estudiantes extrañaron fundamentalmente dos momentos de
CONICET en el Instituto
de Investigaciones de la escuela secundaria: la sociabilidad intrageneracional y el vínculo pedagógico
Ciencias de la Educación intergeneracional. En este marco, es sensato pensar que la pérdida de un espacio
(IICE), Facultad de
Filosofía y Letras, físico como la escuela no solo haya aumentado la desigualdad educativa en términos
Universidad de Buenos objetivos, sino también la experiencia subjetiva de dicha desigualdad.
Aires (UBA). Docente de
la UBA.

8- Licenciada en
Ciencias de la Educación.
Integrante de proyectos
de investigación
del Instituto de El informe de UNICEF, publicado en el año 2022, destaca la persistencia de manifestacio-
Investigaciones de
Ciencias de la Educación nes y padecimientos experimentados en la primera infancia pese a la mejora de los indi-
(IICE), Facultad de
Filosofía y Letras, cadores epidemiológicos. Entre ellos, releva alteraciones en la alimentación, en el sueño
Universidad de Buenos
Aires (UBA). Docente de y dificultades en los procesos comunicativos. En relación a las adolescencias, el estudio
la UBA.
señala una mejoría en el estado anímico respecto a los dos años anteriores, una mayor
9- Estudiante de la
carrera de Ciencias motivación frente al estudio y el reconocimiento de la alegría por volver al espacio escolar.
de la Educación de la
Facultad de Filosofía
y Letras (FFyL) de la
Universidad de Buenos Podemos pensar el padecimiento como una respuesta psíquica frente a la conmoción que
Aires (UBA). Integrante
de proyectos de produjo la pandemia y el consecuente distanciamiento social. Ahora bien, ello no significa
investigación del Instituto
de Investigaciones de que necesariamente y en todos los casos este padecimiento se convierta en enfermedad, en
Ciencias de la Educación
(IICE), FFyL, UBA. una patología referida a la salud mental. Evitar patologizar ese sufrimiento contribuye a que

12.
niñas, niños, adolescentes y jóvenes, acompañadas/os por personas adultas referentes y
significativas, lo tramiten de la mejor manera posible.

Alicia Stolkiner10 y Julián Ferreyra11, en el artículo Psicopatologizar la


Cuarentena, señalan que no es adecuado reducir el sufrimiento a cuadros
patológicos. Tampoco tratar de definir causalidades lineales frente a
una situación hipercompleja, por ejemplo, asignar todo el malestar a las
restricciones y medidas que se toman para prevenir la expansión del contagio,
o a la falta de certezas que nadie puede garantizar. Analizar las dimensiones
subjetivas y la posible producción de sufrimiento “psíquico” requiere de un
abordaje complejo que incluya la articulación entre lo económico, lo institucional
y los dispositivos propios de la vida de los sujetos singulares.

Si bien el atravesamiento por la pandemia ha sido global, existen formas singulares de vi-
virlo. Conocerlas, pensarlas, analizarlas, con el cuidado de no llevar a cabo generalizaciones
“a todas/os nos pasó lo mismo” o de establecer enunciados patologizantes “las/os estudiantes
se encuentran deprimidas/os”, será parte de nuestra tarea y de las formas de acompañar.
Esto equivale a alojar los efectos de los malestares, como un llamado a pensar en las sub-
jetividades contemporáneas dentro de este entramado, y lejos de posiciones esencialistas y
patologizantes.

Los procesos de subjetivación suceden mientras aprendemos, nos vinculamos, nos equi-
vocamos y nos recomponemos; y los conocimientos y saberes circulan, con palabras,
pensamientos, afectividades que se van construyendo en ese tiempo suspendido (Simons
M., Masschelein, 2014) que es la escuela. Tiempo “suspendido” porque se diferencia y se
distancia del tiempo de la producción y de la exigencia del mercado, poniendo una distin-
ción temporal diferente que no se corresponde con la aceleración consumista y con las
formas de desigualdad. En este horizonte, la escuela puede funcionar como ese tiempo
indispensable de formación en donde hacer parte a todas y todos desde una perspectiva
de igualdad.

3. Colocar la pandemia en la historia

Un acontecimiento como la pandemia merece un análisis que nos ayude a encontrar he- 10- Licenciada en
Psicología. Doctora
rramientas para elaborar lo vivido. Lo que sucedió nos ha conmovido de formas diversas Honoris Causa de la
Universidad Autónoma
y singulares. Si bien actualmente hemos retomado muchas de nuestras rutinas cotidianas de Entre Ríos (UADER).
Profesora Titular Regular
previas al inicio de la pandemia, el regreso a la presencialidad en las escuelas no sucedió de Salud Pública y Salud
Mental de la Facultad de
del mismo modo al que estábamos acostumbradas/os. El reinicio en burbujas, dar clases Psicología (UBA) (1985-
2021).
con barbijo, los escenarios híbridos que se fueron configurando, las demandas de las fami-
11- Licenciado en
lias frente a situaciones de contagio, impactaron en las formas de hacer escuela, al mismo Psicología (UBA).
Psicoanalista. Becario
tiempo que visibilizaron diversos padecimientos de las/os estudiantes con los que tuvimos Doctoral CONICET.
Docente en Salud
y tenemos que trabajar y aprender a contener. Pública/Mental II, UBA.

.13
Creo que hay un trabajo de colocar en la historia a la pandemia, eso que aparece
como un acontecimiento, hay que empezar a darle una inteligibilidad histórica
porque la pandemia no es solamente el virus que irrumpe, es un modo de vivirlo, y
tiene un montón de continuidades que ya empezamos a verlas.

Tenemos que empezar a entramarla con lo que estaba antes, con lo que viene
después. (...) No fue una ruptura en el continuum de la historia (...) el gran desafío
ahora es poder pensar lo que sucedió en relación con la pandemia; lo que ha hecho
visible, lo que trajo de nuevo.

Miriam Kriger12

Colocar la pandemia en la historia deviene en una doble necesidad. Por un lado, colocar a
la pandemia en la historia singular de cada quién supone darle una inteligibilidad a lo acon-
tecido en la historia personal, hacerle un lugar y entramarla con el resto de las vivencias,
para poder seguir adelante teniendo en cuenta lo sucedido; porque la pandemia no es
solamente un virus que irrumpe y trastoca la vida, sino que implica encontrar un modo de
transitar aquello que sucedió haciéndolo parte y pensando cómo continuar. Por otro lado,
colocar la pandemia en la historia implica también construir una historia en común, generando
condiciones de resignificación o de significación de la experiencia, de habilitar espacios de
producción de sentido colectivos políticos, sociales, que vuelvan posible a los sujetos posi-
cionarse de un modo activo y solidario frente a los tiempos que nos toca atravesar.

Tramitar socialmente los procesos de duelo colectivos posibilita colocar el proceso de duelo
individual en la esfera de lo público, comprender que se trata de un fenómeno que afecta a
toda la sociedad y asimilar que entraña un dolor colectivo que requiere ser elaborado de for-
ma conjunta. Lo que implica reconocer diversas formas de padecimientos y la aceptación de
que aquellas experiencias dolorosas no culminan de un día para el otro, sino que forman par-
te de un proceso. Dar lugar al dolor en términos colectivos, no solo contribuye a reconstruir
los lazos sociales que indudablemente se vieron afectados por la pandemia sino que también
es necesario para imaginar y construir un proyecto como sociedad presente y futura.

En el caso de quienes egresaron en la pandemia, las/os estudiantes atravesaron


además de otros padecimientos, el dolor por la pérdida de rituales y la resignación
de encuentros vinculados a la finalización de una etapa.
12- Doctora en Ciencias
Sociales. Investigadora Sin embargo, hubo iniciativas que abordaron esta situación tan particular
adjunta del CONICET y promoviendo la posibilidad de que estudiantes puedan resignificar lo sucedido en
docente investigadora
de la Universidad de ese tiempo, recuperar la trayectoria escolar y realizar un cierre colectivo.
Buenos Aires (UBA).
Directora de proyectos
de investigación (ANCyT, Recuperamos, una de ellas, titulada La generación que egresó en pandemia en el
CONICET y UBA), sobre Apartado IV de este material.
jóvenes, cultura y política,
en entornos nacionales.

14.
Podríamos decir, entonces, que la potencia de la escuela está en la producción de relatos
que no son sólo individuales sino también colectivos. Colocar la pandemia en la historia
en esta doble vertiente (en la dimensión sociopolítica y en la dimensión de la trayectoria
personal) implica restituir a las/os estudiantes en un lugar activo, propiciando el armado
de narrativas desde las cuales situarse, brindando sentidos frente a una realidad que apa-
reció intempestivamente y nos llena de incertidumbre.

La pandemia fue un agujero social en cada uno/a. Estamos intentando producir


sentidos. Generar espacios para dar la palabra es fundamental pero me preocupa
que quede restringido al campo de lo privado. Las construcciones, los espacios,
los dispositivos tienen que permitir la producción de sentidos colectivos, políticos,
sociales, tienen que dar condiciones de significación. Por ejemplo, hablar del
capitalismo, hablar del cuidado, del ambiente en relación a estos tiempos, ahí
entonces está entramar con lo sucedido.

Me parece que el mayor riesgo (...) para cualquier acompañamiento que uno piense,
es que quede desligado de la enseñanza misma, porque los recursos subjetivos
que aporta la escuela están en relación a los recursos simbólicos que aporta la
enseñanza.
Laura Kiel13

4. De duelos y transformaciones

Reconocer que algo se perdió me parece fundamental -no sólo las pérdidas de vidas que
a cada una/o nos han afectado de distinto modo, sino también prácticas y modalidades
de encuentros, rutinas y hábitos-, nos deja energía disponible para investir -desear-
otros objetos. Genera un poco de escozor decir que tenemos que hacer duelo por lo que
ya no es, pero yo apuesto a eso, el duelo es necesario como en cualquier pérdida, no
necesariamente la muerte.

María Eugenia Cora14

Resulta necesario reconocer que algo se perdió. Se han perdido vidas, pero también prácti- 13- Licenciada en
Psicología. Integrante
cas y modalidades de encuentros; se han perdido rutinas y hábitos; se han resignado expec- de proyectos de
investigación: UBACyT,
tativas, deseos y proyectos. Hay estudiantes que no pudieron llevar a cabo su ritos de pasaje Conicet y Agencia
Nacional para la
de nivel educativo, hay cumpleaños que no se han podido festejar de la forma que se venían Promoción Científica y
Tecnológica.
haciendo, situaciones significativas a las que hubo que renunciar. Como en toda pérdida, la
14- Licenciada en
afectación es singular, sin embargo, las niñeces y adolescencias merecen especial acompa- Psicología. Analista
miembro de la Escuela
ñamiento para poder realizar un trabajo de duelo sobre aquello que se perdió. La pérdida, de la Orientación
Lacaniana (EOL) y de
en cuanto tal, necesita ser sostenida y elaborada. Poder reconocer esa pérdida, procesarla, la Asociación Mundial
de Psicoanálisis
simbolizarla y/o nombrarla, permite generar condiciones de posibilidad para que esa energía (AMP). Docente de la
Universidad de Buenos
se encuentre disponible y pueda reubicarse en otros espacios, propuestas, actividades. Aires.

.15
...hay una dificultad muy grande para poder sentirse acompañada/o en el desafío
que implica elaborar duelos. Tenemos más de cien mil muertos. Tenemos una
dificultad para pensar lo traumático de determinadas pérdidas que se dieron sin
contacto, sin cercanía, sin encuentro. Son cuestiones que ahora se abren y recién
van a poder empezar a emerger en todas las modalidades de sufrimiento y de los
sentimientos.

Patricia Álvarez15

En ese sentido, la tramitación de la pérdida -siendo reconocida y no negada- nos permite


ir re-conectando con los proyectos, con el deseo. ¿Cómo acompañar la elaboración de la
pérdida en la escuela?

La cercanía de la muerte ha sido permanente y lo es todavía. Yo creo que aún no


nos ha caído la ficha, porque la pérdida en estos tiempos es tremenda. Parece que
hubiera algo de la muerte que se quiere negar. El tema de no querer saber nada
de “eso” está presente, lo constato permanentemente. Entonces, se hace bastante
difícil de pensar qué efectos dejará la pandemia en las subjetividades, más allá de
las secuelas orgánicas.

Mario Goldemberg16

Dentro del trabajo de elaboración de las pérdidas, el hecho de generar preguntas y/o pro-
blematizaciones -que no apunten a una respuesta única- resulta fundamental. Pensar qué
interrogantes pueden incluirse dentro del espacio del aula y formular problemas es clave
para permitir aproximarnos a lo que ya conocemos, pero también a lo que no terminamos
de entender.

Nuestro desafío tiene que ver con reponer sentidos, re-encarar la tarea educativa, ima-
ginar otros modos de enseñar y aprender y convivir, y entramar lo que sucedió con el
15- Doctora en
Psicología. Psicoanalista. futuro. Así, tras este trabajo compartido, de elaboración de la pérdida y simbolización, será
Profesora Facultad
de Psicología UBA. posible generar condiciones para el despliegue de la subjetividad fomentando formas de
Directora del Proyecto
de investigación vincularnos y de aprender de otros modos.
UBACyT: “Procesos
subjetivos y simbólicos
contemporáneos
comprometidos Historizar, entramar, simbolizar, tramitar lo que sucedió, ponerle palabras, requiere de la
en el aprendizaje
escolar: Nuevas articulación conjunta de la comunidad educativa. Para que estos procesos tengan lugar,
conceptualizaciones e
Intervenciones”. cada institución tendrá que configurar diversos dispositivos institucionales en función de
16- Psicoanalista. pensar qué proyectos, propuestas, estrategias pueden ser viables de crear en pos de que
Analista miembro
de la Escuela de la esta tarea se lleve adelante entre todas/os, y no quede circunscripta a la “buena voluntad”
Orientación Lacaniana
(EOL) y de la Asociación de un/a docente en soledad.
Mundial de Psicoanálisis
(AMP), docente de la
Universidad de Buenos
Aires.

16.
5. El cuidado como respuesta colectiva

Es común escuchar hablar de los cuidados en aspectos bien diversos. Se hace mención, por
ejemplo, al cuidado de la salud en referencia a los distintos modos de construir y promover
el bienestar, tanto propio como de las/os demás. Se habla también de cuidado en referencia
a las tareas del hogar, la crianza y los roles de género asociados a esas actividades. A veces
el cuidado funciona como sinónimo de la palabra seguridad, y otras se vincula al cuidado del
ambiente. Podemos ver que el cuidado resulta un concepto polisémico y, como muchos otros,
en continuo proceso de construcción teórica. Históricamente, en la escuela la idea de cuidado
estuvo ligada al concepto de prevención, atravesada por los enfoques higienistas, biologicis-
tas y sanitarios. Lejos de ser paradigmas superados, estos discursos persisten en posiciones
acerca de la función social de la escuela, los contenidos a ser transmitidos, la concepción de
los sujetos de aprendizaje y la construcción del vínculo pedagógico.

Hoy, el concepto de cuidado circula cada vez más en nuestra sociedad y ha tomado especial
lugar desde el comienzo de la pandemia en los discursos sociales, políticos, mediáticos,
educativos. Más allá de sus múltiples significaciones, coincidimos en que es un elemento
central del bienestar humano. No hay nadie que pueda sobrevivir sin ser cuidada/o o sin
haberla/o sido Se trata de una actividad que realizamos para mantener, reparar y conser-
var nuestro mundo, nuestro entorno, nuestros cuerpos, nuestras subjetividades. En este
sentido, las tareas de cuidado no se limitan a cuidados físicos, básicos y vitales, como ali-
mentar y dar refugio, sino que cuidar implica también una dimensión afectiva que entraña
el escuchar, acompañar, habilitar, preocuparse por la/el otra/o, confiar en lo que puede. El
cuidado se configura, de este modo, como una categoría central para la vida cotidiana e
interpela al ámbito educativo animándonos a pensar: ¿Cómo construimos una pedagogía
del cuidado en las escuelas?

La relación entre enseñar y cuidar fue uno de los puntos que más conmovieron a
las instituciones escolares, porque junto con lo pedagógico durante el 2020 y 2021
también estaba esta preocupación por acercar elementos que tuvieran que ver con
resolver situaciones de vulnerabilidad en sus diversos sentidos. La pandemia iba a
ser una cuestión transitoria, pero hubo un momento en que su extensión generó la
necesidad de trabajar activamente para sostener el vínculo y acompañar situaciones
y vivencias complejas.

Beatriz Celada17

Pensar prácticas y vínculos en la escuela desde un enfoque de cuidado requiere fortalecer la


construcción de espacios democráticos, dialógicos y participativos, que propicien la escucha
y la colaboración priorizando el protagonismo de niñas, niños, adolescentes y jóvenes.

La escuela cuida cuando enseña, no solo desde los contenidos sino también a través de 17- Doctora en
Educación. Investigadora
los vínculos y espacios que se constituyen a partir del entramado institucional y que forman en la Universidad del
Comahue. Directora
de Proyectos de
Investigación IPEHCS-
CONICET-UNCo.

.17
parte de la vida cotidiana en las escuelas. En este sentido dar lugar y alojar las vivencias de
cada estudiante se transforma en una oportunidad de aprendizaje para la convivencia con
otras y otras.

Alejandra Barcala18, directora de la investigación publicada en UNICEF Salud Mental


en tiempos de coronavirus, sostiene que si bien no encontramos una infancia y
adolescencia unívoca -claramente no fue igual la vivencia de la cuarentena en
función del género, el territorio, la etnia o la situación socioeconómica- hubo
algo que atravesó a todas y a todos durante el 2020 que fue la necesidad de ser
escuchadas/os. El estudio, entre diversas propuestas que plantea, apuesta a
construir dentro de la escuela espacios que promuevan la organización colectiva
habilitando el ejercicio de una ciudadanía activa.

Considerar la pandemia desde una pedagogía de cuidado implica tener presente que exis-
ten múltiples formas de abordaje sobre lo que sucedió en estos tiempos y que, de acuerdo
al momento, al espacio y al nivel educativo, a veces será necesario ubicar la pandemia
en el centro, otras veces acercarse desde los bordes y, en otros casos, implica generar
espacios para descentrarnos de ella. De este modo, se espera que puedan brindarse opor-
tunidades diversas para significar el mundo, construir narrativas en torno a lo vivido y pro-
mover proyectos a futuro. Teniendo esto en cuenta habrá instancias en las cuales, desde
las instituciones, busquemos nombrar la pandemia intentando abrir sentidos en relación
a ella, tomándola como objeto de reflexión y siendo los saberes curriculares una opor-
tunidad para ello. Así, convocar a las/os estudiantes a trabajar sobre las problemáticas
ambientales o el uso de tecnologías en la vida cotidiana, por ejemplo, serán espacios a
partir de los cuales la pandemia y las transformaciones sociales y subjetivas constituyan
una oportunidad para darle lugar a proyectos interesantes y sugerentes que permitan el
despliegue de nuevos saberes sobre lo ocurrido. A su vez, al igual que en otros procesos
de duelo y pérdida, la pandemia puede ponerse momentáneamente en suspensión, ofre-
ciendo el acercamiento de otros saberes y experiencias que den lugar al deseo de nuevos
objetos de conocimiento, lo cual también resulta necesario para poder tramitarla. De esta
manera, se trataría de generar propuestas en las que niñas, niños, adolescentes y jóvenes
se sientan convocadas/os a través de sus opiniones, intereses y anhelos. Aquí proyectos
ligados al deporte, las expresiones artísticas, experiencias recreativas, toman centralidad
en tanto se transforman en oportunidades donde compartir con otras y otros; vincularse y
comunicarse de formas diversas.

6. Promover la corresponsabilidad
18- Postdoctora en
Ciencias Sociales,
Niñez y Juventud. Otra de las realidades que visibilizó fuertemente la pandemia es que la escuela no puede
Investigadora, directora
de investigaciones, concebirse como una institución aislada, ya que forma parte de un Sistema Integral de
coordinadora de equipos
interdisciplinarios. Protección de Derechos. Por ello, resulta necesario consolidar un trabajo de articulación

18.
permanente con otros actores, instituciones y sectores, a fin de favorecer la garantía de
derechos de niñas, niños, adolescentes y jóvenes.

La ley de Protección Integral de los Derechos de las/os niñas, niños y adolescentes


implicó un viraje respecto a la cuestión de corresponsabilidad en términos de quiénes
son garantes del cumplimiento y el ejercicio de los derechos de niñas, niños y
adolescentes. Desde esta perspectiva, se promueve la labor coordinada entre quienes
se ocupan de la salud, la educación, la seguridad y el bienestar de las/os niñas, niños,
adolescentes y jóvenes, ya que se considera que sus derechos son universales e
indivisibles: universales -porque alcanzan a toda/o niña, niño y adolescente que habite
el suelo argentino- e indivisibles -porque la niña, el niño o la/el adolescente que estudia
es la/el misma/o que necesita amor, alimento, salud y contención-.

Pensar la escuela como parte de una red requiere considerar el principio de corresponsa-
bilidad, para garantizar los derechos de niñas, niños y adolescentes sin perder la especifi-
cidad de cada sector y la contracara del abandono que supone la fragmentación institucio-
nal. Estas articulaciones no surgen espontáneamente y muchas veces requieren de largos
caminos de conocimiento mutuo, que no están exentos de tensiones ya que cada espacio
institucional persigue finalidades diferentes y posee lógicas propias.

Para profundizar sobre esta temática, invitamos a explorar el material


Derechos de Niños, Niñas y Adolescentes. Se trata de una propuesta que invita
la incorporación de este enfoque en nuestras prácticas docentes considerando
a nuestras/os estudiantes como sujetos activos, con quienes trabajamos
cotidianamente para construir ciudadanía en el mundo contemporáneo.

Para que esta red pueda consolidarse es necesario reconocer quiénes son las personas que
se encuentran detrás de los aparatos institucionales. Durante la pandemia los vínculos con
las familias, con los centros de salud, con distintas instituciones del Estado y organizaciones
comunitarias, fueron una parte fundamental del trabajo cotidiano en las escuelas y tuvieron
un lugar central en el sostenimiento de las trayectorias educativas. En muchas ocasiones, la
pandemia nos obligó a realizar un nuevo “reconocimiento” entre las/os distintas/os actores
en función de las nuevas coordenadas. Podemos situar entre ellas, la relación entre familias
y docentes, donde en algunos casos se generaron situaciones de mucha cercanía, y el vínculo
de las escuelas con espacios de salud que se resignificó a partir de la instauración de medi-
das sanitarias para la prevención del contagio. Sin dudas, estas articulaciones requieren un
trabajo sistemático y sostenido en función de pensar estrategias en conjunto.

.19
La apuesta del vínculo salud y educación reside, entre otras cosas, en generar espacios
de co-construcción. Así, sobre temas que siempre ocupan y preocupan a los y las
docentes en las escuelas -y que muchas veces se sienten como una carga porque no
encuentran las herramientas suficientes- se transforman en una oportunidad para pensar
en un abordaje conjunto que incluya una propuesta integral.

Ana Clara Camarotti19

El acompañamiento desde una pedagogía del cuidado implica entonces un trabajo manco-
munado, con un criterio de corresponsabilidad, entre diversas personas, espacios e ins-
tituciones en pos de construir redes que sostengan y promuevan los derechos de niñas,
niños, adolescentes y jóvenes. No solo realizando abordajes discursivos sino generando
espacios para que los derechos se puedan reconocer y ejercer.

7. La ESI como herramienta

Pensar la escuela en tiempos de pandemia y desde una pedagogįa del cuidado, nos lleva
a tener en cuenta la inclusión y la transversalidad de la Educación Sexual Integral (ESI),
lo que involucra la dimensión curricular y el resto de las prácticas institucionales de la
escuela.

Cabe mencionar que las situaciones de violencias por motivos de género y familiares se
acentuaron durante el periodo de confinamiento. Según datos del Ministerio de Mujeres,
Género y Diversidad de la Nación, durante el contexto de aislamiento social, preventivo y
obligatorio, han aumentado significativamente.

Los cinco ejes conceptuales de la ESI: cuidar el cuerpo y la salud; valorar la afectividad; ga-
rantizar la equidad de género; respetar la diversidad y ejercer los derechos han constituido
19- Doctora en Ciencias desde su creación una herramienta conceptual del Programa Nacional (PNESI) elaborada
Sociales. Investigadora
independiente para poder garantizar la integralidad que prescribe la Ley N 26.150. De este modo, se visi-
del CONICET. Co-
coordinadora del Área biliza las distintas dimensiones que articulan los contenidos de la ESI, donde los aspectos
de Salud y Población
e Investigadora biológicos, psicológicos, sociales, afectivos y éticos se entrelazan y por lo tanto el abordaje
del Instituto de
Investigaciones integral es fundamental. En tiempos de pandemia, es posible (re) pensar los cinco ejes
Gino Germani. Es
consultora externa de la como núcleos a ser trabajados y desarrollados en el marco de lo que nos proponemos
Organización de Estados
Americanos. pensar en estas páginas.

20.
LXXVIII

SOBRE LA PSICOLOGÍA DEL COLEGIAL[1667]

1914

EXTRAÑO sentimiento le embarga a uno cuando en años tan avanzados de


la vida se ve una vez más en el trance de tener que redactar una «composición de
idioma alemán» para el colegio. No obstante, se obedece automáticamente, como
aquel viejo soldado licenciado de filas que al oír la orden de «¡firmes!» no puede
menos de llevar las manos a la faltriquera, dejando caer al suelo sus bártulos. Es
curioso el buen grado con que acepto la tarea, cual si durante el último medio siglo
nada importante hubiera cambiado. Sin embargo, he envejecido en este lapso; me
encuentro a punto de llegar a sexagenario, y tanto las sensaciones de mi cuerpo
como el espejo me muestran inequívocamente cuán considerable es la parte de mi
llama vital que ya se ha consumido.

Hace unos diez años aún podía tener instantes en que de pronto volvía a
sentirme completamente joven. Cuando, ya barbicano y cargado con todo el peso
de una existencia burguesa, caminaba por las calles de la ciudad natal, podía
suceder que me topara inesperadamente con uno u otro caballero anciano, pero
bien conservado, al que saludaba casi humildemente, reconociendo en él a un
antiguo profesor del colegio. Pero luego me detenía y, ensimismado, lo seguía con
la mirada: ¿Realmente es él, o sólo alguien que se le asemeja a punto de confusión?
¡Cuán joven parece aún, y tú ya estás tan viejo! ¿Cuántos años podrá contar? ¿Es
posible que estos hombres, que otrora representaron para nosotros a los adultos,
sólo fuesen tan poco más viejos que nosotros?

El presente quedaba entonces como oscurecido ante mis ojos, y los años de
los diez a los dieciocho volvían a surgir de los recovecos de la memoria, con todos
sus presentimientos y desvaríos, sus dolorosas trasmutaciones y sus éxitos
jubilosos, con los primeros atisbos de culturas desaparecidas —un mundo que,
para mí al menos, llegó a ser más tarde un insuperable medio de consuelo ante las
luchas de la vida—; por fin, surgían también los primeros contactos con las
ciencias, entre las cuales creíamos poder elegir aquélla que agraciaríamos con
nuestros por cierto inapreciables servicios. Y yo creo recordar que durante toda esa
época abrigué la vaga premonición de una tarea que al principio sólo se anunció
calladamente, hasta que por fin la pude vestir, en mi composición de bachillerato,
con las solemnes palabras de que en mi vida querría rendir un aporte al humano
saber.
Llegué, pues, a médico, o más propiamente a psicólogo, y pude crear una
nueva disciplina psicológica —el denominado «psicoanálisis»— que hoy embarga
la atención y suscita alabanzas y censuras de médicos e investigadores oriundos de
los más lejanos países, aunque, desde luego, preocupa mucho menos a los de mi
propia patria.

Como psicoanalista, debo interesarme más por los procesos afectivos que
por los intelectuales; más por la vida psíquica inconsciente que por la consciente.
La emoción experimentada al encontrarme con mi antiguo profesor del colegio me
conmina a una primera confesión: no sé qué nos embargó más y qué fue más
importante para nosotros: si la labor con las ciencias que nos exponían o la
preocupación con las personalidades de nuestros profesores. En todo caso, con
éstos nos unía una corriente subterránea jamás interrumpida, y en muchos de
nosotros el camino a la ciencia sólo pudo pasar por las personas de los profesores:
muchos quedaron detenidos en este camino y a unos pocos —¿por qué no
confesarlo?— se les cerró así para siempre.

Los cortejábamos o nos apartábamos de ellos; imaginábamos su


probablemente inexistente simpatía o antipatía; estudiábamos sus caracteres y
formábamos o deformábamos los nuestros, tomándolos como modelos.
Despertaban nuestras más potentes rebeliones y nos obligaban a un sometimiento
completo; atisbábamos sus más pequeñas debilidades y estábamos orgullosos de
sus virtudes, de su sapiencia y su justicia. En el fondo, los amábamos
entrañablemente cuando nos daban el menor motivo para ello; mas no sé si todos
nuestros maestros lo advirtieron. Pero no es posible negar que teníamos una
particularísima animosidad contra ellos, que bien puede haber sido incómoda para
los afectados. Desde un principio tendíamos por igual al amor y al odio, a la crítica
y a la veneración. El psicoanálisis llama «ambivalente» a esta propensión por las
actitudes antagónicas; tampoco se ve en aprietos al tratar de demostrar el origen de
semejante ambivalencia afectiva.

En efecto, nos ha enseñado que las actitudes afectivas frente a otras


personas, actitudes tan importantes para la conducta ulterior del individuo,
quedan establecidas en una época increíblemente temprana. Ya en los primeros
seis años de la infancia el pequeño ser humano ha fijado de una vez por todas la
forma y el tono afectivo de sus relaciones con los individuos del sexo propio y del
opuesto; a partir de ese momento podrá desarrollarlas y orientarlas en distintos
sentidos, pero ya no logrará abandonarlas. Las personas a las cuales se ha fijado de
tal manera son sus padres y sus hermanos. Todos los hombres que haya de conocer
posteriormente serán, para él, personajes sustitutivos de estos primeros objetos
afectivos (quizá, junto a los padres, también los personajes educadores), y los
ordenará en series que parten, todas, de las denominadas imágenes del padre, de la
madre, de los hermanos, etc. Estas relaciones ulteriores asumen, pues, una especie
de herencia afectiva, tropiezan con simpatías y antipatías en cuya producción
escasamente han participado; todas las amistades y vinculaciones amorosas
ulteriores son seleccionadas sobre la base de las huellas mnemónicas que cada uno
de aquellos modelos primitivos haya dejado.

Pero de todas las imágenes de la infancia, por lo general extinguidas ya en la


memoria, ninguna tiene para el adolescente y para el hombre mayor importancia
que la del padre. El imperio de lo orgánico ha impuesto a esta relación con el padre
una ambivalencia afectiva cuya manifestación más impresionante quizá sea el mito
griego del rey Edipo. El niño pequeño se ve obligado a amar y admirar a su padre,
pues éste le parece el más fuerte, bondadoso y sabio de todos los seres; la propia
figura de Dios no es sino una exaltación de esta imago paterna, tal como se da en la
más precoz vida psíquica infantil. Pero muy pronto se manifiesta el cariz opuesto
de tal relación afectiva. El padre también es identificado como el todopoderoso
perturbador de la propia vida instintiva; se convierte en el modelo que no sólo se
querría imitar, sino también destruir para ocupar su propia plaza. Las tendencias
cariñosas y hostiles contra el padre subsisten juntas, muchas veces durante toda la
vida, sin que la una logre superar a la otra. En esta simultaneidad de las antítesis
reside la esencia de lo que denominamos «ambivalencia afectiva».

En la segunda mitad de la infancia se prepara un cambio de esta relación con


el padre, cambio cuya magnitud no es posible exagerar. El niño comienza a salir de
su cuarto de juegos para contemplar el mundo real que lo rodea, y debe descubrir
entonces cosas que minan la primitiva exaltación del padre y que facilitan el
abandono de este primer personaje ideal. Comprueba que el padre ya no es el más
poderoso, el más sabio y el más acaudalado de los seres; comienza a dejar de estar
conforme con él; aprende a criticarle y a situarle en la escala social, y suele hacerle
pagar muy cara la decepción que le produjera. Todas las esperanzas que ofrece la
nueva generación —pero también todo lo condenable que presenta— se originan
en este apartamiento del padre.

En esta fase evolutiva del joven hombre acaece su encuentro con los
maestros. Comprenderemos ahora la actitud que adoptamos ante nuestros
profesores del colegio. Estos hombres, que ni siquiera eran todos padres de familia,
se convirtieron para nosotros en sustitutos del padre. También es ésta la causa de
que, por más jóvenes que fuesen, nos parecieran tan maduros, tan remotamente
adultos. Nosotros les transferíamos el respeto y la veneración ante el omnisapiente
padre de nuestros años infantiles, de manera que caíamos en tratarlos como a
nuestros propios padres. Les ofrecíamos la ambivalencia que adquiriéramos en la
vida familiar, y con ayuda de esta actitud luchábamos con ellos como habíamos
luchado con nuestros padres carnales. Nuestra conducta frente a nuestros maestros
no podría ser comprendida, ni tampoco justificada, sin considerar los años de la
infancia y el hogar paterno.

Pero como colegiales también tuvimos otras experiencias no menos


importantes con los sucesores de nuestros hermanos, es decir, con nuestros
compañeros. Estas empero han de quedar para otra ocasión, pues el jubileo del
colegio orienta hacia los maestros la totalidad de nuestros pensamientos.
W
ha
ts
ap Ale
p: jan
35 dr
17 ía
61 Im

- 215 -
17 pre
27 si
- 3 one
51 s
76
11
63
2
W
ha
ts
ap Ale
p: jan
35 dr
17 ía
61 Im

- 216 -
17 pre
27 si
- 3 one
51 s
76
11
63
2
W
ha
ts
ap Ale
p: jan
35 dr
17 ía
61 Im

- 217 -
17 pre
27 si
- 3 one
51 s
76
11
63
2
W
ha
ts
ap Ale
p: jan
35 dr
17 ía
61 Im

- 218 -
17 pre
27 si
- 3 one
51 s
76
11
63
2
W
ha
ts
ap Ale
p: jan
35 dr
17 ía
61 Im

- 219 -
17 pre
27 si
- 3 one
51 s
76
11
63
2
W
ha
ts
ap Ale
p: jan
35 dr
17 ía
61 Im

- 220 -
17 pre
27 si
- 3 one
51 s
76
11
63
2
W
ha
ts A
ap le
p: jan
35 dr
17 ía
61 Im

- 221 -
17 pre
27 si
- 3 one
51 s
76
11
63
2
W
ha
ts A
ap le
p: jan
35 dr
17 ía
61 Im

- 222 -
17 pre
27 si
- 3 one
51 s
76
11
63
2
W
ha
ts A
ap le
p: jan
35 dr
17 ía
61 Im

- 223 -
17 pre
27 si
- 3 one
51 s
76
11
63
2
W
ha
ts A
ap le
p: jan
35 dr
17 ía
61 Im

- 224 -
17 pre
27 si
- 3 one
51 s
76
11
63
2
W
ha
ts A
ap le
p: jan
35 dr
17 ía
61 Im

- 225 -
17 pre
27 si
- 3 one
51 s
76
11
63
2
W
ha
ts A
ap le
p: jan
35 dr
17 ía
61 Im

- 226 -
17 pre
27 si
- 3 one
51 s
76
11
63
2
W
ha
ts A
ap le
p: jan
35 dr
17 ía
61 Im

- 227 -
17 pre
27 si
- 3 one
51 s
76
11
63
2
W
ha
ts A
ap le
p: jan
35 dr
17 ía
61 Im

- 228 -
17 pre
27 si
- 3 one
51 s
76
11
63
2
W
ha
ts A
ap le
p: jan
35 dr
17 ía
61 Im

- 229 -
17 pre
27 si
- 3 one
51 s
76
11
63
2
W
ha
ts A
ap le
p: jan
35 dr
17 ía
61 Im

- 230 -
17 pre
27 si
- 3 one
51 s
76
11
63
2
W
ha
ts A
ap le
p: jan
35 dr
17 ía
61 Im

- 231 -
17 pre
27 si
- 3 one
51 s
76
11
63
2
W
ha
ts A
ap le
p: jan
35 dr
17 ía
61 Im

- 232 -
17 pre
27 si
- 3 one
51 s
76
11
63
2
W
ha
ts A
ap le
p: jan
35 dr
17 ía
61 Im

- 233 -
17 pre
27 si
- 3 one
51 s
76
11
63
2
W
ha
ts A
ap le
p: jan
35 dr
17 ía
61 Im

- 234 -
17 pre
27 si
- 3 one
51 s
76
11
63
2
W
ha
ts A
ap le
p: jan
35 dr
17 ía
61 Im

- 235 -
17 pre
27 si
- 3 one
51 s
76
11
63
2
W
ha
ts A
ap le
p: jan
35 dr
17 ía
61 Im

- 236 -
17 pre
27 si
- 3 one
51 s
76
11
63
2
W
ha
ts A
ap le
p: jan
35 dr
17 ía
61 Im

- 237 -
17 pre
27 si
- 3 one
51 s
76
11
63
2
W
ha
ts A
ap le
p: jan
35 dr
17 ía
61 Im

- 238 -
17 pre
27 si
- 3 one
51 s
76
11
63
2
W
ha
ts A
ap le
p: jan
35 dr
17 ía
61 Im

- 239 -
17 pre
27 si
- 3 one
51 s
76
11
63
2
W
ha
ts A
ap le
p: jan
35 dr
17 ía
61 Im

- 240 -
17 pre
27 si
- 3 one
51 s
76
11
63
2
W
ha
ts A
ap le
p: jan
35 dr
17 ía
61 Im

- 241 -
17 pre
27 si
- 3 one
51 s
76
11
63
2
W
ha
ts A
ap le
p: jan
35 dr
17 ía
61 Im

- 242 -
17 pre
27 si
- 3 one
51 s
76
11
63
2
W
ha
ts A
ap le
p: jan
35 dr
17 ía
61 Im

- 243 -
17 pre
27 si
- 3 one
51 s
76
11
63
2
W
ha
ts A
ap le
p: jan
35 dr
17 ía
61 Im

- 244 -
17 pre
27 si
- 3 one
51 s
76
11
63
2
W
ha
ts A
ap le
p: jan
35 dr
17 ía
61 Im

- 245 -
17 pre
27 si
- 3 one
51 s
76
11
63
2
W
ha
ts A
ap le
p: jan
35 dr
17 ía
61 Im

- 246 -
17 pre
27 si
- 3 one
51 s
76
11
63
2
W
ha
ts A
ap le
p: jan
35 dr
17 ía
61 Im

- 247 -
17 pre
27 si
- 3 one
51 s
76
11
63
2
SOUTHWELL- Docentes: La tarea de cruzar fronteras y tender puentes
Propósito del texto: Reflexionar sobre cómo está cambiando el oficio docente, tomando
como punto de partida tanto las transformaciones en la estructura del sistema educativo y
las nuevas pedagogías como los cambios sociales, culturales y políticos que envuelven a
las escuelas. Se nos abre, así, la necesidad de pensar el presente y plantear nuevas
pedagogías en diálogo con la sociedad en la que vivimos.
Enseñar: Establecer una relación, es decir, construir una posición que no está situada en
coordenadas predefinidas, fijas y definitivas sino que sufre alteraciones y busca e inventa
respuestas. Esa relación se establece con la cultura, el poder, los saberes y las formas de
su enseñanza; una relación con los otros y lo que ellos generan en uno, con la política y la
sociedad, con el mundo del trabajo y las múltiples estrategias que desarrollamos para
ubicarnos en él.
Una relación con la cultura, una autoridad cultural.
Lo que funda el sentido del trabajo de enseñar es la relación con la cultura, esto es, la
relación propia y la que propiciamos para los otros.
La relación pedagógica: es una relación asimétrica y es necesaria
El Estado y las instituciones: establecen formas de autoridad; el conocimiento científico;
un docente esforzándose por
desarrollar puentes que no sólo son con su saber específico sino también con la sociedad
en la que vivimos y en la que queremos vivir, construye una autoridad.
Escuela moderna:
-Escuela como renovadora y transformadora de la sociedad
-Límites entre el adentro y el afuera, rígidamente marcados
-Docencia como trabajo individual, figura fuerte que encarna una autoridad superior
Escuela actual:
-Hay un declive de las instituciones
-Se hace imposible demarcar el adentro del afuera
-Docencia como profesión

Podemos autorizarnos como intérpretes, como puentes que dibujen otros cruces entre
las generaciones; la escuela sigue siendo, en efecto, un puente valiosísimo para configurar
este y otros mundos posibles
El escenario educativo para las nuevas generaciones: Transmitir saberes que:
● Liguen
● Desliguen y
● Religuen
Método y vocación:
● Comienzo como formación técnica e instrumental
● Prescripción de métodos de enseñanza
● Cambio a la idea de Vocación (Dubet)
Política y sociedad:
● Dimensión política de la educación
● Neutralidad vs. favorecedora de democratización y participación ciudadana
● Escuela como un espacio en donde se ponen en juego la justicia y la ética.
Empleo docente:
● Actualmente mayor motivación por el “empleo”
● Cuestionamientos en torno a la “vocación” o condiciones intelectuales y físicas para la
función
Relación con el mundo del trabajo:
● Profesionalización docente
● Regulación de la tarea de enseñar
● Creciente responsabilidad individual

Los Educadores frente a los desafíos del mundo contemporáneo:


Que haya sujetos que puedan educarse depende de lo que hagamos con ellos en la
escuela, no sólo de lo que hagan la familia o la sociedad: depende de cómo los recibamos y
los alojemos en una institución que los considere iguales, con iguales derechos a ser
educados y a aprender (Dussel y Southwell, 2004)

¿Qué es, entonces el enseñar?


Es un trabajo profesional que posee una tradición en la cual se respalda pero a la que
puede revisar porque entiende que no se trata de un mandato inapelable. Por el contrario, la
docencia toma esa tradición para renovarla, y en esa renovación cultural se autoriza.
La potencialidad que adquiere su voz autorizada puede residir en la necesidad de
investigar el mundo, de interpretarlo y darlo a conocer.
Se trata, en definitiva, de brindar ese conocimiento como un puente, ayudando a cruzar
fronteras en una relación mediada que liga y también habilita para la renovación, que
genera responsabilidades −micro y macro, individuales, colectivas e institucionales− en la
construcción de una posición con otros y con nuestro tiempo.

DUBET- "Declive y mutaciones de la Institución"


Institución escolar: Función de instituir y socializar. Debe volverse más democrática y más
política.
La escuela como institución está en crisis. Transformaciones en relación al otro, producto
de la modernidad que trastoca el ordenamiento simbólico de la socialización. Crisis inscripta
en una mutación profunda del trabajo sobre el otro.
PROGRAMA INSTITUCIONAL: Toma como fundamento la estructura de la iglesia. Tiene
una técnica de socialización con una composición simbólica abstracta, que tiene su
fundamento en la iglesia y se replica luego en la escuela.
● En sus orígenes va aportar 4 características de sus orígenes:
-Conjunto de valores y principios sagrados: Son incuestionables.
-Hay un trabajo sobre el otro como vocación (autoridad apoyada sobre los valores de la
institución. Autoridad que encarna esos valores. Como son la razón, la cultura y la ciencia)
-La idea de Santuario: Concebida como un templo de la cultura, protegido de los
desórdenes, de los intereses y las pasiones de la sociedad, los conflictos del afuera.
Murallas bien definidas del adentro y del afuera.
-La socialización como subjetivación: Cuando explican a sus alumnos que su libertad, su
inteligencia y su espíritu crítico surgirán de una larga sumisión a las disciplinas escolares.
Acatar las normas de la institución permitía la subjetivación. Primero había una socialización
y luego emergía un sujeto que podía ser libre, crítico, pero antes tenía por allí. A través de
una institución escolar podemos llegar a una subjetivación.
Declive El mundo ha cambiado. Declive del programa institucional. Factores: Capitalismo y
sus crisis- medios de comunicación- el congreso han roto la alianza de la escuela con la
sociedad. También se puede pensar como un declive endógeno introducido por los virus de
la nueva modernidad.
● 4 Características en relación a la actualidad:
-Poliarquía de valores: Variedad de valores. Los valores han cambiado)
-El dominio del oficio
-El fin del Santuario: Hay otras problemáticas sociales. Otros dispositivos de la escuela
con otras funciones, la autoridad del docente ha cambiado, el involucramiento de la familia,
influencia de la tecnología y los medios de comunicación. Complejidad de la realidad social.
-La afirmación del sujeto: Hay otras instituciones que contribuyen a la subjetivación del
sujeto. Otros actores, otras realidades. El sujeto ya está constituido como tal. Hay un
cambio de posicionamiento. No es pensar a alguien que se va hacer sujeto, sino que es
alguien que ya viene siendo un sujeto. No es alguien que se viene a formar sino que ya es,
la institución va a contribuir. Con perspectiva de derechos.

Su legitimidad de la institución no puede ya apoyarse en principios sagrados como antaño.


Las instituciones han de enfrentarse a las demandas democráticas y al derecho
soberano de los individuos.
No tienen más remedio que ser más democráticas y más políticas para no caer en
nostalgias de épocas pasadas.
No es que ya no sirvan más, sino que necesitan ciertas transformaciones.
Debe permitir que los individuos elaboren experiencias singulares (enseñanza situada,
que tengan en cuenta el contexto, características particulares de los estudiantes).
La buena escuela no es sólo la escuela más eficaz sino también la más justa y la que
permite a los alumnos construirse de manera singular.
Destaca la importancia del trabajo sobre el otro.

KAPLAN- "La implicación afectiva en tiempos de pandemia y en la postpandemia.


Educar para una sociedad de reciprocidades"
Hincapié de la dimensión de lo afectivo en las instituciones escolares. Implicaciones
afectivas. Tiene efectos muy poderosos en los procesos de enseñanza-aprendizaje.
Se le dió más importancia a las cuestiones curriculares, de los saberes disciplinarios, de la
instrucción teórica.
Experiencia escolar: La pandemia nos interpela una pedagogía del trauma. Debe
posibilitar la tramitación y reparación del sufrimiento social.
La construcción de autoestima es un eje vertebrador de la experiencia escolar.

La escuela debe cumplir con estas funciones debe constituirse en soporte emocional,
funcionar como refugio y habilitar la posibilidad de constitución del lazo social (lugar para el
encuentro).
Aspira a construir una sociedad de reciprocidades, ella se crea mediante la educación para
el reconocimiento y el respeto mutuos que consiste en: cuidar de sí, hacer algo por sí
mismo y ayudar a los demás.

La escuela y sus posibilidades:


El sufrimiento puede ser interpretado como una experiencia comunitaria (miedos, pérdida).
Estar disponible para la escucha creando soportes socio-psíquicos en la socialización
de los sentires. Escuela, único lugar y referente que organiza ciertas cosas ejemplo el
tiempo. Sigue estructurando la vida social y la cotidianeidad de los sujetos.
Educar para la sensibilidad hacia los demás.
A través la trama vincular estructura subjetividad en el marco de una pedagogía del
cuidado.
Es importante que la escuela pueda abrir y trabajar sobre estás cuestiones: el padecimiento,
los miedos.
La escuela constituye un territorio simbólico de esperanza en la medida en que allí o
desde allí pueden tejerse lazos de solidaridad y reconocimiento mutuo.
La institución educativa participa contrarrestando los muros emotivos que nos
dividen, nos separan, nos segregan y nos excluyen. Permite atenuar las marcas de la
desigualdad durante el proceso de escolarización.

En síntesis: La escuela como espacio donde se permiten estás cuestiones para que esas
personas tengan la esperanza de algo mejor. Tengan libertades, democracias, pueda
habitar el respeto. De ahí la importancia de la construcción del autoestima. Es esa la
oportunidad, es ese el momento y la importancia de la función docente ahí, del refugio, del
sostén, del acompañamiento.

Generar las condiciones para que la comunidad educativa pueda objetivar sentimientos de
duelo, de ruptura de vínculos, de pérdida de soportes afectivos, que marcan los sentidos del
proceso de escolarización en el transcurrir de una experiencia traumática.
Se trata de transformar los miedos y los sentimientos de pérdida en esperanza, dado
que la escuela representa algo del orden de lo vital y de proyección del porvenir.

Contexto pandemia:
La mirada protectora de la escuela, incluso en la continuidad pedagógica no
presencial, simboliza un sostén emocional, un lugar simbólico donde
amarrarse subjetivamente.
No teniendo la presencia física del contacto directo, permitió una reparación simbólica.
La LECCIÓN que nos deja esta experiencia inédita es que no se puede escindir lo
académico de lo vincular. Sin afectividad, sin afectación subjetiva, sin movilización
emocional, no hay posibilidad de estructurar una trama que promueva los procesos
colaborativos y fraternales de enseñanza y aprendizaje. La importancia del otro en los
procesos de enseñanza -aprendizaje.
Las emociones como categoría interpretativa permiten acceder al corazón de las prácticas
pedagógicas. El lenguaje de las emociones es una gramática fértil para comprender la
construcción de vínculos y las vivencias subjetivas de los actores de la comunidad, donde
se han transformado las ritualidades y las formas de interacción. Las emociones tienen una
dimensión social.

CORNU- "Transmisión e instituciones del sujeto. Transmisión simbólica, sucesión y


finitud"
Nos habla de la transmisión humana para trasladarlo al ámbito educativo

● Cuando hablamos de transmisión o comunicación podemos identificar 3


elementos:
-Un objeto que se va a transmitir: El conocimiento
-Un transmisor: Al docente
-Alguien a quién se le transmite: El alumno
"Lo que le falta a este esquema son algunos aspectos del proceso: Lo que se construye
entre sujetos se transmite como 'construcción de sujeto' ".
Toma relevancia lo que ocurre entre el transmisor y a quien se le transmite. Va a ser
fundamental y va a constituir la construcción del sujeto.

El TRANSMISOR es un pasador que a la vez recibió.


Este no es que tenga que borrarse para que “eso pase” eso pasa a través de él.
Pero eso que lo inscribió como sucesor le significa, por la misma razón su FINITUD”
En síntesis: El sujeto del educador. Es alguien que recibió a su vez algo. El que está
transmitiendo es alguien que recibió. Es necesario que esté como sujeto, porque alguien
primeramente lo reconoció como sujeto. Como sujeto del saber, podía saber ese
conocimiento y a su vez se lo va a pasar a alguien, al estudiante.
Finitud: se lo tengo que pasar porque se me pasa el tiempo porque hay algo que les
sucede a los seres humanos y es inexorable, la muerte. Es importante la transmisión. Lo
importante es que ocurre en ese proceso. La transmisión no es unidireccional.
En esa transmisión el otro le va a otorgar un sentido propio. No es tal cual, hay una
transformación en esa transmisión.

"Nos focalizamos en los objetos a transmitir: Saberes, maneras, usos, valores. Pero al
focalizarnos en esa forma no vemos que la transmisión es una modalidad de relación con el
objeto". Es una dialéctica y la vemos en los procesos de enseñanza-aprendizaje. No sólo se
transmite información, saberes y conocimiento sino que también se transmite la relación que
tienen los docentes con ese conocimiento e inclusive como se lo pasamos al otro, como lo
hago, como yo preparo esa valija. A través de la pasión, del deseo, de la experiencia de los
saberes, etc.

"Transmitir un saber, un conocimiento, es reconocer en otro sujeto la capacidad de saber


ese saber, de desearlo, de entenderlo, de desarrollarlo".
" 'Construir al sujeto' es reconocerlo sin intentar fabricarlo de pies a cabeza como si fuera un
objeto".
"Es sólo despertar su curiosidad, es instituir como sujeto del conocer"
Acá nos adentramos en el posicionamiento docente. El voto de confianza, confianza mutua,
trabaja en la confianza del que el otro tiene la capacidad de, confiar en las acciones futuras
del otro, es más posible que suceda. La confianza es algo que se contagia.

En educación: Es transmisión democrática aquella que se preocupa no sólo de transmitir


"lo mismo" por tradición, sino la posibilidad de que exista lo "otro"
Uno recibe y la idea no es la reproducción total de eso. Si no que hay en esa transformación
del propio sujeto. No es transmitir lo mismo con clarividencia. Lo importante es que exista lo
otro. Es la idea de transmisión democrática. Es para darle la idea de ser, de ser un sujeto.

La transmisión se hace entonces invitación a tomar lugar, a inventarse la propia


subjetivación, la propia emancipación.
Tomar lugar hace referencia a los recién llegados, ya que tienen la intención o se espera
que tengan la intención de habitar un espacio, de construir a partir de allí. Darle la
posibilidad de tomar su lugar y a su vez, después van a ser quienes transmitan.

CORNU- "La ética de la oportunidad"


La educación está hecha de disponibilidad, de momentos aprovechables y de giros
decisivos.

"Habilitar la oportunidad es buscar y actualizar lo posible, imaginar y darle paso a otra


realidad planteada cómo un ¿Por qué no?"
"En habilitar oportunidades hay decisión, decisión ética, un cambio de mirada que apuesta a
infundir confianza y que busque capacidades vivas de contabilizar déficit, y que con esa
misma apuesta les brinde a esos posibles la oportunidad de cobrar cuerpo".

Habilitar oportunidades de alguna manera es que existe otro posible a eso donde el espacio
educativo debería promover esas cuestiones.
Oportunidad tiene que ver con la acción y la apertura a algo diferente. Tengo la oportunidad
de, todavía no ocurre pero sin embargo, existe la posibilidad de que ocurra, pero para que
eso ocurra tienen que pasar cosas.

ÉTICA DE LA OPORTUNIDAD
● Ocasión: es un instante, es huidiza. Es algo que ocurre en un momento, un imprevisto.
● Oportunidad: (el buen momento) Sentido de lo posible. Apunta más lejos que la ocasión.
Tienen más futuro. Lugar y tiempo favorable. Oportunidad que en este presente me permita
pensar en un futuro
● ÉTICA: responsabilidad en el reconocimiento del otro. Cómo juega esto en el escenario
educativo.

¿Cómo construir un posicionamiento


docente?
★ Se traduce en gestos mínimos, minúsculas maneras de estructurar lo que el otro intenta
decir-hacer. Esto de la apertura, de la posibilidad, del error. Hacer una observación. Cómo
pensarlo de otra forma. Darle la posibilidad de modificar algo que tiene capacidad de
hacerlo.
★ Hospitalidad de la voz. De dar la palabra.
★ Rechazar apriorismos. Resistir categorizaciones como pronósticos de carencias, de
profecías autocumplidas.
Desmentir lo que se espera.

Como docente: La cuestión es pensar e implementar acciones que no sean acciones


“sobre”, “cosificantes”. Esto de posicionamiento desde el yo sé lo que es bueno para el otro.
Actuar sobre el otro.
Acá el DESAFÍO es pensarse como docente y actuar con el otro. Planteando
subjetividades, alentando subjetividades.
“Con”,“subjetividades”, instituyentes y que pongan en acto el reconocimiento del otro como
sujeto y no reducirlo a objeto. Esto tiene que ver con el concepto de la ética que plantea la
autora.

La ética de la transmisión:
➔ Afán de actuar justo. Transmisión instituyente.
➔ Transmisión simbólica. Una invitación a tomar lugar.
➔ Oportunidad de la oportunidad: descubrir, inventar, hacer travesías, navegar hacia
otros destinos. Se trata de generar esas condiciones para que algo suceda, tiene que ver
con la oportunidad, con el momento justo.

MALDONADO- Escritos sobre Psicología y Educación


Educación: Como un objeto complejo, en donde se pueden identificar la interrelación entre
tres componentes (perspectiva de complejidad para entender la educación y los procesos
educativos, teniendo en cuenta la interrelación).
SUJETO DE APRENDIZAJE:
● Siempre y cuando se lo conciba como unos de los términos de la ecuación (E-AP-OC). Se
denomina así porque está dentro de la ecuación, condición necesaria.
● Presenta una serie de características psíquicas (modalidades afectivas y cognitivas) que
caben identificar y conocer
● Conviene postular un sistema de
★ Invariantes: características generales, estructurales, cuestiones psicológicas, cognitivas.
★ Variantes: la particularidad del conocimiento, conocimiento situado, conocimiento propio,
experiencias de cada uno. Lo singular de ese sujeto del aprendizaje (experiencia subjetiva,
historia de vida, el contexto, la institución, la época).
● Generalmente nos encontramos con sujetos que se encuentran en etapas del desarrollo,
lo cual complejiza la mirada.
● “Un alumno es siempre un niño o joven pero un niño o joven NO siempre es un alumno”.
● Participa de una cultura institucional particular (normas-códigos de convivencias-modos
de interacciones)
● En tanto admite una relación asimétrica con quien se denomina enseñante. Asume la
intencionalidad de la cultura escolar.
*Esas dos últimas características son constitutivas.

SUJETO DE LA ENSEÑANZA:
● Otro de los protagonistas de los procesos educacionales. Importancia de las
características psicológicas en relación a la implicancia del vínculo educativo. Importancia
de tener en cuenta los aspectos psicológicos, subjetivos.
● SUJETO (Psicoanálisis) sujeción intrapsíquica y de naturaleza icc (formar parte de la
cultura).
● Sujeción con la función de representar y hacer cumplir el orden social establecido.
● Orden anímico e intelectual como recursos psicológicos en el quehacer educacional
● Mantiene una relación asimétrica con sus alumnos (experienciales, intelectuales,
afectivas, etc). En relación a los derechos había simetría.
● El modo de operar con la asimetría colocándola a favor del aprendizaje dependerá de su
pericia técnica así como la importancia de su estructura afectiva.

Una relación entre relaciones: el alumno, el docente y el conocimiento


Los tres componentes conforman un sistema y su dinámica estará supeditada a las
características y comportamientos de cada uno de aquellos en las distintas situaciones y
unidades de tiempo referencial.
La supremacía ostensible de cualquiera de los componentes genera desestabilizaciones
poco propicias en los procesos educativos.

Supremacías de algún componente:


● Si bien las investigaciones acerca de la infancia y la adolescencia han producido un fuerte
impacto en el campo educativo. No debe ser considerado el único componente a considerar
(supremacía del alumno).
● El exceso de autoridad docente (supremacía del sujeto de la enseñanza), monopoliza la
acción y decisiones, el tiempo, el espacio, la palabra, el silencio.
Autoridad pensada como ubicada en el supuesto a saber, la dominación, el poder sobre el
otro, el control sobre el otro. Paradigma de la disciplina.
● Supremacía de los contenidos, tiranía a la hora de las planificaciones curriculares.
Desatiendo lo procedimental (el saber hacer, el poner en práctica), lo transferencial (lo
subjetivo) y lo concerniente al sujeto que aprende. Sería solo pensar en los contenidos
teóricos desatendiendo todo lo otro.
Acá tendríamos el QUÉ de la enseñanza y el objeto del conocimiento.

LO ESPERADO SERÍA UN EQUILIBRIO CONSTANTE, PERO EN CIERTAS


CIRCUNSTANCIAS LA ALTERNANCIA DE LAS SUPREMACÍAS RESULTA TAN FECUNDA
COMO INEVITABLE

MALDONADO- La exclusión del docente como sujeto psíquico en el proceso


educativo.
Los docentes asumen como importante para el ejercicio de su labor se circunscribe a la
habilidad técnica o se instala como ideal a partir del cual los alumnos deben identificarse.
Escasa consideración que se le asigna a la incidencia de los aspectos psíquicos que
el docente tiene sobre los estudiantes y sus estilos particulares de enseñanza que ellos
desarrollan. Osea; ¿Cual va hacer el impactos de estos aspectos psiquicos en el aula?

Aportes del psicoanálisis a la hora de pensar en los procesos educativos


-Analogía con el analista
● Comprender la vida anímica y los mecanismos involucrados en los vínculos de las
personas
● Procesos subjetivos de naturaleza profunda y compleja
Los ANALISTA (además de los conocimientos teóricos) requieren de un saber de origen
extra académico. La exploración de las vicisitudes de su propia vida psíquica.
● Si este no indaga en sí mismo su práctica tendrá fisuras, puntos ciegos que restan
eficacia a su tarea terapéutica. (análisis personal)

Que ocurre con la enseñante:


●Debe encontrar la distancia óptima en relación a su objeto de análisis.
● Convierte su condición de sujeto psíquico (encuadre de por medio) en un formidable
recurso técnico.
● Los educadores pueden sacar provecho de esos descubrimientos psicoanalíticos

La importancia del SABER SOBRE SÍ MISMO (como sujeto deseante/sujeto epistémico)


puede constituir un punto de apoyo para fortalecer sus actividades de enseñanza.
Contribuye con su propia persona (sujeto psíquico) como para la de los alumnos y los
procesos de enseñanza- aprendizaje.
Objeto deseante. Reconocer cuál es el deseo para ese estudiante y su deseo, porque
quizás no coincide. Poder reconocer el deseo de ese otro
Pensar la dinámica y las estrategias de enseñanza. Como voy a dar ese contenido (sólo de
manera registral?). Qué vinculación voy hacer con ese conocimiento. De qué manera lo voy
a poner en juego para que los estudiantes se relacionen con ese objeto de conocimiento.
Su tarea docente debe inclinarse a:
● Ayudar aprender
● Estimular el Aprender a aprender
Acompañar los aprendizajes de sus alumnos
● Valorar fundamentalmente los procedimientos y actitudes que estos generen.
● Aprender a aprender: Tener en cuenta los diferentes modos de aprender, que no
siempre son los mismos. Y también esto, de que como docentes no sólo brindan
conocimiento, teniendo en cuenta nuestros propios procesos de aprendizaje (vinculación
propia que tuvieron para aprender). Es importante comunicar eso, qué orientaciones se
pueden dar para aprender eso que estamos enseñando como objeto de conocimiento. Lo
importante es brindar esas herramientas como docentes para que esa persona se pueda
desenvolver en ese ámbito académico con ciertos objetos de conocimiento, teniendo en
cuenta la particularidad de cada sujeto.

Docente: Katherine Watson a cargo del Departamento de Historia del Arte de una escuela
de mujeres en Wellesley

No hay respuesta equivocada. Ni tampoco un libro de texto que les diga qué hacer

2 clase: Qué es arte, que lo hace bueno o malo y quién decide


Katherine pretende captar la atención de sus alumnas y moverlas a pensar por ellas mismas al
margen de convencionalismos y de la mentalidad impuesta por la época. Es un claro ejemplo de
experimentación a través de la motivación y la reflexión.
Cada alumna participa del proceso de enseñanza-aprendizaje, con voz y voto.

Alumnas:
Betty Warren
Giselle Levy
Connie Baker
Joan Brandwyn
Profesor de italiano: Bill Dunbar
Profesora de lenguaje/compañera de casa: Nancy Abbey.
Enfermera: Amanda Armstrong
DIDÁCTICA GENERAL - PROGRAMA 2023

EJE TEMÁTICO I

El oficio de enseñar en contexto: Dimensiones política, histórica, social, cultural, institucional,


subjetiva.
La tarea de enseñar a través del tiempo y los desafíos que imponen las condiciones de época. La
escuela secundaria y sus transformaciones. La relevancia de las condiciones de escolarización en las
trayectorias escolares.

● Alterman y Coria - “Cuando de enseñar se trata” CAP. 1

Introducción
El libro que presentamos busca socializar y poner en discusión los resultados de la investigación
“Condiciones de escolarización y transmisión de saberes. El caso de escuelas primarias y secundarias
de Córdoba”.
Los primeros análisis mostraron que tanto los proyectos implementados por escuelas secundarias de
Córdoba durante la reforma de los 90, como la gestión del tramo final del Ciclo de Especialización (CE)
se orientaron a generar condiciones básicas de escolarización desarrollando estrategias diferenciales
de sobrevivencia institucional. El prestigio acumulado en cada escuela, sus tradiciones institucionales,
las condiciones materiales de infraestructura y equipamiento y las disposiciones objetivas de sus
miembros signaron la diferencia. A pesar de ello, la mayoría no pudo sustraerse de problemas
altamente conflictivos.
Después de 10 años de un despliegue de políticas de restitución del Estado como garante del
cumplimiento del derecho a la educación, estamos ante un tiempo de pasaje, donde se ponen en
juego y persisten viejos y conflictivos atravesamientos. En los espacios locales de las prácticas
asistimos a procesos históricos y otros novedosos entre proyectos nacionales, locales, de organismos
no gubernamentales e inversiones de grupos sostenidos por múltiples factores.
En el contexto nacional, estudios pusieron de manifiesto el problema de la falta de sentido de los
procesos de escolarización, sobre todo para adolescentes y jóvenes, dando cuenta de las variaciones
en los contextos familiares y los lazos sociales. Desde aquí se puede interrogar sobre aspectos críticos,
como la distancia entre lo que se consideran esfuerzos escolares por transmitir saberes socialmente
relevantes y prioritarios para la incorporación en los mundos culturales, sociales y del trabajo que
favorezcan un ejercicio pleno de la ciudadanía, y los avatares de los procesos de reconfiguración
identitaria de los sujetos.
Así, surge el interés por estudiar la relación entre transmisión de saberes, sentidos y condiciones de
escolarización en escuelas primarias y secundarias e identificar las problemáticas emergentes que se
revelan como las más acuciantes.
Estudiamos la relación entre condiciones y enseñanza en el marco de una política pública nacional y
provincial que ha formulado nuevas regulaciones en los niveles educativos, ha producido variaciones
sustantivas en el sentido de los procesos de escolarización, ha modificado la forma de organización
institucional y del trabajo docente así como ha previsto cambios en los dispositivos pedagógicos
didácticos y de evaluación tendientes a garantizar la obligatoriedad de la educación secundaria y la
articulación entre los diferentes niveles y ciclos del sistema.
El discurso político-pedagógico oficial nacional (Consejo Federal de Educación) propone como
estrategia la renovación del sentido de la escuela para la resolución de los graves problemas que la
aquejan.
Ante los esfuerzos para cambiar y construir, la indagación da cuenta de una situación compleja y
crítica en escuelas que atienden a sectores marginalizados de la población. En los últimos años se
observa una tendencia que amplía la brecha que separa la calidad y experiencias vividas por los
alumnos en esas escuelas, de la calidad y experiencias ofrecidas en escuelas que atienden a grupos
poblacionales más heterogéneos y cuyas historias, condiciones materiales y simbólicas están menos
expuestas a variaciones contextuales o de coyuntura.
Por esta razón, las condiciones de escolarización constituyen un analizador fundamental tanto en el
plano estructural como simbólico, ya que serían condiciones de posibilidad de cualquier imaginario
pedagógico que opere revisando las condiciones heredadas de la escuela desde su nacimiento en la
Modernidad, que pueden ser productoras de desigualdad pedagógica o reproductores de
desigualdad social, como los mismos procesos de fragmentación educativa.
El estudio puso en foco las condiciones materiales y condiciones de organización del trabajo escolar y
los significados y estrategias que despliegan los sujetos escolares bajo esas condiciones.
Los resultados de investigaciones permitieron comprender que en las escuelas estudiadas, los
procesos de transmisión fueron impactados por problemáticas interpretadas como inicio de crisis de
autoridad. Estos indicios advierten sobre la necesidad de interrogar los dispositivos específicos de
transmisión que se despliegan en instancias donde pareciera no ser suficiente aludir a la pérdida de
sentido de los saberes escolares como principio de explicación. Desde el discurso oficial nacional y
local se proponen novedosas condiciones pedagógicas generales, pero son escasas las huellas
encontradas en los espacios escolares en que desarrollamos nuestro estudio.
En la investigación se reconocen una serie de situaciones que nos permiten aproximar algunas
hipótesis respecto de la relación al objeto de estudio:
- Se observan indicios de una diferencia evidente entre los alumnos que cursan el Ciclo Orientado (CE)
respecto del Ciclo Básico Unificado (CBU) en cuanto el modo de transitar la experiencia escolar, siendo
en este último caso más conflictiva y problemática en sus vínculos con profesores, compañeros y con
la escuela misma, diferencia que estaría asociada al desconocimiento o no apropiación de la historia
institucional. Historia que aparentemente los adultos no pudieron transmitir a los nuevos ingresantes,
perdiendo así la posibilidad de contar con un recurso de inscripción en una identidad institucional
todavía en proceso de constitución, que requiere de un ideario que los aglutine.
- Alumnos de los primeros años de escolaridad que pertenecen a los sectores de alta vulnerabilidad
social, sin un sentido de pertenencia desarrollado, que caen en las categorías de fracaso, repitencia o
(sobre) edad. Es posible que en su accionar cotidiano reproduzcan las formas conocidas de lo escolar,
en particular, las formas más conflictivas que aprendieron en otras escuelas que decidieron darle la
espalda y reubicarlos en otro lugar.
- Habría docentes que manifiestan cierta resistencia para trabajar pedagógicamente con dichos
estudiantes por el nivel de conflictividad que expresan y así se reproduciría el círculo de la negación de
posibilidades de proyección escolar para los chicos que pasarán de la condición de repitencia al
abandono.
- Se advierte una agudización de la conflictividad en los vínculos interpersonales, tanto a nivel de los
adultos, directivos y profesores y entre los mismos profesores, por diferencias de criterios respecto de
la gestión escolar, las normas de disciplina y de convivencia, las estrategias para resolver los
problemas de violencia en la escuela, el nivel de protagonismo estudiantil habilitado de instancia de
gestión, entre otros rasgos relevantes.
- Se observan signos de crisis del lugar de autoridad directiva, agudizada por la figura del “precariato”
que subsiste, reforzado por estrategias ministeriales que oscilan entre el acompañamiento y el control.
- Se generan divergencias de punto de vista que tensionan los vínculos entre los actores. Las peleas
entre alumnos y hechos de violencia física en un tiempo acotado, constituyen escenas dramáticas
cada vez más presentes en contextos escolares, leídas como profunda crisis de autoridad.
- Se han detectado situaciones de fractura en el pasaje entre el nivel primario y el CBU y en la
finalización del Ciclo de Especialización, a pesar de los discursos que sostienen la necesidad de
articulación inter-niveles. Dicha fractura torna a los estudiantes que acceden el primer año del CBU en
un grupo de mayor vulnerabilidad en términos de exposición al fracaso escolar, y de mayor
precariedad para la apropiación de las regulaciones institucionales y demandas evaluativas, y a
quienes acceden al último año, como el grupo que refleja una escasa capacidad de retención.
- Se observan diferencias sustantivas, entre las escuelas primarias estudiadas, en los modos en que
los sujetos se vinculan con la cultura escrita a través de los libros y las bibliotecas escolares.
A partir del año 2004 se observaron algunos giros en las políticas sociales tendientes a superar las
profundas desigualdades, la disminución de los índices de desocupación y trabajo informal, y a
atenuar las injusticias en la distribución del ingreso (Asignación Universal por Hijo).
En el marco de las políticas educativas, la sanción de la Ley Nacional de Educación 26206/06 legítima
procesos de cambio. Se generan alternativas para repensar la escuela en sus distintos niveles a través
de programas, planes y proyectos, aunque con diversos derroteros en las provincias. El Programa
Integral para la Inclusión Educativa para escuelas primarias, el Plan Nacional de Lectura y el
Programa Conectar Igualdad con dotación de computadoras a cada adolescente o joven de Escuela
Secundaria en todo el territorio nacional, son ejemplo de ello.
En la Provincia de Córdoba, algunas propuestas nacionales se materializan en cambios en la
organización de la escolaridad, con efectos incipientes: desde proyectos escolares puntuales como el
14-17 pasando por decisiones políticas que modificaron el sistema de evaluación, como la
trimestralización y la extensión de 2 a 3 asignaturas previas, la incorporación de coordinadores de
curso para asumir el lugar tutorial y definiciones de cambio curricular con variaciones en las
orientaciones del ciclo superior de la escuela secundaria, en la cantidad de asignaturas y la
redefinición de su orden de dictado.
Por otra parte, en un contexto de problematización de políticas y prácticas curriculares dominantes se
advierte una mayor preocupación por la enseñanza y una tendencia a repensar los saberes
prioritarios y los problemas de acceso a la cultura letrada, en particular en las escuelas primarias a
través de la generación de programas específicos, como el Plan Nacional de Lectura.
Sin embargo, insisten en la escuela formas de concreción de la experiencia escolar que se mueven la
tensión entre políticas inclusivas y prácticas de exclusión, entre la incorporación de sectores sociales
antes excluidos y su desconocimiento en tanto sujetos de derechos, entre saberes trazados y
acordados curricularmente y aquellos que circulan en las aulas.

● Bauman - Modernidad Líquida


“Los sólidos tienen una clara dimensión espacial pero disminuyen la significación del tiempo mientras,
los fluidos no conservan una forma durante mucho tiempo y están proclives a cambiarlas: por lo tanto
para ellos lo que cuenta es el flujo del tiempo más que el espacio que puedan ocupar.” “Los fluidos se
desplazan con facilidad. (…) Asociamos “levedad” o “liviandad” con movilidad e inconstancia.”
“Derretir los sólidos del Manifiesto comunista. Aludía a una sociedad que se encontraba demasiado
estancada para su gusto y demasiado resistente a los cambios.” La única manera de separarse de la
mano muerta es derritiendo los sólidos, o sea, disolviendo todo aquello que persiste en el tiempo. Para
ello a su vez se requería la profanación de lo sagrado, la desautorización y la negación del pasado y
por sobre todo de la tradición. -> Esto era para hacer espacio a nuevos y mejores sólidos.
Y uno de los motivos más poderosos que estimulaba la disolución era el deseo de descubrir o inventar
sólidos cuya solidez fuera duradera.
Los primeros sólidos que debían disolverse eran las lealtades tradicionales, los derechos y
obligaciones acostumbradas que ataban la iniciativa.
(Derretir los sólidos para Weber: liberar la iniciativa comercial de los grilletes de las obligaciones
domésticas y de la densa rama de los deberes éticos)
Esa disolución destrabava la compleja red de relaciones sociales dejándola desnuda, desprotegida,
desarmada y expuesta incapaz de resistirse a las reglas del juego. Lo que dejó campo libre a la
invasión y al dominio de (como dijo Weber) la racionalidad instrumental (o como dijo Marx) del rol
determinante de la economía: sus bases.
La disolución de los sólidos condujo a una progresiva EMANCIPACIÓN de la economía de sus
tradicionales ATADURAS políticas, éticas, y culturales. EL NUEVO ORDEN DEFINIDO EN TÉRMINOS
ECONÓMICOS. Este nuevo orden debía ser “más sólido”.
Casi todos los poderes políticos o morales que podían trastocar o reformar el nuevo orden fueron
destruidos o incapacitados porque ese nuevo orden llegó a DOMINAR la TOTALIDAD de la vida
humana, volviendo irrelevante e inefectivo todo aspecto de la vida que no contribuyera a su incesante
reproducción.
Claus Offe: The utopia of the zero option. “Las sociedades complejas se han vuelto tan rígidas que el
mero intento de renovar o pensar normativamente su orden está obturado (tapado) por su inutilidad
esencial. Por LIBRES y VOLÁTILES que sean, individual o grupalmente, los “subsistemas” de ese
orden se encuentran interrelacionados de manera “rígida, fatal y sin ninguna posibilidad de libre
elección.”
El orden general de las cosas no admite opiniones y ni siquiera está claro cuáles serían ellas y menos
todavía cómo podría hacerse real alguna de esas opciones. A diferencia de los casos distópicos este
efecto emergió de la DISOLUCIÓN RADICAL de aquellas amarras acusadas de limitar la libertad
individual de elegir y actuar (ej. Gobiernos dictatoriales, subordinación, opresión, esclavitud) LA
RIGIDEZ DEL ORDEN ES EL ARTEFACTO Y SEDIMENTO DE LA LIBERTAD DE LOS AGENTES
HUMANOS.
“Si ha pasado la época de las revoluciones sistémicas es porque no existen edificios para alojar las
oficinas del sistema y también porque resulta increíblemente difícil e incluso posible imaginar que
podrían hacer los vencedores una vez dentro de esos edificios para revertir la situación y poner fin al
malestar que los impulsó a rebelarse.”
No está en la agenda la tarea de construir un nuevo orden mejor que el viejo y defectuoso, LA
DISOLUCIÓN DE LOS SÓLIDOS, EL RASGO PERMANENTE DE LA MODERNIDAD HA ADQUIRIDO UN
NUEVO SIGNIFICADO, UN NUEVO OBJETIVO, LOS SÓLIDOS DERRITIÉNDOSE EN LA MODERNIDAD
FLUIDA, SON LOS VÍNCULOS ENTRE LAS ELECCIONES INDIVIDUALES Y LOS PROYECTOS Y
ACCIONES COLECTIVOS.
Jonathan Rutherford “modernización de la modernidad” habla de “categorías zombies” y de
“instituciones zombies” que están muertas y todavía vivas. Ej. Familia.
Hoy se produce una redistribución y reasignación de los “poderes de disolución” de la modernidad.
Las configuraciones, estructuras de dependencia y dominación fueron fundidas para ser
remodeladas: esa fue la fase de “romper el molde”. Los individuos ahora se enfrentan a nuevas
pautas que aunque “nuevas y mejores” son tan RÍGIDAS e INFLEXIBLES como antes.
TODOS LOS MOLDES QUE SE ROMPIERON FUERON REEMPLAZADOS POR OTROS.
En las clases, los marcos que (tan inflexiblemente como los ya disueltos estamentos) encuadraban la
totalidad de las condiciones y perspectivas vitales y condicionaban el alcance de los proyectos y
estrategias de vida.
Salimos de la época de los “GRUPOS DE REFERENCIA” para desplazarnos hacia una era de
“COMPARACIÓN UNIVERSAL” en la que el destino de la labor de construcción individual está
endémica (sucede frecuentemente) e irremediablemente INDEFINIDO, NO DADO DE ANTEMANO, y
tiende a pasar por numerosos y profundos cambios antes de alcanzar su único final verdadero: el final
de la vida del individuo.
Cada una de las pautas y configuraciones fueron despojadas de su poder coercitivo o estimulante.
PROCESO DE LICUEFACCIÓN: se descendió del “macronivel” al “micronivel” de la cohabitación social.
La nuestra es una versión privatizada de la modernidad, en la cual el peso de la construcción de
pautas y responsabilidad del fracaso caen sobre los hombros del individuo.
Las pautas de dependencia e interacción son maleables ya que como todos los fluidos NO
CONSERVAN DURANTE MUCHO TIEMPO SU FORMA.

Modernidad fluida: el hecho de que la estructura sistemática se haya vuelto remota + el estado fluido
y desestructurado del encuadre de la política de vida. Esto ha cambiado la condición humana de
modo radical y exige repensar viejos conceptos que hoy en día están vivos y muertos al mismo tiempo.
La modernidad puede evaluarse empleando diferentes parámetros pero es crucial el CAMBIO EN LA
RELACIÓN ENTRE TIEMPO Y ESPACIO. // La modernidad empieza cuando el espacio y el tiempo se
separan de la práctica vital y entre sí. El TIEMPO en la modernidad tiene HISTORIA, gracias a su
“capacidad de contención” que se amplía permanentemente: prolongación de los tramos de espacio
que las unidades de tiempo. EL TIEMPO ADQUIERE HISTORIA CUANDO LA VELOCIDAD DE
MOVIMIENTO A TRAVÉS DEL ESPACIO SE CONVIERTE EN UNA CUESTIÓN DE RECURSOS
HUMANOS (a diferencia del espacio siempre inflexible sin posibilidad de ampliación / reducción)
El TIEMPO MODERNO se ha convertido por su flexibilidad y capacidad de expansión en el ARMA
PARA LA CONQUISTA DEL ESPACIO. El espacio de aspecto sólido, pesado e inerte es capaz de
entablar sólo una guerra defensiva. Mientras que el tiempo es el bando activo y DINÁMICO del
combate.
Durante la modernidad la velocidad de movimiento y el acceso a medios de movilidad más rápidos
ascendieron hasta llegar a ser el PRINCIPAL instrumento de poder y dominación. PANÓPTICO. Michel
Foucault. “La inmovilidad de los internos era muy segura, la más difícil de romper entre todas las
ataduras que condicionan su subordinación. El dominio del tiempo era el secreto del poder de los
jefes. La pirámide de poder estaba construida sobre la base de la velocidad, el acceso a los medios de
transporte y la subsiguiente libertad de movimientos. Era un modelo de confrontación entre los DOS
lados de la RELACIÓN DE PODER. Pero existía tensión entre ambas tareas: los “rutinizadores”
tampoco tenían plena libertad de movimiento y era imposible concebir la idea de “amos ausentes”:
administrar y responsabilizarse requiere presencia y confrontación.” La aceleración de la velocidad ya
ha alcanzado su límite natural, el tiempo requerido se ha reducido a la instantaneidad. En la práctica
el poder se ha vuelto EXTRATERRITORIAL, ya no está atado, ni siquiera detenido por la resistencia del
espacio. Lo cual permite algo sin precedentes: PRESCINDIR DE LOS ASPECTOS IRRITABLES DE LA
TÉCNICA PANÓPTICA DEL PODER.
La etapa actual de la modernidad es POSPANÓPTICA. El fin del panóptico augura EL FIN DE LA ERA
DE COMPROMISO MUTUO.
Nueva técnica de poder puede ser evidenciada en la guerra de hoy. (Ej. guerra de Yugoslavia). “La
fuerza militar y su estrategia bélica de “GOLPEAR Y HUIR” prefiguraban, anunciaron y encarnaron
aquello que realmente estaba en juego en el nuevo tipo de guerra de la época de la modernidad
líquida: ya no la conquista de un nuevo territorio, sino la demolición de los muros que impedían el flujo
de los nuevos poderes globales fluidos. SACARLE DE LA CABEZA AL ENEMIGO TODO DESEO DE
ESTABLECER SUS PROPIAS REGLAS PARA ABRIR DE ESE MODO UN ESPACIO PARA LA
OPERACIÓN DE OTRAS ARMAS (NO MILITARES) DE PODER. LA PROMOCIÓN DEL LIBRE COMERCIO
MUNDIAL POR OTROS MEDIOS.”
El advenimiento de la era moderna significó el ataque consistente y sistemático de “los establecidos”.
Los nómades que menospreciaban las preocupaciones territoriales y que ignoraban los esfuerzos por
establecer fronteras fueron presentados como subdesarrollados que necesitaban ser ilustrados por
su reticencia a seguir el esquema mundial de desarrollo. VENGANZA DEL NOMADISMO: en la etapa
fluida de la modernidad, la mayoría sedentaria es gobernada por una elite nómade y extraterritorial.
Lo pequeño, lo liviano, lo más portable significa ahora mejora y “progreso”. Viajar liviano es ahora el
mayor bien y símbolo de poder. Aferrarse demasiado cargándose de compromisos puede ser
perjudicial, mientras las nuevas oportunidades aparecen en cualquier otra parte.
Una notable INVERSIÓN DE LA TRADICIÓN, los poderosos de hoy son quienes rechazan y evitan lo
durable y celebran lo efímero, mientras los que ocupan el lugar más bajo luchan desesperadamente
para lograr que sus frágiles, vulnerables y efímeras posesiones duren más y les brinden servicios
duraderos.
LA DESINTEGRACIÓN SOCIAL ES TANTO UNA AFECCIÓN COMO UN RESULTADO DE LA NUEVA
TÉCNICA DE PODER QUE EMPLEA COMO PRINCIPALES INSTRUMENTOS EL DESCOMPROMISO Y
EL ARTE DE LA HUIDA. Cualquier trama densa de nexos sociales implica un OBSTÁCULO. La fluidez
es la garantía de la invencibilidad.

● Dubet - Declive y mutaciones de la Institución

Introducción: En este texto utilizaremos la noción de institución en un sentido particular: el que tiene la
función de instituir y de socializar. La institución es definida entonces por su capacidad de hacer
advenir un orden simbólico y de formar un tipo de sujeto ligado a este orden, de instituirlo. Según esta
acepción, la noción de institución no designa solamente un tipo de aparato o de organización, sino
que también caracteriza un tipo específico de socialización y de trabajo sobre el otro.
Si nos interesamos por la escuela, es porque a día de hoy muchas de ellas parecen estar en crisis.
Esta crisis no es sólo una dificultad de adaptación a un entorno en movimiento, sino que es una crisis
del propio proceso de socialización, una crisis inscrita en una mutación profunda del trabajo sobre el
otro.

Socialización y subjetivación.
La socialización “formatea” a los individuos de tal manera que se conducen, sin que a menudo ellos
mismos lo sepan, conforme a las necesidades del sistema. De acuerdo con esto, la integración social
y la integración sistémica vienen juntas (Lockwood, 1964).
Siendo todo social, los actores interiorizan modelos, valores, sentimientos, marcos cognitivos
colectivos, habitus que terminan por “clonar” a los individuos. Pero, las sociedades modernas también
son sociedades individualistas en las que cada uno se percibe como un sujeto libre y autónomo, como
un individuo capaz de ser el autor de sus propios juicios y de sus propias acciones. Así, la socialización
de los individuos está dominada por una ambivalencia fundamental puesto que supone, a la vez, que
los actores estén perfectamente socializados y que, al mismo tiempo, cada uno de ellos disponga de
un sentimiento de libertad fundamental, aquel que lo constituye como sujeto singular y no
simplemente como un caso particular de un modelo social general. Del mismo modo, para Freud
existe una distancia entre el Yo y el Ideal del Yo, distancia que se funda en la autonomía y la
capacidad crítica de los actores: para ser un sujeto, hace falta dejar de identificarse con su padre a fin
de identificarse con los marcos simbólicos con los cuales es, a su vez, identificado su propio padre, es
el pasaje de la identificación particular a la ley simbólica. Según Durkheim, dos grandes desgracias
amenazan al individuo: la anomia cuando la socialización es demasiado débil; el “fatalismo” cuando
es demasiado fuerte. En pocas palabras, el individuo está amenazado por un déficit de socialización
que lo desgarra de la sociedad, y por un exceso de socialización que lo sofoca y lo conduce a la
“neurosis”.
En la sociedad de los individuos, cada uno se constituye como el autor de su propia moral y de sus
propias conductas, incluso cuando las normas que lo guían son las de la sociedad en su conjunto. El
mundo social se presenta entonces como un paisaje de fondo mientras que el individuo se refiere a su
propia moral y a su propia subjetividad.

El programa Institucional.
Si la Iglesia ha “inventado” la escuela, si ella ha racionalizado la pedagogía sacando a los alumnos de
las familias, creando clases, niveles, disciplinas, profesionales de la educación, reuniendo y
racionalizando en un solo lugar lo que los preceptores llevaban a cabo individualmente, fue para crear
un tipo particular de sujeto: un creyente. Desde el punto de vista de la Iglesia, la socialización de los
cristianos no se podía reducir a su adhesión a las costumbres y a los hábitos de las familias, hacía
falta sacar a los niños de la “pequeña sociedad” como decía Durkheim, y elevarlos hasta la gran
sociedad, aquella cuyos valores universales dan forman a los creyentes auténticos capaces de
aplicar los principios del dogma transformando los dogmas generales en fe personal. Para alcanzar
este objetivo, la Iglesia ha inventado “un programa institucional”: una composición simbólica y
práctica al que se pliegan aquellos que están encargados de llevarlo a cabo.
Si fue la Iglesia la que construyó este programa a partir del siglo IX, este programa se ha ido
distanciando progresivamente de su marco inicial hasta el punto de ser retomado por aquellos que
han querido combatir a la Iglesia, como los republicanos franceses o los revolucionarios rusos...

Un conjunto de valores y de principios sagrados


El primer elemento del programa institucional radica en su identificación con un conjunto de principios
y de valores tenidos por sagrados. Como estando fuera del mundo, situados por encima de la
sociedad y que parecen indiscutibles dentro del marco de la institución. La escuela republicana
francesa, al igual que la Iglesia, afirma el carácter “sagrado”, no negociable, de un cierto número de
valores planteados como a priori, los de la Razón, los de la nación, los del progreso y la ciencia. Esta
crítica es siempre interna al orden institucional y adquiere una forma teológica más que pragmática,
se discuten creencias en nombre de estas creencias, principios en nombre de los principios.
Durante mucho tiempo las lenguas muertas, las matemáticas y la filosofía han sido consideradas
como las disciplinas escolares más dignas y más selectivas, mientras que las disciplinas más
enraizadas en la vida social, como la tecnología y las lenguas vivas, tenían una dignidad inferior. La
escuela republicana laica y francesa, de la misma manera que las escuelas religiosas están
encargadas de elevar a los alumnos hacia Dios, ella ha querido elevar a los alumnos hacia la Razón y
la cultura. Eran capaces de fomentar el espíritu crítico y la libertad.

El trabajo sobre el otro como vocación


En el programa institucional, la principal virtud de los maestros es su vocación, el hecho de que crean
en los principios de la institución y que los encarnen. Desde el momento en que enseña, se espera del
maestro y del profesor que representen los valores de la institución sacrificándose a ellos, adoptando
una vida virtuosa y ejemplar.
Al obedecer al maestro, el alumno obedece antes que nada a aquello que el maestro representa y
encarna. Al obedecer el alumno obedece a esos mismos valores. No obedezco al maestro o al médico
porque tenga una personalidad simpática o autoritaria, sino porque actúa en nombre de la Razón, de
la Nación, de la Cultura, de la Ciencia.
Así, el individuo socializado no está preso en una simple relación particular como la que propone la
familia, por ejemplo, sino en una relación que lo eleva hacia los valores que el maestro representa con
una cierta majestad. Esto no impide recurrir ni a la simpatía, ni a veces a la violencia, pero por la
mediación de la vocación, la educación no es ni una pura seducción ni un simple adiestramiento.
Durante un largo periodo, la escuela republicana francesa ha formado a sus maestros como la Iglesia
formaba a sus sacerdotes. Se aseguraba más de sus convicciones y de su moralidad personal que de
sus competencias pedagógicas propiamente dichas. La república se ha opuesto desde hace mucho
tiempo al derecho de los profesores a sindicarse, a hacer valer sus intereses particulares en
detrimento de su vocación.

El santuario
En la medida en la que los valores de la institución están fuera del mundo, hace falta que la institución
misma esté fuera del mundo, que ella sea un santuario protegido de los desórdenes, los intereses y las
pasiones de la sociedad.
La conversión en santuario de la escuela exige que la naturaleza social del alumno sea ella misma
“santuarizada”. Se encuentra la misma separación en la escuela republicana francesa que distinguía
radicalmente a los niños y a los adolescentes, atrapados por su psicología y la sociedad, de los
alumnos, considerados sujetos participantes de la razón e iguales entre sí en la medida en que “la
razón es lo más compartido” del mundo. El maestro encarnando la Razón se dirige a la razón de los
alumnos por encima de sus singularidades psicológicas y sociales y cada actor escolares, dentro de
una cierta medida, santuarizado porque está dividido entre una parte sagrada, el alma o la razón, y
una parte profana, las adscripciones y los intereses sociales.
La escuela republicana francesa ha impulsado muy lejos su santuarización al negarse a que los
padres participen en ella, al negarse a que los empresarios y los sindicatos de asalariados y de
patrones intervengan en los asuntos escolares. De la misma manera, esta escuela, a pesar de estar
fundada en la igualdad, separó mucho tiempo a los alumnos en función de su sexo con el fin de que
los deseos y las “pasiones” no pervirtiesen el orden de un santuario construido al margen de la vida
social “normal”.

La socialización escolar es una subjetivación


A través del entrenamiento de los ritos y las disciplinas se forja una libertad subjetiva, la de la fe que es
un sentimiento personal. Los profesores defienden el mismo razonamiento cuando explican a los
alumnos que su libertad, su inteligencia y su espíritu crítico surgirán de su larga sumisión a las
disciplinas escolares. Es en este sentido que la escuela, como la Iglesia, es una institución, porque
instituye un sujeto arrancado de su propia naturaleza y de su propia naturaleza social.
El programa institucional está adaptado necesariamente a la sociedad y dirigido a producir actores
conformes a sus demandas. Pero también está construido en tensión con esta sociedad y esta
tensión es la que separa la socialización y la subjetivación; generalmente el ejercicio tiene éxito
cuando el alumno es capaz de criticar la escuela y su formación y, sobre todo, cuando lo hace en
nombre de los principios de la escuela porque ésta no será jamás totalmente digna de los principios
que promueve.

La fuerza de un programa

La legitimidad del trabajo sobre el otro


La relación pedagógica implica, a la vez, una autoridad y una voluntad de liberar al otro y de
convertirlo en igual. Los maestros no tienen que exponer su personalidad y sus sentimientos porque
están preservados por la institución y por su rol incuestionable. Además, están protegidos por el
santuario escolar que los dispensa de rendir cuentas a los usuarios, ya que ellos sólo deben
explicaciones a sus superiores e, incluso, a su sola conciencia.
En gran medida, el programa institucional participa de un encanto carismático que silencia la
violencia real de las instituciones porque esta violencia es, en cierta medida, legítima. Ahora bien, hay
que recordar que siempre hay una cara sombría de las instituciones, la de los golpes, las
humillaciones, las exclusiones que durante mucho tiempo han parecido aceptables porque estaban
encubiertas por los valores de la institución.

La capacidad crítica
La fuerza del programa institucional procede también de su plasticidad puesto que no cambia
fundamentalmente de forma incluso cuando cambian los valores que lo dirigen. A priori esta teoría es
un esfuerzo de crítica y un desencantamiento radical de la escuela, definida como un aparato de
reproducción de las desigualdades sociales, de la cultura dominante mediante una especie de
adiestramiento de los individuos.
Si el programa institucional se invierte desde el punto de vista normativo, sin embargo, se mantiene
desde el punto de vista de su forma. En un principio considerado por los profesores franceses como
una crítica intolerable a la escuela porque destrozaba las ilusiones, acababa siendo defendida por los
profesores y los sociólogos más cercanos a la tradición republicana en nombre de la preservación de
un modelo institucional hoy en día amenazado por las mutaciones liberales de la sociedad. Como lo
que importa es la estructura del programa institucional más que el contenido, el alcance crítico del
análisis ha acabado por desdibujarse tras la defensa de una forma institucional, ya que cada uno
sabe que lo esencial está ahí: poco importa que se enseñe, lo esencial es la forma de transmisión.

La relación regulada
El programa institucional es una economía simbólica y no sólo un marco organizado. Las relaciones
entre los roles sociales y la personalidad están establecidos fuertemente con una predominancia de
los roles que se imponen a las personalidades, pero forjan las subjetividades singulares en una tensión
entre la adhesión y la resistencia a los roles sociales. El maestro y el alumno desaparecen detrás de
sus roles y se construyen al abrigo de éstos y bajo su protección.
A la Iglesia no le gustan mucho los creyentes “demasiado” santurrones, al hospital no le gustan los
enfermos “demasiado” pasivos y a la escuela los alumnos “demasiado” buenos.
Desde el punto de vista de los actores, el programa institucional es “económico” en la medida en la
que las conductas de los distintos actores son previsibles. La organización misma es “liviana” porque
los actores se adaptan espontáneamente y porque durante mucho tiempo la división del trabajo en
ella ha sido relativamente relajada. También, cuando decae el programa institucional, se sobrecarga
el peso de la organización, del control y de la burocracia.

El declive del programa institucional

Hoy este programa parece prisionero en un proceso de declive que afecta a la naturaleza misma del
trabajo sobre el otro tal y como se practica en la escuela. El análisis más compartido respecto a esta
cuestión, atribuye esencialmente este declive a factores exteriores a la escuela. El capitalismo y su
crisis, los medios de comunicación, la pobreza y el paro, y la crisis de la familia, han acabado por
romper la alianza entre la escuela y la sociedad. Como en todo relato institucional, la institución es
pura y el mal viene de fuera, del “mundo”. Por ejemplo, es cierto que la convergencia de una
masificación escolar rápida y de una crisis social ha confrontado a la escuela a problemas que hasta
ahora había logrado mantener a distancia. El declive del programa institucional es un proceso
endógeno introducido por los “virus” de la “nueva”, “tardía”, “post” modernidad.

La poliarquía de los valores


Como somos “cada vez más modernos”, el “desencantamiento” con el mundo no proviene del declive
de los valores, sino del declive de su identificación con un conjunto de principios sagrados,
homogéneos y trascendentes que se imponen a todos. Del mismo modo que el espíritu de la
Ilustración ha terminado por debilitar la autoridad de los principios religiosos, la modernidad crítica
erosiona hoy los principios del progreso y de la razón.
La escuela ya no es el templo encargado de construir una nación homogénea cuando las distintas
minorías exigen que sus singularidades sean reconocidas en el espacio escolar, como muestra el
episodio del pañuelo islámico. También desde este punto de vista, se encomienda a la escuela ser
más democrática y menos republicana.
La presión por la igualdad se ha incrementado con el modelo de la igualdad de oportunidades y el
aumento de la lógica competitiva de la escuela. La necesidad de adaptar la escuela a las demandas
del mercado de trabajo ha sido a su vez intensificada, y la institución fundamenta menos su
legitimidad sobre sus valores que sobre su utilidad. Los valores sagrados de la escuela han perdido
algo de su buena imagen frente a la influencia de los medios de comunicación que ofrecen imágenes
del mundo alternativas, desordenadas sin duda, pero extremadamente poderosas. Son vulgares,
mediocres y anárquicos, están aquí por mucho tiempo y proponen a los alumnos otras maneras de
comprender el mundo, distintas de las de la escuela.
Ofrecen hoy otras imágenes del mundo, otros modelos culturales y se sabe que los adultos y los
alumnos dedican más tiempo a la televisión que a los estudios.

De la vocación al dominio del oficio


Cuando decae lo sagrado institucional, la vocación se transforma.
Ya no es la vocación del sacerdote o del monje que se olvida de sí mismo por una causa superior y se
separa por tanto de la sociedad. Entonces la vocación se convierte en profesión y la profesión es
percibida como un modo de realización personal. El individuo quiere estar cualificado y ser
competente y, en un nivel más subjetivo, quiere realizarse con su profesión. Ya no se pregunta a los
profesores si “creen” en Dios o en la Razón, sino si se sienten hechos para este oficio, si les gusta
enseñar, si aguantan a los niños, si piensan que pueden realizarse con este oficio. La autoridad del
profesor no proviene ya de su vocación “sagrada” y del carisma que la acompaña, sino de sus
competencias, de sus cualificaciones, de su eficacia. Está formado para ejercer eficazmente su oficio,
para que los alumnos salgan adelante y debe rendir cuentas con respecto a ello ante los alumnos,
ante los padres y ante la administración.
En todas partes, la división del trabajo escolar se ha visto considerablemente intensificada. La gestión
se ha vuelto mucho más fina y mucho más engorrosa que hace treinta años. “La Orden Regular”
católica o republicana se ha vuelto una gigantesca burocracia profesional en la que trabajan
administrativos, especialistas de la pedagogía, psicólogos, orientadores, expertos como los
sociólogos, políticos... A medida que la escuela deja de ser una institución, se va convirtiendo en una
organización “como las demás”.

El fin del santuario


La masificación escolar ha hecho explotar literalmente el santuario escolar. Los problemas sociales no
se han podido mantener fuera de la escuela y la han invadido.
De manera general, cada vez más alumnos pasan cada vez más tiempo en la escuela, los muros que
protegían la institución se han derrumbado.
La escuela se ha vuelto una gran productora de títulos, hasta el punto de que la ausencia de título
constituye una verdadera desventaja social. Desde entonces la competitividad escolar ha aumentado
por el simple hecho de la multiplicación del número de competidores. Mientras que el programa
institucional conseguía protegerse de la demanda social e imponer sus reglas a los usuarios, el
proceso se invierte y es la escuela la que debe responder a múltiples demandas: las de la economía,
las de las familias que persiguen la mayor rentabilidad escolar, las de las diversas comunidades
culturales que ya no quieren diluirse en el molde institucional.

La afirmación del sujeto


El sacerdote, un mediador entre Dios y el fiel, para dirigirse a los creyentes, los domina hablando
desde el púlpito y los creyentes son definidos como “fieles”, cuyo primer deber es el de sumisión y el de
obediencia. Al arrodillarse delante del sacerdote, los fieles se arrodillan ante Dios. El nuevo rito
instaurado cambia la posición y la definición del fiel: el fiel se convierte en un laico y el sacerdote se
pone frente a él para dar la misa en la lengua de los asistentes; el sacerdote no se sube ya al púlpito y
comparte la prédica con algunos laicos. Es un cambio de fondo en el cual el laico se considera un
sujeto autónomo que tiene una demanda de creencia y de fe, a la cual es importante atender y a
cuyas preguntas hay que responder. El ideal de obediencia del fiel queda sustituido por un modelo de
diálogo, desde luego con jerarquías, pero un diálogo a pesar de todo. Este rito, indica claramente que
nos adentramos en una sociedad donde el sujeto ya no es considerado como el producto de la
obediencia y de la socialización, sino como un individuo que ya existe, que tiene una personalidad, que
tiene necesidades y que debe adherirse libremente a la creencia.
La escuela ha sufrido una evolución similar pero más radical todavía con la transformación del
estatus del niño y del adolescente que ya no se distinguen del de alumnos. Éste pasa a ser
considerado teniendo una personalidad, un juicio, una singularidad que la escuela debe reconocer y a
partir de ahí asegurar su desarrollo. En general, no esperamos de la escuela que discipline a los niños
para hacerles crecer, sino que les ayude a convertirse en lo que ya son. Se insiste sobre la actividad de
los alumnos, sobre su compromiso con el aprendizaje, esperamos que se expresen y que los
conocimientos tengan sentido para ellos. Todas las nuevas pedagogías combaten los aprendizajes
automáticos, la simple memorización y el simple conformismo. Muy a menudo este desplazamiento
hacia una nueva pedagogía no es percibido por los profesores como un proyecto verdaderamente
nuevo, sino como una adaptación a las demandas de los alumnos que ya no aceptan desempeñar el
papel tradicional de alumno “pasivo” y “fiel” porque su educación familiar y las representaciones
ideales del niño y del adolescente creativos e iguales ya no les atraen (Singly, 1993).

Crisis y decepción

El oficio es difícil
Esta larga mutación ha incrementado apreciablemente las dificultades del oficio de profesor. El
profesional del trabajo sobre el otro se convierte en un trabajador encargado de construir él mismo el
marco simbólico de su actividad. No sólo hace falta adaptarse a los alumnos percibidos como cada
vez más dispares, sino que este trabajo ya no está sostenido por el dispositivo simbólico de la
institución. El declive del programa institucional hace que la autoridad del maestro ya no esté
apoyada sobre un principio superior, sobre el hecho de que el maestro represente una cosa más o
menos sagrada a la que se le debe obediencia. Le corresponde al profesor construir él mismo su
autoridad movilizando aquello que, a falta de otro concepto llamamos su personalidad. Ahí donde la
subjetividad del maestro desaparecía tras su rol, se antepone a éste, provocando un replanteamiento
profundo y un agotamiento puesto que la clase ya no es una rutina. Ahora bien, la motivación en el
trabajo escolar, ésta que algunos llaman su sentido, se vuelve incierta cuando la cultura escolar entra
en competencia con otras culturas más seductoras, cuando la utilidad de los estudios ya no está
garantizada o está muy pospuesta, cuando las distintas culturas juveniles y sociales invaden la
escuela. Entonces es cuando el maestro tiene que motivar a los alumnos y también en este caso
poner en juego su personalidad.
Decepciones
A estas dificultades profesionales se suma una profunda decepción ya que la escuela democrática
de masas está lejos de haber cumplido todas sus promesas al tiempo que ha contribuido a destruir el
programa institucional. La esperanza de formar un mundo de cultura y de justicia parece haber sido
traicionada en numerosos países a pesar de que la nación ha consagrado muchos medios a la
educación.
Las desigualdades sociales siguen pesando mucho en las trayectorias escolares e incluso desde
hace algunos años se han acentuando. Y en este asunto, la escuela no puede considerarse
perfectamente inocente, como totalmente víctima de las desigualdades sociales, puesto que las
estrategias de los establecimientos, los itinerarios, las composiciones de clases parecen reforzar las
desigualdades sociales (Duru-Bellat, 2004).
Tanto en la escuela pública como en la privada, los padres se comportan como usuarios racionales en
un mercado escolar y buscan los itinerarios y los establecimientos más prestigiosos, más eficaces y
más rentables. En gran medida también la escuela ha desarrollado un espíritu crítico que se vuelve
contra ella y muchos profesores se sienten profundamente afectados, incluso insultados por las
críticas que la sociedad dirige a la escuela. Criticar el sistema, es cuestionar al personal educativo y
aquí también, el sentimiento de declive del estatus es total. Todo esto engendra un sentimiento de
crisis endémica, una impresión de caída continua que arrastra consigo todas las dimensiones del
estatus de los profesores.
Estos movimientos de profesores se califican a sí mismos de “republicanos” y se oponen a los
“pedagogos” que serían, a sus ojos, los agentes de la destrucción de la escuela. Ellos reclaman la
vuelta a la escuela de antaño, la de la obediencia y el santuario, contra los nuevos métodos y la
presencia creciente de las demandas de la sociedad. Pero la limitación de estos movimientos radica
en que acentúan el cierre de la escuela sobre sí misma y que implican que serán rápidamente
excluidos todos los alumnos que no estén ya dispuestos a seguir el juego de la institución y a creer en
sus valores.

¿“Providencia” o liberalismo?
Muchos actores y sociólogos que analizan las transformaciones de las instituciones explican en
función de la expansión del liberalismo, del mercado y de la mundialización. Sin embargo, este análisis
confunde los efectos con las causas, porque, en lo esencial, el cambio institucional proviene de la
misma modernidad y de la amplia mutación social y cultural que implica. Todo sucede como si el
programa institucional no hubiese resistido a los valores de la modernidad. La mayor parte de las
transformaciones de la escuela, por ejemplo, provienen de la misma escuela. La demanda de
profesionalización proviene de los profesores y de sus sindicatos más que de la sociedad. La nueva
concepción de los niños y de los adolescentes está enraizada en las familias de las clases medias
escolarizadas donde los profesores tienen una posición esencial. La crítica de los valores es el
resultado de un mundo intelectual producido por la escuela y que se vuelve contra ella. Se podría
decir que el programa institucional ha entrado en contradicción con la modernidad a la que
inicialmente servía.
En consecuencia se puede lamentar el declive del programa institucional a la vez que se observa que
tiene algo de “fatal”. Al mismo tiempo, el retorno al programa institucional exigiría sacrificar libertades
y autonomía que pocos de nosotros aceptaríamos de buen grado. Al fin y al cabo, esta es la tragedia
de la condición moderna dominada por el cambio continuo y por la nostalgia que todos los “padres
fundadores” de la sociología habían anunciado y descrito al hablar de “tragedia de la cultura”, de
anomia y de desencantamiento del mundo...

Del programa institucional a la experiencia social

Del rol a la experiencia social


El trabajo de socialización se ha desplazado hacia los actores, en la medida en que deben, en cierto
modo, hacer lo que la institución hacía por ellos. De este modo, la socialización institucional ya no
depende fundamentalmente del aprendizaje de los roles sociales, de los alumnos, de los maestros, de
los enfermos, de los médicos: porque estos roles no cristalizan la totalidad de la institución, o, por
decirlo de otra manera, porque estos roles se fraccionan y tienen que ser habitados por la persona
mientras que se esperaría más bien que el rol constituyese a esta persona. Se puede ejemplificar este
cambio retornando al problema de las motivaciones. Esto no implica el declive de los roles sociales.
No porque haya más roles, sino porque no basta con desempeñar su papel para que el trabajo de
socialización se cumpla. Los individuos deben comprometerse subjetivamente con su trabajo, deben
motivarse y motivar a los otros cuando el sistema de motivaciones no es evidente y unánimemente
compartido.
Las instituciones tenían la capacidad simbólica de ser percibidas como espacios protegidos de las
impurezas del mundo atribuían sus fracasos y debilidades a la sociedad. Por ejemplo, el mundo del
profesorado aceptó fácilmente las teorías que atribuyen los fracasos escolares a las simples
desigualdades sociales porque la escuela seguía siendo “inocente”. Pero cuando el santuario se
erosiona, la culpabilidad se cuela en el trabajo; los usuarios protestan legítimamente y los
profesionales de las instituciones se ven privados de todas las explicaciones que les liberaban de sus
responsabilidades: ¿no hay fallos médicos y fallos pedagógicos detrás de los fracasos?
A fin de cuentas el trabajo de socialización ya no puede ser concebido como consecución de roles
sociales. Exige un compromiso de los actores que tienen que socializarse a través de la construcción
de sus propias experiencias. Por ello, el problema de los actores es el de construir la unidad de su
experiencia y, a partir de ahí, de construirse a sí mismos. Los actores se constituyen a través de
“pruebas” que deben a la vez construir y superar. ¿Cómo movilizarse a sí mismo para lograr hacer su
trabajo más allá de la multiplicidad de roles, de expectativas y de alternativas a las cuales cada uno
se ve confrontado?

¿Qué instituciones?
El declive del programa institucional no significa que salgamos de las instituciones, que éstas se
hayan vuelto inútiles o que sólo puedan ser concebidas salvo como sistemas de regulación de la
acción. Si se admite que estamos irremediablemente comprometidos con un proceso de
desencantamiento del mundo y del declive de lo sagrado, se pensará que las instituciones están hoy
confrontadas al problema que se ha presentado a los regímenes democráticos después de la caída
de los regímenes monárquicos, cuya legitimidad procedía de principios sagrados, en los que el rey
recibía su autoridad de dios y el maestro de escuela, la suya, del rey... No estamos condenados a
renunciar a todo acuerdo sobre los valores y la definición del bien soberano siempre que se piense
que puede haber acuerdos sobre unos principios de justicia que preservan unos intereses particulares
y unas identidades múltiples.
Desde el punto de vista del programa institucional, esta solución da vértigo del mismo modo que la
democracia dio vértigo a los que vieron deshacerse el vínculo entre lo soberano y lo sagrado. Las
instituciones son necesarias porque protegen a aquellos que trabajan en ellas de una inestabilidad y
de un compromiso subjetivo excesivo.
Las instituciones ya no pueden protegerse de las demandas sociales, pero por otro lado, no pueden
ser simples organizaciones de servicios encargadas de satisfacer estas demandas, aunque sólo fuera
porque hay una tensión entre sus principios, su profesionalidad y las demandas de los colectivos y de
los individuos. Por lo tanto es importante redefinir las fronteras del santuario institucional mediante
mecanismos de filtro que, ellos también, deben ser concebidos como procesos políticos.
Esto no supone abolir ni las reglas, ni las disciplinas, pero implica que la institución permita a los
individuos elaborar experiencias singulares. Hoy en día, todos saben que la buena escuela no es sólo
la escuela más eficaz sino que es también la más justa y la que permite a los alumnos construirse de
manera singular. Todos saben también que el buen hospital no es sólo el que cura mejor a los
enfermos, sino el que los trata también como personas y no sólo como cuerpos enfermos.

Conclusión: Esta evolución provoca entonces sentimientos de miedo y de defensa de una tradición
institucional que a veces no es más que nostalgia. Ahora bien, el declive del programa institucional no
significa la muerte de las instituciones, sino la transformación de las instituciones enfrentadas a un
mundo más moderno, más desencantado, más democrático y más individualista.

● Feldman - “Enseñanza y Escuela” CAP. 1

Espacios confusos y espacios ordenados:


El término “escolar” no define lo mismo en distintas épocas y no tiene, con el correr del tiempo, el
mismo significado.
Este cambio en la representación de la escuela ocurre en un período que va, aproximadamente,
desde el s.XVI hasta principios del s.XIX, cuando, cuando las imágenes de las escuelas comienzan a
mostrar progresivamente un escenario reconocible como “escuela” según la experiencia personal de
la mayor parte de nosotros. Así, “la escuela se escolarizó”.

La escolarización es un proceso que, acompañando otros movimientos sociales de envergadura,


crea, ordena y organiza un sistema técnico para educar en gran escala. La comprensión de este
proceso es importante a los fines actuales de lo que podrían llamarse políticas de enseñanza y
políticas curriculares: las formas de dirigir, regular y administrar las actividades y los propósitos
educativos.
-para hacerlo, es necesario recurrir a un enfoque histórico: es decir, comprender qué es la escuela,
significa reconstruir el proceso de su constitución; esta reconstrucción permite, por un lado,
establecer mejor el problema que el dispositivo resultante viene a resolver y, por otro, establecer
vinculaciones entre ese problema, las formas de resolverlo, los recursos sociales y las fuerzas
existentes.

La escuela reúne rasgos de distintas formas de educar provenientes de:


-las universidades medievales, la educación religiosa o dispositivos de caridad emergentes de la
urbanización y la industrialización; recoge las nuevas tecnologías de control personal desarrolladas a
partir de la Reforma y adapta en muchos sentidos la idea de “disciplina”.
Esta constitución no sucedió en un período corto, sino como históricamente se gestan los procesos
sociales que, como la escolarización, marcan los cambios trascendentales.
La escuela es un producto histórico contingente, relacionado con el nuevo papel de la educación en
los Estados Nacionales:
-el producto resultante conjugó tecnologías preexistentes junto con nuevos instrumentos
conceptuales que articularon un nuevo tipo organizacional caracterizado por categorías como
“sistema”, “clase”, “aula”, “currículum” y “método”.

La “selección natural” del dispositivo escolar:


La emergencia de la escolarización, como principal formato educativo, se produce en un proceso de
varios siglos que puede ser descrito como la creación de una tecnología para abastecer las
necesidades crecientes de educación en un marco de reestructuración completa de las relaciones
sociales, las formas de gobierno y los modos de producción.
La forma particular que adopta la escolarización es un invento relativamente reciente, pero que deriva
de construcciones previamente existentes en la educación superior y religiosa: -de hecho, es hacia el
s.XVI que comienzan a generarse mecanismos que limitan los estudios libres, agrupando los alumnos
en “clases” conducidas por un regente, como por la determinación de planes para su formación
(“curso rígido”).

El proceso de escolarización retoma la formulación de la escuela como un espacio cerrado propio de


las formas anteriores, pero organiza, gradúa y clasifica mediante un sistema de aula que desplazada
definitivamente la isla colectiva y desordenada
Entonces, la emergencia de estas nuevas técnicas de educar, acompaña las transformaciones en el
modo de producción, que repercutió en la aceptación de nuevas concepciones sobre el mundo: en
este sentido, encontramos que la Revolución Industrial implicó cambios en todos los ámbitos de la
sociedad, por el pasaje de la economía artesanal a la producción industrial centralizada en las
fábricas.

Mirar la escuela como lo que es:


Con independencia de la adhesión que despierten en cada lector las pedagogías, generalmente
constructivistas, no se puede dejar de reconocer que reducen los márgenes de actuación docente:
dicho de otra manera, procuran controlar su actividad.
-Feldman: desde mi punto de vista, éste no es un problema en sí mismo, sino que el problema reside
en la naturaleza de control y en la orientación que se procura imprimir a la actividad mediante su
ejercicio.
Parece bastante lógico sostener que un criterio al que debería ajustarse un dispositivo didáctico es su
capacidad de diálogo con las capacidades de las escuelas para utilizarlo, incluyendo los problemas
de la masificación de la enseñanza y de construcción de grandes sistemas que obsesionaron a los
educadores de los s.XVIII y XIX.
-es probable que una revisión de métodos y estrategias generales, junto con un adecuado enfoque
instrumental, puedan ayudar frente a las permanentes dificultades para utilizar el sofisticado stock
didáctico en las últimas décadas, que de poco ha servido para mejorar la enseñanza. Puede decirse
que una gran dificultad para el trabajo didáctico consiste en lograr que los productos que desarrolla
sean utilizados efectivamente.

A partir de los 1970’, se fue provocando una notoria separación entre reflexión pedagógica, didáctica
y curricular, y las nuevas configuraciones de la sociedad argentina, que cristalizan descarnadamente
en la década del 1990’.
Lo que vemos es una tensión porque la didáctica y la teorización sobre el currículum separaron su
producción teórica de los campos prácticos en los que debían actuar.

CONCLUSIÓN:
La reflexión sistemática sobre la enseñanza, nace históricamente ligada a la voluntad de “enseñar
todo a todos”. Esta herencia no implica la obligación de un retorno a lo clásico, pero conduce a
preguntarse por su significado para nuestros problemas actuales.
En esta tarea, la necesidad de considerar históricamente los procesos de educar y enseñar, no tiene
como objetivo recuperar una esencia original perdida en el transcurso de su evolución; en cambio,
procura llamar la atención sobre requerimientos de un dispositivo creado para resolver el problema de
la educación y de la instrucción universal, y trata de reevaluar los instrumentos que marcaron el
camino de la escolarización.
Volver a ponerlos en el centro de la escena, de la que paradójicamente fueron desplazados merced a
sus éxitos, ofrece mejores condiciones para clarificar las preguntas a las que responde actualmente el
trabajo de este campo.
Quizá sea necesario volver a apreciar la educación como un dispositivo contingente para cumplir un
propósito y no como un mal resultado de un plan más perfecto.

● Maldonado - “Aportes para mejorar los aprendizajes en la Universidad”

El aprendizaje y sus vicisitudes.


El arte de enseñar goza de respeto y consideración y el arte de aprender recibe apenas una atención
secundaria o menor.
El maestro enseña al alumno el peso del proceso de enseñanza - aprendizaje recae en la acción de
uno de los actores de la ecuación. El maestro es el agente de la oración; el alumno es tan solo el
complemento pasivo.
Relación asimétrica:
Al maestro le asignó el lugar del saber y al alumno el lugar de la ignorancia. Así pues, el enseñar ubica
al maestro en una posición activa y el aprender sitúa al alumno en una posición pasiva. Este
dispositivo que mantiene actualmente una extraordinaria vigencia en los espacios educativos
formales, ha provocado problemas de aprendizajes - enseñanza , lo cuales hoy se constituyen en un
motivo de consulta permanente a los especialistas.
Nos permite comprender por qué el maestro debe procurar ciertas habilidades para participar del
proceso y el alumno a menudo , sólo aporta en posición receptiva, la atención, concentración,
memoria , la disposición para escuchar al docente o lo que muestra. Queda explícita la supremacía
desmesurada que la enseñanza ha tenido sobre el aprendizaje durante los últimos trescientos años y
los inconvenientes que ahora nos toca afrontar, razón por la cual es primordial alentar la emergencia
de otro paradigma más fructífero para lidiar con estas realidades. Se requiere , sin lugar a dudas ,
resignificar el posicionamiento de los actores involucrados en los procesos educativos.
Papert siglo XXI ingresamos a la era del aprendizaje.
También propone una palabra para designar el aprendizaje y sugiere usar el término: matetico
significa "con disposición para el aprendizaje. Desde está visión, urge revitalizar y promover ese
maravilloso concepto piagetiano que es del sujeto epistémico , desde el cual los estudiantes tienen
que ser concebidos como sujetos activos y productores de conocimiento. Cabe resaltar la importancia
de fomentar en los sujetos , dentro y fuera de la escuela, la disposición de aprender, la capacidad de
aprender a aprender.
Este aprender a aprender interpela con decisión aquellas ideas pedagógicas que mantuvieron una
hegemonía casi absoluta durante toda la modernidad y que sobreviven sin mayores objeciones aún en
los primeros años de este nuevo siglo ideas que privilegiaron los contenidos en desmedro de los
procedimientos y la actitudes.
Papert reconoce dos grandes tendencias actuales.
La primera la tecnología , responsable de esa vital necesidad de un aprendizaje mejor también nos
brinda los medios para actuar en forma efectiva, inauguran un amplio abanico de posibilidades para
optimizar la calidad del entorno de aprendizaje ( conjunto de condiciones que contribuyen a que el
aprendizaje se desarrolle vertiginosamente en el trabajo, la escuela, en el juego o la recreación).
La segunda epistemológica. Un aporte fundamental de las nuevas tecnologías a la mejora cualitativa
y cuantitativa de los aprendizajes, se centra en la aparición de herramientas personalizadas capaces
de dar cabida a una amplia gama de estilos intelectuales.
Si los niños y jóvenes adoran las máquinas es por una sencilla razón; cuando las utilizan conquistan un
fuerte protagonismo cognoscitivo y sentimientos de autonomía y libertad epistémica. Evidentemente
los roles tradicionales desempeñados tanto por padres como por docentes deben ser resignificados
paulatinamente.
En la reflexión sobre el aprendizaje es factible abordar el tema desde una perspectiva general y
también desde una restringida. Desde una perspectiva amplia está sobre determinado por múltiples
factores. Una simplificación supone considerar que la inteligencia es el único requisito que garantiza
un aprendizaje satisfactorio; es un recurso esencial, también lo son el deseo de aprender, las
condiciones materiales y simbólicas, la relación docente - alumno, las políticas educativas, etc. El
aprendizaje es un proceso humano insoslayable y constante desde el nacimiento.
La tercera idea clave. Desde una perspectiva general es que se aprende no solo constantemente sino
también en cualquier espacio social. Está afirmación desnaturaliza la concepción de que se aprende
solo en espacios escolares tradicionales. Los humanos aprenden dentro y fuera de esos espacios
específicos establecidos en cada tiempo y lugar histórico.
Desde inicios del siglo XXI conviven tres espacios educativos principales:
El primer espacio es el familiar , ocurren allí aprendizajes que tengan una gravitación extraordinaria y
decisiva para la vida ulterior de cada sujeto. En este contexto primario, de fuertes intercambios
emocionales y reglas explícitas los niños deben aprender cuestiones tanto nodales como triviales. A
controlar los impulsos,a convivir con semejantes y diferentes a prefigurar la identidad sexual y de
género. Control de esfínter, a comer con utensilios, nociones especiales y temporales, lo privado y lo
compartido.
Un segundo espacio sistema educativo formal nuestro actual sistema educativo se origina alrededor
de los 4 años /5 y con frecuencia culmina con la educación media. Este sistema obligatorio , posee dos
actores principales: los aprendientes y los enseñantes. Entre ellos median reglas más o menos
estipuladas y fundamentalmente objetos de conocimiento , currícula ( contenido) fondos y organizado
por especialistas en educación.
El tercer sistema son los medios de comunicación que poseen un peso arrollador en promover la
transmisión cultural. Las pantallas cada vez más son una matriz socializadora. Les cabe a los padres y
docentes acceder a una indispensable alfabetización digital

Aprendizaje y Complejidad

Introducción
En él se establecen una serie de tópicos inherentes al aprendizaje. Procuramos la necesidad de
tratarla desde un enfoque complejo.
Complejidad. El aprender es inherente al experimentar. No deja de insinuar el valor de la escucha y l
mirada pero le dan una significación restringida en comparación con la experimentación ( compleja)
Morin sostiene que se han producido conocimientos sin precedentes sobre el mundo psíquico en
función de una ciencia que promueve sin miramientos los métodos de verificación empírica y lógica.
Ello ha traído como consecuencia que el error y la ignorancia progresen voluminosa mente a la par
que nuestros conocimientos . Dichos errores y cegueras poseen un carácter común que se origina en
el mundo mutilante de organizar el conocimiento, de modo que resulta incapaz de reconocer y
aprehender la complejidad de lo real. Desde Descartes soportamos la tiranía del paradigma de la
simplificación.
El principio disyuntor sería el responsable de aislar con mucho éxito tres grandes campos de
conocimiento científico; la física , la biología y las ciencias del hombre. Una alternativa poco feliz para
neutralizar tal disyunción fue la de forjar otra burda simplificación, esto es, reducir lo complejo a lo
simple.
Este pensamiento simplificante tan usual , es incapaz , enfatiza Morin, de concebir la conjunción de lo
uno y lo múltiple. O unifica abstractamente anulando la diversidad o por el contrario, yuxtapone la
diversidad sin concebir la unidad. En relación con esta cuestión inteligencia ciega, está corroe los
conjuntos y las totalidades , aísla los objetos de sus ambientes y ni siquiera puede, admitir el lazo entre
el observador y el objeto observado.
La imposibilidad para reconocer la complejidad de la realidad antropo social ( el sujeto individual) y su
macro- dimensión ( el conjunto de lo humano)
He aquí el motivo, por el cual resulta imperativo trabajar por el desarrollo del pensamiento complejo. Es
un tejido de constituyentes heterogéneos inseparablemente asociados: presenta la paradoja de lo uno
y lo múltiple. La dificultad del pensamiento complejo es que le toca afrontar lo entramado , la
inexorable interrelación de los fenómenos entre sí, el orden y el desorden, lo subyacente, la
incertidumbre y la contradicción. Asistimos al surgimiento de una ciencia que ya no se limita a
situaciones simplificadas, más nos instala frente a la complejidad del mundo real, que permite la
creatividad humana, como la expresión singular de un rasgo fundamental común en todos los niveles
de la naturaleza.
Morin propone como alternativa el paradigma de la disyunción/ reducción/ unidimensional un
paradigma que considere la distinción/ conjunción, el cual permite distinguir sin desarticular, asociar
sin identificar o reducir. Está modalidad de pensamiento contrarestaria al pensamiento mutilante y
como consecuencia de ello se podrían evitar las acciones cercenantes de él derivadas.
La patología contemporánea del pensamiento. Coincide con la hiper simplificación que reprime o
asfixia la complejidad de lo real. La enfermedad de la teoría está en el doctrinamiento y en el
dogmatismo. La idea deviene del fundamento epistémico que es el conocimiento único.
La patología de la razón está en la racionalización que configura lo real como un sistema de ideas en
principio coherente , pero sin admitir su parcialidad y sin reconocer que algo de lo real es irracionable.
Este posicionamiento tampoco acepta que a la racionalidad le cabe dialogar con aquello que no se
puede razonar y simplemente lo excluya.
En el avance del desarrollo del pensamiento complejo, pueden aparecer dos ilusiones:
La primera es la de suponer que la complejidad conduce a la eliminación de la simplicidad.
La segunda confundir complejidad con completud.el pensamiento complejo busca elucidar las
relaciones entre los dominios disciplinarios fracturados por el pensamiento disgregador. Ambiciona un
conocimiento multidimensional.

Aprendizaje
Papert propone el término matetico de origen griego, con disposición para el aprendizaje. Desde la
comisión de la Unesco al establecer el perfil de la educación s XXI determinó que la misma debería
construirse desde cuatro pilares: aprender a ser, a hacer, a convivir y aprender a aprender.
Cuando Papert propone si concepción matetica . Sostiene que no son suficientes los métodos para
resolver un problema. Resulta fructífero pensar sobre un problema y hablar de él. Está afirmación
subraya dos cuestiones:
Por un lado la dimensión lúdica inherente al acto de aprender; por el otro la significación que tienen los
intercambios y las diversas argumentaciones en relación con un problema dado. Estás dos
alternativas son frecuentemente desatendidas cuando no excluidas en los espacios educativos
formales.
La cuestión del placer parece confrontar con el pretendido control sobre el aprendizaje. Aquello
inherente al placer por el aprendizaje y a las vicisitudes relativas a la apropiación roza lo prohibido. Las
instituciones han desarrollado una cultura que desalienta ambas posibilidades.
Papert sugiere desarrollar la siguiente práctica: hablar libremente sobre nuestras experiencias de
aprendizaje.

Piaget entrevió como pocos la necesidad de superar los reduccionismos y concebir lo real desde una
perspectiva compleja y su insistencia por alentar el trabajo interdisciplinario. En cuanto al desarrollo de
las estructuras cognitivas: no resulta viable explicar el desarrollo ni tampoco el aprendizaje
apoyándose en un único factor como lo insinuaban las posturas maduracionistas o las conductistas.
Piaget con énfasis esas posturas reduccionistas.
Cómo es sabido enuncia cuatro factores: la maduración; la experiencia; la transmisión social y la
equilibración o autorregulación. Cada uno de los factores es fundamental, pero por sí solo insuficiente.
La obra de Piaget en torno a los problemas del conocimiento es uno de los autores que aportaron a la
idea de entender lo real desde una perspectiva compleja.
No obstante, su obra omite la exploración y el esclarecimiento de los aspectos afectivos vinculados al
aprendizaje.
En Psicoanálisis los desarrollos psicoanalíticos resultan imprescindibles a la hora de acceder a esas
vicisitudes ( motor) involucrados en el acto de aprender y también en el de no aprender. Las teorías del
aprendizaje vinculados a los sistemas positivistas. La significación otorgada al enseñante, reducida al
saber disciplinarios y sus habilidades didácticas. Se minimizó el valor de la persona del maestro en
aquel proceso. En cuanto al sujeto que aprende recibió demandas en cuanto al algoritmo cognitivo
(etapas) que el sujeto despliega en la apropiación del conocimiento, la relación con el sentir fue
descartada.
Cuando emergen problemas en cuanto al aprender los afectos , son considerados como causa
exógenos u obstáculos del aprender.
Sin embargo el psicoanálisis aportó ideas cruciales a fin de entender el vínculo: deseos , el de aprender
y el de enseñar. Que muchas veces propician dos tipos de problemas: el de aprender y el de enseñar.
Es factible afirmar que el deseo es el motor del aprendizaje y también de la enseñanza. Aprendientes y
enseñantes son copartícipes de una sucesión de actos sistemáticos a través de los cuales la cultura
se transmite y se recrea continuamente.
Si el enseñante tuviera claro que sus deseos no necesariamente son compatibles con los del
aprendiente y que ambos deseos son a veces incompatible con la exigencia de la cultura, se
despejarian alternativa diversas para mejorar el proceso de enseñanza y aprendizaje.
La Institución educativa mira con recelo el hecho de que el aprendizaje tenga lugar todo el tiempo
más allá de sus fronteras. El aprendizaje es inherente a la vida cotidiana.
El primer aprendizaje es el de la convivencia (currículo oculto) condición para el origen de procesos de
aprendizaje - enseñanza definidos curricularmente. El convivir en una institución también produce
bienestar y deseos de aprender: malestar y bienestar son dos alternativas siempre presente en la vida
institucional y ambas condicionan el aprendizaje escolar.
Cultura virtual la era de información coincide con la era del aprendizaje. Estamos en posición de
sostener que los nuevos aprendizajes están en gran medida supeditado a los avatares de la cultura
virtual. El mundo de la simulación otorga enormes posibilidades al aprendizaje via de la exploración y
experimentación virtual; el aprendiente afronta sin tantos riesgos ni control, un horizonte epistémico
multidimensional.
La experimentación facilitada por medios virtuales generará espacios de posibilidades en el abordaje
de la complejidad de lo real. Y les toca a los maestros construir roles inéditos s din de lograr formas de
cooperación con los estudiantes.

● Merieu - “El significado de educar en un mundo sin referencias”


Comienza afirmando que efectivamente la educación está en crisis, y habla de cuestiones de la
modernidad, como LA INMEDIATEZ, que nos lleva a querer todo y rápido, a pensar que “nuestros
deseos son órdenes”, a una organización del mundo en el corto plazo, sin un futuro posible, y de un
PRINCIPIO DE RESPONSABILIDAD de todo educador, que debe darle a los chicos un futuro que no
esté determinado de antemano, esa debe ser la ambición de todo educador.

REFLEXIONES EN TORNO A EDUCAR EN TIEMPOS DE CRISIS


1) La crisis de la educación está ligada al surgimiento de la democracia: la democracia afirma que
hay un lugar de poder que está VACÍO y que sólo puede ser ocupado PROVISORIAMENTE por
hombres a quienes PROVISORIAMENTE se les confíe el poder. En todos los países en dictadura no
hay reflexión educativa, la crisis de la educación es el precio que pagan las democracias por la
INCERTIDUMBRE que asumen en términos políticos, morales y sociales, y en democracia la
educación no debe tener una dirección única, una trayectoria que sea la misma para todos.
2) FENÓMENOS SOCIOLÓGICOS
Como la aceleración de la historia, la desligación entre generaciones (los padres por ej no pueden
usar con sus hijos los mismos métodos que sus padres utilizaron con ellos). Son problemas novedosos
a los que no podemos hallarles soluciones en nuestra propia historia. Las generaciones se separan
cada vez más unas de otras.
El motor de la economía es el capricho, la pulsión de compra, vivimos en un mundo que nos dice “tus
deseos son órdenes”, mientras que el educador tiene que enseñar a los chicos que sus deseos no son
órdenes.
NIÑO REY: vive la totalidad del mundo de acuerdo a su propia subjetividad, su deseo choca con la
existencia de los demás y deberá aceptar SALIR DE SU OMNIPOTENCIA, en un mundo que lo invita a
ser omnipotente todo el tiempo. (Da el ejemplo del control remoto como FALO de alta tecnología que
nos permite ir de contenido en contenido, y dice que en los países desarrollados los chicos llegan a
clase con un control remoto insertado en su cabeza y lamentan profundamente no poder hacer
zapping en clases, estando obligados a “permanecer en el mismo canal”, generando profunda
frustración.
APRENDER LA ALTERIDAD: algo muy difícil para los niños, entrar en relación con el otro, colaborar,
reconocerlo como su semejante, pero también como un ser distinto. “La educación es aprendizaje
para la RENUNCIA DE LA OMNIPOTENCIA”. El niño cree que su deseo es ley y no soporta que se vea
trabado. (entonces es inquieto en clases, no les interesa nada, se levanta continuamente, porque
siempre está en la INMEDIATEZ. No ha construido el espacio interno entre EL DESEO Y EL ACTO
(donde hay una especie de caja negra que algunos llaman conciencia, alma, razón) Para el educador
esto es EL APLAZAMIENTO DEL ACTO.

La modernidad descubre que el ciudadano es aquél que renuncia a LO INFANTIL, que se toma el
tiempo para examinar las consecuencias de sus actos, que no vive en la inmediatez, que está en el
tiempo de la REFLEXIÓN. No pasa por pedirle al chico que renuncie a sus deseos, sino de enseñarle a
que los examine, los pase por el tamiz de su conciencia, anticipe las consecuencias de sus actos y
examine más allá de su interés individual, el interés colectivo: ES LA RENUNCIA AL NARCISISMO.
Educar al chico es ayudarlo a renunciar a su narcisismo y pensar en el BIEN COMÚN.
3) PARA EL NIÑO EL MUNDO NO EXISTE, EL MUNDO ES LA TV
La ficción y la realidad se mezclan, “existir es salir en la TV” (hoy podrían ser las redes sociales). El
mundo es lo que la TV muestra del mundo, y si no está en la TV es porque no existe. Tenemos que
acompañar al chico en la idea de que Superman no va a llegar a último momento para salvar al
mundo, eso es ficción.

OTROS PUNTOS IMPORTANTES:


“Creo que lo esencial de una educación democrática consiste en aprender a hacer en conjunto, para
descubrir que nuestro deseo no es omnipotente”
Eso impone RENUNCIA, DISCIPLINA, eso es lo que abre las puertas al mundo. “En una patria, en una
clase, en una sociedad, cuando alguien quiere ocupar todo el lugar es porque no tiene un lugar”
Darle lugar a una OBRA COLECTIVA es permitirle ya no tener más voluntad de ocupar todo el lugar. A
la hora de abordar determinadas temáticas (pensemos en violencia de género por ej) tenemos que
ofrecerles objetos culturales donde sus miedos, deseos encuentren FORMAS PSICOLÓGICAMENTE
MANIPULABLES. Por ejemplo: ¿cómo un chico puede pensar las contradicciones del comportamiento
de su propia familia? Solo puede pensarlas si hay objetos culturales que encarnen esas
contradicciones y que le ofrezcan la mediación necesaria. El chico también es un ser violento, PERO
EL OBJETIVO NO ES PROHIBIRLE LA VIOLENCIA, SINO METABOLIZAR LA VIOLENCIA.
Cuando le pedimos al chico que renuncie a ser el centro del mundo, le pedimos como ciudadano que
se inscriba en un colectivo que renuncia a que su comunidad le imponga su ley a lo colectivo.
Renunciar a ser el centro del mundo es a la vez la condición para aprender una lengua extranjera,
historia, geografía, matemática, pero también es la condición para vivir en la sociedad democrática.
Mantener el ESPACIO VACÍO (en relación a la democracia y el poder) diciendo que nadie tiene
derecho a instalarse en el centro del mundo: ni el chico es el centro de la familia ni el tirano el centro de
la ciudad, la democracia es eso.

● Romero Cecilia- Construyendo justicia educativa en “Hacia la innovación en


la escuela secundaria” Ed Comunicarte. Córdoba

- Introducción
El nivel secundario es hoy decisivo para la inclusión de los jóvenes en la vida social, económica y
política, sobre todo en los sectores más pobres. Dicha inclusión, debe ser real y de calidad.

- Los procesos más recientes en el nivel secundario: expansión, democratización aparente y baja
calidad
La expansión del nivel secundario ha sido formidable desde el siglo pasado, generando
heterogeneidad entre sus alumnos debido a que ya no pertenecen exclusivamente a las capas
medias/medias-altas de la sociedad, sino que han ingresado los sectores populares.
Sin embargo, así como creció la matrícula, también han crecido los índices de fracaso, repitencia,
deserción y abandono debido a que la escuela no logra procesar la diversidad que ha dejado entrar.
Asimismo, la baja calidad educativa del nivel secundario, que se evidencia aún más en una
desigualdad en la distribución de los aprendizajes según el nivel socioeconómico de los alumnos.

Existen variadas formas de exclusión educativa, entre las cuáles podemos mencionar:
a. la exclusión total: el no acceso (alrededor de 700.000 jóvenes en Argentina no asisten a la escuela).
Es en cierto modo invalidante, debido a que hoy la educación secundaria es educación básica, y
quedar afuera del básico en la sociedad del conocimiento es quedar afuera de la sociedad.
b. La exclusión temprana: en Argentina, alrededor del 50% de los jóvenes que ingresan a la escuela
secundaria, no termina, fracasa, repite reiteradamente y se va.
c. La exclusión por inclusión sin calidad: sin garantías de aprendizaje, los estudiantes permanecen en
la escuela e incluso se egresan pero tienen un nivel de aprendizaje muy bajo. En Argentina, el 50% de
los jóvenes tiene dificultades para interpretar un texto sencillo.

- Precisiones sobre la desigualdad educativa


Las desigualdades socioeconómicas de las familias son fortalecidas en el sistema educativo.
Reproducción y legitimación de la desigualdad social de las familias por vía de desigualdad educativa
para sus hijos. Eso es precisamente la injusticia educativa.
La distribución del capital físico escolar muestra que las escuelas que tienen nivel socioeconómico
más alto, son las que se les otorga mejores recursos, dando más a los que tienen más. Así, aún
multiplicando los recursos persiste la imposibilidad de generar equidad, reproduciendo desigualdades,
acrecentarlas, legitimarlas.

- El ADN: contrato selectivo, matriz curricular rígida y cultura anárquica


La escuela secundaria no nace con pretensiones de ser universal, sino todo lo contrario debido a que
surge a partir de un compromiso de selección para formar a la élite. Este contrato fundacional
selectivo, propone una propuesta curricular altamente académica que es rígida pero a la vez
fragmentada donde se proponen módulos de 40 minutos de asignaturas que no están relacionadas.
Obliga a una fijación del contenido para un grupo en una localización temporal fija. Asimismo, la
cultura organizacional que se caracteriza por ser una anarquía organizada justifica el trabajo aislado
de los docentes proponiendo soluciones como apoyos o refuerzos que actúan como “curitas”. El
resultado es una currícula del siglo xix, con profesores del siglo xx y con jóvenes del siglo xxi.

- La escuela secundaria en los medios:


Por lo general la educación escolar ha sido tratada como un problema gremial o policial, donde las
noticias informan acerca de lo extraordinario como paros docentes, toma de escuelas, o los diversos
hechos de violencia entre alumnos, profesores y padres. Así, la educación comienza a ganar espacio
como fenómeno. Es necesario apostar a debates informados, que nos permitan una mayor conciencia
general acerca de los problemas de la educación y de los caminos para resolverlo.

- El aporte de las investigaciones y la voz de los alumnos:


El principal motivo por el cuál los alumnos dejan la escuela, es directamente la falta de interés en el
estudio. Plantean que es poco interesante aprender en la escuela. Además, se cuestionan las posturas
que plantean que la asistencia está fundada sólo en la exigencia familiar. Algunos datos que las
investigaciones aportan, es que las mujeres tienen una percepción ligeramente más favorable que los
varones y asimismo, a menor nivel socioeconómico, mejor es la evaluación que la escuela recibe.

- Conclusiones:
El sistema educativo tradicional nos garantizaba ciertas certezas, que ahora cambiaron por
sensaciones de transición, pérdida, mutación. Para superar este estado de desasosiego es necesario
producir un doble compromiso con el deseo de aprender y con el poder enseñar.
Reconstruir la función pedagógica y con ella la autoridad, el liderazgo, pero también el
acompañamiento y la escucha.
La mejora escolar no opera por demolición sino por reconstrucción. Es un proceso que tiene
historicidad y es básicamente una obra de reconstrucción de las prácticas a partir de lo que tenemos.

● Ministerio de Educación Ciencia y Tecnología de la Nación- “Docentes en la tarea de cruzar


fronteras y tender puentes”.

Introducción
Nos gustaría reflexionar sobre cómo está cambiando el oficio docente, tomando como punto de
partida tanto las transformaciones en la estructura del sistema educativo y las nuevas pedagogías
como los cambios sociales, culturales y políticos que envuelven a las escuelas. Se nos abre, así, la
necesidad de pensar el presente y plantear nuevas pedagogías en diálogo con la sociedad en la que
vivimos.
Enseñar es −a riesgo de ser un poco esquemáticos− establecer una relación, es decir, construir una
posición que no está situada en coordenadas predefinidas, fijas y definitivas sino que sufre
alteraciones y busca e inventa respuestas. Esa relación se establece con la cultura, el poder, los
saberes y las formas de su enseñanza; una relación con los otros y lo que ellos generan en uno, con la
política y la sociedad, con el mundo del trabajo y las múltiples estrategias que desarrollamos para
ubicarnos en él.

UNA RELACIÓN CON LA CULTURA, UNA AUTORIDAD CULTURAL


Lo que funda el sentido del trabajo de enseñar es la relación con la cultura, esto es, la relación propia y
la que propiciamos para los otros. Cuando hablamos de relación propia, pensamos que antes que
docentes somos ciudadanos que nos vinculamos a una sociedad y nos insertamos en ella
poniéndonos en diálogo con sus tendencias, sus problemas, sus urgencias, sus dilemas. (Claro está,
cuando decimos diálogo no nos referimos a obedecer un mandato inapelable, sino a una práctica que
involucra la crítica, el aporte propio, el compromiso, las múltiples perspectivas, la ética, etc.) Pero
también es necesario subrayar que a partir de la propia relación habilitamos, facilitamos, abrimos,
acompañamos una relación de los otros −fundamentalmente nuestros alumnos y alumnas− con una
cultura y una sociedad en las que viven y que les pertenecen. Sobre esas bases, asentamos nuestro
trabajo a partir de un sentido que se nutre permanentemente, que genera crecimiento para nosotros
como docentes y también a nuestro alrededor.
La relación pedagógica es una relación asimétrica −y es necesario que así sea−, porque ambos
miembros de la díada no están en igual relación con el saber, las normas, las responsabilidades, los
frutos del trabajo, etc. Además, el trabajo de la enseñanza supone una construcción de formas de
autoridad: el currículum constituye una autoridad cultural; el Estado y las instituciones donde
desarrollamos nuestro trabajo establecen formas de autoridad; el conocimiento científico se
constituye en una autoridad; un docente esforzándose por desarrollar puentes que no sólo son con su
saber específico sino también con la sociedad en la que vivimos y en la que queremos vivir, construye
una autoridad.
La expansión de la escolarización de masas requirió la formación de un cuerpo profesional que fuera
difusor de los nuevos valores del Estado-nación. Esa figura fue absorbiendo también formas de
representación del Estado en una amplia gama de funciones, en competencia con otros actores
sociales.
Se trataba de crear una nueva red institucional local −ya no internacional como la Iglesia−, que
ordenará y regulará los intercambios entre las personas en una forma nueva, con nuevos "apóstoles".
Se institucionalizó, así, una de las piezas clave de la "maquinaria escolar", la formación de docentes,
bajo el imperio del control político del Estado y el control científico de la pedagogía (Diker y Terigi, 1997).
A fines del siglo XIX y comienzos del XX, pensaron que la escuela debía civilizar el mundo, formar
sujetos nuevos −"ciudadanos letrados"− desconociendo aquello que los individuos traían como
experiencia propia, previa y diferente del mundo escolar.
la escuela concebida de esa forma también llevó adelante una utopía transformadora que nos legó
muchas cosas, algunas muy democráticas y otras con consecuencias menos alentadoras.
Dado que se pensaba a la escuela como una institución renovadora y transformadora de la sociedad,
no debe sorprendernos que los límites entre el afuera y el adentro estuvieran allí rígidamente
marcados, y que el adentro se percibiera como superior al afuera.
Es cierto que había un mundo mejor al que se miraba, y era el de una Europa idealizada e inexistente,
el de las letras, y a veces el de las ciencias.
La docencia se pensó como un trabajo individual, personal, en cuya definición los elementos del
carácter y la personalidad eran muy influyentes. El docente se definía por un dominio del saber letrado,
lo que le otorgaba una autoridad legítima e inapelable para ponerse frente al aula y ser digno de imitar.
Además, esta autoridad gozaba de gran prestigio en la sociedad, y sobre esa base se consolidaba
una sólida alianza con las familias en pos de la educación de las nuevas generaciones.
Hoy, con el declive de las instituciones fuertes se hace imposible demarcar el adentro y el afuera de
forma taxativa.
Pensar el presente no significa ni más ni menos que eso: pensar, reflexionar, decidir cómo los docentes
nos ubicamos frente a él y qué selección hacemos de ese presente para transmitir a nuestros
alumnos. Educamos para que ellos aprendan a vivir en el mundo, para que tengan más herramientas
que los ayuden a ser felices, y para que puedan hacer algo mejor con lo que reciben de nosotros. Ahí
es donde cobran sentido las preguntas sobre qué saberes acerca del mundo estamos transmitiendo,
con qué actitud nos posicionamos ante ese mundo, y cuáles de sus rasgos ofrecen potencialidades
nuevas.
Cómo los adultos que trabajamos en la escuela nos podemos constituir en "cruzadores de fronteras"
entre épocas, traduciendo, articulando y ayudando a construir referencias nuevas? Si retomamos las
cuestiones de la asimetría y la autoridad, probablemente uno de los rasgos más productivos para
refundar esas ideas sea un diálogo −con más curiosidad que sospecha− con la cultura
contemporánea. Podemos autorizarnos como intérpretes, como puentes que dibujen otros cruces
entre las generaciones; la escuela sigue siendo, en efecto, un puente valiosísimo para configurar este y
otros mundos posibles.

EL ESCENARIO EDUCATIVO PARA LAS NUEVAS GENERACIONES


Que en las sociedades democráticas, la transmisión implica subversión, creación de un presente y
futuro diferentes. En la medida en que pensemos la educación como el surgimiento de una libertad y
no como inculcación, educación y democracia estarán vinculadas.
La escuela debe transmitir a los niños, entonces, saberes que los liguen, los desliguen y los religuen. En
primer sentido, es necesario integrar a los niños a nuestro mundo en común, transmitirles saberes que
les permitan integrarse a la sociedad.
En segundo lugar, tenemos que transmitirles saberes que les habiliten modos de escapar a toda forma
de influencia, saberes que los desliguen, enseñarles a pensar por sí mismos era una de esas formas.
Hoy, podemos pensar en otros modos de sujeción que están en expansión y de los que es necesario
enseñarles a los niños a desligarse: La educación tiene el rol de promover la liberación de las distintas
formas de sujeción del sujeto, de permitirle interrogarse y emanciparse.
Un tercer momento, inescindible de los otros dos, los docentes deben transmitirles a los alumnos
saberes que permitan construir la humanidad en el hombre, que los religuen. Esto implica trasmitirles
los sentidos de los saberes, darles los medios para perforar el misterio, medios que les permitan
encontrar respuestas parciales a cuestiones antropológicas. ¿Cómo? Hablándole al alumno de sus
preguntas antropológicas: su lugar en el mundo, su relación con los demás, consigo mismo,

MÉTODO Y VOCACIÓN. PENSAR PEDAGOGÍAS PARA ESTE TIEMPO


Los futuros profesores aprenderían a ocupar "el lugar del que sabe, del que vigila, del que es capaz de
contribuir a la producción de saberes en la institución escolar de una manera correcta". Allí es donde
surge la profesionalización de la docencia: la escuela deja de ser un asunto estrictamente eclesiástico
para participar de manera directa del orden público.
El problema del grado de preparación de los docentes, de su situación laboral y de la todavía precaria
acción estatal en ese territorio de la enseñanza, alcanzó entonces su cauce privilegiado: la
prescripción metodológica. Si se lograba establecer y difundir con claridad y precisión cuáles eran las
metodologías adecuadas, el progreso escolar parecía estar garantizado. Esto, además, se unía a la
preocupación ligada a que ningún aspecto del ejercicio docente quedará fuera de control, para
alcanzar una homogeneización cultural y moral puesta en manos de esos funcionarios civilizadores
llamados "maestros".
Esta forma de construir una relación con la prescripción precisa del método a seguir también
establece una relación con la autoridad, con la autonomía profesional ante la serie de aspectos que
deben ser considerados al buscar las mejores condiciones para enseñar, para incluir a los otros en las
maneras más convenientes de participar del conocimiento. Sin embargo, no hay nada de natural,
esencial o "escrito en piedra" acerca de los mejores modos de enseñar. El método no es sino un
artificio que establece una determinada relación del individuo −tanto el alumno como el docente− con
el conocimiento que aprende o que prepara para acercar a otros.
Aquel modo prescriptivo de operar sobre la realidad educativa sintetiza el pensamiento político y
social sobre el cual se basó la escuela moderna. Incluía una mirada particular de las disciplinas
escolares que mostraba, entre otros aspectos, un fuerte relieve moral para la enseñanza, una
concepción del conocimiento que lo entendía de modo acabado y permanente en el tiempo, y una
pretensión de atrapar y "formatear" las diversas complejidades de las realidades educativas. La
producción pedagógica, psicológica y sociológica del siglo XX contribuyó a desarrollar nuevos
modelos e intervenciones sobre esos modelos. De este modo, la formación necesaria para el docente
se concentró en las particularidades metodológicas de su rol y la comunicación con los alumnos, pero
ya no eran igualmente necesarios los conocimientos generales que orientaban las prácticas
pedagógicas.
La escisión entre producción de conocimientos y docencia ha sido uno de los más tenaces esfuerzos
de las didácticas positivistas.. Esto representó una continuidad con la concepción restringida de la
ciencia social propia de los regímenes autoritarios. En suma, la tarea docente era pensada en un
espacio de determinación que reunía las ideas de eficiencia y orden, a través de una propuesta
tecnocrático-moralizadora, sobre la base de algunos valores que se pretendían "salvadores"
(Kaufmann y Doval, 1999). En la actualidad, la cuestión de la formación técnica e instrumental ha
tomado otros caminos.
Hay una creencia instalada en el sentido común, según la cual la práctica −en sus diferentes
denominaciones− es el momento en que el nuevo docente entra en contacto con "la realidad" y esta es
la que −en forma de choque− establece las adecuaciones que esa persona deberá operar sobre su
formación. Es la idea de que la práctica dictamina, y que por lo tanto esa es la experiencia ante la cual
deben subordinarse los demás saberes. Prácticas conservadoras al entender que sólo debe operarse
con "la realidad", entendida esta como "lo que hay", desestimulando así perspectivas más
transformadoras.
El modo de posicionar al docente frente a la tarea que venimos describiendo fue caracterizada −y
continúa siendo caracterizada− por la idea de vocación. Nos interesa ponerla aquí en un marco más
amplio que deje de verla como un atributo individual −frecuentemente como algo con lo que se nace o
no− para mirarla en términos más colectivos, como efecto de una dinámica social.
Este programa institucional considera el trabajo sobre el otro como una mediación entre valores
universales e individuos particulares, entiende el trabajo de socialización como una vocación en tanto
se encuentra fundado en valores, y cree que ese trabajo inculca normas que configuran al individuo y
simultáneamente lo vuelven autónomo y libre. Mirar la tarea de enseñar nutrida de responsabilidades,
saberes diversos y sensibilidad por el mundo y por los otros, nos pone algunos pasos más adelante de
una idea de vocación entendida como don personal que remite a restaurar algunos de los modelos
más conocidos.

LA POLÍTICA Y LA SOCIEDAD CONSTITUYENDO LA TAREA


En el mundo y el país en los que vivimos, donde los saberes circulan y la comunicación crea nuevos
vínculos, nuestras vidas están atravesadas por la desigualdad y la injusticia; En ese territorio, volvemos
a preguntarnos por el rol político −transformador− de las instituciones que tienen como objeto lo
común, lo colectivo, y entre las cuales ubicamos a la escuela.
Para nosotros, resulta hoy obvio que la educación y la enseñanza tienen una dimensión política que
aloja derechos, que produce sujetos, que favorece u obstaculiza la participación, la democratización y
la transformación de las instituciones y la sociedad.
Nos interesa aquí explorar qué vinculación se establece entre política y trabajo docente,
cómo eso incluye perspectivas acerca de quiénes, cómo y para quiénes enseñar en un escenario
social y público más amplio.
¿Puede construirse una relación de enseñanza entre docente y alumno que presente una asimetría
respecto de los saberes y responsabilidades pero que sea de semejanza en los aspectos humanos y
ciudadanos?. Eso implica sostener una mirada cautelosa para que la asimetría de la posición de quien
orienta con respecto a quien es orientado no se deslice hacia una desigualdad irremediable que
elimine toda dignidad. Sobre el vínculo entre escuela, ética y justicia, recordando que nadie −ni jóvenes
ni adultos− puede dentro de la escuela poseer menos derechos que los que tiene fuera de ella
(Southwell, 2004). Este y otros problemas éticos y políticos se ponen en juego cuando buscamos
ayudar a los estudiantes a pensar su futuro de manera más esperanzada e integrada a la sociedad.
están bajo condiciones que no son definitivas ni inexorables, que tienen aspectos ambiguos y abiertos,
continuamente desafiados.
Todos los días, muchas iniciativas y acciones entre familias e instituciones, grupos comunitarios y
docentes, distintas instituciones en asociación, familias y organizaciones sociales, muestran diversas
posibilidades de intervención y cambio, sin rendirse frente a una perspectiva de crisis como pérdida
total o devastación que nos dejaría sin posibilidad de modificar nada. Contrariamente, esas iniciativas
y situaciones concretas muestran las posibilidades que brinda el hecho de buscar alianzas para
organizar a madres y padres, pensar en organizaciones estatales y comunitarias que puedan sumarse
a mejorar el bienestar de la población y crear, en la práctica, otros lazos compartidos, de sostén y
dependencia mutua. Quizás en esa cadena de dependencias mutuas pueda articularse una relación
más igualitaria, donde los unos necesiten de los otros, de manera que en esa mutua protección pueda
funcionar una verdadera sociedad humana.
La escuela y el trabajo docente han estado y están siempre tensionados, preocupados por la
producción y la reproducción de desigualdad e injusticia. La no naturalización de la injusticia es algo
que les toca a las instituciones políticas, entre las que se incluye la escuela. Esto implica no perder de
vista que la escuela es un ámbito específico y particular donde se produce la socialización de las
nuevas generaciones en la cultura letrada y donde se construye una relación con la cultura y la política
no solamente a través de los espacios curriculares destinados a ello, sino además mediante el modo
en que la justicia y la ética circulan por los pasillos y por los patios, sin perder de vista las palabras que
allí se ponen en juego.

EL EMPLEO DOCENTE
Históricamente, los aspirantes al magisterio provenían de sectores sociales heterogéneos,
en general de las primeras generaciones de familias que lograban el acceso al nivel medio de
enseñanza. Una de las motivaciones fue la perspectiva de una movilidad social ascendente. Los
docentes fueron el ejemplo del discurso que transmitían: en ellos se encarnaban la posibilidad del
ascenso social y la posesión de un capital cultural por medio de la educación. Con el tiempo, otro de
los rasgos que distinguieron al trabajo docente fue la estabilidad en el cargo. Al finalizar el siglo XX el
panorama mostraba significativas diferencias: se redujo la heterogeneidad de la población que
accedía a las carreras docentes, con un peso creciente de los sectores más empobrecidos. El
mercado de trabajo regulan los movimientos en el interior del mercado de trabajo docente; el
incremento de la precariedad del empleo ha generado motivaciones en relación con el ingreso en la
carrera docente, así como el retorno de quienes no ejercían (Birgin, 1999). De este modo, el empleo
docente se instaló en una paradoja: por un lado, amenazado por las medidas de ajuste estructural; por
el otro, crecientemente buscado ante la vulnerabilidad producida por el desmoronamiento de la
condición salarial y, fundamentalmente, de la protección social que ella incluía. Se configura allí una
resignificación de las motivaciones "vocacionales"
para la tarea docente (apostolado, voluntarismo, optimismo pedagógico) y su adecuación a los
parámetros más generales que caracterizan la opción por cualquier empleo. Por ello, para las
perspectivas de los aspirantes a la docencia, el trabajo ocupa un nuevo lugar donde se reacomodan lo
vocacional, lo redentor, lo laboral, la enseñanza.
La valoración de lo que sucede en la escuela y del rol de quienes trabajan en ella se hace desde un
peculiar encapsulamiento, y pese a que se reconoce que fuera de las escuelas las dinámicas social,
familiar, de género y laboral se han transformado notoriamente, se sigue proponiendo un "deber ser"
de la escuela y de los docentes, con la idea de reponer un modelo que se estableció y fue eficaz
tiempo atrás.
En esta situación, las instituciones formadoras se lamentan porque los estudiantes no poseen una
"vocación" (a la que consideran indispensable para el ejercicio del rol docente), o porque no cuentan
con las disposiciones intelectuales y físicas que se consideran inherentes a la función. Lo que se busca
identificar es si el estudiante presenta esas disposiciones o no, pero no si la institución las forma (Birgin,
2000). En muchos casos,
en estas instituciones formadoras se construye un discurso alrededor del supuesto déficit que traen
los estudiantes, tanto por sus rendimientos académicos como porque se alejan del patrón cultural y
social que se considera valioso y pertinente para la docencia.
Esto puede devenir discriminación social por lo menos en dos sentidos: por los procesos de
estigmatización que se generan y porque se van construyendo nuevos circuitos en función del perfil de
docente que se visualiza en los alumnos. A su vez, esa perspectiva se absolutiza en una epistemología
academicista que sólo reconoce como conocimientos a aquellos legitimados en el medio escolar y
descarta otras formas de conocer, otras oportunidades, otros saberes mediante los cuales los sujetos
actúan en la sociedad. Es decir, a partir de rígidas clasificaciones que solamente dan lugar al
conocimiento que proviene de la cultura letrada legitimada en el ámbito escolar, los jóvenes son
tipificados (Birgin, 2000). Esta situación acarrea otro problema, que consiste en que estos jóvenes se
forman visualizando sólo determinados circuitos del sistema educativo como los ámbitos de trabajo
deseables y posibles para ellos.
Esto no sólo sucede a partir de la inequidad distributiva sino también como consecuencia de patrones
institucionalizados de interpretación y evaluación que constituyen a alguien como no merecedor de
estima, respeto o legitimidad para ocupar determinada posición (Nancy Fraser, citada por Birgin,
2002).
Probablemente sea una buena ocasión para autorizar la palabra docente, la palabra de mujeres y
hombres que busquen vincularse de modo nuevo, adulto y profundo a los saberes, a la cultura que se
despliega fuera de la escuela, a la sociedad de la que son parte,
mas allá de la sospecha, de rígidos preceptos morales y de relaciones autoritarias con el
conocimiento.

UNA RELACIÓN CON EL MUNDO DEL TRABAJO


El trabajo docente se constituyó a partir de la conformación de una fuerza de trabajo económica y
disciplinada sobre la base de la subordinación de género en la sociedad del siglo XIX y comienzos del
XX. Feminización de la enseñanza, abnegación, sacrificio,entrega, neutralidad, trabajadores/as
baratos/as, fueron rasgos propios del origen de la docencia. Cien años después, las características del
modelo docente que fundó el sistema educativo argentino presentaban importantes cambios. La
pretensión de neutralidad resultó quebrada. Los docentes −hasta ese entonces considerados
misioneros ascéticos del saber− realizaron durante la segunda mitad del siglo XX un proceso de
asunción de su identidad como trabajadores de la educación.
El concepto de profesionalización docente la temática del desarrollo había instalado en las décadas
del 50 y el 60 una preocupación por la expansión del sistema educativo.la expansión el de la formación
y consolidación del sujeto político democrático.
En Argentina, desde la década de 1950,
coincidiendo con los cambios en el modelo de desarrollo, la consolidación de la sociedad salarial y la
difusión del pensamiento económico de la CEPAL −que promulgaba una estrecha relación entre
desarrollo educativo y desarrollo económico−, el Estado diseñó nuevas políticas tendientes a regular la
tarea de enseñar. En 1958 fue sancionado el Estatuto del Personal Docente Nacional, a partir del cual
se reguló el ingreso, ascenso y estabilidad de la carrera docente, se fijó la necesidad de contar con
cursos de perfeccionamiento y se definieron criterios para la remuneración salarial. Los dirigentes
sindicales entendieron la sanción del Estatuto como un avance en materia de organización gremial
por la defensa de los derechos de los docentes. La competencia técnica que les era conferida a los
profesores implicaba un desempeño laboral preciso que se oponía a la idea de un apostolado guiado
por la vocación.
En la década de 1990, la retórica de la profesionalización también supuso una serie de estrategias
para racionalizar y normalizar la formación y el trabajo de los docentes,Por el contrario, la tendencia
fue definir nuevas exigencias para su tarea,en función de incrementar su competencia intelectual y
social y, sobre todo, su responsabilidad por la obtención de resultados favorables (Popkewitz y
Pereyra, 1994). El uso del concepto de accountability, de difícil traducción al castellano, da cuenta de
este cambio que, por una parte, define la necesidad de evaluar la tarea de los profesores en términos
de "rendir cuentas" de su trabajo (Nóvoa, 2002) y, por otra parte, admite la necesidad de regular los
procesos de enseñanza y las identidades de los docentes a fin de conseguir una estandarización
profesional y una creciente responsabilización individual.
Frecuentemente, la profesionalidad está asociada a la defensa de la autonomía, la responsabilidad
por el trabajo y el compromiso social con una mejora en la calidad de la enseñanza. Así, el discurso
profesionalizante ha buscado prescribir lo que significa ser un "buen docente" como un "constructor de
su capacidad" y poseedor de destrezas "no rígidas", o ligadas a la adaptación flexible, como las de ser
administrador y gestor organizacional. Por ello, las políticas docentes impulsadas desde ese consenso
han encerrado una tendencia para regularizar los procesos de enseñanza y las identidades dentro de
la preocupación general por una estandarización profesional y por una instancia de
responsabilización.
La Profesionalización involucra un proceso de normalización que, a su vez, liga una variedad de
presupuestos acerca de disposiciones que deberían formar las competencias de los docentes en
tanto profesionales.
Estos usos remiten a discusiones presentes en el momento de la creación de los sistemas educativos
nacionales, es decir, al carácter vocacional o apostólico de la tarea de enseñar y, consecuentemente,
a la responsabilidad casi mesiánica del docente con la comunidad. Se conjuga así la ya clásica
interpelación por la vocación con la de la profesionalización.
A partir de la consolidación de sociedades de mercado, la responsabilidad por la eficacia y la
eficiencia de la tarea docente −de difícil caracterización y cuantificación− ha comenzado a constituir
nuevas identidades a las que se les exige capacidad para el trabajo colectivo pero también
compromiso individual por los resultados y la calidad de la educación.
se manifestaron en enunciados destinados al ejercicio individual, a la actualización de cada uno de los
docentes, al fortalecimiento de su autonomía y a la gestión del propio riesgo.
Frente a las recomendaciones y prescripciones a favor de este tipo de profesionalismo, los docentes
debieron llevar a cabo su tarea en contextos cada vez más desfavorables, producto de las mismas
políticas económicas que postulaban su responsabilidad aludiendo a su compromiso en la
consecución de una educación de calidad. La evaluación de los docentes, los incentivos ligados a los
resultados obtenidos y la diferencia salarial según el desempeño son respuestas individuales frente a
una situación que implica aspectos colectivos y condiciones socioeconómicas más allá del
desempeño de un profesor aislado.
El siglo XXI ha traído otros enfoques sobre estos problemas. En los últimos años se fue construyendo
otra perspectiva que busca imbricar la formación con la carrera, esto es, construir una perspectiva de
desarrollo profesional que comience con la formación inicial y continúe con el enriquecimiento de
saberes de la experiencia, buscando situar al docente como voz autorizada a través de la recreación
de la herencia cultural y del fortalecimiento de sus vínculos con la política y con el mundo
contemporáneo. Las palabras actúan para ligar una variedad de presupuestos sobre lo que es
socialmente apropiado y sobre cómo un individuo debe comprender su propia competencia en
acciones futuras. Aunque nunca es un asunto monolítico, si uno es considerado "profesional" ello
involucra conjuntos particulares de disposiciones sobre cómo se debe hablar, pensar, actuar y "ver". De
este modo, la profesionalización constituye una interpelación que construye al docente profesional no
sólo como modelo, sino como reivindicación de una tradición siempre existente.
Se trata de un campo de significados que a la vez funciona como mecanismo de regulación. Las
políticas adoptadas en consecuencia establecen una norma, una trayectoria deseable, una serie de
condiciones ideales que caracterizan al docente profesional.

CONCLUSIONES: LOS EDUCADORES FRENTE A LOS DESAFÍOS DEL MUNDO CONTEMPORÁNEO


Hemos hecho referencia a que el trabajo docente, como práctica especializada y referente de
transmisión cultural, ha sido, y es, objeto de interpelación por su inscripción en escenarios atravesados
por profundas e inéditas transformaciones políticas, económicas, sociales y culturales.
Quisiéramos −para cerrar− puntualizar brevemente dos aspectos: los dilemas que plantean la
fragmentación y la igualación social al trabajo docente, y la potencialidad de la tarea de transmisión
de los/as profesores/as y maestros/as en este nuevo tiempo, con las nuevas generaciones. La
situación caracterizada por acelerados cambios, precarización y débiles apoyos, junto a la alta
fragmentación y exclusión social y la configuración de nuevas subjetividades, pone frecuentemente en
cuestión la potencialidad de la escuela −y del trabajo docente− como espacio privilegiado de traspaso
del orden cultural. El sistema educativo argentino abrazó una idea de igualdad que generó un gran
crecimiento y movilidad social para amplios sectores; a la vez, como efecto del mismo proceso,
construyó una concepción de igualdad homogeneizante y autoritaria que dejó fuera de consideración
muchos aspectos humanos culturales y políticos. Para esto, la posición que se construyó para el
docente, mano eficaz de esa imposición, tuvo mucho que ver con el desarrollo efectivo de esa escuela
que se llamaba "común y pública",la igualdad debería empezar a pensarse como una igualdad
compleja, como una igualdad que habilita y valora las diferencias que cada uno porta como ser
humano, por eso convalidar la desigualdad y la injusticia. Observar lo que aprenden los chicos y
construir pronósticos desde una posición preocupada por generar igualdad para que accedan al
patrimonio cultural que les pertenece por derecho, o −a diferencia de esto− pensar que hay
condiciones de partida que los inhabilitan, construye claras posiciones determinantes acerca del éxito
o fracaso y, por lo tanto, acerca del futuro que se les augura.
La imposibilidad de una buena experiencia educativa. Que haya sujetos que puedan educarse
depende de lo que hagamos con ellos en la escuela, no sólo de lo que hagan la familia o la sociedad:
depende de cómo los recibamos y los alojemos en una institución que los considere iguales, con
iguales derechos a ser educados y a aprender los alcances y los límites del trabajo docente y de la
acción escolar.
¿De qué forma se puede hacer conocer el mundo y transmitir la tradición a los "recién llegados"?
¿Cómo se genera la autoridad del maestro? En este sentido, conviene hacer una distinción entre
tradición y mandato. Este último se relaciona con la aceptación y el cumplimiento. En la tradición, en
cambio, hay rasgos en los cuales reconocerse, y es así como se renueva un vínculo de autoridad
respecto del presente y del futuro.
Esa autoridad de la tradición puede construir posiciones conservadoras, renovadoras o críticas, y es a
través de ese gesto, de cualquiera de esas posiciones, que se la vuelve propia. Esto implica que la
tradición es centralmente una transmisión de concepciones, de una mirada del mundo que encierra
valores y les da sentido a las instituciones. Dentro de ellas, modela también posiciones para los
individuos; allí, entonces, la tradición es transmisión para que otros la hagan suya al renovarla.
Qué transmitir, por qué y cómo, son preguntas fundamentales del quehacer de los enseñantes.
La docencia es un trabajo social que se constituye en el entramado de diversas experiencias
−escolares y extraescolares−, y ese diálogo con la cultura de su tiempo es central para los puentes que
los docentes habilitan para sus alumnos. Este es el nuevo suelo en el que puede nutrirse nuestro
trabajo −además de hacerlo en el diálogo con las nuevas generaciones−, con problemas clásicos, con
otros nuevos, y con herramientas que tienen experiencia, que han sido revisadas pero que a la vez
tienen una potencialidad renovada.
Recurrir a nuevas preguntas, a hacer visible lo oculto y permitirse entrar en contradicción con el origen
de las prácticas docentes para incluir nuevas miradas que contemplen la novedad de situaciones, la
pluralidad de infancias, adolescencias y juventudes que caracterizan nuestro tiempo, y estar
preparados además para acompañar circunstancias hasta ahora inéditas.
Tarea compleja que requiere formación y reflexión acerca de la experiencia. Para ello es necesario
reconocer la centralidad de algunas cuestiones en la tarea docente, y redefinir nuestro trabajo y
nuestro papel por medio de la recuperación de la responsabilidad y la importancia del oficio de
enseñar.

¿Qué es, entonces, enseñar? Podemos concluir que es un trabajo profesional que posee una tradición
en la cual se respalda pero a la que puede revisar porque entiende que no se trata de un mandato
inapelable. Por el contrario, la docencia toma esa tradición para renovarla, y en esa renovación
cultural se autoriza. Se trata, en definitiva, de brindar ese conocimiento como un puente, ayudando a
cruzar fronteras en una relación mediada que liga y también habilita para la renovación, que genera
responsabilidades −micro y macro, individuales, colectivas e institucionales− en la construcción de una
posición con otros y con nuestro tiempo.

● Kaplan- La implicación afectiva en tiempos de pandemia y en la postpandemia. Educar para


una sociedad de reciprocidades
RESUMEN
En este ensayo esbozo una serie de reflexiones sobre la necesidad de que la escuela se constituya en
soporte emocional y funcione como refugio para las infancias y juventudes, habilitando la posibilidad
de constitución del lazo social.
Las relaciones afectivas en las sociedades modernas se refieren a las formas de compromiso que
establecen los sujetos entre sí y con su realidad.
La experiencia escolar inédita que se origina a partir de la pandemia por el virus SARS-CoV-2 nos
interpela a una pedagogía del trauma que posibilite la tramitación de narrativas del dolor social. El
dolor es una experiencia humana altamente simbólica. El sentir del dolor, es decir el sufrimiento, es
consecuencia de una relación afectiva y significante con una situación. La relación con el dolor es
siempre una cuestión de significación y de valor. Se entrama en la afectividad, que da la medida de su
intensidad y su tonalidad. Se trata de un hecho situacional, aislable en un sujeto que lo padece, pero
modelizado por la trama social, cultural, relacional que impregna ese sufrimiento (Le Breton, 1999).
El sufrimiento puede ser interpretado, entonces, como una experiencia comunitaria y que puede ser
colectivizado a partir de los relatos biográficos personales. Estar disponible para la escucha y
comprender la perspectiva de los demás es una condición de posibilidad para la socialización de los
sentires hacia la construcción de soportes socio-psíquicos.
El lenguaje y la narración adquieren una importancia fundamental en el proceso de elaboración del
padecimiento de los actores de la comunidad educativa.
Educar para la sensibilidad hacia los demás es un imperativo ético pedagógico en sociedades
desiguales atravesadas por el sufrimiento individual y colectivo que la pandemia agudizó. Si aspiramos
a construir una sociedad de reciprocidades, ella se crea mediante la educación para el respeto mutuo
que consiste en: cuidar de sí, hacer algo por sí mismo y ayudar a los demás (Sennet, 2003). La trama
vincular estructura subjetividades en el marco de una pedagogía del cuidado.
El desafío que interpela a la institución escolar es acerca de cómo contrarrestar el sentimiento de
inferioridad que incide en la construcción de lo que Goffman (2009) ha caracterizado como un proceso
de fabricación de identidades deterioradas. La construcción de la autoestima es un eje vertebrador de
la experiencia escolar.
La escuela constituye un territorio simbólico de esperanza en la medida en que allí o desde allí pueden
tejerse lazos de solidaridad y reconocimiento mutuo. La institución educativa participa
contrarrestando los muros emotivos que nos dividen, nos separan, nos segregan y nos excluyen.
Permite atenuar las marcas de la desigualdad durante el proceso de escolarización.
La reciprocidad en el cuidado del otro constituye un elemento central de los procesos educativos que
se despliegan en una situación de emergencia sanitaria. La escuela es un escenario privilegiado para
la construcción de lazos de pertenencia toda vez que se erige como lugar habitable, sin distinción. Es
el espacio simbólico donde prima la promesa de un “nosotros” que rompe con la instantaneidad de la
individualización (Kaplan, 2017).

LA EMOCIÓN COMO CATEGORÍA PEDAGÓGICA


El lenguaje de las emociones es una gramática fértil para comprender la construcción de vínculos y las
vivencias subjetivas de los actores de la comunidad durante la experiencia inédita de escolarización
en pandemia, donde se han transformado las ritualidades y las formas de interacción. En la
excepcionalidad de la experiencia escolar que nos toca vivir, las coordenadas de espacio y tiempo se
resquebrajaron.
Asumimos que los sentimientos funcionan como organizadores del lazo social. No se trata de
sentimientos sueltos ni apriorísticos, sino que es dable pensar que lo que se conforma es una red o
estructura emotiva que posibilita producir trama escolar. Una cultura afectiva forma un tejido
apretado donde cada sentimiento está situado en perspectiva dentro de un conjunto.
No se pueden comprender las experiencias emocionales sin ponerlas en contacto directo con una
situación específica y con un entramado, con la forma con la cual una cultura afectiva escolar se
mezcla directamente con el tejido social. Las transformaciones en los modos de vivir modelan nuestra
estructura emotiva.
Bajo las consideraciones precedentes, resulta importante puntualizar qué se entiende por las
emociones en la vida escolar desde un enfoque sociocultural e histórico.
Una perspectiva de análisis sobre la construcción social del orden afectivo escolar debe partir de la
premisa de que ninguna de las formas de comportamiento ni las disposiciones para sentir (habitus
emotivo) pueden catalogarse como naturales, aunque así se perciban y se vivencien (característico de
la dinámica de la naturalización). Las disposiciones para sentir son producto del aprendizaje y se
interiorizan bajo la forma del inconsciente social que lleva impreso las marcas de la memoria
biográfica y los signos de época.
La emotividad requiere ser interpretada a partir de la interrelación entre lo biológico y lo social en el
marco de ciertos patrones culturales e históricos. Los comportamientos sociales y las experiencias
emocionales son producto de continuos movimientos inconstantes. De ello se desprende que la
estructura emotiva es cambiante y necesita ser anclada en procesos sociohistóricos y culturales más
amplios. Para pensar la vida afectiva, es preciso considerar que los procesos de construcción y
transformación psicológica y social sólo pueden ser entendidos en conexión; ligando las
transformaciones de largo alcance de la estructura social y la estructura emotiva.
A los fines de alcanzar una comprensión profunda de las experiencias escolares nos posicionamos en
un horizonte epistemológico bajo el supuesto de que ni las emociones pueden ser abordadas sin tener
en cuenta la dimensión estructural material de lo social, ni de que esta última puede ser interpretada si
no se pone en juego la producción de la subjetividad.
Las emociones son construcciones culturales situadas que tienen una historia. En la medida en que
están condicionadas por los contextos sociales, no es posible comprenderlas si no atendemos la
perspectiva relacional de los sujetos. La emoción no es una naturaleza descriptible sin contexto, ni
independiente de las interacciones y significados atribuidos por los actores escolares.
Los soportes simbólicos operan como una protección donde la existencia individual y colectiva se
superponen.
Es indudable el carácter constitutivo de los otros (sean personas, símbolos o instituciones) en la
producción de toda emotividad. Para autoafirmarse el sujeto precisa de la mirada del otro, pues éste
se organiza subjetivamente en la relación interpersonal con otros sujetos, en una relación intercultural
y social (Wieviorka, 2009). De allí la importancia de ayudar desde la escuela a tramitar los sentimientos
que producen las prácticas de humillación y exclusión para fortalecer la valía social.
Es preciso entonces pensar de un modo integral los límites objetivos que marcan de entrada a las y los
estudiantes en su experiencia y trayectoria.
En este proceso de construcción de autoestima o de descrédito, la trama afectiva escolar de
aceptación o rechazo juega un papel estructurante.
Por medio de los juicios, las clasificaciones y los veredictos que la institución educativa realiza, cada
estudiante va incorporando de un modo inconsciente sus límites y sus posibilidades simbólicas,
estableciéndose como efecto simbólico una conciencia de los límites o efecto de destino (Kaplan,
2017). No obstante, instituciones y docentes tienen márgenes de autonomía y creatividad para inclinar
el péndulo a favor de la ampliación de las posibilidades simbólicas con miras a subvertir el orden
injusto.
Se trata de romper ciertas creencias sociales muy naturalizadas sobre la inevitabilidad de ciertos
destinos.
El orden escolar colabora en la producción de ciertas tramas y prácticas de afectividad (Kaplan, 2018).
En un escenario de virtualización forzada y una redefinición de la relación yo-otros en las diversas
formas que asume la presencialidad en las instituciones en el proceso de escolarización en pandemia,
la escuela puede funcionar como un lugar de encuentro. Ello es así debido a que la escuela opera
como una presencia material, pero también, como una presencia simbólica en la constitución del lazo
social.
La mirada escolar posee una innegable fuerza simbólica en la constitución de la autoestima ya que las
y los estudiantes se miran en ese espejo.
Resulta relevante generar las condiciones para que la comunidad educativa pueda objetivar
sentimientos de duelo, de ruptura de vínculos, de pérdida de soportes afectivos, que marcan los
sentidos del proceso de escolarización en el transcurrir de una experiencia traumática. Se trata de
transformar los miedos y los sentimientos de pérdida en esperanza, dado que la escuela representa
algo del orden de lo vital y de proyección del porvenir.
Los sentimientos de pérdida, miedo y soledad se agudizaron en el contexto de la pandemia. Se
observa que en esta experiencia de trauma social se expresa una red emotiva que transita desde el
miedo frente a la posibilidad de perder las oportunidades académicas que prometían las expectativas
subjetivas respecto del ciclo escolar, hasta los miedos frente a la muerte y la enfermedad. La
posibilidad de la pérdida de una persona que ocupa un lugar significativo en el entramado de
relaciones socioafectivas se convierte en un acontecimiento que faculta la pregunta por la propia vida
“la intensidad, el tipo y la estructura de los miedos que laten o arden en el individuo [...] aparecen
determinados siempre por la historia y la estructura real de sus relaciones con otros humanos, por la
estructura de su sociedad y se transforman con ésta” (Elias, 1987: 528).
Al mismo tiempo, la soledad es un sentimiento vinculado a la falta de contacto. Si las profundas
relaciones de dependencia emocional que establecemos en la vida se convierten en una necesidad
vital, el sentimiento de soledad que se experimenta en el mundo social suele estar en la base del miedo
a no existir para los demás. Al mismo tiempo, la soledad es un sentimiento vinculado a la falta de
estima y reconocimiento.
Si las profundas relaciones de dependencia emocional que establecemos en la vida se convierten en
una necesidad vital, el sentimiento de soledad suele estar en la base del miedo a no existir en los ojos
de los otros. El aislamiento emocional entre los individuos estructura la sensación de que la existencia
carece de significación ante la pérdida de vinculaciones afectivas que fundan el apego mutuo en el
tejido humano. De allí que experiencia de escolarización en pandemia y en las condiciones de la
postpandemia se necesita crear y recrear puentes afectivos que operen como anclajes intersubjetivos.

UNA REFLEXIÓN ABIERTA


Si bien es cierto que nada reemplaza a los vínculos que se generan en las modalidades de la
presencialidad, la mirada protectora de la escuela, incluso en la continuidad pedagógica no
presencial, simboliza un sostén emocional, un lugar simbólico donde amarrarse subjetivamente. En el
particular cruce entre la presencialidad y la no presencialidad, así como de otras múltiples formas de
la experiencia escolar que iremos construyendo en lo sucesivo, necesitamos abrazar la idea de
reparación simbólica.
Uno de los mayores desafíos con miras hacia la postpandemia es la de avanzar en la producción de
condiciones educativas, curriculares y socio-afectivas, que contribuyan a revertir las profecías de
fracaso que se ciernen, en especial, sobre niñas, niños y jóvenes de sectores sociales desfavorecidos.
Para ello, ampliando recursos simbólicos que permitan armar nuevas escenas frente a una realidad
compleja de tramitar en soledad, y también, abandonando por fin viejas antinomias entre el contenido
académico y el contenido socializador del proceso de escolarización. Si hay una lección que nos deja
esta experiencia inédita es que no se puede escindir lo académico de lo vincular.
Sin afectividad, sin afectación subjetiva, sin movilización emocional, no hay posibilidad de estructurar
una trama que promueva los procesos colaborativos y fraternales de enseñanza y aprendizaje. Para
ello es preciso reponer una mirada del estudiante que le otorgue valía social. La mirada escolar posee
una innegable fuerza simbólica en la constitución de la autoestima ya que las y los estudiantes se
miran en esa imagen, y tienden a reconocerse o a negarse. La experiencia intersubjetiva de
reconocimiento significa la reivindicación de la identidad, mediante la mirada del otro que produce
autoestima.
La mirada social tiene la capacidad de formular juicios de valor (otorga y quita valor) porque se dirige a
las raíces inconscientes, en un sentido sociológico, de un sentimiento de identidad que depende de la
aprobación de los otros. El reconocimiento, del orden de lo simbólico, implica así la confirmación de la
aceptación y el cuidado del otro, tanto en el plano cognitivo académico, como en el socioafectivo.

Esta pandemia nos deja como lección aprendida la necesidad de trabajar colectivamente
sobre la afectación subjetiva de los procesos de escolarización. Podemos afirmar que las
interacciones escolares se estructuran a través de circuitos afectivos que trazan horizontes de
posibilidad. Si aspiramos a construir una sociedad de reciprocidades, ella se crea y recrea mediante la
educación para el cuidado socioafectivo en la trama escolar. Se trata de educar para que el
sufrimiento del otro nos conmueva y que el otro se movilice por nuestro dolor.

● Ministerio de Educación de la Nación Argentina Pensar los vínculos en tiempos de pandemia : la


escuela como un lugar de cuidado.

Introducción
La escuela ha logrado sobreponerse a importantes crisis y transformaciones sociales, constituyendo
un lugar de resguardo, cuidado, encuentro, enseñanza y aprendizaje.
La irrupción de la pandemia ha generado tal conmoción que puso en el centro la pregunta por la vida
en común y la dimensión colectiva del cuidado.
La pandemia y su atravesamiento nos condujo a pensar de otra manera las condiciones de
enseñanza y aprendizaje, el entramado institucional y las formas de acompañamiento a niñas, niños,
adolescentes, jóvenes y familias instaurando -de algún modo- una reformulación del formato escolar.

“Pensar los vínculos en tiempos de pandemia: la escuela como un lugar de cuidado” Se trata de una
propuesta de acompañamiento a la reflexión y el trabajo en las aulas para docentes, equipos
directivos, equipos de orientación y otras personas que asumen roles pedagógicos en las escuelas
A través de este material, brindar algunas claves que aporten a:
● contribuir en la construcción de una pedagogía del cuidado en las escuelas, fortaleciendo las
propuestas de enseñanza para brindar a las nuevas generaciones saberes necesarios con los
que habitar el mundo;
● propiciar espacios para compartir experiencias;
● elaborar narrativas singulares y comunes, ofreciendo relatos y sentidos a partir de los cuales
significar la pandemia;
● promover el ejercicio de una autoridad pedagógica que facilite la construcción de un porvenir
frente a contextos de incertidumbre;
● favorecer espacios que den lugar a la tramitación de las pérdidas individuales y colectivas.

La escuela en tiempos de pandemia


La pandemia vino a corroborar que la función de la escuela trasciende la acreditación de saberes, por
lo que no se trataría solo de la transmisión de contenidos sino de la oportunidad de construirlos entre
docentes y estudiantes, armando experiencias educativas que resulten significativas.

La pandemia nos vino a mostrar, una vez más, que la escuela co-construye objetos de conocimiento y
deviene, asimismo, en un espacio con tiempos propios para el despliegue de formas singulares en las
que cada estudiante y cada grupo se expresa, aprende y se vincula con otras y otros. En ese sentido,
nuestra responsabilidad como docentes supone el ejercicio de una autoridad pedagógica que
promueva aprendizajes y, a la vez, habilite procesos de subjetivación a partir del ofrecimiento de un
espacio material y simbólico, con los instrumentos necesarios para ocuparlo. Para incluirse en el
mundo las/os niñas, niños, adolescentes y jóvenes precisan de personas adultas que confíen en ellas
y ellos, que las y los desafíen, y que establezcan límites humanizantes. De este modo, nuestra posición
implicaría la responsabilidad de hacerse cargo de otras/os, en el marco de una relación de confianza
que es instituyente.
La propuesta didáctica hizo de puente para saber quién acompañaba a cada estudiante del otro lado
de la pantalla, cómo estaban transitando el confinamiento y qué extrañaban de la cotidianeidad
escolar.

La pandemia lo que hizo fue hacer visible la centralidad del vínculo pedagógico, en el cual reside -por
supuesto- la autoridad pedagógica, pero la autoridad entendida no en el sentido de una imposición, o
de una cuestión que baja desde arriba, sino la autoridad pedagógica entendida como habilitación,
como la posibilidad de abrirle al estudiante y a la estudiante nuevas posibilidades y ofrecerle las
herramientas para conocer el mundo. A partir de la pandemia muchos y muchas docentes advirtieron
que ese vínculo es central en la transmisión del conocimiento, o de los saberes (María Beatriz Greco).

En este sentido, podríamos decir que la escuela demanda el armado de escenarios de ternura (Ulloa,
2005) sobre la base de una pedagogía del cuidado, que permitan el despliegue de experiencias
subjetivantes posibilitando dar sentidos a lo que nos pasó. Desde este enfoque, cuidar no se limita a
resolver situaciones “emergentes” o enseñar contenidos relativos al reconocimiento del propio cuerpo
y de las/os otras/os, sino que se vincula también con garantizar a las nuevas generaciones la
distribución y el acceso a saberes necesarios para conocer y participar del mundo que las/os rodea.

Posicionarse desde una perspectiva de cuidados implica, entonces, un movimiento a partir del cual
la/el otra/o no es un sujeto al que prevenir o del cual prevenirse, sino un sujeto de derecho que habita
la escuela mientras construye su propia identidad. Ello requiere necesariamente de una propuesta
educativa integral que comprenda que la formación, además de avanzar en el dominio de la lectura,
la escritura y el cálculo, también promueve el desarrollo de la conciencia de sí misma o sí mismo, del
propio cuerpo, del cuerpo de las otras personas, de las emociones, de los distintos niveles de
autonomía, del propio lugar en la sociedad. En ese sentido, resulta clave ofrecer experiencias en las
que las/os estudiantes puedan decidir, organizarse, implicarse y participar activamente poniendo a
disposición horizontes culturales diversos y plurales.
De este modo, el acompañamiento, intenta brindar herramientas, cada vez más complejas, para que
ellas/os mismas/os puedan pensar, reflexionar, cuestionar, imaginar, e interpretar sus experiencias.

Se trata de restituir a un sujeto activo. Es decir, no solo acompañar, porque acompañar tiene a veces
ese desliz de que una/o es la/el que acompaña y otra/o es la/el acompañada/o y eso deja al sujeto en
un lugar pasivizado, victimizado. Resulta mucho más interesante poder buscar recursos que
restituyan a un sujeto activo. Es decir, que pueda ser activo también en el lazo con otros y otras, en
armar una red solidaria, tener una función más activa en lo que es ese acto solidario con la/el otra/o
que le devuelva también algo de la vitalidad que se ha visto afectada (Paula Husni).
En este punto cobra relevancia el trabajo sobre la afectividad en las escuelas pensando en el afecto
entramado con el acto pedagógico. Generar espacios de circulación de la palabra y de participación
democrática en los que niñas, niños, adolescentes y jóvenes puedan ser escuchadas/os. Espacios
que rompan con una posición y forma de explicación adultocéntrica y que promuevan el
protagonismo de las/os estudiantes para la narración de sus propias vivencias y experiencias.
Se trata de advertir, en palabras de Sara Ahmed (2016), la relevancia política de los afectos,
distanciándonos de prácticas que pretenden regular, educar e individualizar las emociones,
reconociendo su complejidad y posibilitando su despliegue.
Entonces una de nuestras funciones sería la de intermediar, acompañando a que niñas, niños,
adolescentes y jóvenes construyan interpretaciones y posicionamientos propios, con la prudencia de
no apresurarnos frente a la preocupación misma que nos genera, ni intervenir con preguntas directas
sobre lo que sienten y piensan. Es posible, así, validar y reconocer el tiempo subjetivo en el cual cada
persona puede significar lo que sucedió a través del ofrecimiento de representaciones diversas (como
la escritura, el arte, el cine, la literatura, el juego, proyectos científicos), en soportes y dispositivos
diferentes en pos de promover la tramitación y simbolización de lo ocurrido, de forma individual y
también colectiva.
Tenemos que generar propuestas de participación activa de los y las estudiantes para que transmitan
lo que ha sido la experiencia de la pandemia a través de producciones artísticas, culturales. Espacios
que posibiliten la representación de las niñeces y las adolescencias, que nos permitan corrernos de un
lugar adultocéntrico a través del cual queremos hasta incluso coordinar cómo deberían expresar sus
sentimientos, sus emociones y vivencias (Horacio Paulin).

Subjetividad y escuela, ¿cómo alojar los efectos de la pandemia?


Consideramos a la subjetividad como un proceso de formación de un modo de ser, pensar y hacer, en
vínculo con otras/os dentro de un orden social y discursivo determinado. Es una construcción que se
da en la relación con otras/ os, en tiempos y lugares, en donde circulan saberes, objetos de
conocimiento, pensamientos, sentires. Es la resultante de una construcción socio-histórica que habla
de nosotras/os, de nuestras/os estudiantes, pero también de las formas de vincularnos y las marcas
propias de cada época. Dentro de la escuela las formas de subjetivación refieren a un “entre”, que se
construye en la trama vincular: docentes-docentes, docentes-estudiantes, estudiantes-estudiantes,
docentes-familias, familias-estudiantes.

Toda época presenta características que dejan marcas y moldean los modos de ser, vincularse,
actuar, sentir, a los que podríamos llamar “subjetividades de época”. Este concepto tiene la
potencialidad de referirse tanto a los aspectos singulares de cada persona, como a los construidos
colectivamente.
La situación epidemiológica de la pandemia ha reeditado sentimientos y sensaciones asociadas al
desamparo estructural de la condición humana. Produjeron sufrimientos particulares por las
características del ciclo vital que se encuentran atravesando, niñas, niños, adolescentes y jóvenes.

Durante el año 2020, UNICEF junto a otras organizaciones llevó a cabo un estudio sobre los efectos de
la pandemia en niños, niñas y adolescentes, con el fin de conocer las principales necesidades
emocionales y los cambios en los comportamientos durante ese primer año de pandemia. A través de
la investigación se pudo conocer que:
● Algunas/os niñas, niños y adolescentes atravesaron emociones de soledad, tristeza,
ansiedad, miedo.
● Quienes estuvieron cercanos a la finalización del ciclo secundario, manifestaron vivencias
ligadas a la incertidumbre en relación a proyectos futuros.
● En adolescentes de sectores vulnerables la angustia se profundizó debido a las privaciones
materiales.
● La posibilidad de soñar con un futuro posible se recupera como una cuestión fundamental
para la construcción identitaria.

El trabajo de indagación permitió reconocer también algunos factores asociados a estos


padecimientos como pueden ser deficiencias o insuficiencia del acompañamiento de personas
adultas significativas, déficit habitacional, situaciones de violencia, precaria situación económica,
violencias institucionales, ausencia de espacios donde sentirse escuchadas/os y/o comprendidas/os,
dificultades o insuficiencia de contacto con la escuela, distanciamiento en el vínculo con las y los
docentes, sentimientos de discriminación y estigmatización, entre otros.
El informe de UNICEF, publicado en el año 2022, destaca la persistencia de manifestaciones y
padecimientos experimentados en la primera infancia pese a la mejora de los indicadores
epidemiológicos. Entre ellos, revela alteraciones en la alimentación, en el sueño y dificultades en los
procesos comunicativos. En relación a las adolescencias, el estudio señala una mejoría en el estado
anímico respecto a los dos años anteriores, una mayor motivación frente al estudio y el
reconocimiento de la alegría por volver al espacio escolar.
Podemos pensar el padecimiento como una respuesta psíquica frente a la conmoción que produjo la
pandemia y el consecuente distanciamiento social.

Si bien el atravesamiento por la pandemia ha sido global, existen formas singulares de vivirlo.
Conocerlas, pensarlas, analizarlas, con el cuidado de no llevar a cabo generalizaciones “a todas/os
nos pasó lo mismo” o de establecer enunciados patologizantes “las/os estudiantes se encuentran
deprimidas/os”, será parte de nuestra tarea y de las formas de acompañar. Esto equivale a alojar los
efectos de los malestares, como un llamado a pensar en las subjetividades contemporáneas dentro
de este entramado, y lejos de posiciones esencialistas y patologizantes.

Los procesos de subjetivación suceden mientras aprendemos, nos vinculamos, nos equivocamos y
nos recomponemos; y los conocimientos y saberes circulan, con palabras, pensamientos,
afectividades que se van construyendo en ese tiempo suspendido (Simons M., Masschelein, 2014) que
es la escuela. Tiempo “suspendido” porque se diferencia y se distancia del tiempo de la producción y
de la exigencia del mercado, poniendo una distinción temporal diferente que no se corresponde con la
aceleración consumista y con las formas de desigualdad. En este horizonte, la escuela puede
funcionar como ese tiempo indispensable de formación en donde hacer parte a todas y todos desde
una perspectiva de igualdad.

Colocar la pandemia en la historia


Si bien actualmente hemos retomado muchas de nuestras rutinas cotidianas previas al inicio de la
pandemia, el regreso a la presencialidad en las escuelas no sucedió del mismo modo al que
estábamos acostumbradas/os. El reinicio en burbujas, dar clases con barbijo, los escenarios híbridos
que se fueron configurando, las demandas de las familias frente a situaciones de contagio,
impactaron en las formas de hacer escuela, al mismo tiempo que visibilizaron diversos padecimientos
de las/os estudiantes con los que tuvimos y tenemos que trabajar y aprender a contener.

Creo que hay un trabajo de colocar en la historia a la pandemia, eso que aparece como un
acontecimiento, hay que empezar a darle una inteligibilidad histórica porque la pandemia no es
solamente el virus que irrumpe, es un modo de vivirlo, y tiene un montón de continuidades que ya
empezamos a verlas. Tenemos que empezar a entramarla con lo que estaba antes, con lo que viene
después. (...) No fue una ruptura en el continuum de la historia (...) el gran desafío ahora es poder
pensar lo que sucedió en relación con la pandemia; lo que ha hecho visible, lo que trajo de nuevo
(Miriam Kriger).

Colocar la pandemia en la historia deviene en una doble necesidad. Por un lado, colocar a la
pandemia en la historia singular de cada quién supone darle una inteligibilidad a lo acontecido en la
historia personal, hacerle un lugar y entramarla con el resto de las vivencias, para poder seguir
adelante teniendo en cuenta lo sucedido; porque la pandemia no es solamente un virus que irrumpe y
trastoca la vida, sino que implica encontrar un modo de transitar aquello que sucedió haciéndolo
parte y pensando cómo continuar. Por otro lado, colocar la pandemia en la historia implica también
construir una historia en común, generando condiciones de resignificación o de significación de la
experiencia, de habilitar espacios de producción de sentido colectivos políticos, sociales, que vuelvan
posible a los sujetos posicionarse de un modo activo y solidario frente a los tiempos que nos toca
atravesar.
Tramitar socialmente los procesos de duelo colectivos posibilita colocar el proceso de duelo individual
en la esfera de lo público, comprender que se trata de un fenómeno que afecta a toda la sociedad y
asimilar que entraña un dolor colectivo que requiere ser elaborado de forma conjunta.
Dar lugar al dolor en términos colectivos, no solo contribuye a reconstruir los lazos sociales que
indudablemente se vieron afectados por la pandemia sino que también es necesario para imaginar y
construir un proyecto como sociedad presente y futura.
Podríamos decir, entonces, que la potencia de la escuela está en la producción de relatos que no son
sólo individuales sino también colectivos. Colocar la pandemia en la historia en esta doble vertiente
(en la dimensión sociopolítica y en la dimensión de la trayectoria personal) implica restituir a las/os
estudiantes en un lugar activo, propiciando el armado de narrativas desde las cuales situarse,
brindando sentidos frente a una realidad que apareció intempestivamente y nos llena de
incertidumbre.
Las construcciones, los espacios, los dispositivos tienen que permitir la producción de sentidos
colectivos, políticos, sociales, tienen que dar condiciones de significación. Por ejemplo, hablar del
capitalismo, hablar del cuidado, del ambiente en relación a estos tiempos, ahí entonces está entramar
con lo sucedido. Me parece que el mayor riesgo (...) para cualquier acompañamiento que uno piense,
es que quede desligado de la enseñanza misma, porque los recursos subjetivos que aporta la escuela
están en relación a los recursos simbólicos que aporta la enseñanza (Laura Kiel).

De duelos y transformaciones
Resulta necesario reconocer que algo se perdió. Se han perdido vidas, pero también prácticas y
modalidades de encuentros; se han perdido rutinas y hábitos; se han resignado expectativas, deseos
y proyectos. Hay estudiantes que no pudieron llevar a cabo su ritos de pasaje de nivel educativo, hay
cumpleaños que no se han podido festejar de la forma que se venían haciendo, situaciones
significativas a las que hubo que renunciar. Como en toda pérdida, la afectación es singular, sin
embargo, las niñeces y adolescencias merecen especial acompañamiento para poder realizar un
trabajo de duelo sobre aquello que se perdió. La pérdida, en cuanto tal, necesita ser sostenida y
elaborada. Poder reconocer esa pérdida, procesarla, simbolizarla y/o nombrarla, permite generar
condiciones de posibilidad para que esa energía se encuentre disponible y pueda reubicarse en otros
espacios, propuestas, actividades.
La tramitación de la pérdida -siendo reconocida y no negada- nos permite ir re-conectando con los
proyectos, con el deseo. ¿Cómo acompañar la elaboración de la pérdida en la escuela?
Dentro del trabajo de elaboración de las pérdidas, el hecho de generar preguntas y/o
problematizaciones -que no apunten a una respuesta única- resulta fundamental. Pensar qué
interrogantes pueden incluirse dentro del espacio del aula y formular problemas es clave para permitir
aproximarnos a lo que ya conocemos, pero también a lo que no terminamos de entender

Nuestro desafío tiene que ver con reponer sentidos, re-encarar la tarea educativa, imaginar otros
modos de enseñar y aprender y convivir, y entramar lo que sucedió con el futuro. Así, tras este trabajo
compartido, de elaboración de la pérdida y simbolización, será posible generar condiciones para el
despliegue de la subjetividad fomentando formas de vincularnos y de aprender de otros modos.
Historizar, entramar, simbolizar, tramitar lo que sucedió, ponerle palabras, requiere de la articulación
conjunta de la comunidad educativa. Para que estos procesos tengan lugar, cada institución tendrá
que configurar diversos dispositivos institucionales en función de pensar qué proyectos, propuestas,
estrategias pueden ser viables de crear en pos de que esta tarea se lleve adelante entre todas/os, y
no quede circunscripta a la “buena voluntad” de un/a docente en soledad.

El cuidado como respuesta colectiva


El cuidado resulta un concepto polisémico y, como muchos otros, en continuo proceso de
construcción teórica. Históricamente, en la escuela la idea de cuidado estuvo ligada al concepto de
prevención, atravesada por los enfoques higienistas, biologicistas y sanitarios. Lejos de ser
paradigmas superados, estos discursos persisten en posiciones acerca de la función social de la
escuela, los contenidos a ser transmitidos, la concepción de los sujetos de aprendizaje y la
construcción del vínculo pedagógico. Hoy, el concepto de cuidado circula cada vez más en nuestra
sociedad y ha tomado especial lugar desde el comienzo de la pandemia en los discursos sociales,
políticos, mediáticos, educativos. Más allá de sus múltiples significaciones, coincidimos en que es un
elemento central del bienestar humano. No hay nadie que pueda sobrevivir sin ser cuidada/o o sin
haberla/o sido Se trata de una actividad que realizamos para mantener, reparar y conservar nuestro
mundo, nuestro entorno, nuestros cuerpos, nuestras subjetividades. En este sentido, las tareas de
cuidado no se limitan a cuidados físicos, básicos y vitales, como alimentar y dar refugio, sino que
cuidar implica también una dimensión afectiva que entraña el escuchar, acompañar, habilitar,
preocuparse por la/el otra/o, confiar en lo que puede. El cuidado se configura, de este modo, como
una categoría central para la vida cotidiana e interpela al ámbito educativo animándonos a pensar:
¿Cómo construimos una pedagogía del cuidado en las escuelas?
Pensar prácticas y vínculos en la escuela desde un enfoque de cuidado requiere fortalecer la
construcción de espacios democráticos, dialógicos y participativos, que propicien la escucha y la
colaboración priorizando el protagonismo de niñas, niños, adolescentes y jóvenes.
La escuela cuida cuando enseña, no solo desde los contenidos sino también a través de los vínculos y
espacios que se constituyen a partir del entramado institucional y que forman parte de la vida
cotidiana en las escuelas. En este sentido dar lugar y alojar las vivencias de cada estudiante se
transforma en una oportunidad de aprendizaje para la convivencia con otras y otras.

Considerar la pandemia desde una pedagogía de cuidado implica tener presente que existen
múltiples formas de abordaje sobre lo que sucedió en estos tiempos y que, de acuerdo al momento, al
espacio y al nivel educativo, a veces será necesario ubicar la pandemia en el centro, otras veces
acercarse desde los bordes y, en otros casos, implica generar espacios para descentrarnos de ella.
De este modo, se espera que puedan brindarse oportunidades diversas para significar el mundo,
construir narrativas en torno a lo vivido y promover proyectos a futuro.
Al igual que en otros procesos de duelo y pérdida, la pandemia puede ponerse momentáneamente en
suspensión, ofreciendo el acercamiento de otros saberes y experiencias que den lugar al deseo de
nuevos objetos de conocimiento, lo cual también resulta necesario para poder tramitarla. De esta
manera, se trataría de generar propuestas en las que niñas, niños, adolescentes y jóvenes se sientan
convocadas/os a través de sus opiniones, intereses y anhelos. Aquí proyectos ligados al deporte, las
expresiones artísticas, experiencias recreativas, toman centralidad en tanto se transforman en
oportunidades donde compartir con otras y otros; vincularse y comunicarse de formas diversas.

Promover la corresponsabilidad
Otra de las realidades que visibilizó fuertemente la pandemia es que la escuela no puede concebirse
como una institución aislada, ya que forma parte de un Sistema Integral de Protección de Derechos.
Por ello, resulta necesario consolidar un trabajo de articulación permanente con otros actores,
instituciones y sectores, a fin de favorecer la garantía de derechos de niñas, niños, adolescentes y
jóvenes.
Pensar la escuela como parte de una red requiere considerar el principio de corresponsabilidad, para
garantizar los derechos de niñas, niños y adolescentes sin perder la especificidad de cada sector y la
contracara del abandono que supone la fragmentación institucional. Estas articulaciones no surgen
espontáneamente y muchas veces requieren de largos caminos de conocimiento mutuo, que no
están exentos de tensiones ya que cada espacio institucional persigue finalidades diferentes y posee
lógicas propias.
Para que esta red pueda consolidarse es necesario reconocer quiénes son las personas que se
encuentran detrás de los aparatos institucionales. Durante la pandemia los vínculos con las familias,
con los centros de salud, con distintas instituciones del Estado y organizaciones comunitarias, fueron
una parte fundamental del trabajo cotidiano en las escuelas y tuvieron un lugar central en el
sostenimiento de las trayectorias educativas. En muchas ocasiones, la pandemia nos obligó a realizar
un nuevo “reconocimiento” entre las/os distintas/os actores en función de las nuevas coordenadas.
Podemos situar entre ellas, la relación entre familias y docentes, donde en algunos casos se
generaron situaciones de mucha cercanía, y el vínculo de las escuelas con espacios de salud que se
resignificó a partir de la instauración de medidas sanitarias para la prevención del contagio. Sin
dudas, estas articulaciones requieren un trabajo sistemático y sostenido en función de pensar
estrategias en conjunto.
El acompañamiento desde una pedagogía del cuidado implica entonces un trabajo mancomunado,
con un criterio de corresponsabilidad, entre diversas personas, espacios e instituciones en pos de
construir redes que sostengan y promuevan los derechos de niñas, niños, adolescentes y jóvenes. No
solo realizando abordajes discursivos sino generando espacios para que los derechos se puedan
reconocer y ejercer.

EJE TEMÁTICO II

Los conceptos de Sujeto y Subjetividad.


El sujeto que enseña y el sujeto que aprende: relaciones posibles. Deseo de saber, deseo de enseñar,
procesos transferenciales. Confianza en el maestro, confianza en el discípulo. Enseñar, cuidar,
acompañar.

● Cornú - “Transmisión e institución del sujeto. Transmisión simbólica, sucesión, finitud.”

En la transmisión humana que nos preocupa creemos identificar tres términos estables:
1) el objeto de transmisión 2) el “transmisor” que se piensa decisor y consciente 3) “el receptor” aquel a
quien se le transmite que a veces se lo hace culpable cuando la transmisión parece no hacerse. Pero a
este esquema cómodo le faltan algunos aspectos del proceso, y en particular lo que se le constituye
entre sujetos y que por eso mismo se transmite como “construcción del sujeto”. Se trata entonces de
considerar ese “entre”, porque “entre – dos” revela ser, más que una voluntad unilateral, inductor.
El transmisor es un pasador que a su vez recibió, y que para luego transmitir, no es necesario que él se
borre. Pero eso que lo inscribió como sucesor le significa, por la misma razón, su finitud. Entonces, lo
que distingue la transmisión de la comunicación no es sólo la unilateralidad, sino la inscripción de una
temporalidad irreversible y en lugares disimétricos, ya que dicha transmisión es a la vez temporal y
simbólica. Lo trascendental en dicha transmisión es que el destinatario se transforme a su vez en
sujeto a través de una estructura simbólica y de palabra.
El objeto de la transmisión será transmitido según la manera en la que se haga. Las modalidades de
transmisión son también complejas, transmitir una misiva es construir al otro como destinatario, pero
transmitir un apellido es reconocer a un/a niño/a es humanizarlo. Es reconocer en el otro sujeto ese
saber, de desearlo, de entenderlo, de desarrollarlo. Construir al sujeto es primero plantearlo,
presentarlo, reconocerlo sin justamente pretender fabricarlo de pies a cabeza como si fuera un objeto.
Es no sólo despertar su curiosidad, su espíritu: es instituirlo como sujeto del conocer.
Sucesión, finitud, reconocimiento del otro, requerimiento de sujetos “en su propio nombre” son en
forma muy sumaria, rasgos de la transmisión humana.

- Sujeto, direccionamiento, reconocimiento


El sujeto es el sujeto del verbo que usted conjunta en primera persona o segunda persona.
Pero solo hay “yo” y “tú” si alguien habla, se dirige, actualiza el sentido disponible. Sólo hay sujetos en
primera persona, o segunda persona cuando hay un direccionamiento. Uno se presenta, se plantea
como sujeto cuando se dirige a otro, y otro se dirige a nosotros. No hay sujeto sin reconocimiento
recíproco, sin enunciación dirigida. Es decir, el sujeto es a través de la palabra, de la lengua, de la
lengua estructurada. El sujeto es el sujeto hablante.
Asimismo, no hay sujeto hablante sin la creatividad del uso del lenguaje, es el sujeto que habla y juega
el juego de la lengua.
A medida que habla, él mismo se releva a través de su palabra: ha sido inscripto en el lenguaje, él
mismo ha sido “hablado” antes de hablar, “objeto” de direccionamiento y lugares inconscientes. A
través del lenguaje desarrolla conciencia, constituyendo una vivencia interna como objeto de
comunicación y construir categorías para concebir el mundo.
Posteriormente el sujeto deberá hacerse cargo de aquello que no puede simbolizar con la palabra, lo
que forma parte de su “sombra”. Allí es cuando el sujeto juega con la falta caída, falta significada como
límite que lo proyecta en la función simbólica, y logrará el advenimiento de su subjetivación.

- El afán de actuar justo


Existe una búsqueda ética legítima y necesaria, una búsqueda de una cualidad/calidad del actuar,
búsqueda de actuar justo en sentido de justicia. Esta ética busca justicia en situaciones que sabe
singulares: no es codificable y puede ser silenciosa. Invita a un discurso reflectante, siendo una ética
aplicada a un muestrario de principios vivientes en actos juntos.
Sin embargo, la postura ética siempre puede ser tramposa por la misma racionalidad que implica el
enunciado. La posición ética es asimismo un trámite reflexivo, atento a la experiencia humana que no
es “científica”, ajena y erudita, reservada a los expertos. Es subjetiva en el sentido de remitir no tanto a
pruebas objetivas para ejercerla, sino pruebas personales donde la ética del direccionamiento solo es
“confiada” al sujeto. De allí el surgimiento de la ética en la transmisión, en un afán del actuar justo de
una transmisión instituyente.

- El nombre, el apellido, el decir, el direccionamiento, el responder


El sujeto es el acontecer de su nombre, es el nombre propio que lo hace ocupar un parentesco, lo
acerca a la transmisión de una filiación y la presentación de una singularidad.
Sin embargo, dicha singularidad, está prometida a advenir debido a que será constantemente
reconstruido.
El término identidad es un recurrente en las sociedades “mediáticas”, donde surge la pregunta ¿quién
soy?. Pregunta infinita, identidad indefinida, secreta, siempre por inventar, identidad abierta y narrativa
gestada por el nombre propio y el reconocimiento.
También debe decirse que esas identificaciones cómodas y sumarias son ruinosas para el
reconocimiento de la singularidad y el secreto inclasificable de la identidad del sujeto, debido a que
deben auto limitarse para ser reconocidas, ser asunto de revelación.

- El decir, el direccionamiento
El decir, el nombramiento del sujeto, es instuitirlo en la palabra separadora, la misma que permite
despegarse como sujeto y representarse el mundo. Dirigirse, es portar/gestar lo que tenemos que
significar en una relación de presencia, y no sólo decir que en algún lugar hay leyes escritas. No hay
sujeto sin institución simbólica, sin límites. Ahí es donde “la ética” confirma ser también política, si su
punto de mira son los sujetos “responsables”.

- El responder
Una ética de la responsabilidad se preocupa por las consecuencias de los actos y por la manera en la
que podemos hacernos presentes en lo que sigue, hacernos garantes de otros actos y por la palabra,
a través de iniciativas, inicios que exigen continuación.
Como transmisores, debemos tener la responsabilidad de y por el pasado, que podría verse arruinado,
por ignorancia, en razón de la llegada de los nuevos sobre todo en sociedades mediáticas donde se
hace peligrar lo tradicional por novedades.
Es necesario una transmisión democrática, aquella que se preocupa no sólo de transmitir “lo mismo”
por tradición, sino la posibilidad de que exista “lo otro” permitiendo el relevo y haciendo énfasis en la
libertad. La transmisión se hace entonces invitación a tomar lugar, a inventarse la propia subjetivación,
la propia emancipación.

- El don y la hospitalidad
El don del intercambio simbólico, es un don que circula, que es generoso y su simbólica hace circular
solidaridades, establece relaciones entre personas y/o grupos, a través de un lazo social simbólico. El
don es transgeneracional, no se entrega sino que se “dona”.
En cuanto a la hospitalidad, refiere a una cuestión de acogida y cuestión de lugar.
Entendiendo la ética de la transmisión, de la institución simbólica que se preocupa por los pasajes de
la infancia a la adolescencia y posteriormente a la edad mayor. Aboliendo los ritos de pasaje donde se
somete a los recién llegados para apostar por ritos de pasaje democráticos y profanos, donde
predomine la hospitalidad. Lo político como experiencia de acto común, de acción libre y de
posibilidad para los nuevos de cumplir su libertad, invitando a reconocer la educación como una
dimensión pre política; de espacio y de temporalidad, un espacio “entre”: para respirar, para ser libres,
para darle lugar a lo indeterminado.

___________________________________________________________________________
La transmisión es una problemática vital, es directa o indirecta, es visible o invisible, es finita, es
consciente o inconsciente, invita a tomar un lugar, a inventarse la subjetivación, la emancipación.
Supone objetos frágiles, seres mortales, tiene una estructura simbólica, transforma al sujeto
(quien habla y juega el juego de la lengua) lo instituye como sujeto conocedor, hablante. Hay un
reconocimiento del otro a medida que habla, el Sujeto toma conciencia y tiene que ser pensada en
términos éticos,de responsabilidad.
__________________________________________________________________________

● Cornú - “La ética de la oportunidad”

La ética de la oportunidad propone la noción de habilitación de la oportunidad, entendida como


ocasión de construir un futuro que no sea un callejón sin salida, y sí fue como surgió la idea de pensar
una ética en el campo educativo.
A la oportunidad se la asocia, y a menudo se la reduce, a la idea de ocasión; pero en el campo
educativo implementa un sentido de lo posible, cuya proyección e instrumentación son a la vez éticas y
políticas.

a. La ocasión o nunca:
La ocasión es aquello que nos cae delante en medio del camino, sin que influyamos en términos de
conocimiento o acción. Quien se deja guiar por ella desperdiciaría lo esencial en beneficio de lo
accidental, accesorio y azaroso. Valorizarla supone elegir una inscripción en el tiempo, aceptarla y no
negarla, exigiendo acción. Kairos en griego, es un momento crítico, donde lo que está unido o asociado
se separa para reanudarse o disgregarse. El arte de sacar partido de la ocasión puede hacernos creer
en la suerte; pero apoderarse de la ocasión fugitiva no significa librarse al azar, sino aprovechar la
calidad de ese momento a favor. Paciencia imbuida de vivacidad: capacidad de agarrar el caso al
vuelo, lo que toma en suerte en la ocasión y llega en el accidente.
La acción se teje con decisiones surgidas de razonamiento, juicios en presente, creaciones
instantáneas; siendo irreversible, la acción es acontecimiento de innumerables consecuencias que
desencadenan un proceso infinito. La ciencia de la oportunidad y la búsqueda del orden justo tienden
al mismo fin: se dice que se les asigna a las cosas el lugar más conveniente y apropiado. Tal lugar más
apropiado es precisamente el que constituye la oportunidad de los actos, el buen momento.
Reconocida por su fugacidad temporal, la ocasión incita a buscar lo que vale la pena. La acción se
repliega en prudencia y se produce algo así como un renunciamiento a actuar en lo real. La fugacidad
de la ocasión es un llamado a la clarividencia, a la vivacidad y el coraje que harán virtud: el poder está
al alcance de quien posea los medios de conquistarlo.
¿Puede la educación como la medicina y la política, tomar en serio el sentido de la ocasión? claro que
sí, en clase una pedagogía de la ocasión es la que hace fuego con lo que se presenta y aprovecha
creando ocasiones para instruir; atenta a lo infinitesimal, busca actitudes, atenciones e intentos del
alumno que pudiera pasar desapercibidos. Más improvisadora que progresiva, más disponible que
programada, la educación merecería echarles una pizca de sal, levedad y humanidad a las tercas
pedagogías por objetivos.
El arte de la oportunidad no es aleatorio ni mecánico, sino alta habilidad de encontrar pasajes,
invención de puertos y audacia en la partida, aventura de estrechos y travesías.
Significa la apertura, pasaje, orientación y destino final: todo lo que es ocasión favorable y apertura a
“otra cosa”; decisiones, no pasividad. Ocasión y oportunidad podrían considerarse hermanas, pero no
bien percibimos en la oportunidad la ocasión favorable, el término aporta matices distintivos: se la
juzga favorable, cobra relevancia con relación a un proyecto, obra en ella, hay constructividad y giro a
la apertura.

En educación, habilitar la oportunidad consistiría en descubrir, inventar y hacer que abrigos y travesías
le resulten efectivos a otros, y en las infancias desoladas poner vela al viento hacía nuevos destinos en
busca de puertos y viajes altamente estimados. El “¿por qué no?” calibra el instante que se presenta
con perspectivas a largo viaje, y contra el destino abre y da tiempo para salirse del sendero trazado.

b. Lo posible: pequeño itinerario pensativo con cartografía indicativa


Como la ocasión fue considerada causa secundaria, lo posible sufre la misma metafísica de ser un ser
menor, un ser al que le falta ser real. Lo posible queda en un lugar pasivo, siendo algo por venir,
inacabado, contingente, indeterminado. Como materia, es determinado por la forma, por incierto, está
enteramente dominado por lo cierto, encuadrado por lo previsible y calculado en lo probable.
En el día tras día de los pedagogos, los “factores” hacen sentir impotentes a los actores, su realidad
social parece determinar y limitar lo posible pedagógico. Porque el riesgo es que lo posible se reduzca
a lo “mismo” y se lo piense nada más que como una falta falta de forma, estabilidad, racionalidad.
Todo lo cual, al pasar de “ciencia” a acción, tiene por efecto confirmar la privación y la impotencia. La
idea “general”, “simple” no es reducirlo a lo mismo sino promoverlo reconociendole distinto potencial;
preguntándonos cómo hacer posible otro real. Parece simple pero distinto es hacerlo: lo nuevo siempre
tiene en contra aplastantes posibilidades estadísticas; en el plano teórico implica recuperar poderosas
categorías ontológicas.
Lo posible no es una realidad menor sino una promesa de otra realidad, potencial renovador. Así, el
sentido de lo posible multiplica la realidad, abre a otras realidades o a una distinta. Habilitar la
oportunidad es buscar y actualizar lo posible, imaginar y darle paso a otra realidad planteada como un
¿por qué no?. Para Musil, el hombre de lo posible es el hombre de lo imprevisto y de la improvisación,
del acontecimiento y de sus abundantes consecuencias. El hombre sin atributos es singular y atento a
lo singular, inasible y aprovecha toda ocasión de convertirla en oportunidad.
En el campo de lo educativo, habilitar la oportunidad desarrolla el sentido de la ocasión. Pedirle a un
alumno que observe los aciertos y errores de un ejercicio es muy distinto a decirle que está otra vez
mal o calificarlo de incapaz. Con el primer eco enviado lo posicionamos de otra manera, se le propone
un objeto, se le ofrece la oportunidad de comprobar y se lo alienta a recomenzar. La habilitación de la
oportunidad se adentra en la brecha, ínfima y logra otro efecto por haber buscado otros posibles que
lo probable y lo previsible. Para el educador la habilitación de la oportunidad debe ser más astuta que
la pobre ética de la ocasión de tomar, debe desbaratarla, recordando siempre la regla y tomando ese
gesto ocasional como síntoma de expoliación más radical, operar un cambio de mirada, abrir otra
posibilidad. Habilitar es reconocer, hacer capa, brindar la posibilidad real de evitar la repetición.
Sería una apuesta de confianza donde “alguien” todavía desconocido encuentre una forma que
desmienta aquella en que se lo esperaba. La oportunidad es para un ser el lugar y tiempos favorables
de un posible salto. El alumno no es una cera virgen, sino un sujeto que instituimos y se instituye. Lo
posible - el otro- es acontecimiento de una forma que se cristaliza, reorganiza y aparece en el
acontecer de la forma nueva. Parece casi nada, pero afirmaciones así, que en realidad son decisiones,
desafían a construir otros enfoques y giros del pensamiento.
En el orden del ser y el devenir: el sentido de lo posible implica no considerar lo posible como falta, de
forma y determinación, sino como reserva múltiple, mezclada con formas discernidas que en algunos
casos emergen, prueban, se pierden o refuerzan en función del eco que encuentran.
En el orden de los paradigmas causales: un pensamiento de lo posible acoge el acontecimiento y
admite saltos. No un pensamiento lineal y continúo, sino de interacción y discontinuidad.
En el orden del conocimiento: aquel que admita que pensamiento y vida creativa tienen una parte
secreta que la razón debe reconocer para existir como tal. Lo posible no es irracional sino un
desconocido “aproximable”, “perceptible”.

c. Una ética de importancia política:


¿Es una ética? Ética puede ser el nombre contemporáneo de una búsqueda moral que plantee la
dignidad de la persona humana. Puede ser universal, abstracto, un discurso “moralmente correcto”
capaz de esconder prácticas inescrupulosas. Se necesita una ética atenta, y a veces silenciosa que
sobrepase la jerarquía teórica - práctica, la oposición entre deber y circunstancias.
Se trata de una ética si en ella vemos también búsqueda de justeza, reconocimiento de una dignidad,
asunción de responsabilidad. Se trata de algo más que de ética: de ética política y de una
humanización que lleve en sí la condición política de libertad. Hacerse garante de lo posible es
proteger al porvenir y el lugar de lo desconocido en economías donde todo los lugares están previstos
y sociedades donde ya fueron ocupados.
“Posible” puede incluso entenderse como lo posible de la omnipotencia, el poderío ilimitado, el “todo es
técnicamente posible”. Nuestro pensamiento de lo posible no es omnipotente sino de lo que está en
potencia y justamente es aplastado por la omnipotencia y pide protección. En potencia está lo nuevo y
lo que se juega en abrir oportunidades es la novedades de los recién llegados.

d. Lo nuevo
En la educación, nosotros buscamos resultados tangibles, objetivables y mensurables de la acción,
porque razonamos en términos de medios y fines. De eso llevan huellas la teoría causal y las
distinciones entre lo posible y lo real, entre la potencia y el acto. Actuar es comenzar, hacer posible,
actualizar en la pluralidad humana.
Lo posible no es objeto de fabricación sino de actualización, precisamente en actos que aún siendo
reales no son realizaciones. La acción entonces tiene lugar, aparece, inaugura, muestra su principio. Al
aparecer como comienzo, muestra que es posible comenzar: muestra y al mismo tiempo efectúa lo
posible.
Apostar a la oportunidad comenzaría por resistirnos a clasificar y por el apriorismo de rechazar
apriorismos. Sería una apuesta de confianza donde alguien todavía desconocido encuentre una forma
que desmienta aquello que se esperaba. La oportunidad es para un ser el lugar y tiempo favorables de
un posible salto.
Acción como libertad y como política, que por ser comienzo actualiza la condición humana de
natalidad, la singularidad prometida. Para ello la oportunidad es imprescindible, y en dos niveles, la de
oportunidad en acto, aprovechadas y mostradas, y las artes de la oportunidad cómo educar. Habilitar
oportunidades sería eso: la capacidad de hacer que otro sea capaz de acciones oportunas.
En esta perspectiva, la educación misma es guardiana de posibles, posibles venideros capaces
también de ser arruinados por un “garante” que recuerda lo instituido. Se trata de acción y no de un
hacer previsible: estar seguro de la precariedad del éxito para estar seguros de la imposibilidad de
reverlo todo, razonar en términos de fines y medios, permanecer en la imprevisibilidad propia de la
acción. Actuar con el deseo vivo del paciente, con ciudadanos que son nuestros representados, con
seres que crecen. Son las maneras de ejercer las que actualizan tal o cual posible; el “objeto” de la
acción son sujetos, que tenemos la alternativa de convertir en objetos o reconocer como sujetos.
La cuestión está en pensar e implementar acciones que no sean acciones “sobre”, “cosificantes” sino
“con”, “subjetivantes”, instituyentes y que pongan en acto el reconocimiento del otro como sujeto y no
la reducción del mismo a objeto.
Las dificultades estarán en encontrar la plenitud de lo que significa “institución de niños”, institución
simbólica de sujetos humanos. El límite instituyente, oportuno, es también la finitud, lo cual significa
que la habilitación de la oportunidad no puede hacerse totalmente en el lugar dle otro, que es quien se
apodera de su propia ocasión, emprende esa habilitación, se autoriza y se atreve a comenzar. Pero
hay algo más que una “reforma del entendimiento educativo”, y es modificar las realidades y
tentaciones contemporáneas de una educación enteramente parametrada y funcional.
Lo posible no es virtual sino virtuosidad viviente. Lo posible es capacidad de algo nuevo que se
muestra en actos inaugurales. No hay en él poder de conquistar, destruir y fabricar, sino potencia
como capacidad actualizada de hacer existir un “antes nunca”.

● Freud - “Sobre la psicología del colegial”

Extraño sentimiento le embarga a uno cuando en años tan avanzados de la vida se ve una vez más en
el trance de tener que redactar una «composición» de idioma alemán para el colegio. No obstante, se
obedece automáticamente, como aquel viejo soldado licenciado de filas que al oír la orden de
«¡firmes!» no puede menos de llevar las manos a la faltriquera, dejando caer al suelo sus bártulos. Es
curioso el buen grado con que aceptó la tarea, cual si durante el último medio siglo nada importante
hubiera cambiado. Sin embargo, he envejecido en este lapso; me encuentro a punto de llegar a
sexagenario, y tanto las sensaciones de mi cuerpo como el espejo me muestran inequívocamente
cuán considerable es la parte de mi llama vital que ya se ha consumido. Hace unos diez años aún
podía tener instantes en que de pronto volvía a sentirme completamente joven. Cuando, ya barbicano
y cargado con todo el peso de una existencia burguesa, caminaba por las calles de la ciudad natal
podía suceder que me topara inesperadamente con uno u otro caballero anciano pero bien
conservado, al que saludaba casi humildemente, reconociendo en él a un antiguo profesor del
colegio. Pero luego me detenía y, ensimismado, lo seguía con la mirada: ¿Realmente es él, o sólo
alguien que se le asemeja a un punto de confusión? ¡Cuán joven parece aún, y tú ya estás tan viejo!
¿Cuántos años podrá contar? ¿Es posible que estos hombres, que otrora representaron para nosotros
a los adultos, sólo fuesen tan poco más viejos que nosotros? El presente quedaba entonces como
oscurecido ante mis ojos, y los años de los diez a los dieciocho volvían a surgir de los recovecos de la
memoria, con todos sus presentimientos y desvaríos, sus dolorosas transmutaciones y sus éxitos
jubilosos, con los primeros atisbos de culturas desaparecidas -un mundo que, para mí al menos, llegó
a ser más tarde un insuperable medio de consuelo ante las luchas de la vida-; por fin, surgían también
los primeros contactos con las ciencias, entre las cuales creíamos poder elegir aquélla que
agraciaríamos con nuestros por cierto inapreciables servicios. Y yo creo recordar que durante toda
esa época abrigué la vaga premonición de una tarea que al principio sólo se anunció calladamente,
hasta que por fin la pude vestir, en mi composición de bachillerato, con las solemnes palabras de que
en mi vida querría rendir un aporte al humano saber.
Llegué, pues, a médico, o más propiamente a psicólogo, y pude crear una nueva disciplina
psicológica -el denominado «psicoanálisis»- que hoy embarga la atención y suscita alabanzas y
censuras de médicos e investigadores oriundos de los más lejanos países, aunque, desde luego,
preocupa mucho menos a los de mi propia patria. Como psicoanalista, debo interesarme más por los
procesos afectivos que por los intelectuales; más por la vida psíquica inconsciente que por la
consciente.
La emoción experimentada al encontrarme con mi antiguo profesor del colegio me conmina a una
primera confesión: no sé qué nos embargó más y qué fue más importante para nosotros: si la labor
con las ciencias que nos exponían o la preocupación con las personalidades de nuestros profesores.
En todo caso, con éstos nos unía una corriente subterránea jamás interrumpida, y en muchos de
nosotros el camino a la ciencia sólo pudo pasar por las personas de los profesores: muchos quedaron
detenidos en este camino y a unos pocos -¿por qué no confesarlo?- se les cerró así para siempre. Los
cortejábamos o nos apartábamos de ellos; imaginábamos su probablemente inexistente simpatía o
antipatía; estudiábamos sus caracteres y formábamos o deformábamos los nuestros, tomándolos
como modelos. Despertaban nuestras más potentes rebeliones y nos obligaban a un sometimiento
completo; atisbábamos sus más pequeñas debilidades y estábamos orgullosos de sus virtudes, de su
sapiencia y su justicia. En el fondo, los amábamos entrañablemente cuando nos daban el menor
motivo para ello; más no sé si todos nuestros maestros lo advirtieron. Pero no es posible negar que
teníamos una particularísima animosidad contra ellos, que bien puede haber sido incómoda para los
afectados. Desde un principio tendíamos por igual al amor y al odio, a la crítica y a la veneración. El
psicoanálisis llama «ambivalente» a esta propensión por las actitudes antagónicas; tampoco se ve en
aprietos al tratar de demostrar el origen de semejante ambivalencia afectiva.
En efecto, nos ha enseñado que las actitudes afectivas frente a otras personas, actitudes tan
importantes para la conducta ulterior del individuo, quedan establecidas en una época
increíblemente temprana. Ya en los primeros seis años de la infancia el pequeño ser humano ha fijado
de una vez por todas la forma y el tono afectivo de sus relaciones con los individuos del sexo propio y
del opuesto; a partir de ese momento podrá desarrollarlas y orientarlas en distintos sentidos, pero ya
no logrará abandonarlas. Las personas a las cuales se ha fijado de tal manera son sus padres y sus
hermanos. Todos los hombres que haya de conocer posteriormente serán, para él, personajes
sustitutivos de estos primeros objetos afectivos (quizá, junto a los padres, también los personajes
educadores), y los ordenará en series que parten, todas, de las denominadas imágenes del padre, de
la madre, de los hermanos, etc. Estas relaciones ulteriores asumen, pues, una especie de herencia
afectiva, tropiezan con simpatías y antipatías en cuya producción escasamente han participado;
todas las amistades y vinculaciones amorosas ulteriores son seleccionadas sobre la base de las
huellas mnemónicas que cada uno de aquellos modelos primitivos haya dejado.

Pero de todas las imágenes de la infancia, por lo general extinguidas ya en la memoria, ninguna tiene
para el adolescente y para el hombre mayor importancia que la del padre. El imperio de lo orgánico
ha impuesto a esta relación con el padre una ambivalencia afectiva cuya manifestación más
impresionante quizá sea el mito griego del rey Edipo. El niño pequeño se ve obligado a amar y admirar
a su padre, pues éste le parece el más fuerte, bondadoso y sabio de todos los seres; la propia figura
de Dios no es sino una exaltación de esta imago paterna, tal como se da en la más precoz vida
psíquica infantil. Pero muy pronto se manifiesta el cariz opuesto de tal relación afectiva. El padre
también es identificado como el todopoderoso perturbador de la propia vida instintiva; se convierte en
el modelo que no sólo se querría imitar, sino también destruir para ocupar su propia plaza. Las
tendencias cariñosas y hostiles contra el padre subsisten juntas, muchas veces durante toda la vida,
sin que la una logre superar a la otra. En esta simultaneidad de las antítesis reside la esencia de lo que
denominamos «ambivalencia afectiva». En la segunda mitad de la infancia se prepara un cambio de
esta relación con el padre, cambio cuya magnitud no es posible exagerar. El niño comienza a salir de
su cuarto de juegos para contemplar el mundo real que lo rodea, y debe descubrir entonces cosas
que minan la primitiva exaltación del padre y que facilitan el abandono de este primer personaje ideal.
Comprueba que el padre ya no es el más poderoso, el más sabio y el más acaudalado de los seres;
comienza a dejar de estar conforme con él; aprende a criticarle y a situarle en la escala social, y suele
hacerle pagar muy cara la decepción que le produjera. Todas las esperanzas que ofrece la nueva
generación -pero también todo lo condenable que presenta- se originan en este apartamiento del
padre.
En esta fase evolutiva del joven hombre acaece su encuentro con los maestros.
Comprenderemos ahora la actitud que adoptamos ante nuestros profesores del colegio. Estos
hombres, que ni siquiera eran todos padres de familia, se convirtieron para nosotros en sustitutos del
padre. También es ésta la causa de que, por más jóvenes que fuesen, nos parecieran tan maduros,
tan remotamente adultos. Nosotros les transferíamos el respeto y la veneración ante el omnisapiente
padre de nuestros años infantiles, de manera que caíamos en tratarlos como a nuestros propios
padres. Les ofrecíamos la ambivalencia que adquiriéramos en la vida familiar, y con ayuda de esta
actitud luchábamos con ellos como habíamos luchado con nuestros padres carnales. Nuestra
conducta frente a nuestros maestros no podría ser comprendida, ni tampoco justificada, sin
considerar los años de la infancia y el hogar paterno. Pero como colegiales también tuvimos otras
experiencias no menos importantes con los sucesores de nuestros hermanos, es decir, con nuestros
compañeros. Estas empero han de quedar para otra ocasión, pues el jubileo del colegio orienta hacia
los maestros la totalidad de nuestros pensamientos.

● Lajounquiere - “La educación para la realidad del deseo”

Crítica freudiana a la pedagogía de la época en tres ensayos para una teoría sexual: La educación se
opone a los instintos sexuales.

La educación engaña en el terreno sexual e intimída desde lo religioso. Transmite la moral sexual
cultural a través de la nerviosidad moderna que afecta a los adultos. Pero con el tiempo Freud se da
cuenta que nos es posible la absoluta satisfacción de las pulsiones por ende hay una irreductibilidad
estructural del displacer psíquico

Para Freud la moral no es tan mala como pensaba en algún momento, es resultado de la represión,
posee un elemento orgánico al abandonar zonas sexuales y conquistar la posición erecta.

En proyecto de una psicología para neurólogos Freud plantea que con la represión incluida, la
organización psíquica es una adquisición biológica de barreras contra las amenazas del displacer.

Antes un inevitable displacer (único medio de educación, aparece una defensa normal y una represión
histérica. Por ejemplo con el incesto: La civilización debe renunciar a él para poder existir)
En tres ensayos para una teoría sexual propone que una insatisfacción inherente a la naturaleza
misma del instinto sexual que es muy beneficiosa para la cultura, porque es la que posibilita someterse
a las normas)

En malestar en la cultural sostiene que el displacer es el efecto inevitable de la humanización por tres
razones: la tensión contradictoria inherente a la bisexualidad constitutiva, la relación erótica conlleva
tendencias agresivas directas y la adopción de una postura bípeda lleva a la desvalorización de las
sensaciones olfativas.

Ante la imposibilidad estructural de que exista una satisfacción total y placentera, Freud deja claro que
en la educación (como en otras instituciones) siempre va haber displacer psíquico

Para Freud la educación es contingente, no necesaria.

Cuando la represión de los instintos viene de adentro es espontánea, pero si viene del exterior, no
puede desplegar la metáfora paterna porque no posee las debidas proporciones metafóricas en el
interior del proceso de la evolución psíquica, en consecuencia, la represión espontánea no es lo mismo
que la represión externa. (la moral)

Freud llamó educación para la Realidad a aquella educación en la que los adultos pudiesen dirigir la
palabra a los niños en nombre de algo que no fuese la moral de la época, que vaya más allá del goce
pedagógico hegemónico.

En el malestar en la cultura, también dice que la educación se conduce como si se enviase a una
expedición en el polo norte, vestido de verano con mapas del polo sur. Piensa una educación
psicoanalítica lejos de lo religioso, buscando un camino entre el Laissez Faire (dejar hacer, dejar pasar)
y la frustración para hacer del niño un “hombre sano y eficiente”. Esta educación tiene como meta
pedagógica: alcanzar la Realidad y estimular el vencimiento del principio de placer por la sustitución
del principio de Realidad

La Realidad para Freud: es la realidad del Deseo, es decir, la condena del hombre de estar siempre
algo fuera de foco de sí mismo y por lo tanto, también, la imposibilidad de ser el Otro.

Freud opone y propone una educación científica frente a la educación religiosa porque la religión
cierra, mientras que la ciencia abre, interroga. La religión sutura, doméstica la realidad paradójica del
deseo (motor de la voluntad de querer) en cambio la ciencia reenvía al hombre una y otra vez a su
fragilidad hacia lo transitorio de la existencia, hacia su orfandad real. La religión da consuelo y
anestesia existencial.

Para Freud la religión es justificacionista, obtura la dimensión ética de la acción humana con sus
prohibiciones y moralismo.

“cuando el hombre puede escapar de la ilusión divina puede formularse la pregunta por el deseo que
anima su acto, puede desarrollar una inquietud ética”. La moral es una contingencia de la época, no
de una voz divina. Hay que aceptar la fragilidad como inherente al hombre y no darse respuestas
hipnóticas.

Opone la verdad psicológica que entiende que el hombre está dividido por el deseo, no hay dios que
haga por nosotros y sostiene que las ilusiones son marcas del desamparo existencial a la moral sexual
cultural que produce conflicto psíquico, que reprime violentamente, no permite la tramitación de las
pulsiones, exagera las restricciones comportamentales y que sostiene que las ilusiones con marcas de
trascendencia alguna.

Freud apunta a criticar el justificacionismo (pedagogía religiosa) producto de un esencialismo cultural


(peligroso, moderno) para darle lugar al deseo (la ambivalencia, la ética).

“una educación liberada de las doctrinas religiosas que sirva para que el hombre aprenda a
soportarse preparándose para la muerte”

● Lajounquiere - “La palabra educativa entre el laissez faire y la frustración”.


En la pedagogía empera, desde hace algunas épocas, cierto justificacionismo psicológico: todo lo que
ocurre en la vida junto con los niños -ya sea en casa como en la escuela- es descifrado y justificado, a
partir de una especie de hermenéutica psico-sociológica. El tecnocientíficismo (psico) pedagógico
consuela a padres y a pedagógos, así como anestesia espíritus y corazones adultos, en el intento
siempre vano de saturar el deseo que -a su enigmática manera- anima la vida.
La educación para la realidad que Freud anhelaba apunta sin embargo, al DESEO. Ella se orienta a
evitar que los niños transiten en la vida adulta mal vestidos y carentes de buenos mapas, al punto de
quedar en riesgo de parecer resignados ante la realidad del deseo.
Llevar a los niños a reconocer que el deseo excede las fuerzas de nuestro cuerpo y excede al tiempo
exiguo de nuestras vidas, ¿constituye en efecto una meta pedagógica? y más aún, suponiendo que así
sea: ¿cuáles son los medios para alcanzarla?
La primera merece un “no” como respuesta
La segunda, ninguna respuesta conclusiva
La sujeción de un niño al deseo no constituye una nueva meta educativa. El surgimiento y la operación
de un sujeto -eso que se suele llamar “desarrollo psicológico” , depende de que el bebé sea tomado,
por sus padres, cómo estando sujeto al deseo.
En efecto, los adultos introducen una cuota de familiaridad en el bebe, y así dan el puntapié inicial en
su educación, en el proceso que pretende hacerle un miembro más de la familia. Y lo mismo vale para
la educación escolar. ¿Cuál es el objetivo de los sistemas escolares de enseñanza? Formar una nación
idiosincrásica, es decir, una especie de gran familia, en la que bien podría haber tan solo un
agrupamiento de solipsistas. Educar es transmitir marcas simbólicas que le permitan al niño
conquistar para sí un lugar en una historia, más o menos familiar, y de esa manera, poder lanzarse a
las empresas del deseo.
Pues bien, el hecho de que el Deseo sea el norte de la educación no llega hacer ninguna meta
pedagógica. Si el objetivo pretendido es el deseo, entonces puede haber una educación. Por el
contrario, si la intervención de los adultos junto a los niños está atravesada por el rechazo icc del
deseo, entonces difícilmente habrá educación.
Educar para la realidad o educar para el deseo es aquello que el hombre común sabe hacer -aún sin
saber- desde siempre cuando se trata de niños. Freud, en la ya citada lección XXXIV, afirmó que el
hombre supo hasta ese momento cumplir esa tarea educativa, aún cuando fuese de modo imperfecto.
El obstáculo para ello era, en la época y al parecer de Freud, la religión. Ahora bien, hoy el obstáculo es
-en mi opinión- el tecnocientificismo pedagógico.
¿Cómo cumplir la misión de la educación que la religión y el tecnocientificismo tornan de difícil
acontecimiento? La respuesta es: enseñando o en las palabras del poeta español Antonio Machado
#Caminante no hay camino, se hace camino al andar”.
En dos oportunidades Freud afirmó que la educación era una profesión imposible. Más aún, señaló
que no es la única, pues a ella debemos sumar también la política y el psicoanálisis
La imposibilidad hace que en el psicoanálisis, en la política y en la educación los resultados siempre
dejen de desear, Pues siempre están más acá o más allá de lo que se pretende.
Se trata de profesiones de la falta, en las que siempre algo falta en su lugar. Se trata de profesiones del
habla. Sin palabra no hay educación, no hay familia, no hay nación. Como ya he dicho hace algún
tiempo, la palabra es la herramienta educativa por excelencia.
Freud alertó acerca de la necesidad de reconocer lo imposible en la educación, en la política y en el
psicoanálisis. Así el deseo puede precipita. Es decir que él no es una entidad metafísica existente de
antemano.
¿cómo se educa? dirigiendo la palabra a un niño ¿hablando? ¡Sí! ¿cómo? como un simple mortal, como
alguien que reconoce para sí que no puede no decir -en ciertos momentos- aquello que está diciendo.
Porque en esa precisa palabra se conjugan sus esperanzas y sus miedos. Por lo tanto, si la palabra no
fuese (mal) dicha, el adulto saltaría por encima de su castración.
La palabra con posibilidades de educar es esa palabra marca la sujeción del adulto a la castración.
Esto es, esa palabra testimonio de lo que escapa al conocimiento del sujeto.
Una de las figuras de lo que escapa al adulto cuando le habla a un niño es, justamente, el control de
los efectos de ese decir. Es decir, aquello que escapa de la misma educación. La sabiduría de la
lengua afirma algo en ese sentido: “no puedo aprender por usted, sólo puedo enseñar”.

Cabe concluir, que el psicoanálisis no puede inspirar ninguna pedagogía, ninguna proposición sobre
que decirle y cómo dirigirle la palabra a un niño. El psicoanálisis reconoce que sólo puede decir a
secas: ¡Diga! ¡Hable! Por lo tanto, un educador atravesado por el psicoanálisis, en algún momento
podrá decir a secas -sin oropeles pedagógicos -: ¡Anda! Allí el sujeto tiene posibilidades de aparecer.
Pero no hay garantías, pues en ese instante, la máxima de Goethe, “lo que heredas debes
conquistarlo”, que a Freud le gustaba recordar, entra en escena mostrando todo su valor…. todo su
valor educativo

● Maldonado - “Escritos sobre Psicología y Educación.” CAP 5 y 7.

Desde nuestra óptica consideramos a la educación como un objeto complejo, el cual requiere para su
comprensión y tratamiento un quehacer interdisciplinario

Entendemos a la Psicología como un vasto y heterogéneo campo de conocimiento, generado a partir


de una profusa labor de investigación.

El SUJETO DE APRENDIZAJE (puede ser cualquier sujeto): constituido por otro sujeto, el de la
enseñanza y por el objeto de conocer/conocimiento.
Participa íntimamente en la educación. Asume una cultura institucional específica. Es un Sujeto en
desarrollo, en relación, con propósitos y en contexto.

3 componentes básicos de los procesos educativos: Alumno-Docente y los contenidos. Solo en este
encuadre cabría aludir al sujeto de aprendizaje o al sujeto de enseñanza.

ALUMNO: Protagonista de los procesos educacionales, sujeto que presenta una serie de
características psíquicas (afectivas, cognitivas)- Sujeto particular que asume comportamientos
específicos- existencia de un sujeto genérico o estándar. Si bien se admiten algunos parámetros
establecidos por la psicología de la evolución. NOS PREOCUPA que se utilicen a la manera de una
nomenclador psicopedagógico. CONVIENE postular un esquema de invariantes/variantes. Lo
invariante alude a aquellos aspectos psicológicos de carácter estructural. Se trata de referencias
generales que permiten comprender de manera global la conducta de los estudiantes.
Lo variante pretende insistir en la singularidad y diversidad de los alumnos. Por supuesto que los
sujetos y los aprendizajes varían según el momento de desarrollo, pero también, varían en razón de
determinantes históricos y/o coyunturales ajenos al sujeto (socio-económicos, familiares,
institucionales, pedagógicos, etc.)
El alumno participa de una cultura institucional que tiene entre otras cosas códigos o normas que
regulan la convivencia y la amplia gama de interacciones que en ella se establecen. El alumno lo es en
tanto, se inserta en dicha cultura con propósitos u objetivos formalmente estipulados y que se
encuentran determinados, entre otras cuestiones, por rigurosas coordenadas espaciotemporales.

El alumno es tal en la que admite una relación asimétrica con quien se denomina maestro o
enseñante. Es Alumno en tanto asume la intencionalidad de la cultura escolar, la cual prescribe
aquello que resulta conveniente y preferible incorporar y aquello que está debidamente
desaconsejado.

EL SUJETO DE LA ENSEÑANZA: Otro de los protagonistas de los procesos educacionales. A diferencia


del aprendizaje, es siempre un adulto. Hablamos de un sujeto en sentido psicológico, esto es, de la
sujetación indispensable para constituirse como ser humano. Se trata de una sujeción que podríamos
caracterizar de intrapsíquica y de naturaleza icc.
Una segunda sujeción, tiene que ver con la función que debe cumplir el maestro en tanto
representante de un orden social establecido y dominante. No se alcanza a discernir si debe
considerarse trabajador con obligaciones inherentes a su profesión o un artesano vocacional y
desinteresado.
En cuanto a lo psicológico, estamos en condiciones de asegurar que sus caracteres psíquicos influyen
de manera decisiva (cc o icc), tanto en lo que hace a su práctica pedagógica como lo atinente a su
actuación institucional. Un capitán muy valioso es inherente a sus recursos psicológicos
(particularmente, los del orden anímico e intelectual).
Este sujeto en sí mismo (maestro o profesor) conforma en sí mismo un instrumento sustantivo e
imprescindible en su quehacer educacional.
Este sujeto de la enseñanza mantiene con sus alumnos una relación asimétrica (experienciales,
intelectuales,afectivas) Aunque simétrica en otros.
El maestro está comprometido con sus dispositivos intelectuales y muy especialmente con su
estructura afectiva. Esta última influye tanto en el modo de pensar del profesor como en la
implementación que hace de su destreza pedagógica.

UNA RELACIÓN ENTRE RELACIONES: EL ALUMNO, EL DOCENTE Y CONOCIMIENTO:

Esta triangulación que nos ocupa, constituye uno de los ejes básicos en el cual giran los procesos
educacionales.
Los tres componentes aludidos conforman un sistema y su dinámica estará suspendida a sus
características y comportamientos de cada uno de aquellos en las distintas situaciones y unidades del
tiempo referencial.
No se trata de un sistema estático sino de un sistema altamente dinámico, en los cuales se puede
constatar patrones de funcionamiento diferenciados.

La supremacía ostensible de cualquiera de los componentes genera desestabilizaciones poco


propicias en los procesos educativos. Esto significa que resulta igualmente contraproducente el
exceso protagonismo del alumno, la actuación hegemónica del maestro o la presencia arbitraria o
abusiva de contenidos.
La psicología está en condiciones de aportar en relación a las exigencias psíquicas que la tarea
reclama y también acerca de los efectos que una acción pedagógica en condiciones inapropiadas
(supremacía del docente) produce.
El maestro tiene que saber respecto a sí mismo, a sus motivaciones y a sus anhelos; tiene que procurar
espacios para analizar colectiva e institucionalmente su labor.

El contenido por el contenido impone su tiranía a la hora de las planificaciones curriculares y se


desestima las formas o procedimientos para acceder al saber. A si mismo se desestima lo relativo a la
transferencia de los contenidos idóneos para comprender y transformar las distintas realidades que
los alumnos en cuanto ciudadanos deben afrontar cotidianamente. Tampoco se insiste
suficientemente en lo que hace a la actitud del sujeto en cuanto al conocimiento; se minimiza o
desconoce la vinculación entre apropiación y placer. El interés del sujeto que aprende es escasamente
considerado en este tipo de posiciones hipercontenidistas.

En suma, nuestra reserva básica tiene que ver con la supremacía más o menos permanente de
cualquiera de los componentes.
Se trata entonces, de una relación primordial que conforma una trama a veces muy difícil de
desmontar o precisar, en tanto que cada uno de los componentes presenta matices tangibles y
también dimensionales no explícitas. Las de este orden suelen estar emparentadas con la subjetividad
propia de cada sujeto, se trate de un alumno o un docente.

Resulta imprescindible para el maestro o sujeto de enseñanza conocer al otro con el cual interactúa
cotidianamente en todos sus aspectos, saber que está siempre involucrado en la relación con ese otro
que es el alumno y diseñar un encuadre de trabajo que permita orientar técnicamente su práctica.

LA EXCLUSIÓN DEL DOCENTE COMO SUJETO PSÍQUICO EN EL PROCESO EDUCATIVO


Observamos la escasa consideración que se le asigna a la incidencia que los aspectos psíquicos del
profesor tienen sobre los estudiantes y en los estilos de aprendizaje que ellos desarrollan. El docente
habitualmente asume dicha situación y supone que lo importante para su labor se circunscribe en la
técnica que dispone para ejercer su profesión de enseñar, a sus conocimientos disciplinarios o en el
mejor de los casos, se instala como ideal (positivo) a partir del cual los alumnos deben identificarse.

Algunos aportes de la investigación psicoanalítica : Supone que es necesario un saber extra


académico que pueda ver los procesos subjetivos, profundos y complejos, vía la exploración de las
vicisitudes de su propia vida psíquica. Si no indaga en sí mismo su práctica tendrá fisuras evidentes,
puntos ciegos que restan eficacia a su tarea. Deben alcanzar sus competencias profesionales por una
doble vía: académica y el análisis personal. Esto significa, que para su trabajo tiene notable
importancia tanto los conocimientos disciplinarios como el saber que adquieren sobre sí mismos.
Su subjetividad constituye paradójicamente tanto un obstáculo (cuando se la ignora y
sobredimensiona la calificación teórica-técnica) como una vía regia cuando ha sido y es explorada
hasta sus instancias más recónditas
Hay que encontrar una distancia óptima; ni tan distante que convierta a la práctica en un mero acertijo
intelectual ni una tan cercana que se contamine con sus propios afectos no esclarecidos.
Para que ello funcione tiene que apelar a cierta regulación que consiste en instalar y acogerse un
conjunto de reglas muy precisas y configurar un encuadre sumamente riguroso. En consecuencia
convierten su condición de sujeto psíquico en un formidable recurso técnico.

Aportes de las investigaciones psicológicas sobre las vicisitudes del aprendizaje:


La psicología de la objetividad que aquél le enseñaba, prescribía la total indispensable desvinculación
entre el objeto de conocimiento y objeto cognoscente.
La concepción positivista mantiene una persistente y amplia influencia en el campo educacional.
Quizás ello explique la desconsideración que el docente hace de sus propios procesos de enseñanza e
incluso, permitan justificarlo cuando asevera que dichos conocimientos resultan tan improcedentes
cómo banales en la tarea efectiva del enseñar.
La importancia de las tempranas experiencias de aprendizaje del sujeto, en tanto le permite
conquistar un sentimiento de identidad intelectual. De allí que llama a evitar cualquier desacreditación
desde la mirada adulta.
Las historias de aprendizaje del niño Papert, del niño Piaget, y de otros tantos niños nos revelan el
impacto que los primeros aprendizajes tienen en la obtención de la autonomía e iniciativa intelectual y
cómo su ponderación de análisis proveen excelente información para comprender las vicisitudes del
aprendizaje tanto propias como ajenas. La escuela habitualmente sigue otro camino convocado a
olvidar la singularidades promoviendo modos oficiales y veladamente únicos para acceder al
conocimiento estipulado como correcto
La escuela suele desconfiar de los aprendizajes no oficiales esto es de aquellos que surgen del interés
particular, o bajo estilos novedosos y personales, o que tienen origen extra-áulicos, o que en principio
se manifiestan prematuramente.
Dará por el contrario un fuerte respaldo a lo que llama comprensión intuitiva y enfatiza que la negación
del conocimiento personal ha producido lamentables escisiones entre el aprendizaje de la escuela y el
aprendizaje individual que tiene lugar fuera de ella.
En este sentido deseamos insistir en esa configuración que denominamos estilo de aprendizaje.
Usamos esta noción para indicar que el aprendizaje tiene una conformación heterogénea y que
trasciende el simple mecanismo cognitivo en principio involucrado en un acto de apropiación de la
realidad en general y del contenido curricular en particular.
La noción de estilo nos invita a pensar en los distintos componentes (físicos, psíquicos, sociales) que
participan estructural o coyunturalmente en el hecho de conocer. También nos impulsa a reconocer y
distinguir los aspectos generales pero sobre todo los particulares que tienen lugar en cada situación
epistémica
La formulación busca neutralizar la concepción tan extendida y fácil que prefiere entender el
aprendizaje como un algoritmo estrictamente intelectual. De allí que la expresión que auspiciamos
integra componentes afectivos diversos (históricos y/o actuales), componentes biológico y físico y
componentes socioculturales (género, etnias, posición social, por ejemplo).

La escuela se muestra más interesada por los que (más a fin de la ideología del rendimiento favorecida
por el establishment) que por los cómo (más afín a la idea de producción a cargo del sujeto
cognoscente y en función de su interés) y nuestro autor agrega que aprender a aprender y el teorizar
sobre cómo aprendo tienen un valor superlativo.
Ahora bien, en tanto educador emergente de una cultura complaciente con los modelos epistémicos
positivistas, en lugar del docente debe ser consecuente con ideas afines a la que Lacan define con la
noción de sujeto supuesto saber, lo cual implica una marcada imposibilidad de abrirse al otro, de
escucharlo y al mismo tiempo de escucharse y saber de uno mismo.
Evidentemente estamos en presencia de un sistema cerrado, que no interactúa y se realimenta
magramente con el medio, endogámico según los etnógrafos. Esta posición que se le exige y asume lo
aleja de la extraordinaria posibilidad de aprender, de renovarse y transformarse permanentemente.
Así es factible sorprender que el docente padece algo equivalente a una notable inhibición para el
aprendizaje, para el acceso y/o producción de lo nuevo en el ámbito del conocimiento

La situación en el territorio educativo formal


Educación podría definirse como una práctica social específica y la psicología como una disciplina
científica
Adscribimos a una educación que procure una efectiva igualdad de oportunidades sin desentenderse
de la calidad y equidad. En este marco le cabe estipular, como fin insoslayable, promover la
construcción y la creatividad permanente tanto de los alumnos como de los docentes.
Es sabido que la pedagogía tiene una vocación fuertemente prescriptiva y otorga en la práctica
insuficiente crédito a lo singular y aquellas modalidades que impulsan la creatividad.
Donde la psicología ha tenido escasa repercusión es en todo lo relativo al docente o enseñante.
La educación no le concede, salvo a las propiedades indicadas y a otras equivalentes, un valor
ponderable a la persona del docente en cuanto a su práctica profesional.
El docente no debería excluirse como sujeto en el curso de los procesos educativos que gestiona. Le
conviene saber que siempre se implica y que si reconoce y modera su implicación, su labor puede ser
muy fértil en cuanto a la calidad y equidad de la enseñanza que promueve.
La toma de conciencia del compromiso afectivo que ello conlleva y la búsqueda de la distancia
apropiada para afrontar la cotidianidad operará a favor del mismo docente y de las relaciones
institucionales que establezca.
El admitir y manejar la ansiedad que la inestabilidad laboral y las transformaciones de su práctica le
provoca ayudará a neutralizar el deterioro de su salud mental.
Cuando hablamos de la exclusión del docente como sujeto, lo hacemos en tanto la concebimos como
sujeto deseante y como sujeto epistémico (tiene un estilo cognoscitivo particular que lo caracteriza
dicho estilo es considerado muy vagamente o de manera estereotipada).
El desconocer sus propios modos de relacionarse con el conocimiento y ejercer el aprendizaje podría
interpretarse como la emergencia del punto ciegos que afectan y obstruyen su práctica profesional.
¿Qué significa incluirse como sujeto en su práctica pedagógica o ir más allá de sus habilidades
técnicas? significa que como enseñante siente y piensa, esto es, que también es un sujeto deseante y
sujeto epistémico y no meramente un homofaber didáctico.
Significa que si considera lo que siente cuando enseña, enseñará mejor; si no reprime o reniega lo que
siente cuando produce un proceso educativo tendrá mejores posibilidades de generar un vínculo
apropiado con los alumnos a su cargo y con ello obtendrá ventajas en alcanzar los objetivos
estipulados.
El docente que se excluye se convierte en un docente que se generaliza y entonces no simplemente
hace caso omiso a la singularidad de sus alumnos sino que recíprocamente hace caso omiso a su
propia singularidad y se enajena.
Al docente le beneficia tener conciencia de su sentir singular. Conforma un recurso primordial que
puede utilizar en su profesión de enseñar en tanto provoca réditos para su propia persona, para la de
los alumnos y especialmente para el proceso de enseñanza-aprendizaje en que está laboralmente
involucrado de manera cotidiana.
Algunas consideraciones finales
El maestro seguirá siendo un componente indispensable de los procesos educacionales. Sin embargo,
para que ello ocurra seguramente convendrá que mude sus roles con mucha más frecuencia de la
imaginada por estos tiempos y los optimice de forma continua, casi como una rutina laboral.
Nos parece promisorio que su tarea se incline cada vez más hacia el ayudar a aprender, a estimular el
aprender a aprender, acompañar los aprendizajes de sus alumnos y a valorar fundamentalmente los
procedimientos y actitudes que éstos generen.
Su ejercicio supone cada vez más ingentes esfuerzos (socio-psico-pedagógicos) para el docente.
El saber sobre sí mismo, antes que una flaqueza a disimular o reprimir, puede constituir un punto de
apoyo para fortalecer sus actividades de enseñante. Lo cual implica que si no se excluye
flagrantemente, podrá disponer de mejores recursos para atender sus vicisitudes laborales, esto es,
convivir más placenteramente en la institución escolar, generar vínculos más satisfactorios con sus
colegas, establecer relaciones más productivo con sus alumnos, alentar una distribución más
equitativa y calificada de ese bien social que es la cultura y la educación.
Ella también ayudará a que el enseñarte se valore a sí mismo de otra manera, rompa un tanto con la
consideración incierta que se le otorga desde el afuera y de la cual suele llegar a depender en
extremo. Es decir, que se atreva a construir nuevos perfiles profesionales que no descarten su propia
condición de sujeto psíquico y que como corolario hagan epicentro no simplemente en la función de
docente/instructor sino en la de maestro/formador.

● Skliar- “¿ Y si el otro no estuviera allí? Notas para una pedagogía (improbable) de la diferencia”.

Todo es posible con el cambio en educación, la reconversión, transposición; la pedagogía de las


supuestas diferencias en medio de un terrorismo indiferente y la producción de una diversidad que
apenas se atiende, apenas se entiende, apenas se siente.
En la especialidad y la temporalidad de la modernidad tardía, de la modernidad como riesgo se ha
instalado cómodamente la idea de las transformaciones, de la imagen vertiginosa del mundo.
Afirmamos que estamos frente a un nuevo sujeto. Un nuevo sujeto de la mismidad. Porque se
multiplican sus identidades a partir de unidades ya conocidas.
Y si el otro no estuviera aquí?
Pero ni una palabra sobre las representaciones como miradas, sobre la metamorfosis de las
identidades, sobre la vibración con el otro. El cambio ha sido, entonces, la burocratización del otro , su
inclusión curricular.
¿Es la educación el imperio de la mismidad y la desolación de la alteridad?
La educación institucional, la institución educativa, la escuela, es una invención y un producto de
aquello que denominamos como modernidad. Las conclusiones, sobre la relación entre modernidad
educación y escuela son evidentes: el tiempo de la modernidad y el tiempo de la escolarización suelen
ser, como hojas calcadas , temporalidades que solo desean el orden, que se obsesionan por clasificar,
por producir mismidades homogéneas, íntegras sin fisuras, a salvo de toda contaminación del otro; la
espacialidad de la modernidad y el espacio escolar suelen ser, como hermanas de sangre, que solo
buscan reducir al otro lejos de su territorio, de su lengua, de su sexualidad, de su género, etc.
Modernidad y escuela, donde dos cosas distintas no pueden estar al mismo tiempo y dónde una
misma cosa no puede estar en dos sitios al mismo tiempo. Cosas distintas como mismidad / alteridad.
Y es válido también sustituir dos sitios, por un sitio para la mismidad y otro sitio bien diferente para el
otro . Un sitio estable, ordenado, lineal, para la mismidad. Y otro sitio bien diferente, pero de mucho
mayor orden, de mucho mayor control, de mucho mayor gobierno, un sitio deliberadamente sin tiempo
y sin espacio para los otros.
La tarea de educar se transformó en un acto de fabricar mismidades y allí se detuvo, estableció un
orden, una jerarquía de sumas y restas de sujeto y predicado, de historias e historia, exclusión y de
inclusión, de ángeles y réprobos .
Por otro lado, el acto de educar tomó otro rumbo, sobre el cual nunca se detuvo, pues nunca lo dio por
cumplido, nunca lo dio por acabado. La educación que nunca termina y que nunca se ordena. La
educación como poiesis, es decir, como un tiempo de creatividad y de creación que no puede ni quiere
orientarse hacia lo mismo , hacía la mismidad . La educación como la construcción de otro que
repercuta en la mismidad.
¿podríamos pensar en una pedagogía de la perplejidad?
Una pedagogía de la perplejidad que sea un asombro permanente y cuyos resplandores nos impidan
capturar la comprensión ordenada de todo lo que ocurre alrededor. Una pedagogía del
acontecimiento, es decir, pedagogía discontinua que provoque el pensamiento, que retire del espacio
y del tiempo todo saber ya disponible; que obligue a recomenzar de cero, que haga de la mismidad un
pensamiento insuficiente para decir , sentir, comprender aquello que ha acontecido que enmudezca la
mismidad.
Y qué desorden el orden, la coherencia toda pretensión de significados. Y que posibilite la vaguedad,
la multiplicación de todas las palabras, la pluralidad de todo lo otro . Una pedagogía para un presente
distintivo que puede ser, al mismo tiempo, tres posibles modos de entender la educación:
La pedagogía del otro que debe ser borrado es el nunca – otro y el siempre – otro : el otro permanente.
Siempre existió como otro de lo mismo, como una repetición de la mismidad.
Está pedagogía está cimentada sobre dos principios pedagógicos:
está mal ser aquello que se es o se está siendo la negación del otro en sus propias experiencias de ser
otro, en su devenir , en su lengua en sus temporalidades espacio, acontecimiento. Y mostrarle al otro
que está mal corregirlo, normalizarlo , medicalizarlo, silenciarlo, vociferar lo, producirlo. Obligarlo a qué
se perciba que está mal ser aquello que se es o que está siendo.
Está bien ser aquello que no se es, que no se está siendo y que nunca se podrá o querrá ser. La
mismidad como modelo distante y etérea de toda alteridad. Y mostrarle al otro que está bien ser
aquello que no se es siendo o no podrá: disfrazarlo de diversidad, teñirlo de alteridad, hacerlo divergir
de lo mismo, alejarlo, medirlo, evaluarlo, excluirlo/ incluirlo.
La pedagogía del otro que debe ser borrado una pedagogía para que la mismidad pueda ser siempre
la única temporalidad y especialidad posible.
La pedagogía del otro como huésped es una pedagogía cuyo cuerpo se reforma y / o se autoreforma,
haciendo metástasis sobre lo mismo y sobre el otro; es l ambición del texto de la mismidad que intenta
alcanzar al otro , capturar, domesticar, darle voz para que diga siempre lo mismo, escolarizarlo cada
vez más para que cada vez más pueda parecerse a lo mismo y sea lo mismo.
Es la pedagogía de la diversidad como pluralización de yo mismo y de lo mismo; una pedagogía que
hospeda, que alberga; pero una pedagogía a la cual no le importa quien es su huésped sino que se
interesa por la propia estética del hospedar , del albergar. Que reúne al mismo tiempo, la hospitalidad y
la hostilidad hacia el otro.
Es una pedagogía que se obsesiona , de un lado , con la entrada y con la permanencia a la escuela
regular de aquellos sujetos comúnmente denominados como deficiente. Y que hoy por otro lado ,
estiliza su mismidad atribuyéndole a los otros el carácter de diversidad, de diversos en una relación
cultural, política, lingüística, y comunitaria por lo menos confusa caótica, discriminatorio, violenta,
excluyente, autoritaria, en síntesis de estricta autoridad colonial.
Colonial cuando pensamos en la idea de integrar al otro: esto es reunir en un mismo lugar, juntar lo que
está suelto, aproximar las partes que están separadas y de incluirlo, inclusión puede ser definido ,
entonces, cómo tener como miembro, contener como elemento secundario o menor.
Por eso está pedagogía del huésped debe colocarse en suspenso y ser mirada con desconfianza.
En primer lugar considerarla como una categoría epistemológica, una descripción que emerge a partir
de una etnografía displicente y autosuficiente, como simple dato o hecho de la vida social. Muchas
veces la diversidad es utilizada como un bálsamo tranquilizante con el objetivo de anular o amortiguar
los conflictos culturales y sus efectos; un bálsamo que crea la falsa idea de una equivalencia dentro de
la cultura y entre culturas. La afirmación de la diversidad supone el reconocimiento de contenidos y
costumbres pre- establecidos exentos de mezcla y contaminación.
En segundo lugar los usos de la diversidad: político o gubernamental, empresarial ( lógica del mercado)
, cultural y pedagógicos. Por uso político entiendo una forma de administración y referencia a las
sociedades que se autoproclaman como multiculturales, donde los otros, algunos otros son utilizados
para fijar una imagen satisfactoria completa y no conflictiva. Se mantiene una relación de poder entre
quien hospeda y quién es hospedado que debe en la mayoría de los casos des- vestirse de
tradiciones, des- culturalizarse, des- comunalizarse, des- corporalizarse , des- tituirse como sujeto
para ocupar el lugar de la diversidad.
La pedagogía del otro como huésped de nuestra mismidad hostil. Una pedagogía donde:
• La diversidad es presentada como algo reciente y problemático.
• Diversidad y deficiencia se confunden
• Diversidad y heterogeneidad son sinónimos
• Diversidad es siempre otro, que asume diferentes rostros, nombres, colores, cuerpos.
• Diversidad es todo y nada,, ya que todo diversidad y todos somos diversos.
• La atención a la diversidad está individualizada en los sujetos problemático
• Se reiteran cuestiones de tolerancia, diálogo, respeto, aceptación y reconocimiento del otro
• Estás cuestiones sólo son abordadas como temáticas
• Las expectativas se centran en una mejoría del rendimiento escolar, el avance del dominio del
conocimiento curricular.
La pedagogía del otro que vuelve y reverbera permanentemente es la Pedagogía de un tiempo otro,
de otro tiempo; de una espacialidad otra, de otra espacialidad. Una Pedagogía que tal vez no ha
existido nunca y que tal vez nunca existirá. Si no puede ocultar barbaries y gritos despiadados de los
mismos, que no pueda enmascarar la repetición monocorde y que no pueda tampoco ordenar,
nombrar, definir o hacer congruente los silencios, los gestos y las miradas y las palabras del otro.
Una pedagogía que en el presente, pudiera instalarse pero no acomodarse, entre la memoria y el
porvenir. Porque si el otro no estuviera ahí y allí y aquí – nuestras pedagogía quedarán reducidas a
cenizas , envueltas en borrascas, disueltas en pura mismidad. Nuestras pedagogía no nos dejarán
vibrar con el otro. Pues con tantas reformas nos reformen a nosotros mismos de una vez .

Las siguientes preguntas fueron formuladas como una guía de lectura que recupera
los ejes centrales del material de lectura. “Docentes en la tarea de cruzar fronteras y tender puentes”.

a. ¿Por qué puede hablarse de la "fabricación" del rol docente?


De acuerdo al autor el docente debe transformarse en un constructor y servir como “puente”, para
poder transmitir no sólo el saber científico sino también la cultura en la que vivimos; contraponiéndose
a la idea de la figura fuerte y la autoridad construida en torno a una persona en relación a un trabajo
individual, con un dominio del saber frente al aula, digno de imitar.
La tarea docente gira en torno a pensar, reflexionar y seleccionar qué de ese presente deseamos
transmitir a nuestros alumnos. A partir de esto surge el docente como un “cruzador de fronteras”,
llevando a cabo un proceso en el cual se construye el saber
traduciendo, articulando y ayudando a construir referencias nuevas. Los docentes como interpretes
(puentes) entre generaciones para construir la cultura, entendiendo a la cultura como una
construcción social.

b. ¿Qué implicancias tiene pensar el método como artificio?


En los orígenes de la organización de una carrera profesional para el magisterio, la docencia no era
considerada una profesión liberal sino, una carrera esencialmente burocrática cuya necesidad
dependía de los requerimientos del Estado.
Esta tarea docente era pensada en un espacio de determinación que reunía las ideas de eficiencia y
orden, a través de una propuesta tecnocrático- moralizadora, sobre la base de algunos valores que se
pretendían “salvadores”. A partir de esa idea, pensar el método como artificio implicaba: la carencia
del sentido común, el no tener contacto con la realidad del contexto en el que se desarrollaba el
proceso de enseñanza y la insuficiencia de una dinámica social.

c. ¿Qué procesos intervinieron para que se caracterizará a los docentes como apóstoles en distintos
momentos históricos?
La docencia nace como un apostolado, el docente se caracteriza por su abnegación, entrega, con
bajos salarios, constituyéndose en un verdadero apóstol de la educación.
Nace así el docente con vocación. A mitad del siglo XX se produce la asunción del rol docente basado
en la identidad personal, como trabajador de la educación. En 1950 se establece una relación
estrecha entre desarrollo económico y desarrollo educativo.
A partir de entonces, el Estado diseña políticas educativas tendientes a regular la tarea docente. Por
ello se gesta en 1958, El Estatuto del Docente. Con la aparición de éste, aparece la idea de
profesionalización docente y comienza a gestarse el concepto de que el maestro no sólo nace sino
también se hace.

d. En este fascículo se hacen reiteradas referencias acerca de la mirada sospechosa que la escuela
ha tenido sobre la cultura y los desarrollos innovadores que son contemporáneos a ella. ¿Qué efectos
ha tenido esto? ¿Qué ejemplos actuales de esta tendencia se podrían dar?
En principio la escuela es considerada un templo del saber donde la cultura popular y
la cultura propia del alumno, no tenían cabida en la educación sistemática ni en las enseñanzas
impartidas. Los límites entre el afuera y el dentro estaban estrictamente
marcados y el adentro era percibido como algo superior y más valioso que el afuera, con
la amenaza de contaminar al alumno. esta es la mirada sospechosa de la escuela hacia
la cultura.
En los años 20, el pensador John Dewey indujo al currículum de la escuela temas populares como el
fútbol, el cine, la prensa, los viajes. En la actualidad persiste esta visión positiva o de inclusión de la
cultura en la escuela, incorporando al alumno con todo
su bagaje cultural.
Ejemplo de ello es la enseñanza a partir de la contextualización. El aprendizaje de la lengua se
sustenta teóricamente en la lingüística del texto, que considera a la lengua como un elemento vivo que
se produce en un contexto determinado. Por ello se empiezan a incorporar documentos auténticos
como herramientas de estudio. Otro ejemplo: en Matemática se enseñan temas de estadística a partir
de experiencias vivenciales de la realidad del alumno y de su interés personal, logrando así
aprendizajes significativos.

e. Detengámonos en la siguiente frase: "Profesoras y profesores unidos de


métodos sólidos y homogéneos, aplicando los mismos métodos que sus colegas y persiguiendo
idénticos objetivos". ¿Es factible sostener esta afirmación en la actualidad? ¿Qué la hizo factible en el
pasado? ¿Cuáles serían las consecuencias de sostener esa afirmación hoy?
Enseñar es, a riesgo de ser un poco esquemáticos, establecer una relación, es decir, construir una
posición que no está situada en coordenadas predefinidas, fijas y definitivas sino que sufre
alteraciones y busca e inventa respuestas. El papel del profesor no puede ser pensado hoy como un rol
escrito de antemano. Los educadores de las escuelas
normales, a fines del siglo XIX y comienzos del XX, pensaron que la escuela debía civilizar el mundo,
formar sujetos nuevos −"ciudadanos letrados" Esa forma de escuela era entendida como una especie
de injerto que venía a "civilizar" esas poblaciones, a incorporarlas a la sociedad nacional con la
promesa del bienestar, a costa de expropiar sus territorios, sus saberes y su participación activa. Ésta
autoridad gozaba de gran prestigio en la sociedad, y sobre esa base se consolidaba una sólida alianza
con las familias en pos de la educación de las nuevas generaciones.
Podemos autorizarnos como intérpretes, como puentes que dibujen otros cruces entre las
generaciones; la escuela sigue siendo, en efecto, un puente valiosísimo para configurar este y otros
mundos posibles.
En sus comienzos, la escuela, con su finalidad homogenizadora de la población, contaba justamente
con grupos homogéneos a los cuales transmitir los saberes socialmente relevantes y necesarios para
construir al ciudadano pretendido por el estado nacional. La autoridad y el prestigio del que a nivel
social contaba el docente, respaldaban esta transmisión de forma unificadora. La consideración del
alumno como una tabula rasa, dejaba de lado sus saberes previos e impartía contenidos de una forma
mecánica y acrítica.
Las características del contexto actual, la transformación de la institución educativa y
con ella del rol docente, hacen que no sea posible concebir métodos sólidos, homogéneos y únicos,
sino que se hace necesario plantear una educación inclusiva, abierta a la diversidad de contextos y de
individualidades que hoy definen al sujeto que aprende en nuestras escuelas. Por lo tanto, la
educación debe aceptar el desafío de
flexibilizar su propuesta frente a nuevas construcciones familiares, realidades sociales y económicas
diversas, que implican que la escuela, con la pretensión de ser inclusiva y
obligatoria, se abra a esta multiplicidad de roles y funciones.

f. ¿Qué significa pensar la vocación como efecto de una dinámica social?


Se debe pensar la vocación no como atributo individual para su trabajo sino como una mediación
entre valores universales e individuos particulares. Es un proceso de producción cultural y social, que
se forma entre las urgencias, prioridades y dilemas de
cada momento histórico.

g. ¿Puede construirse una relación de enseñanza que sostenga una asimetría


respecto de los saberes y responsabilidades pero que sea de semejanza en los aspectos humanos y
ciudadanos?
Se cree que sí, ya que implica sostener una mirada cautelosa para que la posición de quien orienta
con respecto a quien es orientado no se deslice hacia una desigualdad irremediable que elimine toda
dignidad. La docencia es un trabajo social que se constituye en el entramado de diversas experiencias
(escolares y extraescolares) y ese dialogo con la cultura de su tiempo es central para los puentes que
los docentes habilitan para sus alumnos.

También podría gustarte